TẬP tài LIỆU CHUYÊN đề bồi DƯỠNG ôn THI đại học các môn học TỈNH VĨNH PHÚC

262 2.3K 0
TẬP tài LIỆU CHUYÊN đề bồi DƯỠNG ôn THI đại học các môn học TỈNH VĨNH PHÚC

Đang tải... (xem toàn văn)

Tài liệu hạn chế xem trước, để xem đầy đủ mời bạn chọn Tải xuống

Thông tin tài liệu

SỞ GD & ĐT TỈNH VĨNH PHÚC TRƯỜNG THPT VĨNH YÊN TẬP TÀI LIỆU CHUYÊN ĐỀ BỒI DƯỠNG ÔN THI ĐẠI HỌC NĂM HỌC 2013 - 2014 Vĩnh Yên, tháng 3 năm 2014 1 TT Môn 1 Toán Đại Văn Hải 2 Toán Ngyễn Thị Thanh Hải 3 Tác giả Vật lý Đào Thị Phương Lan Tên chuyên đề Ứng dụng hàm số trong giải phương trình, hệ phương trình và bất phương trình Các dạng toán về phép đếm Phương pháp giải bài toán viết biểu thức cường độ dòng điện và điện áp trong mạch điện xoay chiều 4 Hóa học Hệ thống Lý Thuyết và bài tập về kim loại Nguyễn Thị Lan Phương kiềm, kiềm thổ và hợp chất quan trọng của chúng 5 Ngữ Văn Lê Thu Hà Việc biên soạn hệ thống đề hướng dẫn cho học sinh ôn thi đại học tác phẩm CHí Phèo của Nam Cao 6 Ngữ Văn Nguyễn Văn Lự Dạy kỹ năng làm văn – một số giải pháp quan trọng giúp học sinh lớp 12 thi đại học đạt kết quả cao 7 Lịch sử Nguyễn Tuyết Mai Lịch sủ thế giới – các nước Á – Phi – Mĩ La Tinh (1945-200) 8 Địa lý Lê Thị Thúy Oanh Địa Lí dân cư – hướng dẫn ôn luyện phần địa lí dân cư lớp 12 bằng phương pháp sử dụng “Sơ đồ” 9 Tiếng Triệu Thị Hằng anh Reported speech 10 Tiếng Đỗ Thị Thu Minh anh TEACHING SKILLS TO DO READING COMPREHENSION TESTS 11 Sinh học Phương pháp giải một số dạng bài tập di truyền liên kết với giới tính Nguyễn Thị Thúy 2 SỞ GIÁO DỤC VÀ ĐÀO TẠO VĨNH PHÚC TRƯỜNG T.H.P.T VĨNH YÊN ˜–µ—™ CHUYÊN ĐỀ ÔN THI ĐẠI HỌC MÔN: TOÁN ỨNG DỤNG HÀM SỐ TRONG GIẢI PHƯƠNG TRÌNH, HỆ PHƯƠNG TRÌNH VÀ BẤT PHƯƠNG TRÌNH GIÁO VIÊN: §¹i V¨n H¶i TỔ: TOÁN – LÍ- KT TRƯỜNG: THPT VĨNH YÊN-TP VĨNH YÊN-VĨNH PHÚC Vĩnh Yên, tháng 3 năm 2014 3 CHUYÊN ĐỀ ÔN THI ĐẠI HỌC ỨNG DỤNG HÀM SỐ TRONG GIẢI PHƯƠNG TRÌNH, HỆ PHƯƠNG TRÌNH VÀ BẤT PHƯƠNG TRÌNH Tác giả: Đại Văn Hải Giáo viên trường THPT VĨNH YÊN Đối tượng học sinh: Lớp 12, Ôn thi ĐH – CĐ Số tiết dự kiến:12 tiết A. ĐẶT VẤN ĐỀ Bài toán giải phương trình, hệ phương trình và bất phương trình là một bài toán cơ bản trong chương trình phổ thông thường giải bằng một trong 4 phương pháp ( phương pháp biến đổi tương đương, đặt ẩn phụ, đánh giá, phương pháp hàm số) và xuất hiện thường xuyên trong các đề thi Đại học-Cao đẳng có lời giải dùng phương pháp hàm số. Phương pháp hàm số thường dựa vào một số tính chất của hàm số và đồ thị hàm số nhờ vào dùng đạo hàm (ở lớp 12) khảo sát chiều biến thiên của hàm số để xét nghiệm phương trình. Chuyên đề này nhằm ôn tập cho các em học sinh các kiến thức về hàm số và kĩ năng giải phương trình, hệ phương trình, bất phương trình bằng phương pháp hàm số. B. NỘI DUNG I. HỆ THỐNG KIẾN THỨC CẦN SỬ DỤNG TRONG CHUYÊN ĐỀ: 1. Định nghĩa: Cho hàm số f(x) xác định trên K a) f: đồng biến trên K ⇔ ∀x1 , x2 ∈ K , x1 < x2 ⇒ f ( x1 ) < f ( x2 ) b) : nghịch biến trên K ⇔ ∀x1 , x2 ∈ K , x1 < x2 ⇒ f ( x1 ) > f ( x2 ) c) Hàm số đồng biến trên K hoặc nghịch biến trên K ta gọi chung là hàm số đơn điệu trên K 2. Các tính chất: Tính chất 1: Cho f (x) đơn điệu trên K, ∀x1 , x2 ∈ K a) f ( x1 ) = f ( x2 ) ⇔ x1 = x2 b) f ( x1 ) > f ( x2 ) ⇔ x1 > x2 c) f ( x1 ) < f ( x2 ) ⇔ x1 < x2 Tính chất 2: Nghiệm phương trình f ( x) = 0 là hoành độ giao điểm của đồ thị hàm số và trục hoành. Tính chất 3: Nếu hàm số f (x) đơn điệu trên khoảng (a; b) thì phương trình f ( x) = 0 có không quá một nghiệm trên khoảng đó. 3. Định lí : Cho hàm số f(x) có đạo hàm trên khoảng (a; b) 4 Nếu f '( x) ≥ 0 (hoặc f '( x) ≤ 0 ) , ∀x ∈ (a; b) và dấu đẳng thức chỉ xảy ra tại hữu hạn điểm trên khoảng (a; b) thì f (x) đồng biến ( hoặc nghịch biến) trên trên khoảng (a; b). II. CÁC DẠNG BÀI TẬP ĐẶC TRƯNG TRONG CHUYÊN ĐỀ: 1. Giải phương trình : Dạng 1: D¹ng F( x ) = 0, víi F( x ) hoÆc ®ång biÕn, hoÆc nghÞch biÕn trªn D. Bước 1: Xét hàm số y = F( x ) Chỉ rõ hàm số y = F( x ) đồng biến hay nghịch biến trên D. Bước 2: Đoán được F ( x0 ) = 0 . Lúc đó phương trình có nghiệm duy nhất x = x0 .  F( x ) ®ång biÕn  G ( x ) nghÞch biÕn Dạng 2: F( x ) = G ( x ) với  Bước 1: Tìm TXĐ: D Bước 2: Xét hai hàm số y = f ( x ) và y = g ( x ) Chỉ rõ hàm số y = F( x ) là hàm đồng biến(nghịch biến) trên D và y = G ( x ) là hàm nghịch biến (đồng biến) trên D. Bước 3: Đoán được F ( x 0 ) = G ( x 0 ) . Lúc đó phương trình có nghiệm duy nhất x = x0 . Dạng 3: D¹ng ph­¬ng tr×nh F(u) = F(v) (*), víi F( x ) hoÆc ®ång biÕn, hoÆc nghÞch biÕn trªn ( a; b ) . Lóc ®ã, (*) ⇔ u = v Bước 1: Xét hàm số: y = F(t ) . Chỉ rõ hàm số đồng biến hay nghịch biến trên ( a; b ) . Bước 2:Với u và v nhận giá trị trên (a; b). Khi đó: F (u) = F(v ) ⇔ u = v 2.Giải hệ phương trình: Bài toán: Giải hệ F(x,y) = 0 G(x,y)= 0 (I) Nếu một trong hai phương trình của hệ đưa về dạng: f(x) = f(y) (1) hoặc ( f u ( x )  = f  v ( y )  và f là một hàm đơn điệu thì: Hệ (I) ⇔ x = y hoặc (I) ⇔ u(x) = v(y) (III) (II) G(x,y)= 0 G(x,y)= 0 3.Giải bất phương trình : D¹ng F ( x ) ≥ 0, víi F( x ) hoÆc ®ång biÕn, hoÆc nghÞch biÕn trªn D. 5 Bước 1: Xét hàm số y = F( x ) Lập bảng biến thiên của hàm số trên D. Bước 2: Đoán được số nghiệm của phương trình F ( x ) = 0 trên D. Bước 3: Từ bảng biến thiên suy ra tập nghiệm của bất phương trình III. CÁC VÍ DỤ MINH HỌA 1.Phương trình: Ví dụ 1: Giải các phương trình sau: a) 3x + 4x = 5x b) 2x = 3 – x c) log2x = 3 – x (Bài tập SGK 12 nâng cao) Hướng dẫn cách giải: Cách 1: - Nhẩm nghiệm - Chứng minh nghiệm duy nhất Cách 2: - Thiết lập hàm số - Dùng tính đơn điệu để suy ra nghiệm của phương trình. Cách giải: a) 3x + 4x = 5x (1) Cách 1: Ta có x = 2 là nghiệm của phương trình 3x + 4x = 5x (1) x x 3  4 (1) ⇔  ÷ +  ÷ = 1 5  5 Vế trái: là hàm số nghịch biến ⇒ Nếu phương trình có nghiệm thì có nghiệm duy nhất Vế phải là hàm hằng Vậy x = 2 là nghiệm của phương trình (1) x x 3 4 Cách 2: (1) ⇔  ÷ +  ÷ = 1 5 5 x x 3  4 Xét f(x) =  ÷ +  ÷ 5 5 x x 3 4 3 4 ⇒ f’(x) =  ÷ ln +  ÷ ln < 0 ∀x ∈ ¡ 5 5 5 5 ⇒ f’(x) nghịch biến trên R và f(2)= 1 ⇒ x = 2 là nghiệm duy nhất của phương trình (1). Các ví dụ b, c giải tương tự Ví dụ 2: Giải phương trình: log 3 x2 + x +1 = x 2 − 3x + 2 2 2 x − 2x + 3 (1) Cách giải: 6 2  u = x + x + 1 Nhận dạng: Nếu đặt  ⇒ v – u = x 2 − 3x + 2 2 v = 2 x − 2 x + 3   u - Do đó (1) ⇔ log 3 = v − u (2) v - Nhận thấy phương trình có nghiệm u = v Thiết lập hàm số: biến đổi phương trình 2 về dạng: log3u + u = log3v + v Xét hàm số f(t) = log3 t + t , t > 0 u = x 2 + x + 1 > 0 ∀x đặt v = 2 x 2 − 2 x + 3 >0 ∀x ⇒ v - u = x 2 − 3x + 2 u v Xét hàm số f(t) = log3 t + t , t > 0 1 f’(t) = 1 + >0 với ∀ t > 0 t ln 3 Phương trình (1) ⇔ log 3 = v − u = log3u + u = log3v + v (2) ⇒f(t) đồng biến với ∀ t > 0 (2) ⇔ f(u) = f(v) ⇔ u = v ⇔ v – u = 0 ⇒ x 2 − 3x + 2 ⇔ x = 1 v x = 2 Vậy phương trình có 2 nghiệm x = 1, x= 2 Ví dụ 3 Giải phương trình: 3 x +1 − 2 1 = 2x + 1 − 3 x + 2 ( x∈¡ ) Cách giải:  x ≥ −1 ĐKXĐ:   x ≠ 13 Phương trình đã cho tương đương với ( x + 2 ) ( ) x + 1 − 2 = 3 2x + 1 − 3 ⇔ ( x + 1) x + 1 + x + 1 = 2x + 1 + 3 2x + 1 (1) 3 2 Xét hàm số f ( t ) = t + t ; f ' ( t ) = 3t + 1 > 0, ∀t Suy ra hàm số f ( t ) liên tục và đồng biến trên ¡ Khi đó: Pt(1) ⇔ f ( ) ( x +1 = f 3 ) 2x + 1 ⇔ x + 1 = 3 2x + 1 1  x≥−  1 2  1 x = 0  x ≥ −  x≥−    2 ⇔ ⇔ ⇔ x = 0 ⇔ 2 x = 1 + 5 ( x + 1) 3 = ( 2x + 1) 2 x3 − x 2 − x = 0    2  x = 1 ± 5   2 7 Đối chiếu ĐKXĐ được nghiệm của phương trình đã cho là: 1+ 5 và x = 0 . x= 2 ) ( ) ( 2 2 Ví dụ 4: Giải phương trình: 3x 2 + 3 + 9 x − ( x + 1) 2 + x + 2 x + 4 = 0 Cách giải: ( ) ( 2 2 Phương trình (1) ⇔ 3x 2 + 3 + (3x) = (x + 1) 2 + 3 + (x + 1) ) (2) Xét hàm số f (t) = t(2 + 3 + t 2 ) , ∀t ∈ ¡ , hàm số liên tục trên ¡ ( ) ( )  t   t2  2 f '(t) = 2 + 3 + t + t  = 2+ 3+ t + > 0 , ∀t ∈ ¡ 2 ÷ 2 ÷  3+ t   3+ t  1 ⇒ f (t) đồng biến trên ¡ . Do đó (2) ⇔ 3x = x + 1 ⇔ x = . 2 1 Vậy nghiệm của phương trình là x = 2 2 2. Giải hệ phương trình: (4 x 2 + 1) x + ( y − 3) 5 − 2 y = 0 Ví dụ 5: Giải hệ phương trình  2 2 (x, y ∈ R). 4 x + y + 2 3 − 4 x = 7 (Đề thi ĐH 2010-KA) 3 4 ĐK : x ≤ . Đặt u = 2x; v = 5 − 2 y Pt (1) trở thành u(u2 + 1) = v(v2 +1) ⇔ (u - v)(u2 + uv + v2 + 1) = 0 ⇔ u = v 3  0 ≤ x ≤ 4 Nghĩa là : 2 x = 5 − 2 y ⇔  2  y = 5 − 4x  2 25 Pt (2) trở thành − 6 x 2 + 4 x 4 + 2 3 − 4 x = 7 (*) 4 25  3 Xét hàm số f ( x) = 4 x 4 − 6 x 2 + + 2 3 − 4 x trên 0;  4  4 4 f '( x) = 4 x(4 x 2 − 3) − −3 (*)  1  2 x − y  4  2 x − y  ( 1) ⇔ 5  ÷ +  ÷  − 1 − 2.22 x − y = 0 (3). 5  5     1 t  4  t  t Xét hàm số f ( t ) = 5  ÷ +  ÷  − 1 − 2.2 trên ¡  5   5     1 t  1   4  t  4   t f ' t = 5  ÷ ln  ÷+  ÷ .ln  ÷ − 2.2 ln 2 < 0 ∀t ∈ ¡ . Suy ra f ( t ) nghịch ( ) ta có 5 5 5 5          biến trên ¡ . Do đó ( 3) ⇔ f ( 2 x − y ) = f ( 1) ⇔ 2 x − y = 1 ⇔ y = 2 x − 1 ( 4 ) . Thế (4) vào(2) ta được 1− x  x+3   x + 3  x −1 = ln  = 0 (5) ÷ ⇔ ln  ÷+ 4 4  2x + 2   2x + 2  ( x + 5) ( x − 1) x + 3  x −1 g ' x = + ( ) x > − 1 với , ta có ÷ 4 ( x + 3 ) ( x + 1) 4  2x + 2   Xét hàm số g ( x ) = ln   x = −5 ∉ ( −1; +∞ ) g '( x) = 0 ⇔  .  x = 1∈ ( −1; +∞ ) Ta có bảng biến thiên của g ( x ) trên ( −1; +∞ ) là: Từ BBT, suy ra g ( x ) = 0 ⇔ x = 1 . Do đó y = 1 . Vậy hệ có nghiệm ( x; y ) = ( 1;1) . Ví dụ 7: (Đề thi thử ĐH Chuyên Hà Tĩnh 2013) x y  (1) 3 − 3 = y − x Giải hệ phương trình:  2 2   x + xy + y = 12 (2) (I) Hướng dẫn cách giải: Học sinh nhận thấy được phương trình (1) có nghiệm x = y Biển đổi phương trình (1) về dạng 3x + x = 3y + y (3) Thiết lập hàm số: f(t) = 3t + t Chứng minh f(t) là hàm đồng biến, (3) ⇔ f(x) = f(y) ⇔ x = y 3x + x = 3 y + y (3) ⇔ Cách giải: (I)  2 2  x + xy + y = 12 Xét hàm số: f(t) = 3t + t ⇒ f’(t) = 3tln3 + 1 >0 ∀ t ∈ ¡ ⇒ f(t) là hàm đồng biến, (3) ⇔ f(x) = f(y) ⇔ x = y x = y Nên (I) ⇔  2 2  x + xy + y = 12 ⇔x = y = ± 2 Vậy hệ có hai nghiệm: (2;2) ; (-2; 2) Ví dụ 8 (Đề thi Ôlimpic 30-4 năm 2012) 9 2 x + 1 = y 3 + y 2 + y  3 2 Giải hệ phương trình: 2 y + 1 = z + z + z 2 z + 1 = x3 + x 2 + x  1 3  2  x = 2 ( y + y + y − 1)  x = f ( y)  1 3 2   Cách giải: Hệ tương đương với  y = ( z + z + z − 1) ⇔  y = f ( z ) 2   z = f ( x)  1 3  2 z = ( x + x + x − 1)  2  Xét hàm số f(t) = f’(t) = 1 3 2 (t + t + t − 1) ( t ∈ R ) 2 1 2 (3t + 2t + 1) > 0∀t ∈ R 2 Suy ra f(t) là HS đồng biến trên R. Do đó: + Nếu x < y thì f(x) < f(y) ⇒ z < x ⇒ f(z) < f(x) ⇒ y < z.Vậy x < y < z . Vô lý. Tương tự nếu y < x cũng vô lý Do đó x = y = z .Thế vào hệ ta có: 2x + 1 = x3 + x2 + x ⇔ (x + 1)(x2 - 1) = 0 x =1 = y = z  x = −1 = y = z Hệ  ⇔ ∈ { ( 1,1,1) ; ( −1, −1, −1) } có nghiệm (x,y,z) Ví dụ 9: (Tạp chí toán học tuổi trẻ tháng 5- 2012)  2 x + 3 + 4 − y = 4 Giải hệ   2 y + 3 + 4 − x = 4 (1) (2) (I) Hướng dẫn cách giải: - Nhận dạng: Đây là hệ phương trình đối xứng loại 2 nên có 1 nghiệm x = y - Lấy (1) – (2) và đưa phương trình về dạng 2 x + 3 − 4 − x = 2 y + 3 − 4 − y 3 2 - Thiết lập hàm số: f(t)= 2t + 3 − 4 − t , t ∈ [- ;4] 3 Cách giải: Điều kiện - ≤ x, y ≤ 4 2 Lấy (1) – (2) và đưa phương trình về dạng 2 x + 3 − 4 − x = 2 y + 3 − 4 − y (3) 3 2 Xét hàm số: f(t)= 2t + 3 − 4 − t , t ∈ [- ;4] 1 1 3 + > 0 ∀ t ∈ (- ;4) 2t + 3 4−t 2 3 ⇒ f(t) đồng biến trên (- ;4) 2 ⇒ f’(t) = 10 (3) ⇔ f ( x ) = f ( y ) ⇔ x = y Suy ra: 2 x + 3 + 4 − x = 4 (pt vô tỉ dạng cơ bản) 11 (thỏa mãn điều kiện) 9  11 11  Vậy hệ có 2 nghiệm (3; 3),  ;  9 9 Giải pt được 2 nghiệm : x=3, x= Ví dụ 10 (Đề thi thử ĐH chuyên Vĩnh Phúc 2013) (  x 2 + 1 − 3x 2 y + 2  Giải hệ phương trình:   x 2 y − x + 2 = 0 )( ) 4 y 2 + 1 + 1 = 8x 2 y3 (1) Hướng dẫn: +) Với y ≤ 0 thì VT ( 1) > 0 , VP ( 1) ≤ 0 ⇒ Hệ phương trình chỉ có nghiệm ( x, y ) với y> 0. +) Vì y > 0 nên từ phương trình (2) của hệ suy ra x > 2 . 2 2 2 Khi đó: ( 1) ⇔ x + 1 − 3 x y + 2 = 2 x y ( ) 4 y 2 +1 −1 ⇔ x2 + 1 + 2 = 2 x2 y 4 y2 + 1 + x2 y (3) Thay 2 = x − x y vào phương trình (3) ta được: 2 x2 + 1 + x = 2 x2 y 4 y 2 +1 + 2 x2 y ⇔ 1 1 1 1 + 2 + = 2 y 4 y2 + 1 + 2 y x x x (2) +) Xét hàm số: f ( t ) = t 1 + t 2 + t với t > 0 f '( t ) = 1+ t 2 + t2 1+ t2 + 1 > 0 với mọi t > 0 1 1 1 ⇒ f ( t ) là hàm đồng biến trên ( 0; +∞ ) . Mà f  ÷ = f ( 2 y ) ⇔ = 2 y ⇔ xy = x 2 x 1 1 +) Thay xy = vào phương trình (2) của hệ ta có : x = 4 ⇒ y = . 2 8 x = 4  Thử lại thấy  1 thỏa mãn hệ phương trình đã cho. y=  8   1 Kết luận : Hệ phương trình đã có nghiệm duy nhất ( x, y ) =  4; ÷  8 Ví dụ 11: (Đề thi ĐH khối A, A1 năm 2012)  x 3 − 3 x 2 − 9 x + 22 = y 3 + 3 y 2 − 9 y  Giải hệ phương trình  2 2 (x, y ∈ R). 1 x + y − x + y =  2  x 3 − 3x 2 − 9 x + 22 = y 3 + 3 y 2 − 9 y  Cách giải: Hệ tương đương với  . 1 2 1 2 ( x − ) + ( y + ) = 1  2 2 11 1 2 1 2 3 45 3 45  3 2 3 2 u − u − u = (v + 1) − (v + 1) − (v + 1) 2 4 2 4 Hệ đã cho thành  u 2 + v 2 = 1  3 45 45 Xét hàm f(t) = t 3 − t 2 − t có f’(t) = 3t 2 − 3t − < 0 với mọi t thỏa t≤ 1 2 4 4 Đặt u = x − ; v = y + v = 0 ⇒ f(u) = f(v + 1) ⇒ u = v + 1 ⇒ (v + 1)2 + v2 = 1 ⇒ v = 0 hay v = -1 ⇒ u = 1   v = −1 u = 0 hay  1   1 −3  3 ⇒ Hệ đã cho có nghiệm là  2 ; − 2 ÷;  2 ; 2 ÷.     Ví dụ 12: Giải các hệ phương trình sau:  x − 1 − y = 1 − x 3  3 + x 2 + 2 x = 3 + y a)  b)  4 ( x − 1) = y  3 + y 2 + 2 y = 3 + x  x 3 + 3 x − 3 + ln x 2 − x + 1 = y   3 2 c)  y + 3 y − 3 + ln y − y + 1 = z  3 2  z + 3z − 3 + ln z − z + 1 = x Hướng dẫn giải:  x − 1 − y = 1 − x 3 x −1 ≥ 0 x ≥ 1 ⇔ a)  (I) . Điều kiện:   4 y ≥ 0 y ≥ 0 ( x − 1) = y 2 3   x − 1 − ( x − 1) = 1 − x Ta có (I) ⇔  4  ( x − 1) = y 2 Từ phương trình : x − 1 − ( x − 1) = 1 − x 3 ⇔ x − 1 = − x 3 + x 2 − 2 x + 2 ( ( ( ) ) ) (1) Ta thấy hàm số f ( x ) = x − 1 là hàm đồng biến trên [ 1;+∞ ) Xét hàm số g ( x ) = − x 3 + x 2 − 2 x + 2 . Miền xác định: D = [ 1; +∞ ) Đạo hàm g / ( x ) = −3x 2 + 2 x − 2 < 0 ∀x ∈ D . Suy ra hàm số nghich biến trên D. Từ (1) ta thấy x = 1 là nghiệm của phương trình và đó là nghiệm duy nhất. Vậy hệ có nghiệm ( 1;0 ) .  3 + x 2 + 2 x = 3 + y b)  (II). Điều kiện:  3 + y 2 + 2 y = 3 + x  3 + x 2 + 2 x = 3 + y Ta có (II) ⇔  3 + x = 3 + y 2 + 2 y x ≥ 0  y ≥ 0 12 Cộng vế theo vế ta có: 3 + x 2 + 3 x + 3 = 3 + y2 + 3 y + 3 (2) Xét hàm số f (t ) = 3 + t 2 + 3 t + 3 . Miền xác định: D = [ 1; +∞ ) t 3 / + + 1 > 0 ∀x ∈ D . Suy ra hàm số đồng biến trên Đạo hàm: f (t ) = 3 + t2 2 t D. Từ (*) ta có f ( x ) = f ( y) ⇔ x = y Lúc đó: 3 + x 2 + x = 3 (3) + VT (3) là hàm số hàm đồng biến trên D. + VP (3) là hàm hằng trên D. Ta thấy x = 1 là nghiệm của phương trình (3) (thỏa điều kiện) Suy ra phương trình có nghiệm x = 1 là nghiệm duy nhất. Vậy hệ có nghiệm ( 1;1) ( ( ( ) )  x 3 + 3x − 3 + ln x 2 − x + 1 = y   3 2 c)  y + 3y − 3 + ln y − y + 1 = z  3 2  z + 3z − 3 + ln z − z + 1 = x 3 2 Xét hàm số f (t ) = t + 3t − 3 + ln t − t + 1 ) ( )  f ( x) = y  Lúc đó hệ có dạng:  f ( y) = z . Miền xác định: D = R  f (z) = x  / 2 Đạo hàm : f ( x ) = 3t + 3 + 2t − 1 2 t2 − t + 1 > 0 ∀x ∈ R . Suy ra hàm số đồng biến trên D Ta giả sử ( x; y; z ) là nghiệm của hệ và x = max { x , y, z} khi đó ta suy ra: y = f ( x ) ≥ f ( y) = z ⇒ z = f ( y ) ≥ f ( z ) = x . Vậy x ≥ y ≥ z ≥ x ⇔ x = y = z . ( ) ( ) 3 2 3 2 Thay vào hệ ta có : x + 3x − 3 + ln x − x + 1 = x ⇔ x + 2 x − 3 + ln x − x + 1 = 0 (3) Ta thấy x = 1 là nghiệm duy nhất của phương trình (vì VT (3) là đồng biến trên R) Vậy hệ có nghiệm ( 1;1;1) Ví dụ 13: Giải hệ phương trình:  x 6 − y 3 + x 2 − 9 y 2 − 30 = 28 y (1) ( x, y ∈ R ).  2 x + 3 + x = y (2)  3 2 ĐK: x ≥ − , ( 1) ⇔ x 6 + x 2 = ( y + 3)3 + ( y + 3) (3) Xét hàm số f(t)= t 3 + t , ∀t ∈ R. Có f ’(t)= t 2 + 1 > 0, ∀t ∈ R ⇒ f(t) đồng biến trên R (3) ⇔ f ( x 2 ) = f ( y + 3) ⇔ x 2 = y + 3 1 2 1 2 Thay y = x 2 − 3 vào (2) ta được 2 x + 3 + x = x 2 − 3 ⇔ ( 2 x + 3 + ) 2 = ( x + ) 2 13 1 1   2x + 3 + 2 = x + 2  2x + 3 = x x = 3 y = 6 ⇔ ⇔ ⇔ (tm) ⇒   y = −1  2x + 3 + 1 = −x − 1 x = − 2  2 x + 3 = − x − 1  2 2 KL: Hệ có 2 nghiệm (x; y) ∈ { (3; 6); (− 2; −1)} .  x3 − x 2 y = x 2 − x + y + 1 Giải hệ phương trình:  3 3 2  x + 2 xy + 2(3 x + y ) − 1 = 3 x + 7 Ta có (1) ⇔ ( x 2 + 1)( x − y − 1) = 0 ⇔ y = x − 1 (1) (2) Với y = x − 1 thế vào (2) ta được: x3 + 2 x 2 + 6 x − 3 = 3 3 x 2 + 7 ⇔ ( x + 1)3 + 3( x + 1) = ( x 2 + 7) + 3 3 x 2 + 7 (3) 3 2 Xét hàm số f (t ) = t + 3t ta có f '(t ) = 3t + 3 > 0, ∀t Suy ra f (t ) đồng biến trên ¡ do đó từ (3) suy ra 3 2 x + 1 = 3 x 2 + 7 ⇔ x + 2 x + 3x − 6 = 0 ⇔ x = 1 ⇒ y = 0 Vậy hệ có nghiệm duy nhất ( x; y ) = (1;0) ln x − ln y = x − y Ví dụ 14: Giải hệ phương trình:  2 2  x − xy + y + x + y = 0 ( 1) ( 2) Giải: ĐK: x > 0; y > 0 Ta có: ( 1) ⇔ x − ln x = y − ln y Xét hàm số : f ( t ) = t − ln t với t ∈ ( 0; +∞ ) ⇒ f ' ( t ) = t − 1 t Ta có: f ' ( t ) = 0 ⇔ t = 1 . Hàm số đồng biến trong khoảng ( 0;1) và nghịch biến trong khoảng ( 1; +∞ ) - Nếu x; y ≥ 1 hoặc x; y ≤ 1 thì ln ( 1 + x ) − x = ln ( 1 + y ) − y ⇔ f ( x ) = f ( y ) ⇔ x = y Thay vào PT: 2 x 2 − 5 xy + y 2 = 0 ta có nghiệm x = y = 0 hoặc x = y = −2 (loại) - Nếu x < 1; y > 1 hoặc x > 1; y < 1 thì ( x − 1) ( y − 1) < 0 ⇔ xy − x − y + 1 < 0 x 2 − xy + y 2 + x + y > 1 do đó phương trình vô nghiệm Vậy hệ vô nghiệm.  x 3 − y 3 + 3 y 2 − 3 x − 2 = 0 Ví dụ 15: Giải hệ phương trình:  2 2 2  x + 1 − x − 3 2 y − y = 0 Giải: ĐK: −1 ≤ x ≤ 1;0 ≤ y ≤ 2 ( x + 1) 3 − 3 ( x + 1) 2 = y 3 − 3 y 2 (1) Hệ đã cho tương đương với:  2  x + 1 − x 2 − 3 2 y − y 2 = 0 (2) 3 2 Xét hàm số: f ( t ) = t − 3t với 0 ≤ t ≤ 2 f ' ( t ) = 3t 2 − 6t ≤ 0∀t ∈ ( 0; 2 ) nên hàm số nghịch biến trên khoảng (0;2) khi đó: ( 3) ⇔ f ( x + 1) = f ( y ) ⇔ x + 1 = y thay vào (2) ta được: 14 2 1 − x2 = x2 ⇔ x = ± 2 2 − 2 ( x; y ) = ( Vậy hệ có nghiệm: )( ) 2 2 − 2; 2 2 − 2 + 1 ; − 2 2 − 2; − 2 2 − 2 + 1 Ví dụ 15: Giải hệ phương trình: ( 23 − 3x ) 7 − x + ( 3 y − 20 ) 6 − y = 0 ( 1)  ( 2)  2 x + y + 2 − 2 y − 3x − 8 = 0 Giải: ĐK: x ≤ 7; y ≤ 6; 2 x + y + 2 ≥ 0; 2 x − 3 y + 8 ≥ 0 Ta có ( 1) ⇔ 3 ( 7 − x ) + 2 7 − x = 3 ( 6 − y ) + 2  6 − y ( 3) Xét hàm số: f ( t ) = ( 3t + 2 ) t với t ≥ 0 3t + 2 > 0, ∀t > 0 nên hàm số đồng biến trên ( 0; +∞ ) 2 t Khi đó: ( 3) ⇔ f ( 7 − x ) = f ( 6 − y ) ⇔ 7 − x = 6 − y ⇔ y = x − 1 thay vào ( 2 ) ta được: f '( t ) = 3 t + 3x + 1 − 6 − x + 3 x 2 − 14 x − 8 = 0 ⇔ ( ) ( ) 3 x + 1 − 4 + 1 − 6 − x + 3x 2 − 14 x − 5 3 1   ⇔ ( x − 5)  + + 3x + 1÷ = 0  3x + 1 + 4 1 + 6 − x  ⇔ x=5 3 1 + + 3 x + 1 > 0 ) Vậy hệ có nghiệm: ( x; y ) = ( 5;5 ) (Do 3x + 1 + 4 1 + 6 − x Loại 3: Vận dụng tính đơn điệu để giải bất phương trình Bài tập 1: Giải các bất phương trình sau: a) x + 9 + 2 x + 4 > 5 b) x 2 − 2 x + 3 − x 2 − 6 x + 11 > 3 − x − x − 1 Hướng dẫn giải: x + 9 ≥ 0 ⇔ x ≥ −2 a) x + 9 + 2 x + 4 > 5 (1). Điều kiện:  2 x + 4 ≥ 0 Xét hàm số y = f ( x ) = x + 9 + 2 x + 4 . Miền xác định : D = [ −2; +∞ ) 1 1 / + > 0 ∀x > −2 . Suy ra hàm số đồng biến trên D . Đạo hàm f ( x ) = 2 x+9 2x + 4 Để ý rằng: f (0) = 5 , do đó: + Nếu x > 0 thì f ( x ) > f (0) ⇔ x + 9 + 2 x + 4 > 5 , nên x > 0 là nghiệm bpt. + Nếu −2 ≤ x ≤ 0 thì f ( x ) ≤ f (5) ⇔ x + 9 + 2 x + 4 ≤ 5 nên −2 ≤ x ≤ 0 không là nghiêm bpt. Đối chiếu với điều kiện, suy ra tập nghiệm của (1) là T = ( 0; +∞ ) . b) x 2 − 2 x + 3 − x 2 − 6 x + 11 > 3 − x − x − 1 (2) 15 x2 − 2x + 3 ≥ 0  2  x − 6 x + 11 ≥ 0 ⇔1≤ x ≤ 3 Điều kiện:  (*) 3 − x ≥ 0   x − 1 ≥ 0 Biến đổi bất phương trình: ⇔ x 2 − 2 x + 3 + x − 1 > x 2 − 6 x + 11 + 3 − x ⇔ ( x − 1)2 + 2 + x − 1 > (3 − x )2 + 2 + 3 − x (3) Xét hàm số f (t ) = t 2 + 2 + t . Ta thấy hàm số đồng biến trên [ 1;3] Từ (3) ta có f ( x − 1) > f (3 − x ) ⇔ x − 1 > 3 − x ⇔ x > 2 Đối chiếu với điều kiện, ta có nghiệm của bất phương trình (2) là T = ( 2;3] . IV. CÁC BÀI TẬP TỰ GIẢI Bài tập 1: Giải các phương trình sau: a) 3 − x + x 2 − 2 + x − x 2 = 1 c) 2x −1 + x2 + 3 = 4 − x m x +6 − 2 4 x +3 m = ( 4 − m 2 ) x + 3m − 6 e) 2 2 g) 1 2 sin x − 1 2 sin x cos2 x = sin 4 x 2 2 2 sin x −3 3 + ( 3sin x − 10 ) .3sin x −2 + 3 − sin x = 0 Bài tập 2: Giải các bất phương trình sau: a) x + x 2 − 1 ≥ 1 c) x + 1 ≤ 1 − 2x + x2 − x3 Bài tập 3: Giải các hệ phương trình sau: x − 3 = − x 3 + 3x 2 + x − 12 1 1 2 x −1 x −1 −e = − d) e 2x −1 x −1 b) f) tan x + 2.3log2 tan x = 3 h) b) x − 1 + x 2 − 1 ≥ ( x + 1) ( 3 − x ) d) x +3 x −3 ≥ 9− x ( )  2x − 2y = y − x a)  2 2  x + xy + y = 12 2   4 x + 1 x + ( y − 3) 5 − 2 y = 0 b)  2 2   4x + y + 2 3 − 4x = 7  3 + x 2 + 2 x + 3 = 5 + y + 3 c)   3 + y 2 + 2 y + 3 = 5 + x + 3 x y = yx d)  2 2  x + 4 y = 25 sin 2 x − 2 y = sin 2 y − 2 x  e) 2 x + 3y = π  x, y > 0   x 2 − 2 x + 6. log ( 6 − y ) = x 3   2 f)  y − 2 y + 6.log3 ( 6 − z ) = y  2  z − 2 z + 6. log3 ( 6 − x ) = z 16  tan x − tan y = y − x g)   y + 1 − 1 = x − y + 8  x − y sin x e = sin y  h) 10 x 6 + 1 = 3 y 4 + 2  π < x , y < 5π  4 ( ) 17 - Tên chuyên đề: CÁC DẠNG TOÁN VỀ PHÉP ĐẾM - Tác giả chuyên đề: NGUYỄN THỊ THANH HẢI - Chức vụ : Giáo viên Toán - Đơn vị công tác: Trường THPT Vĩnh Yên - Đối tượng học sinh bồi dưỡng: lớp 11, lớp 12 - Dự kiến số tiết bồi dưỡng: 10 tiết I. HỆ THỐNG KIẾN THỨC SỬ DỤNG TRONG CHUYÊN ĐỀ: 1. Quy tắc cộng: Một công việc A được chia ra k công việc A1 , A2 ,..., Ak để thực hiện, mỗi công việc độc lập nhau. Trong đó: + Công việc A1 có n1 cách thực hiện 18 + Công việc A2 có n2 cách thực hiện ………………………………….. + Công việc Ak có nk cách thực hiện Khi đó số cách thực hiện công việc A là : ( n1 + n2 + ... + nk ) cách. 2. Quy tắc nhân: Một công việc A được thực hiện lần lượt qua k giai đoạn A1 , A2 ,..., Ak , với mỗi cách thưck hiện ở giai đoạn này không trùng với bất kỳ cách thực hiện nà ở các giai đoạn còn lại. Trong đó: + Giai đoạn A1 có n1 cách thực hiện + Giai đoạn A2 có n2 cách thực hiện + Giai đoạn A3 có n3 cách thực hiện ………………………………….. + Giai đoạn Ak có nk cách thực hiện Khi đó số cách thực hiện công việc A là : ( n1.n2 .n3...nk ) cách. 3. Hoán vị: 3.1. Định nghĩa: - Cho tập hợp A gồm n phần tử ( n ≥ 1) . Mỗi cách sắp xếp có thức tự n phần tử của tập hợp A được gọi là một hoán vị của n phần tử đó. 3.2. Định lý: (Số hoán vị của n phần tử) - Nếu ký hiệu số hoán vị của n phần tử là Pn thì ta có Pn = n ! = 1.2.3... ( n − 1) .n *Chú ý: Quy ước 0! = 1 4. Chỉnh hợp: 4.1. Định nghĩa: - Cho tập hợp A gồm n phần tử. Một bộ gồm k phần tử ( 1 ≤ k ≤ n ) sắp xếp có thức tự của tập hợp A được gọi là một chỉnh hợp chập k của n phần tử của tập A. 4.2. Định lý: (Số chỉnh hợp chập k của n phần tử) - Nếu ký hiệu số chỉnh hợp chập k của n phần tử là Ank thì ta có Ank = n. ( n − 1) . ( n − 2 ) ... ( n − k + 1) 19 *Chú ý: n! k - Có thể viết Ank theo cách khác An = ( n − k ) ! - Nếu k=n thì Ank = Pn - Hai chỉnh hợp khác nhau là hai bộ có ít nhất 1 phần tử khác nhau hoặc các phần tử giống nhau nhưng thứ tự sắp xếp khác nhau. 5. Tổ hợp: 5.1. Định nghĩa: - Cho tập hợp A gồm n phần tử. Mỗi tập con gồm k phần tử ( 1 ≤ k ≤ n ) của tập hợp A được gọi là một tổ hợp chập k của n phần tử của tập A. 5.2. Định lý: (Số tổ hợp chập k của n phần tử) - Nếu ký hiệu số tổ hợp chập k của n phần tử là Cnk thì ta có Cnk = n. ( n − 1) . ( n − 2 ) ... ( n − k + 1) n! = Cn0 = 1 và quy ước k! k !( n − k ) ! *Chú ý: - Hai tổ hợp khác nhau là hai tập con có ít nhất 1 phần tử khác nhau . II. DẤU HIỆU NHẬN BIẾT ĐẶC TRƯNG : 1. Bài toán 1: có sử dụng hoán vị của n phần tử . Chúng ta thường dựa trên dấu hiệu đặc trưng sau: - Tất cả n phần tử đều có mặt. - Mỗi phần tử chỉ xuất hiện 1 lần. - Có sự sắp xếp thứ tự giữa các phần tử. - Khi đó số cách sắp xếp n phần tử là số hoán vị của n phần tử đó. Và có Pn = n! = 1.2.3....( n − 1) .n 2. Bài toán 2: có sử dụng chỉnh hợp chập k của n phần tử . Chúng ta thường dựa trên dấu hiệu đặc trưng sau: - Phải chọn k phần tử từ n phần tử cho trước. - Có sự sắp xếp thứ tự giữa k phần tử đó. 20 - Khi đó số cách chọn k phần tử có sắp xếp thứ tự từ n phần tử là số chỉnh hợp chập k của n phẩn tử đó. n! k Và có An = ( n − k ) ! = n ( n − 1) ... ( n − k + 1) 3. Bài toán 3: có sử dụng tổ hợp chập k của n phần tử . Chúng ta thường dựa trên dấu hiệu đặc trưng sau: - Phải chọn k phần tử từ n phần tử cho trước. - Không có sự sắp xếp thứ tự giữa k phần tử đó. - Khi đó số cách chọn k phần tử không có sắp xếp thứ tự từ n phần tử là số tổ hợp chập k của n phẩn tử đó. k Và có Cn = n! k !( n − k ) ! III. CÁC DẠNG TOÁN THƯỜNG GẶP: DẠNG 1: BÀI TOÁN ĐẾM PHƯƠNG ÁN 1. Bài toán đếm có sự sắp xếp Ví dụ 1: Có bao nhiêu cách sắp xếp 7 bạn học sinh A, B, C, D, E, F, G vào một chiếc ghế dài sao cho a- Bạn D ngồi vào chính giữa 7 bạn ? b- Hai bạn A và G ngồi ở 2 đầu ghế ? *Phân tích: a/Sau khi sắp xếp vị trí ngồi cho D chúng ta thấy rằng 6 bạn còn như 6 phần tử đều có mặt và chỉ xuất hiện 1 lần . Mỗi cách sắp xếp có sự phân biệt thứ tự. Do đó ta sử dụng bài toán 1. 21 b/Tương tư ta thấy: Sau khi sắp xếp vị trí ngồi cho A và G chúng ta thấy rằng 5 bạn còn như 5 phần tử đều có mặt và chỉ xuất hiện 1 lần . Mỗi cách sắp xếp có sự phân biệt thứ tự. Do đó ta sử dụng bài toán 1. *Lời giải: a/Sắp xếp D ngồi vào chính giữa: có 1 cách. Mỗi cách sắp xếp A, B, C, E, F vào 6 chỗ còn lại là một hoán vị của 6 phần tử nên có 6 cách sắp xếp A, B, C, E, F . Vậy có 1 × 6!=720 cách sắp xếp thoả mãn yên cầu bài toán. b/ sắp xếp 2 bạn A và G vào vị trí : có 2 cách. Sắp xếp các bạn còn lại : có 5! cách. vậy có 2! × 5!=240 cách sắp xếp thoả mãn yêu cầu bài toán . Ví dụ 2: Có 5 tem thư khác nhau và 6 bì thư cũng khác nhau. Người ta muốn chọn ra 3 tem thư và 3 bì thư và dán 3 tem thư ấy lên 3 bì thư đã chọn. Một bì thư chỉ dán một tem thư. Hỏi có bao nhiêu cách làm như vậy? *Lời giải : Chọn ra 3 tem thư từ 5 tem => có C53 cách chọn Chọn ra 3 bì thư từ 6 bì thư => có C63 cách chọn Mỗi cách dán 3 tem lên 3 bì thư vừa được chọn ra là một hoán vị của 3 nên có 3! cách dán. Vậy có C53 . C63 .3!=1200 cách làm. Ví dụ 3: Một thầy giáo có 12 quyển sách khác nhau, trong đó có 5 quyển văn học, 4 quyển âm nhạc và 3 quyển hội hoạ. Ông muốn lấy ra 6 quyển và đem tặng cho 6 học sinh khác nhau. Mỗi em chỉ được một quyển. 22 a/ Giả sử thầy giáo chỉ muốn tặng cho các em học sinh trên những quyển sách văn học và âm nhạc. Hỏi có bao nhiêu cách tặng? b/ Giả sử thầy giáo muốn rằng sau khi tặng sách xong, mỗi loại còn ít nhất một quyển. Hỏi có bao nhiêu cách tặng? *Lời giải : a/ Cách 1: Chọn 6 quyển sách bất kỳ từ 5 quyển sách văn học và 4 quyển âm nhạc là một tổ hợp chập 6 của 9 phần tử => có C96 cách chọn Với mỗi cách chọn như vậy sẽ có 6! cách tặng . Vậy số cách tặng là: C96 .6!=60480 cách tặng. Cách 2: Số cách tặng 6 quyển sách theo yêu cầu bài toán là một chỉnh hợp chập 6 của 9 phần tử . Vậy số cách tặng là: A96 =60480 cách tặng. b/ Cách 1: Chọn 6 quyển sách bất kỳ từ 5 quyển sách văn học và 4 quyển âm nhạc là một tổ hợp chập 6 của 9 phần tử => có C96 cách chọn Với mỗi cách chọn như vậy sẽ có 6! cách tặng . Vậy số cách tặng là: C96 .6!=60480 cách tặng. Cách 2: Số cách tặng 6 quyển sách theo yêu cầu bài toán là một chỉnh hợp chập 6 của 9 phần tử . Vậy số cách tặng là: A96 =60480 cách tặng. 2. Bài toán đếm không có sắp xếp Ví dụ 4: 23 Đội thanh niên xung kích của trường X có 12 học sinh, gồm 5 học sinh lớp A, 4 học sinh lớp B và 3 học sinh lớp C. Cần chọn 4 học sinh đi làm nhiệm vụ sao cho: a/ 4 học sinh này thuộc cả 3 lớp trên. b/ 4 học sinh này thuộc không quá 2 trong 3 lớp. Hỏi có bao nhiêu cách chọn như vậy? *Phân tích: Số học sinh được chọn từ 12 học sinh không sắp xếp thứ tự gì nên ta có thể sử dụng bài toán 2. *Lời giải: a/Vì 4 học sinh được chọn cần ở cả 3 lớp nên ta có các trường hợp chia như sau: Học sinh lớp A Học sinh lớp B Học sinh lớp C Số cách chọn tương ứng 2 1 1 1 2 1 1 1 2 C52 .C41 .C31 = 120 cách C51.C42 .C31 = 90 cách C51.C41.C32 = 60 cách Vậy có tất cả là 120+90+60=270 cách. b/ Số cách chọn 4 học sinh từ 12 học sinh đã cho là C124 = 495 Số cách chọn 4 học sinh mà mỗi lớp có 1 học sinh là 270 cách chọn . Nên số cách chọn 4 học sinh thuộc không quá 2 trong 3 lớp phải là : 495-270=225 cách chọn . Ví dụ 5: (ĐH khối B-2004) Trong một môn học, thầy giáo có 30 câu hỏi khác nhau gồm 5 câu hổi khó, 10 câu hỏi trung bìnhvà 15 câu hỏi dễ. Từ 30 câu hỏi đó có thể lập được bao nhiêu đề kiểm tra, mỗi đề gồm 5 câu hỏi khác nhau, sao cho trong mỗi đề nhất thiết phải có đủ 3 loại câu hỏi (khó, trung bình và dễ) và số câu hỏi dễ không ít hơn 2? *Lời giải : 24 Ta có trường hợp như sau: số câu hỏi dễ số câu hỏi TB 2 1 2 2 3 1 số câu hỏi khó 2 1 1 Số cách lập đề dạng này 1 C152 .C10 .C52 = 10500 cách C152 .C102 .C51 = 23625 cách 1 C153 .C10 .C51 = 22750 cách Áp dụng quy tắc cộng có tất cả 10500+23625+22750=56875 đề được lập. Ví dụ 6: (ĐH khối B-2005) Một đội thanh niên tình nguyện có 15 người, gồm 12 nam và 3 nữ. Hỏi có bao nhiêu cách phân công đội thanh niên tình nguyện đó về giúp đỡ 3 tỉnh miền núi sao cho mỗi tỉnh có 4 nam và 1 nữ? *Lời giải : Số cách phân công các thanh niên tình nguyện về tỉnh thứ nhất là C31C124 . Với mỗi cách phân công các thanh niên tình nguyện về tỉnh thứ nhất ta có số cách phân công các thanh niên tình nguyện về tỉnh thứ hai là C21C84 . Với mỗi cách phân công các thanh niên tình nguyện về tỉnh thứ nhất và tỉnh thứ hai ta có số cách phân công các thanh niên tình nguyện về tỉnh thứ ba là C11C44 . Vậy có tất cả các cách phân công thanh niên tình nguyện về ba tỉnh sẽ là: C31C124 × C21C84 × C11C44 =207900 cách. Ví dụ 7 : Một lớp có 30 học sinh gồm 18 nam và 12 nữ. Hỏi có bao nhiêu cách chọn một ban cán sự lớp gồm 5 người: a/ Mọi người đều vui vẻ tham gia. b/ Cậu Tâm và cô Bình không thể rời nhau. c/ Cậu An và cô Hà không thể làm việc chung với nhau. *Lời giải : a/ Vì mọi người đều vui vẻ tham gia nên ta tuỳ ý chọn 5 trong số 30 người vào ban cán sự lớp. Mỗi cách chọn đó là một tổ hợp chập 5 của 30 phần tử . 5 Vậy có tất cả C30 = 30! = 142506 cách chọn . 5!25! 25 b/ Cách 1: -Nếu cả Tâm và Bình cùng có mặt trong ban cán sự thì ta chỉ việc chọn 3 người 3 trong 28 người vào ban cán sự => có C28 cách chọn -Nếu cả Tâm và Bình cùng không có mặt trong ban cán sự thì ta chọn 5 người 5 trong 28 người vào ban cán sự => có C28 cách chọn Vậy có tất cả số cách chọn ban cán sự để Cậu Tâm và cô Bình không thể rời nhau là: 3 5 C28 + C28 =101556 cách chọn. Cách 2: -Chọn 5 người tuỳ ý trong 30 người =>có C305 cách chọn . -Chọn 5 người trong đó có Tâm mà không có Bình =>có C284 cách chọn . -Chọn 5 người trong đó có Bình mà không có Tâm =>có C284 cách chọn . Vậy có tất cả số cách chọn ban cán sự để Cậu Tâm và cô Bình không thể rời nhau là: C305 -( C284 + C284 )=101556 cách chọn. c/ Cách 1: 5 -Chọn 5 người trong đó không có An mà không có Hà =>có C28 cách chọn . -Chọn 5 người trong đó có An mà không có Hà =>có C284 cách chọn . -Chọn 5 người trong đó có Hà mà không có An =>có C284 cách chọn . Vậy có tất cả số cách chọn ban cán sự để Cậu An và Hà không thể làm việc chung với nhau là: 5 C28 + C284 + C284 =139230 cách chọn. Cách 2: -Chọn 5 người tuỳ ý trong 30 người =>có C305 cách chọn . 3 -Chọn 5 người trong đó có cả An và Hà =>có C28 cách chọn . 26 Vậy có tất cả số cách chọn ban cán sự để Cậu An và Hà không thể làm việc chung với nhau là: C305 − C283 =139230 cách chọn. Ví dụ 8: Một đội văn nghệ có 20 người trong đó có 10 nam và 10 nữ. Hỏi có bao nhiêu cách chọn ra đúng 5 người sao cho: a/ có đúng 2 nam. b/ có ít nhất 2 nam và ít nhất 1 nữ. *Lời giải : a/ Chọn bất kỳ 2 nam trong 10 nam => có C102 cách chọn Chọn bất kỳ 3 nữ trong 10 nữ => có C103 cách chọn Vậy số cách chọn 5 người có đúng 2 nam là: C102 . C103 =5400 cách chọn . b/ Cách 1: - Chọn 5 người trong đó có 2 nam và 3 nữ => có C102 . C103 cách chọn - Chọn 5 người trong đó có 3 nam và 2 nữ => có C103 . C102 cách chọn - Chọn 5 người trong đó có 4 nam và 1 nữ => có C104 . C101 cách chọn Vậy có tất cả số cách chọn 5 người sao cho có ít nhất 2 nam và ít nhất 1 nữ là: C102 . C103 + C103 . C102 + C104 . C101 =12900 cách chọn. Cách 2: 5 -Chọn 5 người tuỳ ý trong 20 người =>có C20 cách chọn . -Chọn 5 người trong đó có 1 nam và 4 nữ => có C101 . C104 cách chọn -Chọn 5 người trong đó có 0 nam và 5 nữ => có C105 cách chọn -Chọn 5 người trong đó có 5 nam và 0 nữ => có C105 cách chọn Vậy có tất cả số cách chọn 5 người sao cho có ít nhất 2 nam và ít nhất 1 nữ là: 5 C20 -( C101 . C104 + C105 + C105 )=12900 cách chọn. 27 DẠNG 2: BÀI TOÁN ĐẾM CÓ LIÊN QUAN ĐẾN SỐ TỰ NHIÊN +Số tự nhiên n = ab trong đó a, b ∈ { 0,1, 2,...,9} , a ≠ 0 +Số ab = 10a + b , abc = 100a + 10b + c +Số ab ≠ ba 1. Tính số các số tự nhiên liên quan đến so sánh các số, các chữ số: Ví dụ 9: Từ các chữ số 1,2,3,4,5,6,7,8,9 có thể lập được bao nhiêu số có ba chữ số khác nhau và số đó a) Lớn hơn 400? b) Không nhỏ hơn 666? c) Nhỏ hơn 345? *Lời giải: a/ Gọi số cần tìm là x = abc Vì x>400 nên a ≥ 4 , suy ra a có 6 cách chọn và bc có A82 cách chọn . Vậy có tất cả 6 × A82 =336 số . b/ Gọi số cần tìm là x = abc Từ x ≥ 666 suy ra a ≥ 6 , ta có các trường hợp sau: *TH1: Với a ∈ { 7;8;9} khi đó b,c chọn tuỳ ý. Ta có : a có 3 cách chọn 2 bc có A8 cách chọn Suy ra có 3 × A82 =168 số. *TH2: Với a=6 khi đó b>6 và c chọn tuỳ ý. Ta có: a có 1 cách chọn b có 3 cách chọn c có 7 cách chọn Suy ra có 1.3.7=21 số. Vậy có tất cả 168+21=189 số thoả mãn yêu cầu bài toán . 28 c/ Vì x1) còn các chữ số khác có mặt đúng 1 lần” ⇒ Công thức ( n + k − 1) ! k! số cần tìm BÀi 2: “ Cho n chữ số khác nhau chứa cả chữ số 0, 1 ≤ n ≤ 9 . Hỏi có bao nhiêu số tự nhiên có n+k chữ số trong đó một chữ số được lặp lại k lần ( k>1) còn các chữ số khác có mặt đúng 1 lần” ⇒ Công thức ( n + k − 1) !− ( n + k − 2 ) ! k! số cần tìm 5.Bài toán đếm liên quan đến tổng các chữ số và tính tổng các số tự nhiên vừa tìm được: Ví dụ 21: Từ các chữ số 1,2,3,4,5 có thể lập được bao nhiêu số tự nhiên có 5 chữ số khác nhau đôi một được tạo thành từ các chữ số trên. Hãy tính tổng tất cả các số vùa tìm được. *Lời giải : 38 Cách 1: Mỗi số cần tìm là một hoán vị của 5 phần tử do đó số các số cần tìm là 5!=120 số. Nhận thấy: Có 24 số có dạng n= a1a2 a3a41 Có 24 số có dạng n= a1a2 a3a4 2 Có 24 số có dạng n= a1a2 a3 a4 3 Có 24 số có dạng n= a1a2 a3a4 4 Có 24 số có dạng n= a1a2 a3a4 5 ⇒ Tổng các chữ số ở hàng đơn vị là (1+2+3+4+5).120=360 Tương tự : Tổng các chữ số ở hàng chục là (1+2+3+4+5).10.120=3600 Tổng các chữ số ở hàng đơn vị là (1+2+3+4+5)100.120=36000 Tổng các chữ số ở hàng đơn vị là (1+2+3+4+5)1000.120=360000 Tổng các chữ số ở hàng đơn vị là (1+2+3+4+5)10000.120=3600000 Vậy tổng của 120 số n là: 360+3600+36000+360000+3600000=3999960 Cách 2: Trong số 120 số n ta luôn tìm được cặp số n,n’ sao cho tổng của chúng n+n’=66666. chẳng hạn như 12345+54321=66666. Do đó tống tất cả 120 số vùa tìm được là: 66666. 120 =3999960 2 Ví dụ 22: Từ các chữ số 1,2,3,4,5,6 có thể lập được bao nhiêu số tự nhiên có 5 chữ số, mỗi số gồm 6 chữ số khác nhau và tổng các chữ số hàng chục, hàng trăm, hàng nghìn bằng 8? *Lời giải : Gọi số cần tìm là abcdef . 39 Theo giả thiết c+d+e=8. Suy ra c, d , e ∈ { 1;2;5} hoặc c, d , e ∈ { 1;3;4} . +Với c, d , e ∈ { 1;2;5} : Ta có 3! Cách chọn cde . Chọn 3 chữ số a,b,f trong 9 - 3=6 chữ số : có A63 cách chọn . Vậy có 3! × A63 =720 số + Với c, d , e ∈ { 1;3;4} : Tương tự ta cũng có 720 số. Vậy có tất cả là 720+720=1440 số thoả mãn yêu cầu bài toán . DẠNG 3: BÀI TOÁN ĐẾM LIÊN QUAN ĐẾN HÌNH HỌC Ví dụ 23: a/ Cho một đa giác lồi n cạnh. Hỏi có bao nhiêu đường chéo? b/ Một đa giác lồi có bao nhiêu cạnh để số đường chéo bằng 35? *Phân tích: Mỗi đoạn thẳng tương ứng với 2 điểm thuộc n điểm , trong đó có cả cạnh và đường chéo của đa giác.Và ngược lại 2 điểm thuộc n điểm tạo được 1 đoạn thẳng có thể là cạnh hoặc đường chéo của đa giác. *Lời giải : a/ Đa giác có n cạnh => có n đỉnh. Mỗi đường chéo được tạo thành từ 2 đỉnh trong n đỉnh đó và ngược lại. Trừ đi n cạnh của đa giác. n − 1) n ( n! n 2 − 3n Vậy có tất cả là : C − n = 2! n − 2 ! − n = 2 − n = 2 (đường chéo) ( ) 2 n b/ theo kết quả của phần a. ta có n∈N , n ≥3 Cn2 − n = 35 ⇔ n 2 − 3n − 70 = 0 ¬  → n = 10 Vậy đa giác lồi có 10 cạnh sẽ có 35 đường chéo. Ví dụ 24: 40 Cho 2 đường thẳng a, b song song với nhau. Trên đường thẳng a có 10 điểm phân biệt, trên đường thẳng b có 15 điểm phân biệt. Hỏi có bao nhiêu tam giác có đỉnh thuộc 25 điểm kể trên? *Phân tích 1: Mỗi tam giác tương ứng với 3 điểm không thẳng hàng thuộc 25 điểm , nếu 3 điểm cùng thuộc một đường thẳng thì không hình thành được tam giác. Lời giải 1: Chọn 3 điểm bất kỳ trong 25 điểm kể trên ta có: C253 cách chọn . Chọn 3 điểm cùng nằm trên đường thẳng a có : C103 cách chọn . Chọn 3 điểm cùng nằm trên đường thẳng b có : C153 cách chọn . Vậy có tất cả là : C253 - C103 - C153 =1725 tam giác. *Phân tích 2: Mỗi tam giác tương ứng với 3 điểm không thẳng hàng thuộc 25 điểm, bằng cách chọn 3 điểm không cùng thuộc một đường thằng nào ta có các trường hợp như sau: Lời giải 2: Tam giác tạo bởi một điểm nằm trên a và hai điểm nằm trên b ta có : C101 C152 tam giác Tam giác tạo bởi hai điểm nằm trên a và một điểm nằm trên b ta có: C102 C151 tam giác Vậy có tất cả số tam giác là: C101 C152 + C102 C151 =1725 tam giác. Ví dụ 25: Trong mặt phẳng cho 20 đường thẳng phân biệt a1 , a2 ,..., a20 song song với nhau từng đôi một và 30 đường thẳng phân biệt b1 , b2 ,..., b30 cùng vuông góc ( ) với các đường thẳng ai i = 1,20 . Tính số hình chữ nhật được tạo nên từ 50 đường thẳng đó? *Lời giải : Ta thấy cứ bốn đường thẳng gồm 2 đường thẳng ai , a j ( 1 ≤ i, j ≤ 20, i ≠ j ) và 2 đường thẳng bm , bn ( 1 ≤ m, n ≤ 30, m ≠ n ) cắt nhau tạo thành một hình chữ nhật . 41 Ngược lại, mỗi hình chữ nhật đều được tạo thành từ 4 đường thẳng gồm 2 đường thẳng ai , a j và 2 đường thẳng bm , bn . Do đó số hình chữ nhật cần tìm bằng số bộ bốn đường thẳng gồm 2 đường thẳng ai , a j và 2 đường thẳng bm , bn . Có C202 cách chọn đường thẳng ai , a j . Có C302 cách chọn đường thẳng bm , bn . Vậy có C202 × C302 =82650 hình chữ nhật. Ví dụ 26: Cho đa giác đều A1 A2 ... A2 n nội tiếp trong đường tròn tâm O. Biết rằng số tam giác có đỉnh là 3 trong 2n điểm A1 , A2 ,..., A2 n gấp 20 lần so với số hình chữ nhật có đỉnh là 4 trong 2n điểm A1 , A2 ,..., A2 n . Tìm n? Lời giải : 3 Số tam giác có các đỉnh là 3 trong 2n điểm A1 , A2 ,..., A2 n là C2n . Ta thấy ứng với hai đừơng chéo đi qua tâm O của đa giác A1 A2 ... A2 n cho tương ứng một hình chữ nhật có đỉnh là 4 điểm trong 2n điểm A1 , A2 ,..., A2 n và ngược lại mỗi hình chữ nhật như vậy sẽ cho tương ứng hai đường chéo đi qua tâm O của đa giác. Mà số đường chéo đi qua tâm của đa giác là n . Do đó số hình chữ nhật có đỉnh là 4 trong 2n điểm A1 , A2 ,..., A2 n là Cn2 . Theo giả thiết C23n = 20Cn2 ⇔ ( 2n ) ! 3!( 2n − 3) ! = 20 n! 2!( n − 2 ) ! 2n ( 2n − 1) ( 2n − 2 ) 20n ( n − 1) = 6 2 ⇔ n =8 ⇔ Ví dụ 27 : Cho lục giác lồi ABCDEF. a/Có bao nhiêu tam giác có đỉnh là đỉnh của lục giác đã cho? 42 b/ Trong các tìm được câu a/ có bao nhiêu tam giác có canh không phải là cạnh của lục giác? Lời giải : a/ Nhận thấy rằng cứ 3 đỉnh của hình lục giác thì tạo thành một tam giác và ngược lại một tam giác được tạo thành từ 3 đỉnh của hình lục giác, do đó có : C63 = 6! = 20 tam giác. 3!3! b/ Số tam giác có ít nhất 1 cạnh là cạnh của đa giác gồm 2 loại:  số tam giác chỉ có 1 cạnh của đa giác là C61  số tam giác có 2 cạnh của đa giác là C61.C21 Vậy số tam giác có 1 hoăc 2 cạnh là cạnh của lục giác là: C61 + C61.C21 =18 tam giác. Vậy có tất cả số tam giác có cạnh không phải là cạnh của lục giác sẽ là : 20-18 =2 tam giác. *Ví dụ 28: Có thể trả lời câu hỏi như trên với bát giác ABCDEFGH. a/Có bao nhiêu tam giác có đỉnh là đỉnh của bát giác ABCDEFGH? b/ Trong các tìm được câu a/ có bao nhiêu tam giác có cạnh không phải là cạnh của bát giác ABCDEFGH? Đáp số: 3 a/ có C8 = 8! = 56 tam giác có đỉnh là đỉnh của bát giác. 3!5! b/ có C81 + C81.C41 = C81.C51 = 40 tam giác có ít nhất 1 cạnh là cạnh của bát giác. Và có tất cả : 50 – 40 = 10 tam giác có cạnh không phải là cạnh của bát giác . Từ đó giải quyết bài toán Tổng quát như sau: “ Cho đa giác lồi n cạnh A1 A2 ... An . a/Có bao nhiêu tam giác có đỉnh là đỉnh của đa giác A1 A2 ... An ? 43 b/ Trong các tìm được câu a/ có bao nhiêu tam giác có cạnh không phải là cạnh của đa giác A1 A2 ... An ? ” Lời giải : a/ Số tam giác có đỉnh là đỉnh của đa giác A1 A2 ... An là số tổ hợp chập 3 của n phần tử Cn3 = n! ( n − 2 ) ( n − 1) n = 3!( n − 3) ! 6 b/ Số tam giác có ít nhất 1 cạnh là cạnh của đa giác A1 A2 ... An gồm 2 loại:  số tam giác chỉ có 1 cạnh của đa giác là Cn1  số tam giác có 2 cạnh của đa giác là Cn1 .Cn1−4 Vậy số tam giác có 1 hoăc 2 cạnh là cạnh của đa giác là: Cn1 + Cn1 .Cn1−4 = n + n ( n − 4 ) = n ( n − 3) = nCn1−3 = Cn1 .Cn1−3 Vậy có tất cả số tam giác có cạnh không phải là cạnh của đa giác sẽ là : Cn3 − Cn1 .Cn1−3 = = = ( n − 2 ) ( n − 1) n − n 6 ( n − 3) ( n − 2 ) ( n − 1) n − 6n ( n − 3) 6 n ( n 2 − 9n + 20 ) 6 Ví dụ 29: Cho n điểm trong không gian (không có điểm nào trùng nhau). Trong đó có m điểm đồng phẳng số còn lại không có 4 điểm nào đồng phẳng. Dựng tất cả các mặt phẳng chứa 3 trong n điểm đó. a/ Hỏi có bao nhiêu mặt phẳng khác nhau? b/ Hỏi có bao nhiêu tứ diện ? *Lời giải: a/ Mỗi mặt phẳng chứa 3 điểm trong n điểm kể trên nên số mặt phẳng là tổ hợp chập 2 của n : Cn3 44 Nhưng trong số đó có m điểm đồng phẳng khi đó m điểm này xác định chỉ 1 mặt phẳng . Số mặt phẳng tạo ra từ m điểm này là Cm3 và ta coi chúng chỉ là một mặt phẳng. Do đó số mặt phẳng cần tìm là: Cn3 - Cm3 +1 (mặt phẳng). b/ Chọn 4 điểm bất kỳ trong n điểm đã cho là tổ hợp chập 4 của n phần tử có: Cn4 Trong đó sẽ có chứa Cm4 không phải là tứ diện. Vì m điểm này đồng phẳng. Vậy có tất cả số tứ diện cần tìm là: Cn4 - Cm4 IV.CÁC BÀI TẬP TỰ GIẢI: 1. Lớp 11A có 40 học sinh , trong đó có 22 học sinh nam và 18 học sinh nữ. Giao viên chủ nhiệm muốn chọn ra một ban cán sự lớp gồm 1 lớp trưởng, một bí thư, một lớp phó văn nghệ, một lớp phó lao động. Hỏi có bao nhiêu cách chọn Ban cám sự lớp nếu: a. Ban cán sự lớp được chọn bất kỳ? b. Ban cán sự toàn con trai? c. Lớp trưởng phải là học sinh nam và lớp phó văn nghệ phải là học sinh nữ? 2. Một thầy giáo có 10 quyển sách khác nhau, trong đó có 4 quyển sách Toán,3 quyển sách Lý, 3 quyển sách Hoá. Thầy muốn lấy ra 5 quyển và tặng cho 5 học sinh A,B,C,D,E mỗi em một quyển. Hỏi thầy có bao nhiêu cách tặng nếu: a. Chỉ tặng cho học sinh các quyển sách Toán hoặc Hoá? b. Có ít nhất 1 quyển sách Toán được tặng? c. Sau khi tặng sách xong, mỗi loại còn ít nhất 1 quyển? 3. Một đội Văn nghệ có 18 người gồm 10 nam và 8 nữ. Hỏi có bao nhiêu cách lập 1 nhóm đồng ca gồm 6 người, biết rằng: a. Trong nhóm đó toàn là nữ? b. Trong nhóm đó có đúng 2 nam? c. Trong nhóm đó có ít nhất 5 nữ? d. Trong nhóm đó có ít nhất 1 nam? 45 4. Từ các chữ số 1,2,3,4,5,6 có thể lập được bao nhiêu số tự nhiên có 6 chữ số khác nhau và trong mỗi số đó tổng của 3 chữ số đầu nhỏ hơn tổng 3 chữ số cuối 1 đơn vị? 5. Từ các chữ số 0,1,2,3,4,5 có thể lập được bao nhiêu số tự nhiên mà mỗi số có 6 chữ số khác nhau sao cho a. Hai chữ số 2 và 3 đứng cạnh nhau? b. Hai chữ số 2 và 3 không đứng cạnh nhau? 6. Có bao nhiêu số gồm 5 chữ số sao cho tổng các chữ số của mỗi số là số lẻ? 7. Từ các chữ số 0,1,6,7,8,9 . Hãy tìm tất cả các số chẵn có 4 chữ số khác nhau và lớn hơn 5000? 8. Từ các chữ số 0,1,2,3,4,5. Hãy tìm tất cả các số có 5 chữ số khác nhau sao cho a/ chữ số hàng trăm là 2. b/ Luôn có mặt chữ số 4 và chữ số hàng nghìn là 5. 9. Có bao nhiêu số chẵn gồm 6 chữ số khác nhau đôi một trong đó chữ số đầu tiên là chữ số lẻ? 10. Có bao nhiêu số gồm 6 chữ số khác nhau đôi một trong đó có đúng 3 chữ số lẻ và chữ số chẵn? 11. Cho các chữ số 0,1,2,3,4. Hỏi có thể lập được bao nhiêu số có 7 chữ số trong đó chữ số 4 có mặt đúng 3 lần còn các chữ số khác có mặt đúng 1 lần? 12. Có bao nhiêu số tự nhiên gồm 6 chữ số đôi một khác nhau trong đó có mặt chữ số 0 nhưng không có mặt chữ số 1? 13. Có bao nhiêu số tự nhiên gồm 9 chữ số đôi một khác nhau biết rằng chữ số 2 có mặt đúng 2 lần , chữ số 3 có mặt 3 lần và các chữ số khác có mặt không quá 1 lần. 14. Từ các chữ số 0,1,2,5,6,7,8. Hãy tìm tất cả các số có 4 chữ số khác nhau sao cho a/ không tận cùng bằng 6. b/chia hết cho 5. 46 15. Trong mặt phẳng cho n điểm ( n ≥ 4 ), trong đó không có 3 điểm nào thẳng hàng và trong tất cả các đường thẳng nối 2 điểm bất kỳ, không có hai đường thẳng nào song song, trùng nhau hoặc vuông góc. Qua mỗi điểm vẽ các đường thẳng vuông góc với các đường thẳng được xác định bởi 2 trong n-1 điểm còn lại. Hỏi số giao điểm của các đường thẳng vuông góc giao nhau là bao nhiêu ? 16. Cho hình vuông ABCD. Trên mõ cạnh lấy 10 điểm phân biệt và không trùng với đỉnh. Hỏi có bao nhiêu tam giác mà ba đỉnh của nó chọn từ các điểm kể trên a. kể cả 4 đỉnh của hình vuông? b.Không tính các đỉnh hình vuông ? 17. Tìm số giao điểm tối đa của : a. 10 đường thẳng phân biệt? b. 6 đường tròn phân biệt? c. 10 đường thẳng và 6 đường tròn trên? 47 PHƯƠNG PHÁP GIẢI BÀI TOÁN VIẾT BIỂU THỨC CƯỜNG ĐỘ DÒNG ĐIỆN VÀ ĐIỆN ÁP TRONG ĐOẠN MẠCH ĐIỆN XOAY CHIỀU Tác giả: Đào Thị Phương Lan Chức vụ : Giáo viên Đơn vị : Trường THPT Vĩnh yên – Vĩnh Phúc Đối tượng học sinh bồi dưỡng : Lớp 12 Dự kiến số tiết bồi dướng : 3 tiết 1.HỆ THỐNG KIẾN THỨC SỬ DỤNG TRONG CHUYÊN ĐỀ : 1. 1. Các đoạn mạch mạch xoay chiều cơ bản : 1.1.1. Đoạn mạch chỉ có điện trở thuần : R A B Đặt vào hai đầu đoạn mạch điện áp xoay chiều biến thiên điều hòa u R = U 0 R cos(ωt + ϕu ) thì cường độ dòng điện tức thời qua mạch có phương trình i = I 0 cos(ωt + ϕu ) Với I 0 = U 0R R Quan hệ giữa về pha giữa điện áp và cường độ dòng điện: uR cùng pha với i 1.1.2. Đoạn mạch chỉ có cuộn dây thuần cảm L: A L B Đặt vào hai đầu đoạn mạch điện áp xoay chiều biến thiên điều hòa u L = U 0 L cos(ωt + ϕu ) π 2 thì cường độ dòng điện tức thời qua mạch có phương trình i = I 0 cos(ωt + ϕu − ) U U 0L 0L Với I 0 = Z = ω L L Quan hệ giữa về pha giữa điện áp và cường độ dòng điện: i góc π ) 2 π 2 uL sớm pha hơn hay i trễ pha hơn uL góc 48 1.1. 3. Đoạn mạch chỉ có tụ điện C: C A B Đặt vào hai đầu đoạn mạch điện áp xoay chiều biến thiên điều hòa uC = U 0C cos(ωt + ϕu ) π 2 thì cường độ dòng điện tức thời qua mạch có phương trình i = I 0 cos(ωt + ϕu + ) U 0C Với I 0 = Z = U 0C .ωC C Quan hệ giữa về pha giữa điện áp và cường độ dòng điện: π góc 2 uC trễ pha hơn i hay i sớm pha hơn uL góc π ) 2 1.1.4 Đoạn mạch có R, L, C mắc nối tiếp, cuộn dây thuần cảm : A U C L R M B N U 0 0 Với I 0 = Z = 2 R + ( Z L − ZC )2 ϕ là độ lệch pha giữa điện áp ở hai đầu đoạn mạch và dòng điện, được xác định theo biểu thức 1 Z L − ZC ωL − tanϕ = = ωC ; Với ϕ = ϕu − ϕi R R 1.1.5. Đoạn mạch có R, L, C mắc nối tiếp, cuộn dây có điện trở r : A R L,r C B M N Đặt vào hai đầu đoạn mạch điện áp xoay chiều biến thiên điều hòa u = U 0 cos(ωt + ϕu ) thì cường độ dòng điện tức thời qua mạch có phương trình i = I 0 cos(ωt + ϕu − ϕ ) I0 = U0 U0 = 2 Z ( R + r ) + ( Z L − ZC )2 ϕ là độ lệch pha giữa điện áp ở hai đầu đoạn mạch và dòng điện, được xác định theo biểu thức 1 Z L − ZC ωL − tanϕ = = ωC . Với ϕ = ϕu − ϕi R+r R+r Khi ZL = ZC thì u cùng pha hơn i ( ϕ = 0; đoạn mạch cộng hưởng ). 49 Khi ZL > ZC thì u nhanh pha hơn i (ϕ > 0; đoạn mạch có tính cảm kháng). Khi ZL < ZC thì u trễ pha hơn i ( ϕ < 0; đoạn mạch có tính dung kháng). 1.2. Phương pháp số phức : 1.2.1. Số phức : Trục ảo Số phức là số có dạng x = a + bi a: phần thực b: phần ảo i: đơn vị ảo , i2 = -1 1.2.2. Dạng lượng giác : b r α x = a + bi = r.cosα+(r.sinα)i = r (cosα+i. sinα) a Trục thực r = a 2 + b 2 môđun α : acgumen, tan α = b a 1.2.3. Dạng tọa độ cực x = r ∠α 1.2.4. Biểu diễn một hàm biến thiên điều hòa dưới dạng số phức Hàm điều hòa x = A cos(ωt + ϕ ) (*) Nếu biểu diễn dưới dạng véc tơ quay ,tại thời điểm t =0 ta có : ur  A = A u r t =0 x = A cos(ωt + ϕ ) ¬   →A :  uuur (0x, 0A) = ϕ Ta nhận thấy : a =Acosφ b = Asinφ y ur A b (*) có thể biểu diến bởi số phức x = a + bi =A(cos φ + i.sinφ) = A∠φ ϕ 0 a x 50 2. PHƯƠNG PHÁP GIẢI BÀI TOÁN VIẾT BIỂU THỨC DÒNG ĐIỆN VÀ ĐIỆN ÁP 2.1. Sử dụng phương pháp đại số giải các bài toán viết biểu thức dòng điện và điện áp : 2.1.1 Bài toán 1: Cho biểu thức điện áp ở hai đầu đoạn mạch AB, viết biểu thức cường độ dòng điện . A. Phương pháp: Đặt vào hai đầu đoạn AB điện áp uAB = U0 cos (ωt + ϕu), viết biểu thức i L C R A B M N Bước 1: Xác định các linh kiện có mặt trong đoạn mạch, giá trị các linh kiện Xác định tổng trở của đoạn mạch AB : ZAB Bước 2: Viết biểu thức định luật Ôm cho đoạn mạch I = U AB Z AB  I0 = U 0 AB Z AB Với ZAB= R(2AB ) + ( Z L ( AB ) − Z C ( AB ) )2 (1.1) Bước 3: Xác định độ lệch pha giữa điện áp ở hai đầu đoạn mạch và dòng điện tanϕAB = Z L ( AB ) − Z C ( AB ) R( AB ) (1.2) Lưu ý : Trong đoạn mạch AB không có linh kiện nào thì coi giá trị của linh kiện đó bằng không trong biểu thức (1.1);(1.2) Bước 4: Căn cứ vào biểu thức uAB từ đó viết biểu thức dòng điện i = I0 cos (ωt+ϕ u - ϕ AB) B. Bài tập vận dụng : Ví dụ 1: Một đoạn mạch điện xoay chiều gồm một điện trở thuần R = 80Ω, một cuộn dây thuần cảm có độ tự cảm L = 64mH và một tụ điện có điện dung C = 40 µ F mắc nối tiếp. Đoạn mạch được đặt vào điện áp xoay chiều có biểu thức u = 282cos314t (V). Lập biểu thức cường độ tức thời của dòng điện trong đoạn mạch. Hướng dẫn: Bước 1: Cảm kháng: Z L = ω L = 314.64.10−3 ≈ 20Ω 1 1 = ≈ 80Ω Dung kháng: Z C = ωC 314.40.10−6 Tổng trở: Z = R 2 + ( Z L − Z C ) = 802 + ( 20 − 80 ) = 100Ω 2 2 Bước 2: Cường độ dòng điện cực đại: U 282 Io = o = = 2,82 A Z 100 Bước 3: Độ lệch pha của hiệu điện thế so với cường độ dòng điện: 51 tan ϕ = Z L − Z C 20 − 80 3 = = − ⇒ ϕ ≈ −0,6435rad R 80 4 Bước 4: Vậy i = 2,82cos ( 314t + 0,6435 ) (A) +Ví dụ 2: Điện áp giữa hai đầu một đoạn mạch điện xoay chiều chỉ có tụ có điện dung C= 10−4 ( F ) có biểu thức u= 200 2 cos(100π t )(V ) . Viết biểu thức của π cường độ dòng điện trong mạch: Hướng dẫn: Bước 1: Tính Z C = 1 ω.C =100Ω, Bước 2: Tính Io hoặc I = U /.ZL =200/100 =2A; Bước3: i sớm pha góc π/2 so với u hai đầu tụ điện; Bước4: π 2 i = 2 2 cos(100π t + )( A) Ví dụ 3: Hiệu điện thế giữa hai đầu một đoạn mạch điện xoay chiều chỉ có cuộn π 3 1 π cảm có độ tự cảm L= ( H ) có biểu thức u= 200 2 cos(100πt + ) (V ) . Biểu thức cường độ dòng điện trong mạch là : Hướng dẫn: Bước 1: Tính Z L = ω L = 100π.1/π =100Ω, Bước 2: Tính I0 hoặc I = U /.ZL =200/100 =2A; Bước 3: u ở hai đầu cuộn cảm thuần sớm pha góc π/2 so với i ⇒ ϕ = π/2 Bước4: π 3 π 2 π 6 i = 2 2 cos(100π t + − ) ( A) = 2 2 cos(100π t − ) ( A) Ví dụ 4: Cho mạch điện xoay chiều như hình vẽ, R = 100Ω, L là độ tự cảm của 10−4 cuộn dây thuần cảm, C = F, RA ≈ 0. Điện áp u AB = 50 2 cos100π t (V). Khi 3π K đóng hay khi K mở, số chỉ của ampe kế không đổi. a. Tính độ tự cảm L của cuộn dây và số chỉ không đổi của ampe kế. b. Lập biểu thức của cường độ dòng điện tức thời trong mạch khi K đóng và khi K mở. Hướng dẫn: 52 Bước 1: a. Theo đề bài, điện áp và số chỉ ampe kế không đổi khi K đóng hay khi K mở nên tổng trở Z khi K mở và khi K đóng bằng nhau 2 Z m = Z d ⇔ R 2 + ( Z L − Z C ) = R 2 + Z C2 ⇒ ( Z L − Z C ) = Z C2 2  Z L − Z C = Z C ⇒ Z L = 2Z C ⇒ Z L − ZC = −ZC ⇒ Z L = 0 Ta ⇒ L= có: Z L 346 = ≈ 1,1 H ω 100π ZC = 1 = ωC (Loại) 1 = 173Ω 10−4 ; 100π . 3π ⇒ Z L = 2 Z C = 2.173 = 346Ω Bước 2: Số chỉ ampe kế bằng cường độ dòng điện hiệu dụng khi K đóng: U U 50 I A = Id = = = = 0,25 A Zd R 2 + Z C2 1002 + 1732 b. Biểu thức cường độ dòng điện: Bước 3: Khi K đóng: −Z −173 π = − 3 ⇒ ϕ d = − rad Độ lệch pha : tan ϕd = C = R 100 3 Khi K mở: Z − Z C 346 − 173 π = = 3 ⇒ ϕm = Độ lệch pha: tan ϕm = L R 100 3 Bước4: Vậy biểu thức của cường độ dòng điện khi K đóng π  id = 0,25 2 cos 100π t + ÷ (A). 3  Vậy biểu thức của cường độ dòng điện khi K mở π  im = 0,25 2 cos 100π t − ÷ (A). 3  2.1.2 Bài toán 2: Cho biểu thức dòng điện qua đoạn mạch, viết biểu thức điện áp hai đầu đoạn mạch AB. A. Phương pháp: Cho dòng điện chạy qua đoạn mạch AB là i = I0 cos (ωt+ϕi), viết biểu thức điện áp ở hai đầu đoạn mạch AB A C L R M N B 53 Bước 1: Xác định các linh kiện có mặt trong đoạn mạch, giá trị các linh kiện Xác định tổng trở của đoạn mạch AB : ZAB Bước 2: Viết biểu thức định luật Ôm cho đoạn mạch I = U AB Z AB  U 0 AB = I 0 .Z AB Với ZAB= R(2AB ) + ( Z L ( AB ) − Z C ( AB ) )2 (1.3) Bước 3: Xác định độ lệch pha giữa điện áp ở hai đầu đoạn mạch và dòng điện tanϕAB = Z L ( AB ) − Z C ( AB ) R( AB ) (1.4) Lưu ý : Trong đoạn mạch AB không có linh kiện nào thì coi giá trị của linh kiện đó bằng không trong biểu thức (1.3);(1.4) Bước 4: Căn cứ vào biểu thức dòng điện i từ đó viết biểu thức điện áp ở hai đầu đoạn mạch AB uAB = U0AB .cos (ωt+ϕ i +ϕ AB) B. Bài tập vận dụng : Ví dụ 1: Mạch điện xoay chiều gồm một điện trở thuần R = 50Ω, một cuộn thuần cảm có hệ số tự cảm L = 1 ( H ) và một tụ điện có điện dung π 2.10−4 C= ( F ) mắc nối tiếp. Biết rằng dòng điện qua mạch có dạng π i = 5cos100π t ( A ) .Viết biểu thức điện áp tức thời giữa hai đầu mạch điện. Hướng dẫn: Bước 1: 1 = 100Ω ; π 1 1 ZC = = = 50Ω 2.10−4 Dung kháng: ωC 100π . π Cảm kháng: Z L = ωL = 100π . Tổng trở: Z = R 2 + ( Z L − Z C ) = 502 + ( 100 − 50 ) = 50 2Ω 2 2 Bước 2: Định luật Ôm : Với Uo= IoZ = 5.50 2 = 250 2 V; Bước 3: Tính độ lệch pha giữa u hai đầu mạch và i: Z − Z C 100 − 50 π tan ϕ = L = = 1 ⇒ ϕ = (rad). R 50 4 Bước 4: Biểu thức điện áp tức thời giữa hai đầu mạch điện: π  u = 250 2 cos 100π t + ÷ (V). 4  Ví dụ 2: Một mạch điện xoay chiều RLC không phân nhánh có R = 100 Ω ; C= 1 2 .10−4 F ; L= H. cường độ dòng điện qua mạch có dạng: i = 2cos100 π t (A). π π 54 Viết biểu thức tức thời điện áp của hai đầu mạch và hai đầu mỗi phần tử mạch điện. Hướng dẫn : Bước 1: 2 π Cảm kháng : Z L = L.ω = 100π = 200Ω ; Dung kháng : ZC = 1 = ω .C 1 10−4 = 100 Ω 100π . π Tổng trở: Z = R 2 + ( Z L − ZC )2 = 1002 + ( 200 − 100 )2 = 100 2Ω Bước 2: HĐT cực đại :U0 = I0.Z = 2. 100 2 V =200 2 V Bước 3: Độ lệch pha: tan ϕ = Z L − ZC 200 − 100 π = = 1 ⇒ ϕ = rad ; R 100 4 Bước 4: π 4 Biểu thức HĐT : u = U 0 cos(ωt + ϕ ) = 200 2 cos(100π t + ) (V) - HĐT hai đầu R :uR = U0Rcos (ωt + ϕuR / i ) ; Với : U0R = I0.R = 2.100 = 200 V; Trong đoạn mạch chỉ chứa R : uR cùng pha i: uR = U0Rcos (ωt + 0) = 200cos 100πt V HĐT hai đầu L :uL = U0Lcos (ωt + ϕu ) Với : U0L = I0.ZL = 2.200 = 400 V; L/i π π : => ϕuL / i = rad 2 2 π uL = U0Lcos (ωt + ϕuL / i ) = 400cos (100πt + ) V 2 ( ω t + ϕ ) -HĐT hai đầu C :uC = U0Ccos uC / i Với : U0C = I0.ZC = 2.100 = 200V; π π Trong đoạn mạch chỉ chứa C : uC chậm pha hơn cđdđ : => ϕuL / i = − rad 2 2 π uC = U0Ccos (ωt + ϕuC /i ) = 200cos (100πt − ) V 2 Trong đoạn mạch chỉ chứa L: uL nhanh pha hơn cđdđ Ví dụ 3: Mạch điện xoay chiều gồm một điện trở thuần R = 40Ω, một cuộn 0 ,8 2 ( H ) và một tụ điện có điện dung C = .10−4 F π π i = 3cos(100 π t )( A ) mắc nối tiếp. Biết rằng dòng điện qua mạch có dạng thuần cảm có hệ số tự cảm L = Viết biểu thức điện áp tức thời giữa hai đầu điện trở, giữa hai đầu cuộn cảm, giữa hai đầu tụ điện, giữa hai đầu mạch điện. Hướng dẫn: Bước 1: Cảm kháng: Z L = ωL = 100π . 0,8 = 80Ω ; π 55 Dung kháng: ZC = 1 = ωC 1 = 50Ω 2.10−4 100π . π Tổng trở: Z = R 2 + ( Z L − Z C ) = 402 + ( 80 − 50 ) 2 Bước 2: Vì uR cùng pha với i nên : ϕuR/I = 0 π π Vì uL nhanh pha hơn i góc nên: ϕuL/I= 2 2 2 = 50Ω π π nên: ϕuC/I= 2 2 Z − Z C 80 − 50 3 = = ; ⇒ ϕ ≈ 37o Áp dụng công thức: tan ϕ = L R 40 4 37π ⇒ϕ = ≈ 0,2π (rad). 180 Bước 3: Với UoR = IoR = 3.40 = 120V Với UoL = IoZL = 3.80 = 240V; Với UoC = IoZC = 3.50 = 150V; Với Uo= IoZ = 3.50 = 150V; Bước 4: Căn cứ vào biểu thức của i u R = U oR cos100π t = 120cos(100πt) ( V) π π   u L = U oL cos 100π t + ÷= 240cos 100π t + ÷(V) 2 2   π π   uC = U oC cos 100π t − ÷=150cos 100π t − ÷(V) 2 2   Vì uC chậm pha hơn i góc Biểu thức hiệu điện thế tức thời giữa hai đầu mạch điện: u = U o cos ( 100π t + ϕ ) ; Vậy u = 150cos ( 100π t + 0,2π ) (V). 2.1.3 Bài toán 3: Cho biểu thức điện áp ở hai đầu đoạn mạch AB, viết biểu thức điện áp ở hai đầu đoạn mạch AN A. Phương pháp: Đặt vào hai đầu đoạn AB điện áp uAB = U0 cos (ωt + ϕu), viết biểu thức uAN L C R A B M N Bài toán kết hợp bước giải của hai bài toán 1 và bài toán 2 Bước 1: Xác định các linh kiện có mặt trong đoạn mạch, giá trị các linh kiện Xác định tổng trở của đoạn mạch AB : ZAB Bước 2: Viết biểu thức định luật Ôm cho đoạn mạch 56 I = U AB Z AB  I0 = U 0 AB Z AB Với ZAB= R(2AB ) + ( Z L ( AB ) − Z C ( AB ) )2 Bước 3: Xác định độ lệch pha giữa điện áp ở hai đầu đoạn mạch và dòng điện tanϕAB = Z L ( AB ) − Z C ( AB ) R( AB ) Bước 4: Căn cứ vào biểu thức uAB từ đó viết biểu thức dòng điện i = I0 cos (ωt+ϕ u - ϕAB) Bước 5: Xác định tổng trở của đoạn mạch AN, tìm điện áp U 0AN, theo biểu thức định luật Ôm U0AN = I0.ZAN Bước 6: Xác định độ lệch pha giữa điện áp ở hai đầu đoạn mạch AN và dòng điện tanϕAN ⇒ ϕAN. Bước 7: Căn cứ vào biểu thức dòng điện i từ đó viết biểu thức điện áp ở hai đầu đoạn mạch AN uAN = U0AN .cos (ωt+ϕ u - ϕ AB+ϕ AN.) B. Bài tập vận dụng : Ví dụ 1: Sơ đồ mạch điện có dạng như hình vẽ, điện trở R = 100Ω, cuộn thuần cảm L = 1 10−4 H, tụ điện C= F. Biểu thức điện áp giữa hai đầu AF là uAF = 200 cos100π t π 2π (V). Viết biểu thức điện áp giữa hai đầu đoạn mạch AB A R C L F Hướng dẫn: Bước 1: Cảm kháng: Z L = ω L = 100π . 1 = 200Ω Dung kháng: 10−4 100π . 2π Tổng trở của đoạn AF: Z AF = R 2 + Z L2 = 1002 + 1002 = 100 2Ω Bước 2: U 200 ⇒ I o = oAF = = 2 A Z AF 100 2 Bước 3: ZC = 1 = ωC 1 = 100Ω ; π Độ lệch pha giữa hiệu điện thế đoạn mạch AF và dòng điện ϕ AF : Z 100 π tan ϕ AF = L = = 1 ⇒ ϕ AF = rad R 100 4 Bước 4: 57 B π  i = 2 cos 100π t − ÷(A) 4  Bước 5: Tổng trở của toàn mạch: Z = 1002 + ( 100 − 200 ) = 100 2Ω 2 ⇒ U o = I o Z = 2.100 2 = 200 V Bước 6: Độ lệch pha giữa hiệu điện thế đoạn mạch AB và dòng điện ϕ AB : Z − Z C 100 − 200 π tan ϕ AB = L = = −1 ⇒ ϕ AB = − rad R 100 4 Bước 7: π π π  Vậy u AB = U 0 AB cos 100π t − − ÷ = 200cos(100π t − ) (V) 4 4 2  1 Ví dụ 2: Cho mạch điện như hình vẽ. Biết L = H, 10π 10−3 C= F và đèn ghi (40V- 40W). Đặt vào 2 điểm A và N 4π một hiệu điện thế u AN = 120 2 cos100π t (V). Các dụng cụ đo không làm ảnh hưởng đến mạch điện. a. Tìm số chỉ của các dụng cụ đo. b. Viết biểu thức cường độ dòng điện và điện áp toàn mạch. Hướng dẫn: Bước 1: 1 = 10Ω ; a.Cảm kháng: Z L = ω L = 100π . 10π 1 1 ZC = = = 40Ω Dung kháng: 10−3 ωC 100π . 4π 2 U đm 402 = = 40Ω Điện trở của bóng đèn: Rđ = Pđm 40 Tổng trở đoạn mạch AN: Z AN = Rđ2 + Z C2 = 402 + 402 = 40 2Ω Số chỉ của vôn kế: U AN = U oAN 120 2 = = 120 V 2 2 Bước 2: Số chỉ của ampe kế: I A = I = Io = I 2 = U AN 120 3 = = ≈ 2,12 A Z AN 40 2 2 3 . 2 = 3A 2 58 Bước 3: b. Ta có : tan ϕ AN = −ZC 40 = − = −1 ⇒ ϕ AN = − π rad Rđ 40 4 Bước 4: Biểu thức cường độ dòng điện có dạng: π  i = I o cos ( 100π t − ϕ AN ) = 3cos 100π t + ÷ (A). 4  Bước 5: Tổng trở của đoạn mạch AB: Z AB = Rđ2 + ( Z L − Z C ) = 402 + ( 10 − 40 ) = 50Ω 2 2 ⇒ U o = I o Z AB = 3.50 = 150 V Bước 6: Z L − Z C 10 − 40 3 = =− Ta có: tan ϕ AB = Rđ 40 4 ⇒ ϕ AB = − 37π rad 180 Bước 7: Biểu thức hiệu điện thế giữa hai điểm A, B có dạng: π   u AB = U o cos  100π t + + ϕ AB ÷ (V) 4   π 37π  π  Vậy u AB = 150cos 100π t + − ÷ = 150cos(100π t + ) (V) 4 180  20  2. 2. Sử dụng máy tính Casio giải bài toán viết biểu thức dòng điện, điện áp trong đoạn mạch điện xoay chiều 2.2.1. Chọn chế độ cài đặt cho máy tính casio FX-570ES: Chọn chế độ Chỉ định dạng nhập / xuất toán Thực hiện phép tính số phức Hiển thị dạng toạ độ cực: r∠θ Hiển thị dạng đề các: a + ib. Chọn đơn vị đo góc là độ (D) Chọn đơn vị đo góc là Rad (R) Nhập ký hiệu góc ∠ Nút lệnh Bấm: SHIFT MODE 1 Bấm: MODE 2 Bấm: SHIFT MODE  3 2 Bấm: SHIFT MODE  3 1 Bấm: SHIFT MODE 3 Bấm: SHIFT MODE 4 Bấm SHIFT (-) Ý nghĩa- Kết quả Màn hình xuất hiện : Math. Màn hình xuất hiện chữ CMPLX Hiển thị số phức dạng: A ∠ϕ Hiển thị số phức dạng: a+bi Màn hình hiển thị chữ D Màn hình hiển thị chữ R Màn hình hiển thị ∠ 59 Nhập ký hiệu phần ảo i Bấm ENG Màn hình hiển thị i 2.2.2. Các đại lượng điện xoay chiều biểu diễn dạng phức: Đại lượng điện Công thức Dạng số phức trong máy tính FX-570 ES ZL i (Chú ý trước i có dấu cộng là ZL ) Z L = L.ω Cảm kháng ZL Dung kháng ZC ZC = Tổng trở: Z = R 2 + ( Z L − ZC ) Cường độ dòng điện Điện áp Định luật Ôm - ZC i (Chú ý trước i có dấu trừ là Zc ) 1 ; ω .C 2 Z = R + ( Z L − Z C )i = a + bi ( với a=R; b = (ZL -ZC ) ) -Nếu ZL >ZC : Đoạn mạch có tinh cảm kháng -Nếu ZL u = i.Z => Z = i Z Chú ý: Z = R + ( Z L − Z C )i Tổng trở phức Z có gạch trên đầu: + R là phần thực, + (ZL -ZC ) là phần ảo) Cần phân biệt chữ i sau giá trị b = (ZL -ZC ) là phần ảo 2.3.1 Áp dụng máy tính FX-570 ES giải bài toán viết biểu thức i ,u 2.3.1 Bài toán 1: Cho biểu thức điện áp ở hai đầu đoạn mạch AB, viết biểu thức cường độ dòng điện . A. Phương pháp: Đặt vào hai đầu đoạn AB điện áp uAB = U0 cos (ωt + ϕu), viết biểu thức i L C R A B M N Bước 1: Xác định các linh kiện có mặt trong đoạn mạch, tính giá trị các linh kiện R; ZL = ω.L ; ZC = 1 ωC ; ( ZL-ZC) Bước 2: Chọn chế độ mặc định cho máy : 60 Bấm MODE 2 màn hình xuất hiện: CMPLX. Bấm SHIFT MODE  3 2 : dạng hiển thị toạ độ cực:( r∠θ ) -Chọn đơn vị đo góc là độ (D), bấm: SHIFT MODE 3 màn hình hiển thị D -Chọn đơn vị đo góc là radian (rad) bấm: SHIFT MODE 4 màn hình hiển thị R Bước 3: Thực hiện phép chia số phức U 0 ∠ϕ u u Ta có : i = = Z R + ( Z L − Z C )i Nhập U0  SHIFT (-) ϕu : ( R + ZL-ZC ENG ) = Hiển thị: I0 ∠ ϕ i Vậy : Biểu thức tức thời cường độ dòng điện qua mạch là: i = I0cos(100πt +ϕ i) B. Bài tập áp dụng : Ví dụ 1: Cho đoạn mạch xoay chiều có R=100 Ω , L= π 3 1 10−4 (H), C= (F), mắc π 2π nối tiếp điện áp 2 đầu mạch u=200 2 cos(100 π t+ ) (V), Cường độ dòng điện qua mạch Hướng dẫn giải Bước1: Xác định các linh kiện có mặt trong đoạn mạch, tính giá trị các linh kiện 1 1 ZC = = = 200Ω 1 Ω. 10−4 ω .C R =100 Ω ; Z L = L.ω = 100π = 100Ω ; 100π . π 2π Ω Và ZL-ZC =100 Bước2: Chọn chế độ mặc định cho máy : Đơn vị đo góc : rad Bước 3: Thực hiện phép chia số phức U 0 ∠ϕ u u Ta có : i = = Z R + ( Z L − Z C )i Nhập 200 2  SHIFT (-) π/3 : ( 100 - 100 ENG ) = Hiển thị: 2∠ 7π 12 Vậy : Biểu thức tức thời cường độ dòng điện qua mạch là: i = 2cos(100πt + 7π ) (A). 12 Ví dụ 2: Một đoạn mạch điện gồm điện trở R = 50Ω mắc nối tiếp với cuộn thuần cảm L = 0,5/π (H). Đặt vào hai đầu đoạn mạch một điện áp xoay chiều u = 100 2 cos(100πt- π/4) (V). Biểu thức của cường độ dòng điện qua đoạn mạch là: A. i = 2cos(100πt- π/2)(A). B. i = 2 2 cos(100πt- π/4) (A). 61 C. i = 2 2 cos100πt (A). D. i = 2cos100πt (A). Hướng dẫn giải Bước 1: Xác định các linh kiện có mặt trong đoạn mạch, tính giá trị các linh kiện R = 50Ω Z L = L.ω = ZC = 0 0 ,5 100π = 50Ω π Và ZL- ZC =50 Ω Bước 2: Chọn chế độ mặc định cho máy : Đơn vị đo góc: độ Bước 3: Thực hiện phép chia số phức U 0 ∠ϕ u u 100 2∠− 45 = . Ta có : i = = Z ( R + Z Li ) ( 50 + 50i ) Nhập 100  SHIFT (-) - 45 : ( 50 + 50 ENG ) = Hiển thị: 2∠- 90 Vậy : Biểu thức tức thời cường độ dòng điện qua mạch là: i = 2cos( 100πt - π/2) Chọn A 2 Ví dụ 3: Một đoạn mạch điện xoay chiều gồm một điện trở thuần R = 80Ω, một cuộn dây thuần cảm có độ tự cảm L = 64mH và một tụ điện có điện dung C = 40 µ F mắc nối tiếp. Đoạn mạch được đặt vào điện áp xoay chiều có biểu thức u = 282cos314t (V). Lập biểu thức cường độ tức thời của dòng điện trong đoạn mạch. Hướng dẫn giải Bước 1: Xác định các linh kiện có mặt trong đoạn mạch, tính giá trị các linh kiện Điện trở R = 80Ω Cảm kháng: Z L = ω L = 314.64.10−3 ≈ 20Ω 1 1 = ≈ 80Ω Dung kháng: Z C = ωC 314.40.10−6 ZL- ZC = - 60 Ω Bước 2: Chọn chế độ mặc định cho máy : Đơn vị đo góc rad Bước 3: Thực hiện phép chia số phức U 0 ∠ϕ u u 282∠0 = . Ta có : i = = Z ( R + Z Li ) ( 80 − 60i ) Nhập 282  SHIFT (-) 0 : ( 80 - 60 ENG ) = Hiển thị: 2,82∠0,6435 Vậy : Biểu thức tức thời cường độ dòng điện qua mạch là: i = 2,82cos( 100πt +0,6435) 2.3.2 Bài toán 2: Cho biểu thức dòng điện qua đoạn mạch, viết biểu thức điện áp hai đầu đoạn mạch AB. 62 A. Phương pháp: Cho dòng điện chạy qua đoạn mạch AB là i = I0 cos (ωt+ϕi), viết biểu thức điện áp ở hai đầu đoạn mạch AB A C L R M N B Bước 1: Xác định các linh kiện có mặt trong đoạn mạch, tính giá trị các linh kiện R; ZL = ω.L ; ZC = 1 ωC ; ( ZL-ZC) Bước 2: Chọn chế độ mặc định cho máy : Bấm MODE 2 màn hình xuất hiện: CMPLX. Bấm SHIFT MODE  3 2 : dạng hiển thị toạ độ cực:( r∠θ ) -Chọn đơn vị đo góc là độ (D), bấm: SHIFT MODE 3 màn hình hiển thị D -Chọn đơn vị đo góc là radian (rad) bấm: SHIFT MODE 4 màn hình hiển thị R Bước 3: Thực hiện phép nhân số phức Ta có : u = i.Z = I 0∠ ϕ i x ( R + ( Z L − ZC )i ) Nhập I0  SHIFT (-) ϕi x ( R + ZL-ZC ENG ) = Hiển thị: U0AB ∠ ϕ u Vậy : Biểu thức tức thời điện áp ở hai đầu đoạn mạch : uAB = U0AB cos(ωt +ϕ u) B. Bài tập áp dụng : Ví dụ 1: Mạch điện xoay chiều gồm một điện trở thuần R = 50Ω, một cuộn thuần cảm có hệ số tự cảm L = 1 ( H ) và một tụ điện có điện dung π 2.10−4 C= ( F ) mắc nối tiếp. Biết rằng dòng điện qua mạch có dạng π i = 5cos100π t ( A ) .Viết biểu thức điện áp tức thời giữa hai đầu mạch điện. Hướng dẫn giải Bước 1: Xác định các linh kiện có mặt trong đoạn mạch, tính giá trị các linh kiện Z L = ωL = 100π . 1 1 = 100Ω ; Z C = = .... = 50Ω . π ωC Và ZL-ZC =50 Ω Bước 2: Chọn chế độ mặc định cho máy : -Với máy FX570ES : Bấm MODE 2 màn hình xuất hiện: CMPLX. -Bấm SHIFT MODE  3 2 : dạng hiển thị toạ độ cực:( r∠θ ) -Chọn đơn vị đo góc là độ (D), bấm: SHIFT MODE 3 màn hình hiển thị D Bước 3: Thực hiện phép nhân số phức Ta có : u = i.Z . = I 0 .∠ ϕ i x ( R + (Z L − ZC )i = 5∠0 x( 50 + 50i ) ( Phép NHÂN hai số phức) 63 Nhập máy: 5 SHIFT (-) 0 x ( 50 + 50 ENG ) = Hiển thị: 353.55339∠45 = 250 2 ∠45 Vậy biểu thức tức thời điện áp của hai đầu mạch: u = 250 2 cos( 100πt +π/4) (V). Ví dụ 2: Một mạch điện xoay chiều RLC không phân nhánh có R = 100 Ω ; C= 1 2 .10−4 F ; L= H. Cường độ dòng điện qua mạch có dạng: π π π i = 2 2 cos(100 π t- )(A). Viết biểu thức điện áp tức thời của hai đầu mạch? 6 Hướng dẫn giải Bước 1: Xác định các linh kiện có mặt trong đoạn mạch, tính giá trị các linh kiện Z L = L.ω = 2 1 100π = 200Ω ; Z C = = ........= 100 Ω . Và ZL-ZC =100 Ω π ω .C Bước 2: Chọn chế độ mặc định cho máy : -Với máy FX570ES : Bấm MODE 2 màn hình xuất hiện: CMPLX. -Bấm SHIFT MODE  3 2 : Cài đặt dạng toạ độ cực:( r∠θ ) -Chọn đơn vị đo góc là độ (rad), bấm: SHIFT MODE 4 màn hình hiển thị R Bước 3: Thực hiện phép nhân số phức Ta có : u = i.Z = I 0∠ ϕ i x ( R + (Z L − ZC )i = 2 2 >∠ số phức) Nhập máy: 2 400∠ 2 π 12  SHIFT (-) − π 6 −π x ( 100 + 100i ) ( Phép NHÂN hai 6 x ( 100 + 100 ENG ) = Hiển thị: Vậy biểu thức tức thời điện áp của hai đầu mạch: u = 400cos( 100πt +π/12) (V). Ví dụ 3: Một mạch điện xoay chiều RLC không phân nhánh có R = 100 Ω ; C= 1 2 .10−4 F ; L= H. Cường độ dòng điện qua mạch có dạng: π π π i = 2 2 cos100 π t- )(A). Viết biểu thức điện áp tức thời của hai đầu mạch RC? 6 Hướng dẫn giải Bước 1: Xác định các linh kiện có mặt trong đoạn mạch, tính giá trị các linh 2 π kiện Z L = L.ω = 100π = 200Ω ; Z C = 1 = ........= 100 Ω . ω .C Bước 2: Chọn chế độ mặc định cho máy : -Với máy FX570ES : Bấm MODE 2 màn hình xuất hiện: CMPLX. -Bấm SHIFT MODE  3 2 : Cài đặt dạng toạ độ cực:( r∠θ ) -Chọn đơn vị đo góc là rad (R), bấm: SHIFT MODE 4 màn hình hiển thị R Bước 3: Thực hiện phép nhân số phức 64 Ta có : uRC = i.Z RC = I 0 .∠ ϕ i x (R − ZC i) = 2 2 >∠ − số phức) Nhập máy: 2 400∠ - 2  SHIFT (-) − 5π 12 π 6 π X ( 100 − 100i ) ( Phép NHÂN hai 6 x ( 100 - 100 ENG ) = Hiển thị: Vậy biểu thức tức thời điện áp của hai đầu mạch: u = 400cos( 100πt - 5π ) (V). 12 2.3.3 Bài toán 3: Cho biểu thức điện áp ở hai đầu đoạn mạch AB, viết biểu thức điện áp ở hai đầu đoạn mạch AN A. Phương pháp: Đặt vào hai đầu đoạn AB điện áp uAB = U0 cos (ωt + ϕu), viết biểu thức uAN L C R A B M N Bài toán kết hợp bước giải của hai bài toán 1 và bài toán 2 Bước 1: Xác định các linh kiện có mặt trong đoạn mạch AB , tính giá trị các linh kiện trong đoạn mạch AB. R; ZL = ω.L ; ZC = 1 ωC ; ( ZL-ZC) Xác định các linh kiện có mặt trong đoạn mạch AN, tính giá trị các linh kiện trong đoạn mạch AN : R; ZL = ω.L Bước2: Chọn chế độ mặc định cho máy : Bấm MODE 2 màn hình xuất hiện: CMPLX. Bấm SHIFT MODE  3 2 : dạng hiển thị toạ độ cực:( r∠θ ) -Chọn đơn vị đo góc là độ (D), bấm: SHIFT MODE 3 màn hình hiển thị D -Chọn đơn vị đo góc là radian (rad) bấm: SHIFT MODE 4 màn hình hiển thị R Bước 3: Thực hiện phép chia số phức U 0 AB ∠ϕ u u AB = Ta có : i = Z AB R + ( Z L − Z C )i Nhập U0AB  SHIFT (-) ϕu : ( R + ZL-ZC ENG ) = Hiển thị: I0 ∠ ϕ i Vậy : Biểu thức tức thời cường độ dòng điện qua mạch là: i = I0cos(100πt +ϕ i) Bước 4: Thực hiện phép nhân số phức Ta có : u AN = i.Z AN = I 0∠ ϕ i x ( R + Z L .i) ) Nhập I0  SHIFT (-) ϕi x ( R + ZL ENG ) = Hiển thị: U0AN ∠ ϕ uAN Vậy : Biểu thức tức thời cường độ dòng điện qua mạch là: uAN= U0AN cos(ωt +ϕ uAN) B. Bài tập áp dụng : Ví dụ 1(ĐH 2009): Khi đặt hiệu điện thế không đổi 30V vào hai đầu đoạn mạch gồm điện trở thuần mắc nối tiếp với cuộn cảm thuần có độ tự cảm L = 1/4π (H) thì cường độ dòng điện 1 chiều là 1A. Nếu đặt vào hai đầu đoạn mạch 65 này điện áp u =150 2 cos120πt (V). Viết biểu thức điện áp giữa hai đầu cuộn dây . Hướng dẫn giải Bước 1: Khi đặt hiệu điện thế không đổi (hiệu điện thế 1 chiều) thì đoạn mạch chỉ còn có R: R = U/I =30Ω Z L = L.ω = 1 120π = 30Ω ; 4π Bước2: Chọn chế độ mặc định cho máy : Với máy FX570ES : -Bấm MODE 2 màn hình xuất hiện: CMPLX. -Bấm SHIFT MODE  3 2 : Cài đặt dạng toạ độ cực: ( r∠θ ) -Chọn đơn vị góc là độ (D), bấm: SHIFT MODE 3 màn hình hiển thị D Bước 3: Thực hiện phép chia số phức u Z i= = 150 2∠0 (30 + 30i) ( Phép CHIA hai số phức) Nhập máy: 150  : ( 30 + 30 ENG ) = Hiển thị: 5∠- 45 Vậy: Biểu thức tức thời cường độ dòng điện qua mạch là: i = 5cos( 120πt π/4) (A). Bước 4: Thực hiện phép nhân số phức 2 Nhập 5  SHIFT (-) −π x 4 ZL ENG = Hiển thị: 150∠ π 4 Vậy: Biểu thức tức thời của điện áp ở hai đầu cuộn dây là: i = 150cos( 120πt +π/4) Ví dụ 2:Sơ đồ mạch điện có dạng như hình vẽ, điện trở R = 40Ω, cuộn thuần 3 10−3 cảm L = H, tụ điện C = F. Điện áp u AF = 120cos100π t (V). 10π 7π Hãy lập biểu thức của điện áp hai đầu mạch AB. Hướng dẫn giải Bước 1: Điện trở : R= 40Ω 3 = 30Ω 10π 1 1 ZC = = = 70Ω Dung kháng: 10−3 ωC 100π . 7π Đoạn mạch AF: R và ZL Đoạn mạch AB : R ; ZL và ZC. Bước2: Chọn chế độ mặc định cho máy : Với máy FX570ES : -Bấm MODE 2 màn hình xuất hiện: CMPLX. Cảm kháng: Z L = ω L = 100π . 66 -Bấm SHIFT MODE  3 2 : Cài đặt dạng toạ độ cực:( r∠θ ) -Chọn đơn vị góc là độ (rad), bấm: SHIFT MODE 4 màn hình hiển thị R Bước 3: Thực hiện phép chia số phức Nhập máy: 120  : ( 40 + 30 ENG ) = Hiển thị: 12 ∠ − 0, 6435 5 Vậy: Biểu thức tức thời cường độ dòng điện qua mạch là: 37π   12 i = cos( 100πt – 0,6435) (A)= 2,4cos 100π t − ÷ 5 180   Bước 4: Thực hiện phép nhân số phức Nhập 2,4  SHIFT (-) - 37π x ( 40 180 - 40 ENG ) Hiển thị: 96 2 ∠ - 41π 90 Vậy biểu thức của điện áp hai đầu mạch Ablà : 41π   u AB = 96 2 cos 100π t − ÷ (V) 90   Ví dụ 3 : Cho đoạn mạch điện xoay chiều AB không phân nhánh gồm một cuộn cảm thuần, một tụ điện có điện dung C thay đổi được, một điện trở hoạt động π 3 100Ω. Giữa AB có một điện áp xoay chiều luôn ổn định u=110cos(120πt- ) (V). Cho C thay đổi, khi C = 125 μF thì điện áp giữa hai đầu cuộn cảm có giá trị lớn 3π nhất. Biểu thức của điện áp giữa hai đầu cuộn cảm là A. π 6 π u L =110 2cos(120πt+ ) (V). 6 B. u L =264cos(120πt+ ) (V). π 6 C. u L =220cos(120πt+ ) (V). π 2 D. u L =110 2cos(120πt+ ) (V). Hướng dẫn giải Bước 1: Điện trở : R= 100Ω khi thay đổi C để ULmax thì Z L = Z C 1 1 Z L = ZC = = Cảm kháng: ωC 100π .125 .10−6 =240Ω 3π Bước2: Chọn chế độ mặc định cho máy : Với máy FX570ES : -Bấm MODE 2 màn hình xuất hiện: CMPLX. -Bấm SHIFT MODE  3 2 : Cài đặt dạng toạ độ cực:( r∠θ ) -Chọn đơn vị góc là độ (rad), bấm: SHIFT MODE 4 màn hình hiển thị R Bước 3: Thực hiện phép chia số phức Nhập máy: 110  SHIFT (-) − π : ( 100 3 ) = Hiển thị: 11 π ∠− 10 3 Vậy: Biểu thức tức thời cường độ dòng điện qua mạch là: π 3 i = 1,1 cos( 100πt − ) (A) Bước 4: Thực hiện phép nhân số phức 67 Nhập 1,1  SHIFT (-) − π π x ( 240 ENG ) = Hiển thị: 264∠ 3 6 Biểu thức của điện áp giữa hai đầu cuộn cảm là π u L =264cos(120πt+ ) 6 4. MỘT SỐ CÂU HỎI TRẮC NGHIÊM THAM KHẢO Câu 1. Cho đoạn mạch R, L, C (cuộn dây thuần cảm ) mắc nối tiếp với R= 100 1 10 −4 Ω , L= H, C= F. Đặt điện áp xoay chiều vào giữa hai đầu đoạn mạch u R ,L = π 2π π 200 2 cos(100πt + ) (V). Biểu thức điện áp ở hai đầu đoạn mạch có dạng: 2 u = 200 cos( 100 π t )V A. B. u = 200 2 cos(100πt )V π π C. u = 200 cos(100πt + )V D. u = 200 2 cos(100πt + )V 3 4 Câu 2. Cho đoạn mạch R, L, C (cuộn dây thuần cảm ) mắc nối tiếp với R=50 Ω , 1 L= H. đặt điện áp xoay chiều u = U 2 cos(100πt )V vào hai đầu đoạn mạch thì π π u L = 100 cos(100π t + ) . Biểu thức uc là: 4 π π A. uc = 50 cos(100πt − ) (V) B . uc= 50 2 cos(100πt − ) (V) 2 4 3π 3π C. uc= 50 cos(100πt − ) D. uc = 50 2 cos(100πt − ) 4 4 Câu 3: Một đoạn mạch gồm một tụ điện có dung kháng Z C = 100Ω và cuộn dây có cảm kháng ZL = 200Ω mắc nối tiếp nhau. Hiệu điện thế tại hai đầu cuộn cảm π 6 có dạng u L = 100 cos(100πt + )V . Biểu thức hiệu điện thế ở hai đầu tụ điện có dạng như thế nào? A. u C = 50 cos(100πt − π )V 3 C. u C = 100 cos(100πt + π )V 6 B. uC = 50 cos(100πt − 5π )V 6 D. uC = 100 cos(100πt − π )V 2 Câu 4. Cho đoạn mạch R, L, C (cuộn dây thuần cảm ) mắc nối tiếp u = 240 V, R = 40Ω, ZC = 60Ω , ZL= 20 Ω.Viết biểu thức của dòng điện trong mạch A. i = 3 2 cos(100π t ) A B. i = 6 cos(100π t ) A π 4 C. i = 3 2 cos(100π t + ) A π 4 D. i = 6 cos(100π t + ) A Câu 5. Cho mạch điện R,L,C cho u = 240 2 cos(100πt) V, R = 40 Ω, ZL = 60 Ω , ZC = 20Ω, Viết biểu thức của cường độ dòng điện trong mạch A. i = 3 2 cos(100π t ) A. B. i = 6 cos(100π t ) A. π 4 C. i = 3 2 cos(100π t − ) A π 4 D. i = 6 cos(100π t − ) A 68 Câu 6. Cho mạch R,L,C, R = 40Ω, ZL = ZC = 40 Ω, u = 240 2 cos(100πt). Viết biểu thức i A. i = 6 2 cos(100πt )A B. i = 3 2 cos(100πt)A C. i = 6 2 cos(100πt + π/3)A D. 6 2 cos(100πt + π/2)A Câu 7. Cho mạch R,L,C, u = 120 2 cos(100πt)V. R = 30 Ω, ZL = 10 3 Ω , ZC = 20 3 Ω, xác định biểu thức i. A. i = 2 3 cos(100πt)A B. i = 2 6 cos(100πt)A C. i = 2 3 cos(100πt + π/6)A D. i = 2 6 cos(100πt + π/6)A Câu 8: Mạch điện xoay chiều gồm tụ điện C = 10 −4 F, cuộn dây thuần cảm L = π 1 H mắc nối tiếp. Biết cường độ dòng điện là i = 4cos(100πt) (A). Biểu thức 10π điện áp hai đầu mạch ấy là như thế nào? π 2 π π C. u = 220sin(100πt - ) (V) D. u = 360cos(100πt - ) (V) 2 2 0, 04 Câu 9: Điện áp giữa hai đầu một cuộn dây có r =4 Ω ; L= (H) có thức: π π u = 200 2 cos(100πt + )(V ) . Biểu thức của cường độ dòng xoay chiều trong mạch 3 A. u = 36 2 cos(100πt -π) (V) là: π )(A) 12 π C. i = 50cos(100πt - )(A) 12 A. i = 50cos(100πt + B. u = 360cos(100πt + ) (V) π )(A) 12 π D. i = 50 2 cos(100πt + )(A) 12 B. i = 50 2 cos(100πt - Câu 10: Cho đoạn mạch xoay chiều AB gồm hai đoạn mạch AN và NB mắc nối tiếp. Đặt vào hai đầu đoạn mạch AB một điện áp xoay chiều ổn định u AB = 200 2 cos(100πt + π / 3) (V ) , khi đó điện áp tức thời giữa hai đầu đoạn mạch NB là u NB = 50 2 sin(100πt + 5π / 6) (V) . Biểu thức điện áp tức thời giữa hai đầu đoạn mạch AN là A. u AN = 150 2 sin(100πt + π / 3) (V) . B. u AN = 150 2 cos(120πt + π / 3) (V) . C. u AN = 150 2 cos(100πt + π / 3) (V) . D. u AN = 250 2 cos(100πt + π / 3) (V) . Câu 11: Cho mạch điện xoay chiều có R=30 Ω , L= 1 10 −4 (H), C= (F); điện áp π 0.7π 2 đầu mạch là u=120 2 cos100 π t (V), thì cường độ dòng điện trong mạch là π  π i = 2cos(100π t − )( A) 4   A. i = 4 cos 100π t + ÷( A) 4 C. π 4 π D. i = 2cos(100π t + )( A) 4 B. i = 4cos(100π t − )( A) 69 Câu 12:Cho đoạn mạch gồm R, L, C mắc nối tiếp; R = 10 3Ω ; L = 0,3 / π (H); C = 10−3 / 2π (F). Đặt vào hai đầu đoạn mạch một hiệu điện thế u = 100 2 cos ( 100π t ) (V). a) Viết biểu thức cường độ dòng điện trong mạch A. i = 5 2cos ( 100π t − π / 6 ) (A) B. i = 5 2cos ( 100π t + π / 6 ) (A) C. i = 5cos ( 100π t − π / 6 ) (A) D. i = 5cos ( 100π t + π / 6 ) (A) b) Viết biểu thức hiệu điện thế hai đầu mỗi phần tử R; L; C A. uR = 86,5 2 cos ( 100π t + π / 6 ) ; uL = 150 2 cos ( 100π t + π / 3) ; uC = 100 2 cos ( 100π t − 2π / 3) B. A. u R = 86,5 2 cos ( 100π t − π / 6 ) ; uC = 100 cos ( 100π t − 2π / 3) u L = 150 cos ( 100π t + π / 3) ; C. uR = 86,5 2 cos ( 100π t − π / 6 ) ; uL = 150 2 cos ( 100π t + π / 3) ; uC = 100 2 cos ( 100π t − 2π / 3) D. uR = 86,5 2 cos ( 100π t + π / 6 ) ; uL = 150 2 cos ( 100π t + π / 3) ; uC = 100 2 cos ( 100π t + 2π / 3) Câu 13: Cho mạch xoay chiều có R, L, C mắc nối tiếp có R=30 Ω , C= 10−4 (F) , π L thay đổi được cho hiệu điện thế 2 đầu mạch là U=100 2 cos100 π t (V) , để u π rad thì ZL và i khi đó là: 6 5 2 π cos(100π t − )( A) A. Z L = 117,3(Ω), i = B. 6 3 π 5 2 π Z L = 100(Ω), i = 2 2cos(100π t − )( A) cos(100π t + )( A) C. Z L = 117,3(Ω), i = 6 6 3 π C. Z L = 100(Ω), i = 2 2cos(100π t + )( A) 6 nhanh pha hơn i góc Câu 14: Một mạch gồm cuộn dây thuần cảm có cảm kháng bằng 10 Ω mắc nối 2 π tiếp với tụ điện có điện dung C = .10−4 F . Dòng điện qua mạch có biểu thức π ) A . Biểu thức hiệu điện thế của hai đầu đoạn mạch là: 3 π π A. u = 80 2co s(100π t − ) (V) B. u = 80 2 cos(100π t + ) (V) 6 6 π 2π C. u = 120 2co s(100π t − ) (V) D. u = 80 2co s(100π t + ) (V) 6 3 R = 40 Ω Câu 15: Mạch điện xoay chiều gồm điện trở ghép nối tiếp với cuộn u = 80 co s100π t và điện áp hiệu cảm L. Hiệu điện thế tức thời hai đầu đoạn mạch dụng hai đầu cuộn cảm U L =40V Biểu thức i qua mạch là: i = 2 2 cos100π t + A. i = 2 π 2 π co s(100π t − ) A B. i = co s(100π t + ) A 2 4 2 4 70 π 4 π 4 C. i = 2co s(100π t − ) A D. i = 2co s(100π t + ) A Câu 16: Một đoạn mạch điện gồm điện trở R = 50Ω mắc nối tiếp với cuộn thuần cảm L = 0,5/π (H). Đặt vào hai đầu đoạn mạch một điện áp xoay chiều u = 100 2 cos(100πt - π/4) (V). Biểu thức của cường độ dòng điện qua đoạn mạch là: A. i = 2cos(100πt - π/2) (A). B. i = 2 2 cos(100πt - π/4) (A). C. i = 2 2 cos100πt (A). D. i = 2cos100πt (A). Câu 17: Khi đặt điện áp không đổi 30V vào hai đầu đoạn mạch gồm điện trở thuần mắc nối tiếp với cuộn cảm thuần có độ tự cảm 1 (H) thì dòng điện trong 4π đoạn mạch là dòng điện một chiều có cường độ 1 A. Nếu đặt vào hai đầu đoạn mạch này điện áp u = 150 2 cos120πt (V) thì biểu thức của cường độ dòng điện trong đoạn mạch là π 4 π 4 A. i = 5 2 cos(120πt − ) (A). π 4 B. i = 5cos(120πt + ) (A). C. i = 5cos(120πt − ) (A). π 4 D. i = 5 2 cos(120πt + ) (A). Câu 18: Cho đoạn mạch xoay chiều LRC mắc nối tiếp hai đầu AB, L mắc vào AM, R mắc vào MN, C mắc vào NB. Biểu thức dòng điện trong mạch i = I 0 cos 100 π t (A). Điện áp trên đoạn AN có dạng u AN = 100 2cos ( 100π t + π / 3) (V) và lệch pha 900 so với điện áp của đoạn mạch MB. Viết biểu thức uMB ? 100 6 π  cos 100π t − ÷ 3 6  100 6 π  = cos 100π t + ÷ 3 6  A. uMB = B, uMB = 100cos ( 100π t ) C. uMB D. uMB = 100cos 100π t − ÷ 6   Câu 19: Đặt điện áp xoay chiều u = Uocos(100πt + cuộn cảm thuần có độ tự cảm L= π  π ) (V) vào hai đầu một 3 1 (H). Ở thời điểm điện áp giữa hai đầu cuộn 2π cảm là 100 2 V thì cường độ dòng điện qua cuộn cảm là 2 A. Biểu thức của cường độ dòng điện qua cuộn cảm là π ) (A). 6 π C. i = 2 2 cos(100πt + ) (A). 6 A. i = 2 3 cos(100πt + π ) (A). 6 π D. i = 2 3 cos(100πt - ) (A). 6 B. i = 2 2 cos(100πt - Câu 20: Xét đoạn mạch gồm một điện trở hoạt động bằng 100Ω, một tụ điện 3 50 µ F và một cuộn cảm thuần có độ tự cảm H mắc nối tiếp. π π Nếu đặt vào hai đầu một điện áp u = 200cos100π t (V) thì điện áp giữa hai đầu có điện dung C = điện trở hoạt động có biểu thức 71 π 4 B. uR = 100 2 cos(100π t ) (V). π 4 D. u R = 100 2 cos(100π t − ) (V). A. uR = 200cos(100π t − ) (V). C. uR = 200cos(100π t + ) (V). π 4 72 HỆ THỐNG LÝ THUYẾT VÀ BÀI TẬP VỀ KIM LOẠI KIỀM, KIỀM THỔ VÀ HỢP CHẤT QUAN TRỌNG CỦA CHÚNG Người viết: GV. Nguyễn Thị Lan Phương Chức vụ: Tổ phó - Nhóm trưởng chuyên môn Đơn vị công tác: Trường THPT Vĩnh Yên Đối tượng bồi dưỡng: HS lớp 12 Thời lượng: 8 tiết A. MỤC TIÊU CỦA CHUYÊN ĐỀ - Giúp học sinh hệ thống lại lý thuyết, so sánh sự giống và khác nhau về tính chất của đơn chất cũng như hợp chất của các kim loại nhóm IA, IIA. - Giúp HS nắm được được các dạng bài tập thường gặp đồng thời xây dựng được phương pháp giải cho từng dạng thuộc phần lý thuyết dang xét. - Rèn luyện kỹ năng và tốc độ làm bài thông qua hệ thống bài tập tự luyện. B. NỘI DUNG Trong chuyên đề này tôi chia thành 2 phần: Phần 1: Hệ thống lý thuyết và bài tập về phản ứng của đơn chất kim loại với nước và dung dịch nước; phản ứng của dung dịch axit mạnh với dung dịch bazo mạnh. Phần 2. Phương pháp giải bài toán sục khí CO2, SO2 vào dung dịch kiềm và phản ứng của muối cacbonat với dung dịch axit. Tương ứng với mỗi dạng toán tôi chỉ xin giới thiệu phương pháp giải chung và chỉ giải một số bài tiêu biểu, còn lại học sinh tự giải và tự nghiên cứu. Tất cả các bài tập tham khảo đều có đáp án. 1. HỆ THỐNG LÝ THUYẾT VÀ BÀI TẬP VỀ PHẢN ỨNG CỦA ĐƠN CHẤT KIM LOẠI VỚI NƯỚC VÀ DUNG DỊCH NƯỚC. PHẢN ỨNG CỦA DUNG DỊCH AXIT MẠNH VỚI DUNG DỊCH BAZO MẠNH 1.1. HỆ THỐNG LÝ THUYẾT VỀ KIM LOẠI KIỀM, KIỀM THỔ VÀ HỢP CHẤT QUAN TRỌNG CỦA CHÚNG Nội dung lý thuyết được tiến hành nhanh gọn bởi sau khi học xong nội dung kim loại nhóm IA, IIA, giáo viên đã yêu cầu học sinh về nhà tổng hợp các nội dung kiến thức theo mẫu (phụ lục I). Đến lớp, GV chiếu bảng tổng kết kiến thức của phần này lên bảng và HS tự bổ sung, hoàn thiện bảng tổng kết của mình. (Riêng với cột kiến thức của phần Al, khi nào học xong lý thuyết trên lớp, giáo viên lại yêu cầu học sinh điền nốt các nội dung còn trống). 73 74 Ýnh chÊt ho¸ häc TÝnh chÊt vËt lý VÞ trÝ, cÊu t¹o 1.1.1. ĐƠN CHẤT KIM LOẠI NHÓM A DÊu hiÖu so s¸nh Nhãm IA - Thµnh phÇn: 3Li, 11Na, 19K, 54Rb, 68Cs, 87Fr - CÊu h×nh e ho¸ ns1 trÞ: +1 - Sè oxh ®Æc trng: lptk - CÊu t¹o ®¬n chÊt: - Trạng thái tồn tại - R¾n, mµu tr¾ng b¹c. - Tnc, Ts vµ khèi l- - Tnc, Ts vµ khèi lîng riªng, îng riªng, ®é cøng ®é cøng thÊp nhÊt trong sè c¸c kim lo¹i. - §é dÉn ®iÖn cao - Độ dẫn điện - P víi O2 : - P víi Hal2: - P víi axit: - P víi níc: - P víi dd kiÒm ®Æc: * P víi oxit kim lo¹i: TÝnh khö m¹nh, t¨ng dÇn tõ Li → Cs 4Li + O2  → 2Li2O 2Na + O2  → Na2O2 M + O2  → MO2 (M: K, Rb, Cs) 2M + X2  → 2MX 1 H2 2 1 M +H2O → MOH + H2 2 M + H+  → M+ + - Nhãm IIA 4Be, 12Mg, 20Ca, 38Sr, 56Ba ns2 +2 Be, Mg (lp); Ca, Sr (lptd); Ba (lptk). - R¾n, mµu tr¾ng hoÆc x¸m nh¹t. - Ts, Tnc vµ ®é cøng thÊp (trõ Be); khèi lîng riªng thÊp nhng cao h¬n so víi KLK. - §é dÉn ®iÖn cao. TÝnh khö m¹nh, t¨ng dÇn tõ Be → Ba 2M + O2  → 2MO M + O2  → MO2 (M: Ca, Sr, Ba) Nhãm IIIA (Al) t M + X2 → MX2 + M + 2H  → M2+ + H2 t Mg + H2O → MgO + H2 M + H2O → M(OH)2 + H2 (M: Ca, Ba, Sr) 0 0 75 d, ®iÒu chÕ * øng dông: Dïng ®Ó chÕ t¹o hîp kim cã Tnc thÊp, ... * §iÒu chÕ 2MCl(l) ®fnc 2M(r) + Cl2(k) Mg ®îc dïng ®Ó chÕ t¹o hîp kim nhÑ, ®fnc cøng, bÒn... MCl2(l) M(r) + Cl2(k) Hîp chÊt cña nh«m Oxit 1.1.2. HỢP CHẤT QUAN TRỌNG CỦA KIM LOẠI NHÓM A DÊu hiÖu so Hîp chÊt cña nhãm IA Hîp chÊt cña nhãm IIA s¸nh - T/c - R¾n, cã Ts, Tnc vµ ®é bÒn nhiÖt gi¶m - MO lµ chÊt r¾n, mµu tr¾ng. vật lý dÇn tõ Li2O ®Õn Cs2O. - Lµ nh÷ng oxit baz¬ ®iÓn h×nh vµ ®Òu - MO tan ®îc trong níc trõ (BeO, t¸c dông ®îc víi oxi: MgO) vµ lµ oxit baz¬ (trõ BeO): - T/c M2O + O2  → M2O2 (trõ Li2O) hóa - §iÒu chÕ: học t M(OH)2 ↓ → MO + H2O t HoÆc: MCO3 → MO + CO2 0 0 76 Hi®roxit Muèi -R¾n, dÔ tan trong níc (trõ LiOH), bÒn -R¾n, kh¶ n¨ng tan trong níc vµ ®é víi nhiÖt (trõ LiOH): bÒn víi nhiÖt t¨ng dÇn tõ Be(OH)2 t → Ba(OH)2. 2LiOH → Li2O + H2O - M(OH)2 ®Òu cã tÝnh baz¬ (trõ - MOH lµ c¸c baz¬ ®iÓn h×nh Be(OH)2) * §iÒu chÕ * §iÒu chÕ: + Trong c«ng nghiÖp: MO + H2O → M(OH)2 2MCl + 2 H2O ®fdd m.n 2 MOH + Cl2 + H2 (M: Na, (M: Ca, Sr, Ba) K) M2+ + 2OH- → M(OH) 2 + PTN: M2O + H2O → 2MOH (M: Be, Mg) Kh¸i Tån t¹i ë thÓ r¾n, ®a sè ®Òu kh«ng mµu, Tån t¹i ë thÓ r¾n, ®é tan kÐm h¬n so dÔ tan trong níc, dÉn ®iÖn khi nãng ch¶y víi muèi cña c¸c KLK t¬ng øng; qu¸t c¸c M2+ ®Òu kh«ng mµu. vÒ c¸c muèi - MX ®Òu lµ hîp chÊt ion (trõ LiF). - MX2 cã cÊu tróc tinh thÓ kh«ng * Muèi ⇒ C¸c MX cã Tnc, Ts cao; c¸c gi¸ trÞ nµy t- gièng nhau. Ts, Tnc kh¸ cao. halog ¬ng øng gi¶m tõ LiX → CsX. enua 0 77 * Muèi cacbo nat - MHCO3 vµ M2CO3 ®Òu ë thÓ r¾n, tan tèt trong níc (trõ NaHCO3 vµ Li2CO3) thuû ph©n cho m«i trêng baz¬. - MHCO3 kÐm bÒn víi nhiÖt, nhng M2CO3 bÒn víi nhiÖt: T M2CO3 t> → M2O + CO2 øng dông: - NaHCO3: dïng lµm thuèc ch÷a bÖnh d¹ dµy, g©y xèp cho c¸c lo¹i b¸nh… - Na2CO3: lµ nguyªn liÖu trong s¶n xuÊt thuû tinh, xµ phßng… C¸c muèi ®Òu tan. 0 * Muèi sunfat nc - M(HCO3)2, MCO3 lµ chÊt r¾n, kÐm bÒn víi nhiÖt. M(HCO3)2 tan tèt nhng MCO3 Ýt tan vµ thùc tÕ: MCO3 + CO2 + H2O M(HCO3)2 * Níc cøng: - NÕu níc chøa nhiÒu ion Ca2+, Mg2+ ⇒ gäi lµ níc cøng. - Ngêi ta chia ra: níc cøng t¹m thêi, níc cøng vÜnh cöu, níc cøng toµn phÇn. Khö ®é cøng cña níc b»ng c¸ch lµm gi¶m nång ®é c¸c cation Ca2+, Mg2+ - CaSO4, BaSO4 Ýt tan. Quan träng nhÊt lµ th¹ch cao CaSO4.2H2O, khi nung nãng biÕn thµnh th¹ch cao nung 2CaSO4.H2O råi th¹ch cao khan CaSO4. øng dông: Th¹ch cao nung dïng ®Ó nÆn tîng, lµm khu«n ®óc, vËt liÖu x©y dùng vµ bã chØnh h×nh 78 Việc làm này giúp HS tự so sánh, hệ thống hóa được sự giống và khác nhau về tính chất của các kim loại nhóm IA và nhóm IIA cũng như hợp chất của chúng, đồng thời, phát hiện ra những lỗ hổng kiến thức và bổ sung kịp thời. Đây cũng đồng thời tạo một tiền đề thuận lợi để học sinh giải các bài tập lý thuyết cũng như bài tập tính toán. 1.2. BÀI TOÁN VỀ ĐƠN CHẤT KIM LOẠI NHÓM IA, IIA TÁC DỤNG VỚI NƯỚC VÀ DUNG DỊCH NƯỚC 1.2.1. Đặc điểm của bài toán về đơn chất kim loại nhóm IA, IIA tác dụng với dung dịch nước Dung dịch nước gồm : dung dịch axit, dung dịch bazo, dung dịch muối. Như vậy, khi cho đơn chất kim loại tác dụng với nước hoặc dung dịch nước thì cần lưu ý : - Tác dụng với nước : Các kim loại đứng trước Mg tác dụng được với nước ở nhiệt độ thường tạo dung dịch kiềm và giải phóng khí H2. Na + H2O → NaOH + VD: 1 H2 ↑ 2 Ca + H2O → Ca(OH)2 + H2 ↑ Tổng quát M + nH2O → M(OH)n + n H 2 ↑ (n là hóa trị của kim loại, n = 1, 2) 2 Bản chất: là quá trình kim loại khử H+ trong nước: 2H2O + 2e → 2OH- + H2 ↑ Lưu ý tỉ lệ: n e =n OH =2n H - 2 Thường áp dụng: • Tính khối lượng bazơ thu được: m hhbazo =m hhkl +m OH - • Trung hòa dung dịch thu được bằng dung dịch axit, yêu cầu tính thể tích (nồng độ) của dung dịch axit: n H =n OH =2n H + - 2 - Một hỗn hợp hai kim loại tan được trong nước có thể xảy ra 2 khả năng: hoặc cả 2 kim loại đều tác dụng với nước ở điều kiện thường, hoặc chỉ có một kim loại tác dụng với nước tạo dung dịch kiềm, kim loại còn lại tác dụng với dung dịch kiềm mới sinh ra. - Tác dụng với dung dịch axit: Kim loại đứng trước Mg khi tác dụng với dung dịch axit sẽ phản ứng với axit trước, nếu dư kim loại mới có phản ứng với nước. - Tác dụng với dung dịch muối: Kim loại nhóm IA, IIA (trừ Mg, Be) tác dụng với dung dịch muối thì chúng sẽ phản ứng với nước trước, sau đó xét đoán khả năng phản ứng của phản phẩm tạo thành với dung dịch muối. Ví dụ 1: Viết PTPƯ xảy ra khi hòa tan Na vào dung dịch KNO3. 2Na + 2H2O NaOH + KNO3 2NaOH + H2 X Ví dụ 2: Viết PTPƯ xảy ra khi hòa tan Na vào dung dịch Cu(NO3)2. 79 2Na + 2H2O 2NaOH + H2 NaOH + Cu(NO3)2 Cu(OH)2 + 2NaNO3 - Tác dụng với dung dịch bazo: Cho kim loại nhóm IA, IIA (trừ Be, Mg) tác dụng với dung dịch bazo thì xảy ra phản ứng của kim loại với nước. 1.2.2. Một số ví dụ minh họa Ví dụ 1: Hỗn hợp X gồm Na, K, Ba hòa tan hết trong nước dư tạo dung dịch Y và 5,6 lít khí (ở đktc). Tính V ml dung dịch H2SO4 2M tối thiểu để trung hòa Y. ĐS: 125 ml Giải: Sơ đồ phản ứng: + +; 2+ Y (Na ; K Ba ; OH ) + H2 (Na, K, Ba) + H2Od­ mol 0,5 0,25 H+ + OH-  → H2O Y + H2SO4: ⇒ n H + = nOH − = 0,5(mol ) ⇒ n H 2 SO4 = 0,25(mol ) → V = 0,125 lít hay 125 ml → đáp án A Ví dụ 2: Cho m gam hỗn hợp X gốm Na và Al vào nước dư thu được dung dịch X, 5,376 lít H2 (đktc) và 3,51 gam chất rắn không tan. Nếu oxi hóa m gam X cần bao nhiêu lít khí Cl2 (đktc) ? ĐS: 9,744 lít Giải: Sơ đồ phản ứng: NaAlO2 + H2 + Ald­ 0,13 mol x 0,24 (Na, Al) + H2Od­  n Alpu = x(mol ) ⇒ ; n Napu = x(mol ) Áp dụng định luật bảo toàn electron: x + 3x = 0,24.2 ⇔ x = 0,12(mol ) Ví dụ 3. Hòa tan hoàn toàn 22,85 gam hỗn hợp kim loại gồm Na, Ba trong 200 ml dung dịch HCl 1M, sau phản ứng thu được 4,48 lít khí H2 (đktc) và dung dịch X. Cô cạn dung dịch X thu được m gam chất rắn. Giá trị của X là: ĐS: 33,35 gam nHCl = 0,2 mol; n H = 0,2(mol ) Giải: 2 Quá trình nhận electron: 2H+ + 2e mol 0,2 H2 0,1 H+ hÕt ; KL d­ vµ H2O ph¶n øng ⇒ H2O + 2e H2 + 2OH- mol 0,1 0,2 80 Vậy: dung dịch X gồm: (Ba2+; Na+; 0,2 mol OH-; 0,2 mol Cl-) và mX = mKL + mOH + mCl ⇒ Chọn A. − − 1.3. DUNG DỊCH AXIT MẠNH TÁC DỤNG VỚI DUNG DỊCH BAZO MẠNH 1.3.1. Đặc điểm của bài toán và phương pháp làm bài - Khi pha trộn hỗn hợp X(nhiều dung dịch bazơ) với hỗn hợp Y(nhiều dung dịch acid) chúng thường có chung phương trình ion thu gọn: OH- + H+ → H2O Khi đó, chỉ cần chú ý đền ion OH- trong hỗn hợp X và ion H+ trong hỗn hợp Y: - Ta luôn có :[ H+][ OH-] = 10-14 và [ H+]=10-a ⇔ pH= a hay pH=-log[H+] + Tính khối lượng muối thu được sau trung hòa: mmuối = mkl + mgốc axit. → BaSO4 ↓ + 2H2O, tùy vào từng trường hợp cụ - Lưu ý, phản ứng: Ba(OH)2 + H2SO4  thể nên tách ra thành 2 phương trình ion riêng biệt cho đơn giản:  H + + OH −  → H 2 O  2+ 2− → BaSO4  Ba + SO4  1.3.2. Một số ví dụ minh họa Ví dụ 1: Cho 200 ml dung dịch A chứa HCl 0,15M và H2SO4 0,05M trung hoà hết bao nhiêu ml dung dịch bazơ B chứa NaOH 0,2 M và Ba(OH)2 0,1 M ? Giải: Gọi V (lít) là thể tích dung dịch B cần dùng. Ta có: ∑n H + ( A) = 0,05(mol ); ∑ nOH − ( B ) = 0,4V (mol ); Phản ứng: H+ + OH⇒ ∑n H + ( A)  → H2O = ∑ nOH − ( B ) ⇔ VB = 0,125 (lit) hay 125 (ml) Ví dụ 2: Cho 200 ml dung dịch A chứa HCl 1 (M) và HNO3 2(M) tác dụng với 300 ml dung dịch B chứa NaOH 0,8 (M) và KOH (chưa rõ nồng độ) thu được dung dịch C. Biết rằng để trung hoà 100 ml dung dịch C cần 60 ml dung dịch HCl 1 M, tính : a. Nồng độ ban đầu của KOH trong dung dịch B. b. Khối lượng chất rắn thu được khi cô cạn toàn bộ dung dịch C. Giải: Ta có sơ đồ sau: ∑ n + = 0,6(mol ) ∑ nOH − = (0,8 + x).0,3(mol ) H   ddA nCl − = 0,2(mol ) + ddB  n K + = 0,3 x(mol )  → dd C + HCl (đủ)  n − = 0,4(mol )  n Na + = 0,24( mol ) 0,06 (mol)   NO3 ⇒ Dung dịch C chứa OH- dư; H+ phản ứng hết. mol Vậy: nOH − d (C ) H+ + OH0,6 0,6 H2O = (0,8 + x).0,3 – 0,6 = 0,06 (mol) ⇔ x = 1,4 M 81 + + Do ®ã: dd C(K , Na , Cl , NO3 , OH ) mol 0,42 0,24 0,2 0,4 0,06 VËy: ∑m CR = ∑ mcation + ∑ manion = 54,82 gam Ví dụ 3: a. Cho dung dịch NaOH có pH = 13 (dung dịch A). Để trung hoà 10 ml dung dịch A cần 10 ml dung dịch B chứa 2 axit HCl và H2SO4. Xác định pH của dung dịch B ? b. Trộn 100 ml dd A với 100 ml dung dịch Ba(OH)2 x (M), thu được dung dịch C. Để trung hoà dung dịch 500 ml dung dịch C cần 350 ml dung dịch B. Xác định x a. pHB = 1; b. x = 2.10-2 (M) Ví dụ 4. Trộn 100 ml dung dịch gồm Ba(OH)2 0,1M và NaOH 0,1M với 400 ml dung dịch gồm H2SO4 0,0375M và HCl 0,0125M, thu được dung dịch X. Giá trị pH của dung dịch X là ĐS: pH = 2. 1.4. BÀI TẬP VỀ NHÀ 1.4.1. Bài tập lý thuyết o Câu 1: Trong các hang động của vùng núi đá vôi có phản ứng:Ca(HCO 3)2 t → CaCO3 + H2O + CO2. Tìm phát biểu đúng A. Phản ứng này giải thích sự tạo thành các dòng suối trong hang động B. Phản ứng này giải thích sự thành thạch nhũ ở hang động@ C. Phản ứng này giải thích sự xâm thực của nước mưa đối với đá vôi D. Tất cả đều sai Câu 2: Trong các kim loại sau: Na, Mg, Al, Fe. Kim loại có tính khử mạnh nhất là: A. Na B. Mg C. Al D. Fe Câu 3: Cho dung dịch KHSO4 vào lượng dư dung dịch Ba(HCO3)2. A. Không hiện tượng gì vì không có phản ứng hóa học xảy ra B. Có sủi bọt khí CO2, tạo chất không tan BaSO4, phần dung dịch có K2SO4 và H2O C. Có sủi bọt khí, tạo chất không tan BaSO4, phần dung dịch có chứa KHCO3 và H2O@ D. Có tạo hai chất không tan BaSO4, BaCO3, phần dung dịch chứa KHCO3, H2O Câu 4: Hiện tượng xảy ra khi thả mẩu Na và dung dịch CuSO4 A. Xuất hiện ↓ Cu màu đỏ. B. Không hiện tượng. C. Có khí thoát ra và ↓ màu xanh D. Xuất hiện ↓ Cu màu đỏ và có khí thoát ra. Câu 5: Để bảo quản natri, người ta phải ngâm natri trong A. nước. B. rượu etylic. C. phenol lỏng. D. dầu hỏa. Câu 6: Trong công nghiệp để điều chế kim loại kiềm và kiềm thổ người ta dùng phương pháp: A. Dùng kim loại mạnh như Al để đẩy Kim loại kiềm, kiềm thổ ra khỏi muối của chúng 82 B. Dùng chất khử mạnh ( Al, C, CO, H2) để khử các oxi kim loại, kiềm, kiềm thổ. C. Điện phân dung dịch muối halogenua của kim loại kiềm, có màng ngăn. D. Điện phân muối halogenua nóng chảy. Câu 7: Để sát trùng, tẩy uế tạp xung quanh khu vực bị ô nhiễm, người ta thường rải lên đó những chất bột màu trắng có tính oxi hóa mạnh. Chất đó là: A. Ca(OH)2 B. CaO C. CaCO3 D. CaOCl2@ Câu 8: Kim loại nhóm IIA tác dụng với dung dịch HNO 3 loãng , theo phương trình hóa học sau: 4M + 10 HNO3 → 4 M(NO3)2 + NxOy + 5 H2O . Oxit nào phù hợp với công thức phân tử của NXOY A. N2O@ B. NO C. NO2 D. N2O4 Câu 9: Thông thường khi bị gãy xương tay, chân, … người ta phải bó bột lại vậy họ đã dùng hoá chất nào A. CaSO4 B. CaSO4.2H2O C. CaSO4.0,5H2O@ D. CaCO3 Câu 10: Phản ứng nào sau đây: Chứng minh nguồn gốc tạo thành thạch nhũ trong hang động A. Ca(OH)2 + CO2 → Ca(HCO3)2 B. Ca(HCO3)2 → CaCO3 + CO2 + H2O@ C. CaCO3 + CO2 + H2O → Ca(HCO3)2 D. Ca(OH)2 + CO2 → CaCO3 Câu 11: Có 4 dd trong 4 lọ mất nhãn là: Amoni Sunfat, Amoni Clorua, Nat tri Sunfat, Natri Hiđroxit. Nếu chỉ được phép dùng một thuốc thử để nhận biết 4 chất lỏng trên ta có thể dùng thuốc thử nào sau đây A. dung dòch AgNO3 B. dung dòch Ba(OH)2 @ C. dung dòch KOH D. dung dòch BaCl2 Câu 12: Đolomit là tên gọi của hỗn hợp nào sau đây: A. CaCO3. MgCl2 B. CaCO3. MgCO3@ C. MgCO3. CaCl2 D. MgCO3.Ca(HCO3)2 Câu 13: Chỉ dùng một hóa chất nào sau đây để nhận biết các kim loại Ba, Mg, Fe, Ag, Al trong các bình mất nhãn A. H2SO4 loãng@ B. HCl C. H2O D. NaOH Câu 14: Ion Na bị khử khi người ta thực hiện phản ứng: A. Điện phân NaOH nóng chảy B. Điện phân dung dịch NaOH C. Điện phân dung dịch NaCl D. K tác dụng với dung dịch NaCl + Câu 15: Ion Na thể hiện tính oxi hóa trong phản ứng nào: A. 2NaCl  dpnc → 2Na + Cl2 0 t C. 2 NaNO3 → 2NaNO2 + O2 B. NaCl + AgNO3  → NaNO3 + AgCl D. Na2O + H2O  → 2NaOH Câu 16: Phương pháp quan trọng để điều chế kim loại kiềm là 83 A. Điện phân nóng chảy muối halogen của kim loại kiềm. B. Điện phân dung dịch muối halogenua của kim loại kiềm, giữa hai cực có màng ngăn xốp. C. Điện phân dung dịch muối halogenua của kim loại kiềm, giữa hai cực không có màng ngăn xốp. D. Tất cả đều đúng. Câu 17: Cho dây Pt sạch nhúng vào hợp chất của natri rồi đốt trên ngọn lửa đèn cồn, ngọn lửa có màu A. vàng. B. xanh. C. tím. D. đỏ. Câu 18: Nhận biết hợp chất của natri bằng phương pháp A. thử màu ngọn lửa. B. tạo ra chất kết tủa. C. tạo ra bọt khí. D. sự thay đổi màu sắc của các chất. Câu 19: Một hỗn hợp rắn gồm: Canxi và Canxi Cacbua. Cho hỗn hợp này tác dụng vói nước dư nguời ta thu đuợc hỗn hợp khí gì A. H2 B. C2H2 và H2@ C. H2 và CH4 D. Khí H2 và C2H4 Câu 20: Cho Ba vào các dung dịch sau: HCl; H 2SO4 loãng; FeCl3 ; (NH4)2SO4; NaHCO3 . Số phản ứng tạo kết tủa A. 2 B. 3 C. 4@ D. 5 Câu 21: Cho Ba vào dung dịch H2SO4 loãng thu dung dịch X. Dung dịch X có khả năng làm quỳ tím hóa xanh. Do trong X có. A. kết tủa BaSO4 B. BaSO4 và H2SO4 dư. C. Ba(OH)2 và H2SO4 dư D. Ba(OH)2 @ Câu 22: Kim loại kiềm thổ (trừ Be) tác dụng được với: A. Cl2 , Ar ,CuSO4 , NaOH B. H2SO4 , CuCl2 , CCl4 , Br2 C. Halogen, H2O , H2SO4 , O2 , Axit@ D. Kiềm , muối , oxit và kim loại Câu 23: Không gặp kim loại kiềm thổ trong tự nhiên ở dạng tự do vì: Hãy chọn đáp án đúng A. Thành phần của chúng trong thiên nhiên rất nhỏ B. Đây là kim loại hoạt động hóa học rất mạnh@ C. Đây là những chất hút ẩm đặc biệt D. Đây là những kim loại điều chế bằng cách điện phân. Câu 24: Na cháy trong khí oxi khô tạo ra: A. Na2O B. Na2O2@ C. Na2S D. NaCl Câu 25: Các kim loại kiềm thường dễ bị oxi hóa trong không khí, nên để bảo quản kim loại kiềm ta cần cách li chúng với không khí bằng cách ngâm chúng trong: A. nước B. phenol lỏng C. rượu etylic D. dầu hỏa. Câu 26: Natrihiđroxit (NaOH) được điều chế bằng cách 84 A. Điện phân nóng chảy NaCl. B. Điện phân dung dịch NaCl có màng ngăn. C. Điện phân dung dịch NaCl không có màng ngăn. D. Tất cả đều đúng. Câu 27: Dung dịch NaOH tác dụng được với những chất trong dãy nào sau đây A. ZnCl2, Al(OH)3, AgNO3, Ag. B. HCl, NaHCO3, Mg, Al(OH)3. C. CO2, Al, HNO3, CuO. D. CuSO4, SO2, H2SO4, NaHCO3. Câu 28: Hấp thụ hoàn toàn 1 mol khí CO2 vào dung dịch có chứa 1,5 mol NaOH, thu được dung dịch X. Muối trong dung dịch X gồm A. Na2CO3. B. NaHCO3. C. Na2CO3 và NaHCO3. D. Na2CO3 và NaOH. Câu 29: Cho khí CO2 , dd MgCl2 lần lượt tác dụng với các dd : NaHCO 3 , Na2CO3 , NaOH . Số ptpư hóa học xảy ra là : A. 2 B. 3 C. 4 D. 5 Câu 30: Cấu hình electron của Ba2+ là A. [Ar] 4s24p6 B. [Kr] C. [Xe]@ D. [Kr] 4d105s25p6 Câu 31: Muối nào sau đây tan được trong nước A. Ca3(PO4)2 B. MgCO3 C. BaHPO4 D. Ca(H2PO4)2@ Câu 32: Trong phản ứng Mg tan trong dung dịch HNO3 thu muối magie ; amoni và nước. Thì số phân tử axit bị khử và tạo muối lần lượt là. A. 4 và 8 B. 1 và 9@ C. 2 và 10 D. 1 và 8 Câu 33: Cho sơ đồ chuyển hoá: CaCO3 → A → B → C → CaCO3. A, B, C là những chất nào sau đây: 1. Ca(OH)2 2. Ba(HCO3)2 3. KHCO3 4. K2CO3 5. CaCl2 6.CO2 A. 2, 3, 5 B. 1, 3, 4 C. 2, 3, 6 D. 6, 2, 4@ Câu 34: Cho sơ đồ chuyển hóa sau: A MgCl2 C MgCl2 B E MgCl2 D MgCl2 F . Thứ tự các chất (A., (B., (C., (D., (E) ,(F) lần lượt là A. Mg, MgO, MgSO4, Cl2, HCl, BaCl2 B. Mg, Cl2, HCl, MgO, MgSO4, BaCl2 C. Mg, Cl2, MgO, HCl, MgSO4, BaCl2 @ D. Mg, Cl2, MgO, HCl, BaCl2, MgSO4 Câu 35: Cho sơ đồ chuyển hóa sau: 85 A CaCO3 C CaCO3 B E CaCO3 D CaCO3 F . Thứ tự các chất (A., (C., (E) lần lượt là A. CaO, Ca(OH)2, CaCl2 B. CO2, KHCO3, K2CO3 C. CO2, NaHCO3, Na2CO3 D. Cả A, B, C @ Câu 36: Có 4 chất rắn: Na2CO3, Na2SO4, CaCO3, CaSO42H2O. Để phân biệt được 4 chất rắn trên chỉ dùng A. Nước và dung dịch NaOH B. Nước và dung dịch NH3 C. Nước và dung dịch HCl@ D. Nước và dung dịch BaCl2 Câu 37: Cho các dung dịch muối : NaHCO3, NaCl, Na2CO3, CaCl2, Ba(NO3)2. Dung dịch muối àm quỳ tím hoá xanh A. NaHCO3. B. CaCl2 C. Na2CO3; Ba(NO3)2 D. NaHCO3 ; Na2CO3 @ Câu 38: Sục khí CO2 dư vào dd Ca(OH)2 sẽ có hiện tượng gì xảy ra A. Có kết tủa trắng B. Có kết tủa sau đó kết tủa tan dần@ C. Dung dịch vẫn trong suốt D. Có kết tủa xanh lam Câu 39: Cho Sr vào dung dịch HNO3 không thấy khí thoát ra. Tìm phát biểu đúng A. Phương trình phản ứng: Sr + 2 HNO3 → Sr(NO3)2 + H2 B. Phương trình phản ứng: 4Sr + 10 HNO3 → 4Sr (NO3)2 + NO2 + 5 H2O C. Phương trình phản ứng: 4 Sr + 10 HNO3 → 4 Sr(NO3)2 + NH4NO3 + 3H2O@ D. Sr bị thụ động với HNO3 nên không xảy ra phản ứng o Câu 40: Cho phản ứng nhiệt phân 4 M(NO3)x t → 2 M2Ox + 4xNO2 + xO2. M là kim loại nào sau đây A. Na B. K C. Mg@ D. Ag Câu 41: Cho 3 dd NaOH, HCl, H2SO4. Thuốc thử duy nhất để phân biệt 3 dd là: A. CaCO3 @ B. Na2CO3 C. Al D. quỳ tím Câu 42: Có 4 dung dịch trong suốt, mỗi dung dịch chỉ chứa một loại cation và một loại anion. Các loại Ion trong cả 4 dd gồm: Ba 2+ , Mg 2+ , Pb 2+ , Na + , SO42− , Cl − , NO3− , CO32- . Đó là dung dịch gì A. BaCl2, MgSO4, Na2CO3, Pb(NO3)2@ B. BaCO3, MgSO4, NaCl, Pb(NO3)2 C. BaCl2, Mg(NO3)2, Na2CO3, PbSO4 D. BaSO4, MgCl2, Na2CO3, Pb(NO3)2 − Câu 43: Trong cốc nước chứa a mol Ca 2+ , b mol Mg 2+ , c mol Cl − và d mol HCO3 . Biểu thức liên hệ giữa a, b, c, d A. a + b = c + d B. 3a + 3b = c + d C. 2a + 2b = c + d@ D. 2a + 2b + d = c 86 Câu 44: Canxi có trong thành phần của các khoáng chất : Canxit, thạch cao, florit. Công thức của các khoáng chất tương ứng là A. CaCO3, CaSO4, Ca3(PO4)2 B. CaCO3, CaSO4.2H2O, CaF2@ C. CaSO4, CaCO3, Ca3(PO4)2 D. CaCl2, Ca(HCO3)2, CaSO4 Câu 45: Trong các phát biểu sau đây về độ cứng của nước: Chọn phát biểu đúng.1. Đun sôi nước ta chỉ loại được độ cứng tạm thời.2. Có thể dùng Na 2CO3 để loại cả 2 độ cứng tạm thời và độ cứng vĩnh cửu.3. Có thể dùng HCl để loại độ cứng của nước.4. Có thể dùng Ca(OH)2 với lượng vừa đủ để loại độ cứng của nước. `A. 2 B. 1, 2, 4 C. 1,2@ D. 4 1.4.2. Bài tập tính toán Câu 1. Hoà tan hỗn hợp A gồm 13,7g Ba và 5,4g Al vào một lượng nước có dư thì thể tích khí thoát ra ở điều kiện tiêu chuẩn là: A. 6,72 lít B. 4,48 lít C. 13,44 lít D. 8,96 lít@ Câu 2. Cho hỗn hợp chứa 0,5 mol Ba và x mol Al vào dung dịch chứa 0,5 mol HCl và 0,5 mol H2SO4 (loãng). Sau khi các phản ứng xảy ra hoàn toàn thu được 13,44 lít khí H 2 (ở đktc). Phát biểu đúng là A. Cả hai axit hết, kim loại còn dư. B. Các kim loại tan hết, cả hai axit còn dư.@ C. Các kim loại hết, chỉ axit HCl còn dư. D. Các kim loại hết, chỉ axit H2SO4 còn dư. Câu 3. Trộn lẫn V ml dung dịch (gồm NaOH và Ba(OH)2 ) có pH=12 với V ml dung dịch gồm HCl 0,02 M và H2SO4 0,005M được 2V ml dung dịch Y. Dung dịch Y có pH là A. 2.@ B. 4. C. 1. D. 3. Câu 4. Dung dịch X có chứa a mol (NH 4)2CO3 . Thêm a mol Ba kim loại vào X và đun nóng dung dịch, sau khi phản ứng xảy ra hoàn toàn thu đợc dung dịch A. có Ba2+, OH−. B. có NH4+, OH−. C. chỉ có H2O.@ D. có NH4+, CO32−. Câu 5. Chia 39,9 gam hỗn hợp gồm Na, Al, Fe thành 3 phần bằng nhau. Phần 1: Tác dụng với H2O dư, giải phóng 4,48 lít khí H2. Phần 2: Tác dụng với dung dịch NaOH dư, giải phóng 7,84 lít khí H2. Phần 3: Tác dụng với dung dịch HCl dư, giải phóng V lít khí H2. Biết rằng các khí đo ở đktc. Giá trị của V là: A. 7,84 lít B. 12,32 lít C. 10,08 lít@ D. 13,44 lít Câu 6: Một hỗn hợp gồm Na, Al, Fe (với tỷ lệ mol Na và Al tương ứng là 5:4) tác dụng với nước dư thu được V lít khí, dung dịch Y và chất rắn Z. Cho Z tác dụng với dung dịch H2SO4 loãng dư thì thu được 0,25V lít khí (các khí đo cùng điều kiện). Thành phần % theo khối lượng của Na trong hỗn hợp X là A. 14,4 B. 33,63@ C. 20,07 D. 34,8 87 Câu 7. Hỗn hợp X gồm Ba và Al. Cho m gam X vào nước dư, sau khi các phản ứng xảy ra hoàn toàn, thu được 8,96 lít khí H 2 (đktc). Mặt khác, hòa tan hoàn toàn m gam X bằng dung dịch NaOH, thu được 15,68 lít khí H2 (đktc). Giá trị của m là A. 24,5@ B. 29,9 C. 19,1 D. 16,4 Câu 8. Cho 8,5 gam hỗn hợp X gồm Na và K vào 100 ml dung dịch Y gồm H 2SO4 0,5M vaf HCl 1,5M. Sau khi kết thúc phản ứng thu được 3,36 lít khí (đktc) và dung dịch Z. Cô cạn dung dịch Z thu được m gam chất rắn khan. Giá trị của m là: A. 19,475g@ B. 20,175g C. 18,625g D. 17,975g Câu 9. Cho 7,8 gam hỗn hợp Mg và Al tác dụng hết với dung dịch HCl dư. Sau phản ứng thấy khối lượng dung dịch tăng thêm 7 gam. Số mol HCl đã tham phản ứng là A. 0,8 mol. @ B. 0,7 mol. C. 0,6 mol. D. 0,5 mol. Câu 10: Trộn 500 ml dung dịch A chứa HNO3 0,4M và HCl 0,2M với 100 ml dung dịch B chứa NaOH 1M và Ba(OH)2 0,5M thì dung dịch C thu được có tính gì? A. Axit@ B. Bazơ C. Trung tính D. không xác định được Câu 11: Cho 200 ml dung dịch A chứa HCl 1M và HNO3 2M tác dụng với 300 ml dung dịch chứa NaOH 0,8M và KOH (chưa biết nồng độ) thì thu được dung dịch C. Biết rằng để trung hòa dung dịch C cần 60 ml HCl 1M. Nồng độ KOH là? A. 0,7M B. 0,5M C. 1,4M @ D. 1,6M Câu 12: 100 ml dung dịch X chứa H2SO4 2 M và HCl 2M trung hòa vừa đủ bởi 100ml dung dịch Y gồm 2 bazơ NaOH và Ba(OH)2 tạo ra 23,3 gam kết tủa. Nồng độ mol mỗi bazơ trong Y là? A. [NaOH]=0,4M;[Ba(OH)2]=1M B. [NaOH]=4M;[Ba(OH)2]=0,1M C. [NaOH]=0,4M;[Ba(OH)2]=0,1M D. [NaOH]=4M;[Ba(OH)2]=1M@ Câu 13: Trộn 100 ml dung dịch gồm Ba(OH)2 0,1M và NaOH 0,1M với 400 ml dung dịch gồm H2SO4 0,0375M và HCl 0,0125M thu được dung dịch X. Gía trị pH của dung dịch X là? A. 2@ B. 1 C. 6 D. 7 Câu 14: Hỗn hợp A gồm 2 kim loaị kiềm M và M’ nằm ở 2 chu kỳ kế tiếp nhau.Lấy 3,1g A hoà tan hết vào nước thu được 1,12 lít hidro (đktc). M và M’ là 2 kim loại nào: A. Li, Na B. Na, K@ C. K, Rb D. Rb, Cs Câu 15: Cho 3,9 g kali vào 101,8 g nước thu được dung dịch KOH có khối lượng riêng là 1,056 g/ml. Nồng độ % của dung dịch KOH là bao nhiêu ? A. 5,31% B. 5,20% C. 5,30% @ D. 5,50% Câu 16. Trộn 100 ml dung dịch hỗn hợp gồm H2SO4 0,05M và HCl 0,1M với 100 ml dung dịch hỗn hợp gồm NaOH 0,2M và Ba(OH)2 0,1M, thu được dung dịch X. Dung dịch X có pH là 88 A. 12,8. B. 13,0.@ C. 1,0. D. 1,2. Câu 17. Hoà tan hoàn toàn 1,36 gam hỗn hợp 2 kim loại kiềm kế tiếp nhau trong nhóm IA vào nước được 0,56 lít khí H2 (đktc). 2 kim loại đó là A. Li, Na. B. Na, K.@ C. K, Rb. D. Rb, Cs. Câu 18. Dung dịch X chứa hỗn hợp NaOH a mol/l và Ba(OH)2 b mol/l. Để trung hoà 50 ml dung dịch X cần 60 ml dung dịch HCl 0,1M. Mặt khác, cho lượng dư dung dịch Na2CO3 vào 100 ml dung dịch X được 0,394 gam kết tủa. Giá trị của a, b lần lượt là A. 0,1; 0,01. B. 0,1; 0,08. C. 0,08; 0,01. D. 0,08; 0,02.@ Câu 19: Cho 5 gam hỗn hợp Na, Na2O và tạp chất trơ tác dụng với H2O được 1,875 lít khí (đktc). Trung hoà dung dịch sau phản ứng cần 100 ml dung dịch HCl 2M. Phần trăm tạp chất trơ là A. 2%. B. 2,8%. @ C. 5,6%. D. 1,1%. Câu 20: Cho 10 gam một kim loại kiềm thổ tác dụng với nước, thu được 6,11 lít khí H2 (25oC và 1 atm). Kim loại kiềm thổ đó là A. Sr. B. Ca.@ C. Mg. D. Ba. Câu 21: 1 lít dung dịch hỗn hợp X gồm HCl và H2SO4 loãng được trung hoà bằng dung dịch 0,4 mol NaOH. Nếu cho 1 lít dung dịch hỗn hợp X tác dụng hết với Mg thì số mol H2 sinh ra là A. 0,4. B. 0,3. C. 0,2. @ D. 0,1. Câu 22: Cho m gam hỗn hợp Mg và Al vào 250ml dung dịch X chứa hỗn hợp acid HCl 1M và acid H2SO4 0,5M thu được 5,32 lít H2 (đktc) và dung dịch Y (coi thể tích dung dịch không đổi). Dung dich Y có pH là? A. 7 B. 1@ C. 2 D. 6 Câu 23. Cho hỗn hợp các kim loại kiềm Na, K hòa tan hết vào nước được dung dịch A và 0,672 lít khí H2 (đktc). Thể tích dung dịch HCl 0,1M cần để trung hòa hết một phần ba dung dịch A là A. 100 ml. B. 200 ml. C. 300 ml. D. 600 ml.@ Câu 24. Cho 6,08 gam hỗn hợp NaOH và KOH tác dụng hết với dung dịch HCl tạo ra 8,30 gam hỗn hợp muối clorua. Số gam mỗi hidroxit trong hỗn hợp lần lượt là: A. 2,4 gam và 3,68 gam. B. 1,6 gam và 4,48 gam. @ C. 3,2 gam và 2,88 gam. D. 0,8 gam và 5,28 gam. Câu 25. Điện phân muối clorua kim loại kiềm nóng chảy thu được 1,792 lít khí (đktc) ở anot và 6,24 gam kim loại ở catot. Công thức hoá học của muối đem điện phân là A. LiCl. B. NaCl. C. KCl. @ D. RbCl. 89 2. BÀI TOÁN SỤC KHÍ CO2, SO2 VÀO DUNG DỊCH KIỀM. PHẢN ỨNG CỦA MUỐI CACBONAT VỚI DUNG DỊCH AXIT 2.1. BÀI TOÁN SỤC KHÍ CO2, SO2 VÀO DUNG DỊCH KIỀM 2.1.1. Đặc điểm của bài toán xuôi và phương pháp làm bài 2.1.1.1 Đặc điểm của bài toán: cho thể tích khí, cho số mol OH-, yêu cầu tính khối lượng muối. 2.1.1.2. Phương pháp làm bài - Khí XO2 (CO2, SO2) khi được sục vào dung dịch kiềm thì xảy ra phương trình phản ứng: − XO2 + OH-  → HXO 3 2− XO2 + 2OH-  → XO 3 + H2O Đặt k = nOH − n XO2 1 2 k thµnh phÇn dd sau p­ XO2d HXO3 - HXO3, - HXO3 2- XO3 2- XO3 2- XO3 OH - Với loại bài tập sục khí CO2/SO2 vào dung dịch kiềm/hỗn hợp dung dịch kiềm, yêu cầu tính khối lượng kết tủa thì thứ tự làm như sau: + Tính ∑n OH − , và lập tỉ số k để biết sinh ra sản phẩm gì. + Viết phương trình phản ứng dưới dạng ion thu gọn/sơ đồ phản ứng + So sánh số mol CO32 − ( SO32 − ) và số mol Ca2+/Ba2+ để tính khối lượng kết tủa thu được. - Áp dụng định luật bảo toàn nguyên tố, định luật bảo toàn khối lượng và định luật bảo toàn điện tích để giải 2− − - Trường hợp sinh ra hỗn hợp 2 muối ( CO3 , HCO3 ) (ví dụ với trường hợp sục khí CO2 vào dung dịch kiềm): − CO2 + OH-  → HCO 3 x x x 2− CO2 + 2OH-  → CO 3 + H2O y 2y y Dễ dàng chứng minh được công thức: nCO = ∑ nOH − nCO 2− 3 − 2 2.1.1.3. Các ví dụ minh họa Ví dụ 1: Hấp thụ hoàn toàn 2,24 lít CO2 (đktc) vào dung dịch nước vôi trong có chứa 0,075 mol Ca(OH)2. Tính khối lượng muối thu được. 90 Giải: Có k = 1,5 ⇒ Sản phẩm gồm HCO3− ,CO32− CO32- a mol   HCO3- b mol  CO2 + OHmol 0,1 0,15 Áp dụng định luật bảo toàn nguyên tố đối với C, định luật bảo toàn điện tích, ta có hpt:  a + b = 0,01 a = 0,05mol ⇔ ⇒ (CaCO3, Ca(HCO3)2) ⇒ mmuối = 9,05g  0,025 2a + b = 0,15 b = 0,05mol 0,05 mol mol − 2− hoặc: Có k = 1,5 ⇒ Sản phẩm gồm HCO3 ,CO3 ⇒ nCO32 − = ∑ nOH − − nCO2 = 0,15 – 0,05 mol ⇒ (CaCO3, Ca(HCO3)2) ⇒ mmuối = 9,05g 0,05 mol 0,025 Ví mol dụ 2. : Sục 4,48 lít (đktc) CO2 vào 100ml hỗn hợp dung dịch gồm KOH 1M và Ba(OH)2 0,75M. Sau khi khí bị hấp thụ hoàn toàn thấy tạo m g kết tủa. Tính m A. 23,64g B. 14,775g C. 9,85g @ D. 16,745g Ví dụ 3: Hòa tan một mẫu hợp kim K-Ba có số mol bằng nhau vào nước dư, được dung dịch X và 6,72 lít khí (đktc). Sục 0,025 mol CO 2 vào dung dịch X thu được m gam kết tủa. Tính m. Giải: Sơ đồ phản ứng: (K, Ba) + H2O d­ mol x x dd (K+, Ba2+, OH-) + H2 x 0,6 x 0,3  + 0,45 mol CO2 m gam - Áp dụng định luật bảo toàn electron: x + 2x = 0,3.2 ⇔ x = 0,2 mol. Mặt khác, ta có: k = nOH − nCO2 = 0,6 = 1,333 ⇒ SP ( CO32 − ; HCO3− ). 0,45 Khi đó, nCO = ∑ nOH − nCO = 0,6 – 0,45 = 0,15 mol 2− 3 − 2 Ba2+ + CO320,15 mol 0,2 BaCO3 0,15 Vậy, khối lượng kết tủa thu được là: m↓ = 0,15.197 = 29,55 gam Chú ý quan trọng: + Khi bài toán cho cả 2 oxit CO2 và SO2 thì gọi công thức chung của 2 oxit là XO2 để tính toán. 91 + Nếu CO2 tác dụng với dung dịch hỗn hợp gồm OH - và CO32 − thì có thể có các phản ứng sau: − CO2 + OH-  → HCO 3 2− CO2 + 2OH-  → CO 3 + H2O 2− − CO2 + CO 3 + H2O  → 2HCO 3 do đó, khối lượng muối CO 32− sau phản ứng sẽ thay đổi. Khi đó nên: 2− - Quy đổi: CO3 → 2OH- + CO2 - Tính ∑n rồi đưa về dạng cơ bản. OH − Ví dụ 4. Cho 28 gam hỗn hợp X gồm CO2 và SO2 (có tỷ khối so với oxi là 1,75) lội chậm qua 500 ml dung dịch hỗn hợp gồm NaOH 0,7M và Ba(OH)2 0,4M. Sau khi các phản ứng kết thúc thì thu được m gam chất rắn. Tính m (ĐS: 41,80 gam) Giải: Gọi công thức chung của hai oxit là: XO2. Ta có: M XO = 1,75.32 = 56 ⇔ X + 32 = 56 ⇔ X = 24 2 Mặt khác : n XO = 2 28 = 0,5(mol ) 56 Sơ đồ phản ứng: XO2 + dd (Na+, Ba2+, OH-) mol 0,5 Mặt khác, ta có: k = nOH − n XO2 = 0,35 0,2 m gam 0,75 0,75 = 1,5 ⇒ SP ( XO32− ; HXO3− ). 0,5 Khi đó, n XO = ∑ nOH − n XO = 0,75 – 0,5 = 0,25 mol 2− 3 − 2 Ba2+ + XO320,25 mol 0,2 BaXO3 0,2 Vậy, khối lượng kết tủa thu được là: m↓ = 0,2.206 = 41,80 gamVí dụ 4: Sục 4,48 lít CO2 (đktc) vào 200 ml dung dịch chứa Na2CO3 0,5M và NaOH 0,75M thu dung dịch X. Cho BaCl2 dư vào dung dịch X. Tính khối lượng kết tủa thu được? n Na2CO3 = 0,1(mol ) ; nNaOH = 0,15 mol; nCO2 = 0,2(mol ) Giải: Quy ®æi: Na2CO3 mol 0,1 2NaOH + CO2 0,2 0,1 Khi đó, bài toán trở thành: 92 CO2 + dd (NaOH) mol 0,3 Xét: k = nOH − nCO2 = dd BaCl2 d­ dd X m gam 0,35 0,35 = 1,17 ⇒ SP ( CO32 − ; HCO3− ). 0,3 Khi đó, nCO = ∑ nOH − nCO = 0,35 – 0,3= 0,05 mol 2− 3 − 2 Ba2+d­ + CO320,05 mol BaCO3 0,05 Vậy, khối lượng kết tủa thu được là: m↓ = 0,05.197 = 9,85 gam Ví dụ 5. Sục V lít CO2 (đktc) vào 300 ml dung dịch chứa NaOH 0,5M và Na 2CO3 0,4M thu được dung dịch X chứa 29,97 gam hỗn hợp muối. Tính V. (ĐS: 5,04 lít) 2.1.2. Đặc điểm của bài toán nghịch và phương pháp làm bài 2.1.2.1. Đặc điểm của bài toán nghịch Cho khối lượng muối (khối lượng kết tủa) và số mol OH - yêu cầu tìm thể tích khí hoặc cho khối lượng muối và số mol khí, tìm số mol OH-. Với loại bài tập này, nên vẽ đồ thị ra để dễ hình dung. 2.1.2.2. Phương pháp làm bài Ví dụ: Sục khí CO2 vào dung dịch Ba(OH)2. Mô tả hiện tượng, biện luận số mol kết tủa theo số mol Ba(OH)2. → BaCO3 ↓ + H2O (1) Ba(OH)2 + CO2  → Ba(HCO3)2 (2) BaCO3 ↓ + CO2 + H2O  n 2 A n c® I1 O - §o¹n th¼ng OA: øng víi ph¶n øng (1), ®o¹n AB øng víi ph¶n øng (2). - Sè mol kÕt tña cùc ®¹i: n ↓ c® = n Ba (OH ) + NÕu n ↓ (gthiÕt) < n ↓ c® th× ®êng th¼ng y = n ↓ c¾t ®å thÞ t¹i hai ®iÓm I1, I2 øng víi 2 trêng hîp riªng biÖt. * TH1: Ba(OH)2 dư, đường thẳng y = n ↓ c¾t ®å thÞ t¹i hai ®iÓm I1: nCO = n↓ * TH2: CO2 dư sau (1), kết tủa bị hoàn tan một nOHphần theo (2). Đường thẳng y = n ↓ c¾t ®å thÞ 2− − t¹i hai ®iÓm I2; sản phẩm gồm ( CO3 ; HCO3 ). I2 y=n B 2 Khi đó: nCO = ∑ nOH − nCO 2.1.2.3. Các ví dụ minh họa Ví dụ 1: Hấp thụ hoàn toàn V lít CO2 (đktc) vào 2,5 lít dung dịch Ba(OH)2 nồng độ 0,2M, thu được 15,76 gam kết tủa. Gía trị của V là? Đs: V = 1,792 lít hoặc V = 20,608 lít Giải: 2− 3 − 2 93 n Ba ( OH ) 2 = 0,5(mol ) > n ↓= 0,08(mol ) ⇒ Xảy ra 2 trường hợp. * TH1. Ba(OH)2 dư, đường thẳng y = n ↓ c¾t ®å thÞ t¹i hai ®iÓm I1 nCO2 = n↓ = 0,08 (mol) ⇒ VCO2 ( đktc ) = 1,792(lít ) * TH2: CO2 dư sau (1), kết tủa bị hoàn tan một phần theo (2). Đường thẳng y = n ↓ c¾t ®å 2− − thÞ t¹i hai ®iÓm I2; sản phẩm gồm ( CO3 ; HCO3 ). Khi đó, sử dụng nCO = ∑ nOH − nCO 2− 3 ⇔ nCO2 = ∑ nOH − − nCO32 − = ∑n OH − − 2 − n↓ = 0,5.2 – 0,08 = 0,92 (mol) ⇒ VCO2 ( đktc ) = 20,608(lít ) Ví dụ 2. Cho V lít CO2 (đktc) hấp thụ hoàn toàn vào 200 ml dung dịch hỗn hợp KOH 1M và Ba(OH)2 0,75M thu được 27,58 gam kết tủa. Giá trị lớn nhất của V là ? ĐS: 8,064 lít Giải: n Ba ( OH ) 2 = 0,15(mol ) > n↓ = 0,14(mol ) ⇒ Xảy ra 2 trường hợp. Nhưng vì đề n KOH = 0,2(mol )  Theo giả thiết:  yêu cầu tính giá trị lớn nhất của V nên chỉ xét trường hợp 2 (cắt tại I2) Ta có: nCO = ∑ nOH − nCO 2− 3 ⇔ nCO2 = ∑ nOH − − nCO32 − = ∑n OH − − 2 − n↓ = 0,5 – 0,14= 0,36(mol) ⇒ VCO2 ( đktc ) = 8,064(lít ) Ví dụ 3. Trong một bình kín chứa 0,02 mol Ba(OH) 2. Sục vào bình a mol CO2. Tính khoảng biến thiên của kết tủa thu được nếu 0,005 < a < 0,024 mol. ĐS: 0,985 gam < m↓ < 3,94g 2.1.3. Bài toán liên quan tới sự tăng, giảm khối lượng của dung dịch 2.1.3.1. Đặc điểm của bài toán và phương pháp làm bài * Đặc điểm của bài toán - Khi sục khí CO2 hoặc SO2 vào dung dịch bazo: ví dụ Ba(OH) 2 thì dù sinh ra muối nào thì khối lượng dung dịch trước và sau phản ứng cũng có sự thay đổi. → BaCO3 ↓ + H2O (1) Ba(OH)2 + CO2  Ba(OH)2 + CO2 + H2O  → Ba(HCO3)2 (2) * Phương pháp làm bài Áp dụng định luật bảo toàn khối lượng, có: mdd trước pư + mCO = mdd sau pư + m↓ 2 ⇔ ∆mdd = mdd trước pư - mdd sau pư = m↓ - mCO2 = a gam + Nếu ∆mdd > 0 ⇒ Khối lượng dung dịch sau phản ứng giảm a gam. 94 + Nếu ∆mdd < 0 ⇒ Khối lượng dung dịch sau phản ứng tăng a gam. Chú ý: Nếu đề hỏi, khối lượng bình đựng dung dịch nước vôi trong thay đổi như thế nào sau phản ứng thì: Khối lượng bình sau phản ứng luôn tăng; ∆mb↑ = mdd trước pư + mCO 2 2.1.3.2. Ví dụ minh họa Giải lại ví dụ 1 mục 2.1.2.3 Ví dụ: Hấp thụ hoàn toàn V lít CO2 (đktc) vào 2,5 lít dung dịch Ba(OH) 2 nồng độ 0,2M, thu được 15,76 gam kết tủa. Khối lượng dung dịch sau phản ứng thay đổi như thế nào ? * Trường hợp 1: V = 1,792 lít ∆mdd = mdd trước pư - mdd sau pư = m↓ - mCO2 = 15,76 – 0,08.44 = 12,24 gam > 0 Vậy, sau phản ứng khối lượng dung dịch giảm 12,24 gam. * Trường hợp 2: V = 20,608 lít ∆mdd = mdd trước pư - mdd sau pư = m↓ - mCO2 = 15,76 – 0,92.44 = - 24,72 gam < 0 Vậy, sau phản ứng khối lượng dung dịch tăng 24,72 gam. 2.2. PHẢN ỨNG CỦA MUỐI CACBONAT VỚI DUNG DỊCH AXIT 2.2.1. Cho dung dịch axit vào dung dịch muối a. Cho từ từ CO 32− vào dung dịch H+ 2− Phản ứng: CO 3 b. Cho từ từ H+ vào dung dịch CO 32− 2− − + 2H+  → CO2 ↑ + CO 3 + H+  → HCO 3 (1) H2 O HCO 3− + H+  → CO2 ↑ + H2O (2) - Nếu sau TN có CO 2 ↑ thì pư (1) đã xảy ra xong. - Nếu sau TN không có CO2 ↑ thì pư (2) không xảy ra. 2.2.2. Cho dung dịch muối vào dung dịch axit Tổng quát: Cho dung dịch CO 32− (x mol), HCO 3− (y mol) tác dụng với dung dịch axit: 2− − 2− a. Cho từ từ CO 3 , HCO 3 vào dung dịch b. Cho từ từ H+ vào dung dịch CO 3 , H+ − HCO 3 Pư: CO 32− + 2H+  → CO2 ↑ + H2O (*) − HCO 3 + H+  → CO2 ↑ + H2O (**) − CO 32− + H+  → HCO 3 (3) x x x+y 95 Coi (*), (**) xảy ra đồng thời. HCO 3− + H+  → CO2 ↑ + H2O (4) n H + = 2 a + b  nCO32 − pu = a (mol )  ⇒ a x Đặt n = = b ( mol )  HCO3− pu  b y x+y 2− - Nếu CO 3 dư sau (3) thì pư (4) không xảy ra. - Nếu H+ dư thí (4) xảy ra hoàn toàn. 2.2.3. Các ví dụ minh họa Ví dụ 1: Có 2 cốc. Cốc 1 chứa Na 2CO3 (0,2 mol), NaHCO3 (0,3 mol); cốc 2 chứa dung dịch HCl (0,5 mol). a. Rót từ từ cốc 2 vào cốc 1. Xác định thành phần của dung dịch sau phản ứng và thể tích khí bay ra ở đktc (nếu có) b. Rót từ từ cốc 1 vào cốc 2. Xác định thành phần của dung dịch sau phản ứng và thể tích khí bay ra ở đktc (nếu có) ĐS: a. VCO = 6,72lít ; n HCO = 0,2mol ; − 3 2 b. VCO = 8lít ; nCO = 0,057 mol ; n HCO = 0,086mol 2− 3 2 2− 3 Giải: a. Rót từ từ cốc 2 vào cốc 1. Tính thể tích khí CO2(đktc) và nCO ; n HCO sau phản ứng: 2− 3 n H + = 0,5 (mol) n CO 2- = 0,2 (mol)  3  n HCO3- = 0,3 (mol)  − 3 Thứ tự phản ứng: − → HCO 3 (1) CO 32− + H+  mol 0,2 0,2 0,2 − + → CO2 ↑ + H2O (2) HCO 3 + H  mol 0,5 0,3 0,3 Vậy, sau phản ứng: VCO2 ( đktc ) = 6,72lít ; n HCO3− = 0,2mol ; nCO 2 − = 0 d 3 b. Rót từ từ cốc 1vào cốc 2. Tính thể tích khí CO2(đktc) và nCO ; n HCO sau phản ứng: 2− 3 − 3  nCO32 − pu = a (mol ) n CO 2- = 0,2 (mol)  a 0,2 2 3 ⇒ = = hay 3a – 2b =  Gọi  n HCO3 = 0,3 (mol) b 0,3 3 n HCO3− pu = b( mol )  n 2− H + = 0,5 (mol) − 0(*) Cho từ từ CO 32− , HCO 3− vào dung dịch H+, các phản ứng sau xảy ra đồng thời: Pư: CO 32− + 2H+  → CO2 ↑ + H2O (3) mol a 2a a − + HCO 3 + H  → CO2 ↑ + H2O (4) mol b b b Nếu CO 3 , HCO 3 phản ứng hết theo (3), (4) thì: 96 ∑n H+ = 0,2.2 + 0,3 = 0,7(mol ) > 0,5( mol ) Chứng tỏ H+ hết, muối dư. Từ (3), (4) có: ∑n H+ = 2a + b = 0,5(mol ) (**)  3a − 2b = 0  a = 0,14286(mol ) ⇔ b = 0,21428(mol ) 2a + b = 0,5 Từ (*) và (**) có:  VCO2 ( đktc ) = 8lít ; nCO 2 − d = 0,2 − a = 0,057 mol ; n HCO 2 − d = 0,3 − b = 0,086mol 3 3 Nhận xét: Tùy vào cách tiến hành thí nghiệm khác nhau thì thể tích khí CO2 thu được và thành phần dung dịch sau phản ứng là khác nhau. Ví dụ 2: Cho từ từ 100ml dung dịch hỗn hợp Na2CO3 1M với KHCO3 aM vào 200ml dung dịch HCl 1M, sau phản ứng thu được 2,688 lít CO2 (ở đktc). Giá trị của a là ĐS: 0,5M Ví dụ 3. Cho rất từ từ dd chứa x mol HCl vào dd chứa y mol NaCO3 thu được dung dịch A. Cho A tác dụng với dung dịch BaCl2 dư thu được 1,97 gam kết tủa. Cho A tác dụng với dung dịch Ba(OH)2 dư thì thu được 3,94 gam kết tủa. Tính số mol các ion trong dung dich A (cho rằng không có sự thủy phân xảy ra), từ đó tìm x, y. ĐS: x = 0,01 mol; y = 0,02 mol. Ví dụ 4: (TSĐH – Khối A- 2009). Dung dịch X chứa hỗn hợp gồm Na2CO3 1,5M và KHCO3 1M. Nhỏ từ từ từng giọt cho đến hết 200 ml dung dịch HCl 1M vào 100 ml dung dịch X, sinh ra V lít khí (ở đktc). Giá trị của V là ĐS: V = 1,12. Ví dụ 5. Cho từ từ 150 ml dd HCl 1M vào 500 ml dung dịch A gồm Na 2CO3 và NaHCO3 thì thu được 1,008 lít khí (đktc) và dung dịch B. Cho dung dịch B tác dụng với dung dịch Ba(OH) 2 dư thì thu được 29,55 gam kết tủa. Nồng độ mol của Na2CO3 và NaHCO3 trong dung dịch A lần lượt là: ĐS. 0,21M và 0,18M 2.3. BÀI TẬP VỀ NHÀ Câu 1. Hấp thụ 1,12 lít khí CO2 (đktc) vào 200 ml dung dịch chứa hỗn hợp NaOH 0,3M và K2CO3 0,2M được dung dịch X. Cho X tác dụng với dung dịch CaCl2 dư, đun nóng được khối lượng kết tủa là: A. 9,0g B. 11,0g C. 7,0g@ D. 8,0g Câu 2: (TSĐH- Khối A- 2007). Cho từ từ dung dịch chứa a mol HCl vào dung dịch chứa b mol Na2CO3 đồng thời khuấy đều, thu được V lít khí (ở đktc) và dung dịch X. Khi cho dư nước vôi trong vào dung dịch X thấy có xuất hiện kết tủa. Biểu thức liên hệ giữa V với a, b là: 97 A. V = 11,2(a - b). B. V = 22,4(a - b).@ C. V = 22,4(a + b). D. V = 11,2(a + b). Câu 3: (TSĐH – Khối A- 2010). Nhỏ từ từ từng giọt đến hết 30ml dung dịch HCl 1M vào 100ml dung dịch chứa Na2CO3 0,2M và NaHCO3 0,2M, sau phản ứng thu được số mol CO 2 là: A. 0,02 B.0,03 C.0,015 D.0,01@ Câu 4. Nhỏ từ từ từng giọt dung dịch chứa 0,05 mol HCl vào dung dịch chứa 0,06 mol Na2CO3. Thể tích khí CO2 thu được (đktc) thu được bằng: A. 0 lít @ B.0,56lít C.1,12lít D. 1,344lít Câu 5. Nhỏ từ từ 200ml dung dịch HCl vào 100ml dung dịch X chứa Na2CO3, K2CO3, NaHCO3 ( trong đó NaHCO3 có nồng độ 1M), thu được 1,12 lít CO2 (đktc) và dung dịch Y. Cho nước vôi trong dư vào dung dịch Y thu được 20 gam kết tủa.Nồng độ mol/l của dung dịch HCl là: A. 1,25 M B.0,5M C.1,0M @ D. 0,75M Câu 6: Nhỏ từ từ 200ml dung dịch X (K2CO3 1M và NaHCO3 0,5M) vào 200ml dung dịch HCl 2M thì thể tích khí CO2 thu được (đktc) là: A.4,48lít B.5,376lít @ C.8,96lít D.4,48lít Câu 7. Cho 0,14 mol CO2 hấp thụ hết vào dung dịch chứa 0,08mol Ca(OH) 2. Ta nhận thấy khối lượng CaCO3 tạo ra nhỏ hơn khối lượng CO2 đã dùng nên khối lượng dung dịch còn lại tăng là bao nhiêu? A. 2,08 gam B. 1,04 gam C. 4,16 gam@ D. 6,48 gam Câu 8. Hấp thu hết CO2 vào dung dịch NaOH được dung dịch A. Biết rằng: Cho từ từ dung dịch HCl vào dung dịch A thì phải mất 50ml dd HCl 1M mới thấy bắt đầu có khí thoát ra. Mặt khác cho dd Ba(OH) 2 dư vào dung dịch A được 7,88 gam kết tủa. Dung dịch A chứa? A. Na2CO3 B. NaHCO3 C. NaOH và Na2CO3@ D. NaHCO3, Na2CO3 Câu 9. Dẫn 5,6 lít CO2 (đktc) vào bình chứa 200ml dung dịch NaOH nồng độ a M; dung dịch thu được có khả năng tác dụng tối đa 100 ml dung dịch KOH 1M. Giá trị của a là? A. 0,75 B. 1,5 C. 2@ D. 2,5 Câu 10. (Đại học khối A năm 2007). Hấp thụ hoàn toàn 2,688 lít CO 2 (đktc) vào 2,5 lít dung dịch Ba(OH)2 nồng độ a mol/l, thu được 15,76 gam kết tủa. Gía trị của a là? A. 0,032 B. 0,048 C. 0,06 D. 0,04@ Câu 11. Hấp thụ toàn bộ 0,3 mol CO 2 vào dung dịch chứa 0,25 mol Ca(OH) 2. khối lượng dung dịch sau phản ứng tăng hay giảm bao nhiêu gam? A. Tăng 13,2gam B. Tăng 20gam 98 C. Giảm 16,8gam D. Giảm 6,8gam @ Câu 12. (Đại học khối B-2007). Nung 13,4 gam 2 muối cacbonat của 2 kim loại hóa trị II, được 6,8 gam rắn và khí X. khí X sinh ra cho hấp thụ vào 75 ml dung dịch NaOH 1M, khối lượng muối khan sau phản ứng là? A. 5,8gam B. 6,5gam C. 4,2gam D. 6,3gam@ Câu 13. Thổi CO2 vào dd chứa 0,02 mol Ba(OH)2. Giá trị khối lượng kết tủa biến thiên trong khoảng nào khi CO2 biến thiên trong khoảng từ 0,005 mol đến 0,024 mol A. 0 gam đến 3,94g B. 0,985 gam đến 3,94g@ C. 0 gam đến 0,985g D. 0,985 gam đến 3,152g Câu 14. HÊp thô 3,36 lÝt SO2 (®ktc) vµo 0,5 lÝt hçn hîp gåm NaOH 0,2M vµ KOH 0,2M. C« c¹n dung dÞch sau ph¶n øng thu ®îc khèi lîng muèi khan lµ A. 9,5gam B. 13,5g C. 12,6g D. 18,3g@ Câu 15. Cho 6,72 lit khí CO2 (đktc) vào 380 ml dd NaOH 1M, thu được dd A. Cho 100 ml dd Ba(OH)2 1M vào dd A được m gam kết tủa. Gía trị m bằng: A. 19,7g @ B. 15,76g C. 59,1g D.55,16g Câu 16. Hấp thụ hết 0,672 lít CO 2 (đktc) vào bình chứa 2 lít dung dịch Ca(OH) 2 0,01M. Thêm tiếp 0,4gam NaOH vào bình này. Khối lượng kết tủa thu được sau phản ứng là? A. 1,5g B. 2g@ C. 2,5g D. 3g Câu 17. Cho 0,14 mol CO2 hấp thụ hết vào dung dịch chứa 0,11 mol Ca(OH) 2. Ta nhận thấy khối lượng CaCO3 tạo ra lớn hơn khối lượng CO2 đã dùng nên khối lượng dung dịch còn lại giảm bao nhiêu? A. 1,84 gam@ B. 3,68 gam C. 2,44 gam D. 0,92 gam Câu 18. Cho 0,14 mol CO2 hấp thụ hết vào dung dịch chứa 0,08mol Ca(OH) 2. Ta nhận thấy khối lượng CaCO3 tạo ra nhỏ hơn khối lượng CO2 đã dùng nên khối lượng dung dịch còn lại tăng là bao nhiêu? A. 2,08 gam B. 1,04 gam C. 4,16 gam@ D. 6,48 gam Câu 19. V lít khí CO2 (đktc) vào 1,5 lít Ba(OH)2 0,1M được 19,7 gam kết tủa. Gía trị lớn nhất của V là? A. 1,12 B. 2,24 C. 4,48@ D. 6,72 Câu 20. Dẫn 8,96 lit CO2 (đktc) vào V lit dd Ca(OH)2 1M, thu được 40g kết tủa.Gía trị V là: A.0,2 đến 0,38 B. 0,4 C. < 0,4 D. >= 0,4@ Câu 21. Thổi V ml (đktc) CO2 vào 300 ml dd Ca(OH)2 0,02M, thu được 0,2g kết tủa.Gía trị V là: A. 44.8 hoặc 89,6 B.44,8 hoặc 224@ C. 224 D. 44,8 99 Câu 22. Thổi V lit (đktc) CO2 vào 100 ml dd Ca(OH)2 1M, thu được 6g kết tủa. Lọc bỏ kết tủa lấy dd đun nóng lại có kết tủa nữa. Gía trị V là: A.3,136@ B. 1,344 C. 1,344 hoặc 3,136 D. 3,36 hoặc 1,12 Câu 23. Dẫn V lít CO2 (đkc) vào 300ml dd Ca(OH)2 0,5 M. Sau phản ứng được 10g kết tủa. V bằng: A. 2,24 lít B. 3,36 lít C. 4,48 lít D. Cả A, C đều đúng Câu 24. Hấp thụ toàn bộ x mol CO2 vào dung dịch chứa 0,03 mol Ca(OH) 2 được 2 gam kết tủa. Gía trị x? A. 0,02mol và 0,04 mol @ B. 0,02mol và 0,05 mol C. 0,01mol và 0,03 mol D. 0,03mol và 0,04 mol Câu 25. Tỉ khối hơi của X gồm CO2 và SO2 so với N2 bằng 2.Cho 0,112 lít (đktc) X qua 500ml dd Ba(OH)2. Sau thí nghiệm phải dùng 25ml HCl 0,2M để trung hòa Ba(OH) 2 thừa. % mol mỗi khí trong hỗn hợp X là? A. 50 và 50 B. 40 và 60@ C. 30 và 70 D. 20 và 80 Câu 26. Cho 5,6 lít hỗn hợp X gồm N 2 và CO2 (đktc) đi chậm qua 5 lít dung dịch Ca(OH)2 0,02M để phản ứng xảy ra hoàn toàn thu được 5 gam kết tủa. Tính tỉ khối hơi của hỗn hợp X so với H2. A. 18,8@ B. 1,88 C. 37,6 D. 21 Câu 27. Hấp thụ hết V lít CO2 (đktc) vào 300 ml dung dịch NaOH x mol/l được 10,6 gam Na2CO3 và 8,4 gam NaHCO3. Gía trị V, x lần lượt là? A. 4,48lít và 1M@ B. 4,48lít và 1,5M C. 6,72 lít và 1M D. 5,6 lít và 2M Câu 28. Sôc CO2 vµo 200 ml hçn hîp dung dÞch gåm KOH 1M vµ Ba(OH) 2 0,75M. Sau khi khÝ bÞ hÊp thô hoµn toµn thÊy t¹o 23,6 g kÕt tña. TÝnh VCO2 ®· dïng ë ®ktc A. 8,512 lÝt @ B. 2,688 lÝt C. 2,24 lÝt D. C¶ A vµ B ®óng Câu 29. Dung dịch X chứa hỗn hợp gồm Na2CO3 1,5M và KHCO3 1M. Nhỏ từ từ từng giọt cho đến hết 350 ml dung dịch HCl 1M vào 100 ml dung dịch X, sinh ra V lít khí ( ở đktc). Giá trị của V là A. 1,12. B. 3,36. C. 2,24. D. 4,48.@ Câu 30. Hấp thụ V lít CO2 (đktc) vào 200 ml dung dịch NaOH 1M, thu được dung dịch X. Khi cho CaCl2 dư vào dung dịch X thì thu được kết tủa và dung dịch Y, đun nóng Y lại thấy kết tủa xuất hiện. Giá trị của V là: A. 1,12 < V < 2,24 B. 2,24 < V < 4,48@ C. 4,48 ≤ V D. V ≤ 1,12 Câu 31: Khi đốt m gam hỗn hợp X gồm Na, Ba thu 21,5g hỗn hợp oxit Na 2O, BaO. Nếu cho m gam X vo nước dư thu 4,48lit khí (đktc). và 0,5 lit dung dịch Y. Giá trị của m là A. 81,3 B. 3,18 C. 18,3@ D. 38,1 100 Câu 32: Cho a mol Al và 0,15 mol Mg phản ứng vừa đủ với b mol Cl 2 và 0,2mol O2 thu được 32,3g chất rắn . Gía trị a, b lần lượt là A. 0,2; 0,3 B. 0,15; 0,15 C. 0,3; 0,2@ D. 0,1; 0,3 Câu 33: Cho 2,13g hỗn hợp X gồm Mg, Cu, Al tác dụng hoàn toàn với oxi thu được hỗn hợp Y gồm các oxit có khối lượng 3,33g. Thể tích dung dịch HCl 2M vừa đủ để phản ứng hết với Y là A. 57ml B. 50ml C. 75ml@ D. 90ml Câu 34: Hoà tan hết 7,74 gam hỗn hợp bột Mg, Al bằng 500 ml dung dịch hỗn hợp HCl 1M và H2SO4 0,28M thu được dung dịch X và 8,736 lít khí H2 (ở đktc). Cô cạn dung dịch X thu được lượng muối khan là A. 38,93 gam. @ B. 103,85 gam. C. 25,95 gam. D. 77,86 gam Câu 35: Chia 2,290 gam hỗn hợp Mg, Al, Zn thành 2 phần bằng nhau. Phần 1 hoà tan hoàn toàn trong dung dịch HCl vừa đủ thu được 1,456 lít H 2 (đktc) và tạo ra x gam muối. Phần 2 cho tác dụng với O2 dư, thu được y gam 3 oxit.Giá trị của x là A. 6,955. B. 6,905. C. 5,890. D. 5,760.@ Câu 36: Chia 2,290 gam hỗn hợp Mg, Al, Zn thành 2 phần bằng nhau. Phần 1 hoà tan hoàn toàn trong dung dịch HCl vừa đủ thu được 1,456 lít H 2 (đktc). và tạo ra x gam muối. Phần 2 cho tác dụng với O2 dư, thu được y gam 3 oxit. Giá trị của y là A. 2,185.@ B. 3,225. C. 4,213. D. 3,33. Câu 37: Hoà tan hoàn toàn 8,862 gam hỗn hợp gồm Al và Mg vào dung dịch HNO 3 loãng, thu được dung dịch X và 3,136 lít (ở đktC. hỗn hợp Y gồm hai khí không màu, trong đó có một khí hóa nâu trong không khí. Khối lượng của Y là 5,18 gam. Cho dung dịch NaOH (dư) vào X và đun nóng, không có khí mùi khai thoát ra. Phần trăm khối lượng của Al trong hỗn hợp ban đầu là A. 19,53%. B. 12,80%. @ C. 10,52%. D. 15,25%. Câu 38: Cho 2,16 gam Mg tác dụng với dung dịch HNO3 (dư). Sau khi phản ứng xảy ra hoàn toàn thu được 0,896 lít khí NO (ở đktC. và dung dịch X. Khối lượng muối khan thu được khi làm bay hơi dung dịch X là A. 8,88 gam. B. 13,92 gam.@ C. 6,52 gam. D. 13,32 gam. Câu 39: Hỗn hợp X gồm Mg và MgO được chia thành 2 phần bằng nhau. Cho phần 1 tác dụng hết với dung dịch HCl thu được 3,136 lít khí (đktc); cô cạn dung dịch và làm khô thì thu được 14,25g chất rắn khan A. Cho phần 2 tác dụng hết với dung dịch HNO 3 thì thu được 0,448 lít khí Y (đktc), cô cạn dung dịch và làm khô thì thu được 23 gam chất rắn khan B.Công thức phân tử của Y là A. NO2. B. NO. C. N2O. D. N2.@ 101 Câu 40: 500 ml dung dịch hỗn hợp A gồm: HCl 0,2M – H2SO4 0,4M – HNO3 0,6M được trung hòa vừa đủ bởi dung dịch hỗn hợp B gồm: Ba(OH)2 0,6M – NaOH 2M. Khối lượng kết tủa thu được sau phản ứng trung hòa là A. 46,6g B. 139,8g C. 27,96g D. 34,95g@ Câu 38: Cho 20,6g hỗn hợp gồm muối cacbonat của kim loại kiềm và của kim loại kim thổ tác dụng hết với dung dịch HCl dư thu 4,48lit khí (đktc) .Cô cạn dung dịch sau phản ứng rồi cho muối khan điện phân nóng chảy thu m gam kim loại . Giá trị m là. A. 8,6g@ B. 8,7g C. 8,8g D. 8,9g Câu 39: Hai kim loại A, B đều có hóa trị II. Hòa tan hết 0,89 gam hỗn hợp hai kim loại này, trong dung dịch HCl. Sau phản ứng thu được 448 ml khí H 2 (đktc). Hai kim loại A, B là A. Mg, Ca B. Zn, Fe C. Ba, Fe D. Mg, Zn @ Câu 40: Hoà tan 1,7 gam hỗn hợp kim loại A và Zn vào dung dịch HCl thu được 0,672 lít khí ở điều kiện tiêu chuẩn và dung dịch B. Mặt khác để hoà tan 1,9 gam kim loại A thì cần không hết 200ml dung dịch HCl 0,5M. M thuộc phân nhóm chính nhóm II. Kim loại M là A. Ca@ B. Cu C. Mg D. Sr Câu 41: Cho hỗn hợp gồm 3,36 gam Mg và 0,4 gam MgO tác dụng với dung dịch HNO 3 loãng dư thu được 0,448 lít khí N2 là sản phẩm khử duy nhất (đktc). Cô cạn dung dịch sau phản ứng thu được 23 gam chất rắn khan. Số mol HNO3 đã phản ứng là: A. 0,32 mol B. 0,28 mol C. 0,34 mol D. 0,36 mol@ Câu 42. Nhỏ từ từ dung dịch H2SO4 loãng vào dung dịch X chứa 0,1 mol Na2CO3 và 0,2 mol NaHCO3, thu được dung dịch Y và 4,48 lít khí CO2 (đktc). Tính khối lượng kết tủa thu được khi cho dung dịch Ba(OH)2 dư vào dung dịch Y? A. 66,30 gam B. 54,65 gam@ C. 46,60 gam D. 19,70 gam Câu 43. Hấp thụ hết V lít khí CO2 vào dung dịch chứa 0,39 mol Ca(OH)2 thu được a gam kết tủa. Tách lấy kết tủa, sau đó thêm tiếp 0,4V lít khí CO2 nữa, thu thêm 0,2a gam kết tủa. Thể tích các khí đo ở đktc. Giá trị của V là: A. 7,84 lít. B. 5,60 lít. C. 8,40 lít. D. 6,72 lít.@ Câu 44. Hòa tan hết m gam hỗn hợp Na, Ba vào dung dịch Al2(SO4)3 5,6 lít khí H2 (đktc); dung dịch X và 57,52 gam kết tủa. Sục khí CO2 đến dư vào dung dịch X, lọc kết tủa và đem nung trong không khí đến khối lượng không đổi được 1,02 gam chất rắn. Phần trăm khối lượng kim loại Na có trong hỗn hợp ban đầu là: A. 7,44%@ B. 15,448% C. 12,460% D. 1,370% 102 Câu 45: A là hỗn hợp khí gồm SO 2 và CO2 có tỷ khối hơi so với H 2 là 27. Dẫn a mol hỗn hợp khí A qua bình đựng 1 lít dung dịch NaOH 1,5a M, sau phản ứng làm bay hơi cẩn thận dung dịch thu được m gam muối. Biểu thức liên hệ giữa m và a là A. m=103,5a B. m=105a@ C. m=141a D. m=116a C. KẾT LUẬN Hiện nay do giáo viên có nhiều công việc kiêm nhiệm và được phân công giảng dạy ở nhiều khối lớp khác nhau nên thời gian để soạn các chuyên đề ôn thi còn nhiều hạn chế. Qua chuyên đề này, tôi muốn nhấn mạnh tới việc HS phải tự tóm tắt, tổng hợp kiến thức lý thuyết để tạo tiền đề cho việc nắm vững các dạng bài tập cũng như các phương pháp giải bài tập trong mỗi chuyên đề. Thêm vào đó, giáo viên căn cứ vào chất lượng học sinh, lựa chọn hệ thống bài tập phù hợp và động viên, khích lệ học sinh một cách kịp thời để khơi dậy niềm tin cũng như khát vọng học tập của học sinh nhằm đạt mục tiêu học tập. Trên đây là một cách triển khai của cá nhân tôi khi đứng lớp dạy các chuyên đề ôn thi đại học cho học sinh lớp 12; tôi thấy việc tổng kết lý thuyết và xây dựng phương pháp giải toán cho mỗi dạng với mỗi chủ đề được các em học tập tốt hơn. Cuối cùng, với mong muốn được chia sẻ và học hỏi, rất mong nhận được sự góp ý của quý thầy cô để chuyên đề được hoàn thiện hơn. 103 1.Tác giả chuyên đề: Lê Thu Hà Tổ trưởng: Tổ Văn – Sử - Địa – GDCD Trường THPT Vĩnh Yên. 2.Đối tượng học sinh bồi dưỡng: lớp 12, dự kiến số tiết bồi dưỡng 10 3.Hệ thống kiến thức sử dụng trong chuyên đề -Kiến thức cơ bản: +Tác giả Nam Cao. +Tác phẩm Chí Phèo. +Bài giảng về tác phẩm Chí Phèo -Kiến thức nâng cao: Các đề thi, các bài viết, các nhận định về các nhân vật(Bá Kiến, Chí Phèo, thị Nở…); về tác phẩm(chi tiết, giá trị nội dung, giá trị nghệ thuật…) và một số các tác phẩm khác liên quan(Đời thừa, Vợ nhặt…) 4.Hệ thống các dạng bài tập của chuyên đề: -Hệ thống câu hỏi 2 điểm. -Hệ thống câu hỏi 5 điểm. 5.Hệ thống phương pháp để giải các bài tập trong chuyên đề: -Tái hiện kiến thức. -Phân tích, cảm nhận chi tiết, nhân vật… -Bình luận, đánh giá… 6. Hệ thống các ví dụ, bài tập cụ thể cùng lời giải minh họa cho chuyên đề. 104 PHẦN MỘT: ĐẶT VẤN ĐỀ Trong những năm gần đây, cùng với một số môn học khác, thực trạng dạyhọc môn Văn được đề cập nhiều trên các phương tiện thông tin, trong đó đặc biệt nhấn mạnh tỉ lệ học sinh yếu ở bộ môn này ngày càng cao, kéo theo kết quả không mấy khả quan qua các kỳ thi Đại học, Cao đẳng. Vì vậy, vấn đề làm thế nào để có thể nâng cao kết quả học tập và ôn thi đại học môn Ngữ Văn cho học sinh THPT thật sự là vấn đề thiết yếu và được quan tâm hàng đầu hiện nay. Có thể khẳng định, từ khi tiến hành cải cách chương trình và sách giáo khoa bậc THPT (năm 2006) đến nay, nhiều giáo viên đã rất nỗ lực trong việc dạy - học để mang lại cho học sinh những phương pháp học Văn tích cực cùng với sự hỗ trợ của các phương tiện công nghệ thông tin ngày càng hiện đại, giúp các tiết học Văn đạt hiệu quả cao hơn, song việc học sinh học yếu môn Văn hiện vẫn đang là một tồn tại mà bất cứ ai quan tâm đến nền giáo dục của nước nhà cũng có thể thấy. Khách quan mà nói, điều đó một phần là do vẫn có giáo viên chưa quan tâm đúng mức, chưa giúp đỡ kịp thời đối với học sinh trong quá trình dạy - học nên để các em có những lỗ hổng kiến thức cơ bản. Một phần không nhỏ là do chính bản thân các em không thích học (kể cả không chịu học) các môn xã hội nói chung, môn Ngữ Văn nói riêng; kể cả việc có em không biết cách học như thế nào cho có hiệu quả nên dẫn đến kết quả học tập của các em ngày càng thấp so với yêu cầu và của mặt bằng xã hội nói chung. Từ thực tế trên, vấn đề được quan tâm hiện nay là làm thế nào để có thể nâng cao chất lượng, kết quả học tập môn Ngữ Văn nói riêng, các môn xã hội nói chung qua các kỳ thi hàng năm. Vấn đề trên đã trở thành mục tiêu hàng đầu trong mọi chương trình nghị sự khi bàn về giáo dục, nhất là trong những năm học tới có thể các em học sinh phải thi đại học bắt buộc hai môn thi: Toán và văn thì vấn đề dạy ôn đại học môn văn càng được xã hội quan tâm nhiều hơn. Bất kể nguyên nhân do đâu, việc giúp đỡ các em học sinh lớp 12 học và ôn tập thi đại học môn Ngữ Văn đạt hiệu quả là việc làm hết sức cần thiết, cần được nhà trường và đặc biệt là người giáo viên Ngữ Văn quan tâm nhiều nhất trong tình hình hiện nay. Từ những lí do trên, tôi xin trao đổi kinh nghiệm của bản thân về việc biên soạn hệ thống đề hướng dẫn cho học sinh ôn thi đại học tác phẩm Chí Phèo của Nam Cao.Qua đây muốn được chia sẻ với các bạn đồng nghiệp để cùng nhau làm tốt hơn nhiệm vụ dạy học của mình, hướng dẫn học sinh ôn thi đại học đạt kết quả tốt hơn. 105 PHẦN HAI: NỘI DUNG. I-Khi giảng dạy và hướng dẫn học sinh ôn tập tác phẩm Chí Phèo, mỗi người có một hướng tiếp cận riêng. Với cá nhân tôi, khi viết chuyên đề này tôi dựa trên cơ sở thực tiễn sau: 1. Đúc rút kinh nghiệm từ bản thân trong quá trình giảng dạy môn ngữ văn ở trường THPT (Từ bài giảng đến việc biên soạn hệ thống đề hướng dẫn cho học sinh ôn thi đại học tác phẩm Chí Phèo của Nam Cao) 2. Học hỏi kinh nghiệm của đồng nghiệp trong giảng dạy, ra đề, hướng dẫn học sinh ôn tập. 3.Tham khảo một số đề đã ra về tác phẩm Chí Phèo trong những kỳ thi đại học. 4.Căn cứ vào thực tế việc học ôn của học sinh đó là: a. Trong bộ môn Ngữ văn ở trường phổ thông trung học nhiều năm nay cho thấy vẫn còn có hiện tượng học sinh học theo kiểu cũ: đọc thuộc, sao chép, nói lại ý sách vở thầy cô mà không hoặc ít có sự sáng tạo khi tiếp xúc tác phẩm văn chương. b. Hiện tượng ít tập trung suy nghĩ, ít tìm tòi ở học sinh. c. Thị trường sách hiện nay: Sách in ấn nhiều, giảng giải cụ thể tác phẩm, học sinh mua về chép lại một cách máy móc mà không suy nghĩ, sáng tạo dẫn đến tình trạng học tủ, học vẹt. d. Khi tiếp xúc với tác phẩm văn học, học sinh chỉ hiểu theo một chiều, ít chịu khó phát hiện, vốn từ ngữ nghèo, diễn đạt kém. Vì vậy, không đạt hiệu quả cao khi cảm nhận tác phẩm văn chương. II-Từ thực tế trên tôi tự rút ra giải pháp giúp học sinh học và ôn thi đại học đạt hiệu quả như sau: 1.Chuẩn bị bài giảng và giảng tác phẩm Chí Phèo. -Muốn để học sinh yêu thích môn văn, thích học văn thì giáo viên văn cũng phải thật sự yêu thích, say mê môn học này. Sẽ là chuyện hoang tưởng nếu thầy cô đến với văn học bằng một trái tim hững hờ, một sự “giảng cho hết bài” mà lại mong có học trò yêu thích học văn. Tất nhiên yêu thích nó , say mê nó nhưng để truyền niềm yêu thích ấy sang cho học trò còn phải có thêm một số yếu tố khác nữa như khả năng truyền đạt, sự phối hợp các phương pháp dạy học và tổ chức học sinh học tập…nhưng yêu thích, say mê thậm chí si mê văn học, say mê dạy văn cũng như cái đẹp trong văn chương là yếu tố đầu tiên để thầy và môn văn chinh phục được lòng người đọc nói chung, học sinh nói riêng -Do đặc điểm riêng của đối tượng học sinh nên khi soạn bài thầy nên soạn bài, chọn phương pháp, phương tiện thích hợp phục vụ cho bài giảng. Khi giảng bài cố gắng đơn giản hóa kiến thức (mà không sơ sài, không cắt bớt) bằng cách chọn các từ ngữ giản dị, thậm chí nôm na để các em có thể hiểu được những kiến thức cơ bản nhất. 2.Biên soạn hệ thống câu hỏi, đáp án hướng dẫn học sinh ôn thi đại học đạt hiệu quả: 106 Sau bài dạy những giờ ôn tập môn học này thường là những giờ giáo viên phải đổ nhiều công sức mà kết quả thu được còn rất hạn chế. Dạy Văn cũng đòi hỏi sự sáng tạo của người giáo viên. Người thầy dạy Văn giỏi vừa là một nhà sư phạm lại đồng thời phải có phẩm chất của người nghệ sĩ. Trong giờ ôn tập, tương tác giữa thầy và trò giúp người dạy tránh được vai trò độc diễn khô khan, học trò bớt được cảm giác thụ động, giờ học thoát khỏi không khí nặng nề. Việc hướng dẫn học sinh ôn tập nếu làm tốt sẽ mang lại những hiệu quả vô cùng thiết thực. Phương pháp ôn tập: Trong quá trình ôn tập, cần giúp học sinh ôn cả hai mảng kiến thức lẫn kỹ năng làm bài. Kiến thức đóng vai trò nền tảng, bởi "không có bột, không thể gột nên hồ”nhưng kỹ năng yếu thì cũng không áp dụng được. Việc hướng dẫn học sinh ôn kiến thức: Có nhiều cách để hệ thống hóa kiến thức đã học. Chúng tôi bám sát vào cấu trúc đề thi đại học, cách hỏi, ra đề của Bộ giáo dục trong những năm gần đây để hướng dẫn học sinh ôn thi đại học theo các cách sau: Thứ nhất: Biên soạn và hướng dẫn học sinh ôn tập các câu hỏi 2 điểm liên quan đến tác phẩm( tái hiện kiến thức, chi tiết và ý nghĩa chi tiết…) Với tác phẩm Chí Phèo của Nam Cao có thể có các câu hỏi như sau: Câu 1: Tác phẩm Chí Phèo cuả Nam Cao đã có những tên gọi như thế nào ? Anh/chị hãy cho ý kiến nhận xét về những tên gọi ấy. ( CĐSP hà Tĩnh 2000) Dàn ý: 1. Các tên gọi của tác phẩm Chí Phèo : - Nhan đề đầu tiên 1940: “ Cái lò gạch cũ ” - Khi in thành sách 1941,NXB Đời mới đổi thành “Đôi lứa xứng đôi” - Mãi đến năm 1946, tác giả mới đổi thành “Chí Phèo” khi in trong tập “ Luống cày ” 2. Nêu nhận xét : - Cái lò gạch cũ : + Cách gọi này dựa vào hình ảnh cái lò gạch bỏ hoang ở phần đầu và được lặp lại ở câu kết của tác phẩm . + Ý nghĩa : Nhấn mạnh tích chất quy luật của hiện tượng “ Chí Phèo ” ,tố cáo và lên án xã hội đương thời . Tạo nên những ám ảnh trong lòng người đọc về vòng đời quẩn quanh , bế tắc của người nông dân + Hạn chế : Cái nhìn bi quan của tác giả về số phận của người nông dân - Đôi lứa xứng đôi : + Cách gọi này dựa vào mối tình giữa Chí Phèo - Thị Nở,nhắm gợi trí tò mò của một số độc giả đương thời + Hạn chế:Chưa khái quát được ý nghĩa của tác phẩm vì mối tình Chí Phèo – Thị Nở chỉ có giá trị như một tình huống tạo nên bước ngoặt quan trọng trong cuộc đời Chí Phèo, bộc lộ một khía cạnh tư tưởng nhân đạo của tác phẩm.Tên gọi này đã 107 biến mối tình của hai nhân vật thành trò cười và gây ra một hướng tiếp cận sai lệch về tác phẩm. - Chí Phèo + Cách gọi này thống nhất với tác phẩm khác của Nam Cao - lấy tên nhân vật chính để đặt tên truyện : Lão Hạc,Dì Hảo , Lang Rận,… +Ý nghĩa : Với nhan đề này , tác giả muốn tạo sự chú ý của người đọc vào diễn biến cuộc đời và số phận của nhân vật trung tâm . Từ đó , tác giả giúp cho người đọc thấy được giá trị hiện thực và giá trị nhân đạo lớn lao của tác phẩm Câu 2: Phân tích ý nghĩa hình ảnh “ cái lò gạch cũ ” trong tác phẩm Chí Phèo của Nam Cao Dàn ý: a) Trình bày được hình ảnh “ cái lò gạch cũ” nơi Chí Phèo ra đời bị bỏ rơi,trần truồng và xám ngắt ,…và hình ảnh cái lò gạch Thị Nở nghĩ đến sau khi Chí Phèo chết . Thể hiện được bi kịch đau khổ của người nông dân nghèo bị đàn áp trước cách mạng b) Ý nghĩa tư tưởng hình ảnh “ cái lò gạch cũ ” - Chí chết ,vẫn còn đó chiếc lò gạch cũ ,vẫn còn sẽ có nhiều sản phẩm như Chí trong cái vòng lẩn quẩn không lối thoát . Vì thế để không còn những số phận bi kịch như vậy thì phải xóa bỏ những cái lò gạch cũ , cũng có nghĩa là xóa bỏ cái xã hội cũ đó . Đó là lời cảnh báo của nhà văn , cũng chính là ý nghĩa toát ra từ tác phẩm - Hình ảnh “ cái lò gạch cũ” cũng thể hiện cái nhìn bế tắc của Nam Cao ,cũng là bế tắc về tư tưởng lịch sử trong thời điểm đó(Tắt đèn-NTT) c) Ý nghĩa nghệ thuật : Trong tác phẩm ,Chi tiết này chủ yếu xuất hiện 2 lần tạo nên kết cấu đầu cuối tương ứng ,kết cấu vòng tròn . Kết cấu này nằm trong ý đồ nghệ thuật của nhà văn và là một sáng tạo nghệ thuật đặc sắc Câu 3: Truyện ngắn Chí Phèo của Nam Cao có chi tiết nghệ thuật đặc sắc: Tiếng chửi của Chí Phèo. Hãy phân tích chi tiết đặc sắc ấy. Dàn ý: 1. Giới thiệu tác giả ,tác phẩm và chi tiết nghệ thuật - Tác giả ,tác phẩm - Giới thiệu chi tiết nghệ thuật 2. Phân tích chi tiết * Với chi tiết “ tiếng chửi của Chí Phèo”cần phân tích để làm nổi bật tính chất bi kịch của nhân vật được thể hiện qua các khía cạnh như sau : - Tiếng chửi mở đầu thiên truyện một cách bất ngờ và giới thiệu nhân vật một cách ấn tượng 108 - Đó là tiếng chửi của một kẻ say nhưng cũng có cái gì đó tỉnh táo ( vì nó có gì đó “văn vẻ’ , lớp lang : “ trời”- “đời ”-“ cả làng Vũ Đại ”- “ cha đứa nào không chửi nhau với hắn ” – “ đứa chết mẹ nào đẻ ra thân hắn ” ) - Đối tượng của tiếng chửi: cái xã hội dửng dưng lạnh lùng , tàn nhẫn đã sinh ra kiếp sống Chí Phèo - Ý nghĩa tiếng chửi : + Đó là phản ứng của Chí Phèo với toàn bộ cuộc đời : Bộc lộ tâm trạng bất mãn của một con người ít nhiều ý thức được mình đã bị xã hội phi nhân tính gạt ra khỏi thế giới loài người à Tiếng chửi là phương thức duy nhất làm cho Chí có đủ dũng khí để giao tiếp với xã hội loài người + Đó là dấu hiệu tuyệt vọng về một kiếp sống cô độc của người nông dân bị tha hóa , không còn được làm người + Tiếng chửi đầu tác phẩm như mở ra 2 bi kịch day dẳng của Chí Phèo : Bi kịch tha hóa và bi kịch bị từ chối quyền làm người -Lời trần thuật nửa trực tiếp rất độc đáo : Sự hòa trộn giữa lời người kể chuyện và lời của nhân vật đã làm cho chi tiết tiếng chửi tăng thêm sức bộc lộ trong trạng thái bi phẫn,bế tắc,kiếp sống cô đơn cùng cực của Chí Phèo.Trong giọng điệu này như nghe thấy tiếng chì chiết ,đay nghiến tình đời , tình người của tác giả . 3. Nhận xét về tác dụng nghệ thuật của chi tiết ấy Chi tiết nghệ thuật đặc sắc là chi tiết có vai trò nhất định trong việc thể hiện chủ đề tư tưởng tác phẩm hay nhằm nhấn mạnh một khía cạnh quan trọng . Cụ thể ở đây là vai trò thể hiện các tư tưởng nhân văn , giá trị hiện thực của tác phẩm Chí Phèo. Câu 4: Truyện ngắn Chí Phèo của Nam Cao có chi tiết nghệ thuật đặc sắc: bát cháo hành của Thị Nở. Hãy phân tích chi tiết đặc sắc ấy. Dàn ý: 1. Giới thiệu tác giả, tác phẩm và chi tiết nghệ thuật - Tác giả ,tác phẩm - Giới thiệu chi tiết nghệ thuật 2. Phân tích chi tiết * Với chi tiết “ bát cháo hành của Thị Nở cần làm nổi bật giá trị nhân đạo mà Nam Cao gửi gắm : thiên lương,khao khát hướng thiện trong con người không thể bị dập tắt ,bất chấp hoàn cảnh khắc nghiệt nhất, nó vẫn luôn tiềm ẩn, sẽ bùng lên mãnh liệt khi có cơ hội .Chỉ ra được các khía cạnh sau : - Nam Cao không miêu tả nhiều về bát cháo hành của Thị Nở (chỉ vài ba câu chữ miêu tả cụ thể như: “cháo hành còn nóng nguyên”, “ bát cháo hành còn bốc khói”, “ trời ơi cháo mới thơm làm sao ! ”…) -Tác giả tập trung miêu tả phân tích ý nghĩ ,cảm giác ,cảm tưởng của người cho là Thị Nở và nhất là người ăn cháo là Chí Phèo *Suy nghĩ của người cho cháo(Thị Nở) 109 + “ Cái thằng liều lĩnh ấy kể ra thì đáng thương ,còn gì đáng thương bằng đau ốm mà nằm còng queo một mình … ” + “ Đang ốm thế thì chỉ ăn cháo hành ,ra được mồ hôi thì là nhẹ nhõm người ngay đó mà … ” +Thằng này rất ngạc nhiên, mắt hình như ươn ướt vì lần đầu tiên Chí Phèo được người đàn bà cho ăn. + Hắn nhận ra “ những người suốt đời không ăn cháo hành không biết rằng cháo hành ăn rất ngon…Hắn tự hỏi rồi lại tự trả lời:có ai nấu cho mà ăn đâu?Mà còn ai nấu cho mà ăn nữa!Đời hắn chưa bao giờ được săn sóc bởi một bàn tay “đàn bà”….. ” Hắn nhớ đến bà ba “ Hắn chỉ thấy nhục,chứ yêu thương gì.Không,hắn chưa được một người đàn bà nào yêu cả…Hắn có thể tìm bạn được,sao chỉ lại gây kẻ thù ” +Phát hiện “trông thị thế mà có duyên” + Đó là hương vị mộc mạc mà ngọt ngào pha lẫn cay đắng của hạnh phúc và tình người cảm động mà Chí Phèo chưa bao giờ được hưởng. Nó là cầu nối đưa Chí trở về với cõi người. Bọn thống trị giam cầm Chí hơn 20 năm trời,nhưng chỉ một bát cháo hành đã hóa giải hết các “ phép thuật ” độc ác,thâm hiểm. 3. Nhận xét về tác dụng nghệ thuật của chi tiết ấy Chi tiết nghệ thuật đặc sắc là chi tiết có vai trò nhất định trong việc thể hiện chủ đề tư tưởng tác phẩm hay nhằm nhấn mạnh một khía cạnh quan trọng . Cụ thể ở đây là vai trò thể hiện các tư tưởng nhân văn , giá trị nhân đạo của tác phẩm Chí Phèo. Câu 5:Trong tác phẩm Chí Phèo (Nam Cao), sau khi Chí Phèo giết bá Kiến và tự sát, mọi người có thái độ như thế nào trước cái chết của Chí Phèo ? Qua đó, nhà văn thể hiện tư tưởng gì ? Dàn ý: 1.Thái độ của mọi người trước cái chết của Chí Phèo: - Người dân làng Vũ Đại : Họ bàn tán rất nhiều về vụ án không ngờ ấy. Có kẻ mừng thầm, có kẻ mừng ra mặt, có người nói xa xôi, có người nói toạc : "Thằng nào chứ hai thằng ấy chết thì không ai tiếc",... - Mừng nhất là bọn kì hào ở trong làng. Họ tuôn đến để hỏi thăm, nhưng chính là để nhìn lí Cường bằng những con mắt thỏa mãn và khiêu khích. Đội Tảo thì nói toang toang ngay ngoài chợ : "Thằng bố chết, thằng con lớp này không khỏi người ta cho ăn bùn". - Những người biết điều thì hay ngờ vực; họ chép miệng nói : "Tre già măng mọc",... -Bà cô thị Nở đay nghiến cháu, còn thị Nở cười, nói lảng, nghĩ thầm "sao có lúc nó hiền như đất" và thị nhớ lại những lúc ăn nằm với Chí Phèo, đột nhiên thị thấy thoáng hiện ra cái lò gạch cũ bỏ không, xa nhà cửa và vắng người lại qua... 2.Ý nghĩa: - Nhà văn vạch trần, tố cáo bản chất của giai cấp thống trị. Một mặt chúng thờ ơ, lạnh lùng trước cái chết của người nông dân hiền lành, lương thiện bị đẩy vào bước 110 đường cùng trong guồng quay của sự tha hóa và phải tìm đến cái chết để bảo vệ nhân phẩm. Mặt khác, chúng tìm cách trừng trị lẫn nhau, cưỡi lên đầu lên cổ nhau, làm cho nhau lụi bại và cho nhau "ăn bùn". - Thể hiện nỗi đau của Nam Cao trước sự vô tình của người dân làng Vũ Đại khi chứng kiến cái chết của đồng loại và thái độ bi quan của nhà văn về sự tồn tại, tiếp diễn cái xấu, cái ác và sự tha hóa. Mặt khác, nhà văn còn thể hiện niềm tin vào bản chất của người nông dân lương thiện qua thái độ của thị Nở. Tất cả mọi người đều mừng ra mặt trước cái chết của Chí Phèo chỉ riêng thị Nở biết hắn "hiền như đất". Chí Phèo đã trở lại với bản tính lương thiện của một con người và dám chết để bảo vệ nhân phẩm ấy. Thứ hai: Biên soạn câu hoi và hướng dẫn học sinh ôn tập các câu hỏi 5 điểm liên quan đến tác phẩm *Hỏi về nhân vật, một khía cạnh giá trị của tác phẩm: Câu 1: Phân tích nhân vật Bá Kiến trong truyện ngắn Chí Phèo của nhà văn Nam Cao. Dàn ý: I. Vài nét về tác giả tác phẩm và giới thiệu nhân vật Chí Phèo - Nam Cao - Nam Cao (1917-1951) là nhà văn hiện thực có tư tưởng nhân đạo vừa sâu sắc vừa mới mẻ.Ông am hiểu khá tường tận về cuộc sống của những người nông dân nghèo sau luỹ tre làng. Vì vậy, viết về tầng lớp này, ông đã có hàng loạt tác phẩm rất có giá trị, tiêu biểu nhất trong số đó là Chí Phèo (1941) xứng đáng là kiệt tác của dòng văn học hiện thực phê phán. - Với ngòi bút hiện thực sâu sắc, qua Chí Phèo, Nam Cao tập trung vạch trần mối mâu thuẫn giai cấp đối kháng ngàn đời chưa thể điều hoà. Đó là mâu thuẫn giữa người nông dân lao động nghèo khổ bị áp bức với bọn cường hào thống trị. Điển hình cho thế lực cường hào thống trị tàn bạo ấy là Bá Kiến. II. Phân tích nhân vật 1. Lai lịch - Bá Kiến xuất thân trong một gia đình bốn đời làm lí trưởng. -Bản thân ông ta làm lí trưởng, rồi chánh tổng. - Bằng mưu mô và thủ đoạn khôn khéo, hắn lần lượt leo lên đỉnh cao của danh vọng: tiên chỉ làng Vũ Đại, Bá hộ, Chánh hội đồng kỳ hào, huyện hào, Bắc kỳ nhân dân đại biểu, “hắn khét tiếng đến cả trong hàng huyện”. Ở làng Vũ Đại, nơi có cái thế đất “quần ngư tranh thực”, nghĩa là cả bầy cá tranh mồi, Bá Kiến trở thành con cá lớn. 2. Những nét tính cách của Bá Kiến Khác với Nghị Lại trong Bước đường cùng của Nguyễn Công Hoan, Nghị Quế trong Tắt đèn của Ngô Tất Tố, những tên địa chủ trọc phú, hủ lậu, Bá Kiến là loại cường hào, cáo già rất lọc lõi. Đây là nhân vật mang rõ nét phong cách nghệ thuật độc đáo đầy tài năng của Nam Cao.Khắc hoạ nhân vật này, Nam Cao không 111 nhấn mạnh khía cạnh bóc lột người nông dân mà khắc sâu một nét bản chất đặc biệt của lão. Đó là một tên cường hào có nghệ thuật thống trị, đàn áp người nông dân rất thâm hiểm. Vì thế, Bá Kiến có những nét tính cách tiêu biểu sau đây: a.Bá Kiến là tên cường hào ác bá, đặc biệt là gian hùng, nham hiểm - Đây là nét tính cách mang ý nghĩa bản chất nhất của Bá Kiến. Nam Cao đã mô tả nét bản chất ấy qua những chi tiết hết sức sinh động. + Đó là cái giọng quát rất sang, “bao giờ cũng quát để thử dây thần kinh của người ta”. +Bá Kiến có cái cười Tào Tháo và “giọng nói ngọt nhạt mà thâm hiểm chết người”. - Đặc biệt, Nam Cao để cho nhân vật Bá Kiến độc thoại nội tâm, nhằm phơi bày những suy nghĩ tỉnh táo, nham hiểm: Rút ra được những phương châm, thủ đoạn thống trị người nông dân rất khôn ngoan và hiệu quả mà Bá Kiến đã đúc rút từ bốn đời làm tổng lý: + “trị không được thì cụ dùng” + “dùng những thằng đầu bò để trị những thằng đầu bò” + “mềm nắn rắn buông” + “bám thằng có tóc chứ không ai bám kẻ trọc đầu”, với triết lý “thứ nhất sợ kẻ anh hùng, thứ hai sợ kẻ cố cùng liều thân”. - Bá Kiến là tên cường hào “khôn róc đời” : + Con người “thét ra lửa” ấy đã biết ngọt ngào, dịu giọng, xử nhũn với những kẻ cố cùng, liều thân như Chí Phèo để biến hắn thành công cụ đắc lực trong bàn tay thống trị độc ác của lão. + Biết mềm, biết cứng, biết thu dụng những kẻ bạt mạng, không sợ chết và không sợ bị tù, rất được việc trong chuyện đối phó với bất cứ anh nào không nghe mình à Tập hợp được một phe cánh, bè đảng xung quanh lão đầy thế lực. Sức mạnh của Bá Kiến lấn át tất cả, làm cho bọn cường hào đối địch trong làng phải nể sợ, kiêng dè. b. Bá Kiến là tên cường hào thống trị, lọc lừa, gian dối, luôn luôn biết ném đá giấu tay Đọc tác phẩm Chí Phèo, độc giả thấm thía những suy nghĩ về mưu kế ứng xử khôn khéo mà hiểm độc của lão: - “hãy ngấm ngầm đẩy người ta xuống sông, nhưng rồi dắt nó lên để nó đền ơn. - “Hãy đập bàn đập ghế đòi cho được năm đồng, nhưng được rồi thì vứt trả năm hào vì thương anh túng quá”. Vì thế, nhận cho ra bộ mặt thật của tên cáo già Bá Kiến không phải điều dễ dàng. c. Về phương diện bóc lột người nông dân : Bá Kiến là kẻ già đời trong nghề đục khoét. - Một mặt lão tìm cách bóp nặn đám dân hiền lành và yên phận vào những vụ thuế - Lão thu dụng những tên “bạt mạng” sinh chuyện với những kẻ có máu mặt trong làng để mà ăn, mà kiếm. d. Bá Kiến là tên cường hào có nhân cách xấu xa, dâm đãng, đồi bại Dù đã ngoài 60 tuổi, một lúc có đến bốn vợ, hay ghen bóng ghen gió với những trai trẻ và rất sợ vợ nhưng chính lão cáo già háo sắc và thích chơi “trống 112 bỏi” ấy lại bí mật đi lại với vợ Binh chức đã có bốn con, rồi còn lên tỉnh ngồi chung xe, chơi bời trác táng. e. Bá Kiến là kẻ đẩy bao người nông dân lương thiện vào con đường tha hóa, cùng quẫn Chính tên cáo già lọc lõi, gian hùng ấy đã đưa biết bao người dân lương thiện như Binh chức, Năm thọ vào tù, vào con đường tha hoá, bị đày đoạ, và khi cần thì sẵn sàng thí mạng những con người khốn khổ đó. Việc Bá Kiến xúi Chí Phèo đi đòi tiền Đội Tảo chẳng khác nào cài sẵn một cái bẫy để mượn tay Đội Tảo giết chết Chí Phèo. III. Kết luận - Bá Kiến là tên cường hào điển hình sắc sảo, vừa có những nét cá tính sinh động vừa có những nét chung phổ biến, tiêu biểu cho bọn cường hào, địa chủ gian ác. - Nhân vật Bá Kiến cũng là một hình tượng mang rõ nét tài năng của Nam Cao. Trong dòng văn học hiện thực phê phán lúc bấy giờ, ít có ngòi bút nào khắc hoạ được một hình tượng sinh động và có chiều sâu như thế ! Câu 2: Phân tích diễn biến tâm trạng Chí Phèo ( truyện ngắn: “Chí Phèo” của Nam Cao) từ buổi sáng sau khi gặp Thị Nở đến khi kết thúc cuộc đời để làm nổi rõ bi kịch của nhân vật này. ( ĐH Khối D – 2004 ) Dàn ý: 1. Khái quát tác giả, tác phẩm và bi kịch nhân vật a. Tác giả : -Nam Cao (1915 – 1951) là nhà văn hiện thực lớn có tư tưởng nhân đạo vừa sâu sắc, vừa mới mẻ, độc đáo. Sáng tác của ông trước Cách mạng xoay quanh hai đề tài chính là trí thức tiểu tư sản và người nông dân cùng khổ. Điều làm ông day dứt đến đau đớn là tình trạng nhân cách con người bị hủy hoại. - Là nhà văn có biệt tài phân tích tâm lí, Nam Cao đã khám phá ra những diễn biến nội tâm nhân vật vừa tất yếu, vừa bất ngờ rất thú vị. Qua việc tìm hiểu diễn biến tâm trạng Chí Phèo từ buổi sáng sau khi gặp Thị Nở đến khi kết thúc cuộc đời, chúng ta cũng có thể thấy rõ điều đó. b.Tác phẩm : -“Chí Phèo” là kiệt tác của Nam Cao về đề tài người nông dân và là kết tinh khá đầy đủ cho tài năng của Nam Cao. Tác phẩm đi sâu vào tấn bi kịch tinh thần của nhân vật chính Chí Phèo. - Bi kịch của Chí Phèo gồm hai bi kịch nối tiếp nhau. + Thứ nhất, bi kịch bị tha hóa, đầy đọa lăng nhục, từ một con người nông dân lương thiện bị xã hội biến thành một kẻ bất lương, thậm chí thành “con quỷ dữ”. + Bi kịch thứ hai là bị từ chối quyền làm người. Đoạn mô tả Chí Phèo từ buổi sáng sau khi gặp Thị Nở đến khi kết thúc cuộc đời thuộc bi kịch thứ hai. 113 2. Phân tích cụ thể diễn biến tâm trạng Chí Phèo. a. Trước hết là sự thức tỉnh. Bắt đầu là tỉnh rượu. - Tỉnh rượu : “Sau những cơn say vô tận”, “bây giờ thì hắn tỉnh” sau đêm gặp Thị Nở, Chí Phèo đã sống lại những cảm xúc đầy nhân tính. + Hắn cảm nhận được không gian xung quanh với “cái lều ẩm thấp mới chỉ lờ mờ”. +Đặc biệt hắn đã cảm nhận được những âm thanh quen thuộc của cuộc sống quanh mình: “Tiếng cười nói của những người đi chợ; tiếng gõ mái chèo đuổi cá, tiếng chim hót ngoài kia vui vẻ quá!”. Những âm thanh bình dị ấy ngày nào chả có, nhưng xưa nay, vì say hắn bị xã hội làm cho “mù điếc cả tâm hồn”, không nghe được. +Cùng với sự cảm nhận bức tranh cuộc sống xung quanh, Chí Phèo cũng đã cảm nhận được một cách thấm thía về tình trạng thê thảm của bản thân mình (già nua, cô độc, trắng tay):“Chí Phèo dường như đã trông thấy trước tuổi già của hắn, đói rét, ốm đau và cô độc, điều này còn đáng sợ hơn đói rét và ốm đau”. + Chí nhận ra thực tế đau lòng là mình chưa từng được chăm sóc như thế:Hắn “rất ngạc nhiên”, “mắt hắn hình như ươn ướt”, bởi vì “đây là lần thứ nhất hán được người ta cho cái gì”. Hắn nhận ra “Trời ơi, chào mới thơm làm sao!”. ( Chú ý đến chi tiết bát cháo hành và Chí Phèo khóc – Cần thấy đó là những dấu hiệu của nhân tính bị vùi lấp đang trở về ) Chí Phèo hi vọng. -“Trời ơi! Hắn thèm lương thiện, hắn muốn làm hòa với mọi người biết bao! Thị Nở sẽ mở đường cho hắn”, mọi người sẽ lại nhận hắn vào cái xã hội bằng phẳng của những con người lương thiện. Chúng sẽ làm thành một cặp rất xứng đôi. Chúng nhất định sẽ lấy nhau. Câu trả lời của Thị Nở lúc này sẽ quyết định số phận của Chí: được kết nạp lại xã hội loài người hay vĩnh viễn bị đày đọa trong kiếp sống thú vật? - Chí Phèo hồi hộp hi vọng. Nhưng cánh của hi vọng vừa hé mở thì đã bị đóng sầm ngay lại. Vì bà cô không cho Thị Nở “đâm đầu” đi lấy “thằng chỉ có một nghề là rạch mặt ăn vạ”. c. Tiếp đó là niềm thất vọng và đau đớn : Bà cô không cho Thị Nở lấy Chí . Thị Nở từ chối . Chí chạy theo nắm lấy tay Thị Nở như là nổ lực cuối cùng để níu Thị Nở lại với mình . Thị Nở đẩy Chí ngã , tỏ sự cắt đứt dứt khoát . Đau đớn và căm hận mù quáng, Chí nguyền sẽ giết chết bà cô và Thị Nở . d) Cuối cùng là trạng thái phẫn uất và tuyệt vọng : - Hắn lôi rượu ra uống. “Nhưng càng uống càng tỉnh ra! Chao ôi! Buồn”. Hắn cứ thoảng lấy hơi cháo hành - hơi của tình yêu hạnh phúc đang sắp tuột khỏi bàn tay cố níu kéo của Chí và “ôm mặt khóc rưng rức”. Đây chính là đỉnh điểm của bi kịch tinh thần trong Chí Phèo. - Quằn quại trong tuyệt vọng, Chí Phèo lại xách dao ra đi. Nhưng hắn không rẽ vào nhà Thị Nở như đã dự định ban đầu (đến đâm chết con đĩ Nở và con khọm già kia) mà đến thẳng nhà Bá Kiến. Trong cơn say, hắn càng thấm thía tội ác của kẻ đã cướp đi hình người và hồn người của hắn. Chí Phèo đã vung lưỡi dao căm thù lên 114 giết chết Bá Kiến và quay lại tự kết liễu cuộc đời. Chí Phèo chết vì không tìm ra lối thoát, vì xã hội không cho hắn sống. Tác phẩm kết thúc, Chí Phèo Chết. Nhưng dường như hiện tượng Chí Phèo - hiện tượng hàng vạn người nông dân lương thiện bị xã hội đẩy vào con đường tha hóa, lưu manh hóa và khi ý thức nhân phẩm trở về thì bị xã hội lạnh lùng cự tuyệt vẫn chưa chấm dứt. Chi tiết cái lò gạch bỏ không, vắng người qua lại hiện ra ở cuối tác phẩm khi Thị Nở nhìn nhanh xuống bụng, bỗng tưởng tượng ra hình ảnh này đã nói với ta điều gì đó. 3. Kết luận Quá trình diễn biến tâm trạng nói trên của Chí Phèo đã làm nổi rõ bi kịch: “Sinh ra là người mà không được làm người”. Qua đó, Nam Cao đã bày tỏ sự cảm thông sâu sắc với khát vọng lương thiện trong con người và sự bế tắc của hiện thực xã hội bấy giờ. Câu 3: Phân tích những nét đặc sắc về nghệ thuật của tác phẩm Chí Phèo Dàn ý: MB: - Gthiệu vài nét về tgiả Nam Cao và sự nghiệp văn học của ông - Gthiệu tác phẩm Chí Phèo và đặc sắc nghệ thuật cảu tác phẩm TB: Đặc sắc nghệ thuật: - NT điển hình hoá của Nam Cao thể hiên qua cách miêu tả, phân tích tâm kí của n/v Chí Phèo, Chí Phèo đại diện cho những ng` nông dân bị bần cùng hoá tới mức đọ tột cùng vừa bị đẩy và con đường lưu manh hoá, tha hoá tớy mức đánh mất cả nhân hình lẫn nhân tính. Tuy nhiên Nam Cao cũng tẩo hình ảnh nhân vật Chí Phèo đọc đáo, tiêu biểu và có thể nói là duy nhất trong nần văn học Việt Nam, điều đó đc thể hiện trong nthuật khắc họa khuôn mặt của Chí - Nam Cao miêu tả nhân vật trong quá trình vận đọng và phát triển của tính cách, của sự phát triển về tâm lí, khiến nhân vật trở nên có sức sống, sinh đônngj và do đó nhân vật trở thành điển hình của văn học. => Thành công trong việc xdựng n/v điển hình mang tính cáh điển hình. - Qua Chí Phèo, Nam Cao rất thành công trong việc tổ chức tác phẩm: + NC tạo nên một cốt truyện có tính kịch tính cao: Cốt truyện đc dẫn dắt bằngcác nút thắt kịch tính để dẫn tớy một kết thúc hợp lí mà về hình thức tưởng chừng đó là 1 kết thúc ngẫu nhiên + Cốt truyện của NC đc đặt trong khung time hiện tại trong đó có sự đảo chiều, có quay ngược time kể, PHần mở đầu và kết thúc thụoc time hiện tại, tức là gắn vs những gì đang diễn ra trc' mắt ng` kể chuyện, twong ứng vs những gì ng` kể chuyện đnag quan sát đc. Phần giữa có sự dảo chiều time, nhân vật người kể chuyện đang về quá khứ để chỉ ra gốc gác của Chí Phèo rồi quay lại kể theo trình tự qkhứ- hiện tại để nối liền mạch kể. Sự thay đổi thời gian kể gắn liền với thay đôi điểm nhìn trần thuật, tạo nên tiếng nói đa âm trong câu chuyện, cụ thể là câu chuyện về cđời chí Phèo ko chỉ đc tái hiện đơn giản qua cách kể, lời kể của n/v ng` kể chuyện mà 115 còn qua điểm nhìn của chính Chí Phèo, Thị Nở, bá Kiến,.... các điểm nhìn này tạo sự đa dạng trong nghệ thuật trần thuật, tạo ra sự phối âm, hoà điệu trong tác phẩm - Nam Cao rất thành công trong việc sử dụng ngôn ngữ kể chuyện, bao gồm ngôn ngữ kể và tả của nhân vật ng` kể chuyện, ngôn ngữ mang tính cá thể và đc cá thể hoá của các nhân vật trong truyện + kết hợp đối thoại và đọc thoại, lời kể gián tiếp + lời kể nửa trực tiếp + Độc thoại mang dấu ấn độc thoịa nội tâm như đoạn Chí Phèo tỉnh rwouj ngồi ôn lại qkhứ, cảnh Bá Kiến ngồi đợi bà Ba + Kiểu đối thoại một chiều mà bên phát tín hiệu thì cứ phát, bên nhận tính hiệu thì ko có phản ứng rep như cảnh Chí Phèo - Thị Nở gặp nhau sau trận ốm - Khi tạo ra các nhân vật, Nam Cao trung thành với nguyên tắc phản ánh hiện thực song nhà văn ko phóng đại cực đoan phần bản năng, thú tính trong con người, không hạ thấp, không xóa bỏ nét đẹp mang tính người của các nhân vật, đây chính là nét đặc sắc trong nghệ thuật vị nhân sinh của Nam Cao. KB: Khẳng định ngòi bút tài hoa của Nam Cao trong tác phẩm Chí Phèo Câu 4: Tư tưởng nhân đạo cuả Nam Cao thể hiện trong khát vọng làm người cuả nhân vật Chí Phèo. Trong truyện , mấy lần Chí Phèo ý thức về nhân phẩm cuả mình ? Diễn biến mỗi lần ? Dàn ý: I. Khái niệm nhân phẩm : - Nhân phẩm là phẩm giá cuả con người , tức là những giá trị cuả một người được xã hội công nhân. Nhân phẩm được đánh giá theo chuẩn mực đạo đức, đồng thời cũng được pháp luật bảo vệ . - Chuẩn mực cuả nhân phẩm là phẩm chất và giá trị cuả người lao động lương thiện.Những kẻ phạm pháp không còn nhân phẩm , người ta có thể bắt giam , đánh đập ,rồi tử hình . Những cô gái mại dâm không còn nhân phẩm phải đưa vào trường phục hồi nhân phẩm giáo dục những phẩm chất và giá trị làm người lương thiện . * Tình trạng nhân phẩm cuả Chí Phèo: - Chí ở tù ra , đã mất một phần nhân phẩm , rồi bị tha hoá mất hẳng nhân hình , nhân tính , bị mọi người coi là “ con quỷ dữ làng Vũ Đại “ à Chí không còn được xã hội coi Chí là một con người . - Thực ra Chí cũng không ý thức về nhân phẩm vì Chí say đến nỗi không còn biết mình có mặt trên đời. Chí sống với mọi người nhưng lại vi phạm vào những giá trị làm người : giá trị đạo đức ,luật pháp ( say rượu ăn vạ, đốt nhà mẹ bán rượu , cưỡng hiếp Thị Nở .. ) II. Tư tưởng nhân đạo cuả Nam cao 116 Thể hiện ở khát vọng làm người lương thiện cuả Chí , mà cốt lõi là ý thức về nhân phẩm.Trong tp , Chí 3 lần ý thức về nhân phẩm : 1. Lần thứ nhất: - Sau khi tỉnh rượu , được Thị Nở cho ăn cháo hành , Chí xúc động vì lần đầu tiên được săn sóc bởi một người đàn bà . Chí nhớ lại việc mình bị bà ba vợ bá Kiến bắt lên bóp chân“ cái con quỷ cái hay bóp hắn bóp chân mà lại bóp lên trên , trên nưã nó chỉ nghĩ đến sao cho thoả nó..Bị một con đàn bà gọi lên bóp chân ! Hắn thấy nhục hơn là thích “ vì “ con vợ chủ sai hắn làm một việc không chính đáng “ , “ thấy hắn dung dằng , bà mắng xơi xới vào mặt , hắn chỉ thấy nhục “ * Nhục là trạng thái nhân phẩm bị xúc phạm , vì phải “ làm những việc không chính đáng”. Công việc cuả Chí là làm canh điền như những người nông dân khác .Chí cũng từng ước mơ “ Hình như có một thời hắn đã ao ước có một gia đình nho nhỏ . Chồng cuốc mướn cày thuê , vợ dệt vải , chúng lại bỏ một con lợn nuôi để làm vốn liếng .Khá giả thì mua dăm ba sào ruộng làm ” - Trạng thái nhục cuả Chí và cử chỉ dùng dằng không làm theo lời bà Ba , giúp người đọc hiểu nhân phẩm cuả Chí lúc ấy là nhân phẩm một người nông dân lương thiện . Chí ý thức rõ về nhân phẩm cuả mình .Cũng vì điều này. Chí bị BK ghen , cho đi ở tù 2.Lần thứ hai: - Sau khi tỉnh rượu , nhìn Thị Nở mang cháo hành đến cho mình , Chí ngạc nhiên . “ hết ngạc nhiên thì hắn thấy mình hình như ươn ướt “ , bởi vì đây là lần thứ nhất Chí được một người đàn bà cho . “Xưa nay hắn chỉ sống bằng giật cướp và doạ nạt “ .” hắn thấy vưà vui vưà buồn . và một cái gì nưã , giống như là ăn năn ..Người ta hay hối hận về tội ác khi không còn đủ sức mà ác nưã “ - Chí ý thức rõ về nhân phẩm cuả mình , bây giờ Chí là kẻ ác , kẻ làm ác , Chí không còn nhân phẩm . Chí ăn năn : đó là trạng thái phục hồi nhân phẩm . “ Chí thèm lương thiện , hắn muốn làm hoà với mọi người biết bao ..Họ sẽ nhận hắn vào cái xã hội bằng phẳng , thân thiện cuả những người lương thiện “ ( cái xã hội mà Chí đã làm xáo trộn, bị mọi người xa lánh ) Chí nghĩ rằng Thị Nở sẽ mở đường cho Chí , hắn nhìn Thị Nở cười mà thấy tự nhiên nhẹ người . - Năm ngày sống với Thị Nở , Chí đã thực sự thực hiện được ướ`c mơ cuả mình : có một gia đình , có nhà có vườn.Chí sống lương thiện như mọi người lương thiện khác. Điều này lộ ra bản chất cuả Chí là người lương thiện , khát vọng cháy bỏng cuả Chí là trở về cuộc đời lương thiện 3.Lần thứ 3 : - Thị Nở đi gặp bà cô . bà bảo thị “ Đàn ông chết hết cả rồi sao , mà lại đâm đầu đi lấy một thằng không cha . Ai lại đi lấy thằng chỉ có một nghề là rạch mặt ra ăn vạ. Trời ơi nhục nhã ơi là nhục nhã “, “ đã nhịn được đến bằng này tuổi thì nhịn hẳn , ai lại đi lấy thằng Chí Phèo “ .Nghe thế thị lộn ruột , Thị chạy sang nhà Chí Phèo , chửu hắn một trận “ trút vào mặt hắn tất cả lời bà cô”, hắn nghĩ ngợi một tí rồi hình như hiểu . Hắn bỗng ngẩn người .Sau khi Thị Nở gạt hắn ngã xuống đất rồi bỏ đi . Hắn uống rượu .”càng uống lại càng tỉnh ra. Tỉnh ra , chao ôi buồn. Hắn cứ 117 thoang thoảng thấy hơi cháo hành.Hắn ôm mặt khóc rưng rức , rồi lại uống , lại uống “ * Tuy Nam Cao không miêu tả trực tiếp Chí đang nghĩ gì , Chí hiểu gì , vì sao Chí khóc , nhưng người đọc hiểu được rằng Chí ý thức rõ về thân phận một hằng không cha không mẹ , chỉ có nghề rạch mặt ăn vạ, con đường nhờ Thị Nở mở đường cho Chí trở về đời sống lương thiện đã đóng lại. Chí thương thân và tuyệt vọng ..Chí ý thức rõ “ không thể là người lương thiện ‘ tức là ý thức mình bị tước đoạt quyền sống làm người lương thiên . - Trong trạng thái bi phẫn , Chí xách dao đi .Ngay cả khi đã chết miện Chí vẫn còn không nguôi khát vọng làm người lương thiện II. Việc miêu tả Chí ý thức về nhân phẩm Trong quá trình miêu tả sự vận động tâm lý cuả Chí , Nam Cao tập trung miêu tả nguyên nhân làm cho Chí bị tha hoá và sáng tạo tình huống Chí gặp Thị Nở , được Thị Nở chăm sóc , tình yêu thương cuả Thị Nở giúp Chí thức tỉnh . Trong quá trình thức tỉnh , điều quan trọng là Chí phục hồi ý thức về nhân phẩm Chính nhờ việc Nam Cao miêu tả Chí ý thức về nhân phẩm mà người đọc nhận ra bản chất cuả Chí là lương thiện. Tình cảnh cuả Chí là đáng thương. Cái bi kịch cuả Chí lộ ra giá trị hiện thực và giá trị nhân đạo cuả tác phẩm . Tiếng nói nhân đạo cuả Nam Cao trong tác phẩm là sâu sắc , vươn tới tầm tư tưởng. Câu 5: Sau khi ở tù về , Chí Phèo đến nhà Bá Kiến mấy lần ? Thuật lại ngắn gọn những gì xảy ra mỗi lần Chí Phèo đến nhà Bá Kiến. Phân tích ý nghiã tư tưởng cuả lần đến sau cùng. Qua đó nếu suy nghĩ về giá trị cuả tác phẩm Dàn ý: I . Những lần Chí Phèo đến nhà Bá Kiến 1.Lần I : - Ở tù ra hôm trước , hôm sau Chí ngồi ở ngoài chợ uống rượu với thịt chó từ trưa đến xế chiều - Hắn xách một cái vỏ chai đến cổng nhà Bá Kiến gọi tên tục cụ Bá ra chửi . “ hắn định đến đây nằm vạ “ , “ Chí nói bới BK : “ Tao chỉ liều chết với bố con nhà mày đấy thôi “ .Bá Kiến mời Chí vào nhà uống nước , xử nhũn với hắn , giết gà mua rượu cho hắn uống , đãi thêm đồng bạc để về uống rượu . - Ở nhà BK ra về , Chí vô cùng hả hê , hắn loạng choạng vưà đi vưà cười . * Ý nghiã : Chí còn tỉnh táo , chỉ mượn rượu để gây sự , Chí nhìn rõ kẻ thù và quyết tâm trả thù . Nhưng Bá Kiến rút kinh nghiệm với Năm Thọ , Binh Chức và đời làm tổng lý cuả hắn để xử nhũn với Chí . Chí không biết , vui vẻ ra về . Bá Kiến hoá giải ý định trả thù cuả Chí. 2.Lần 2 : 118 - Hắn uống rượu được 3 ngày , đến ngày thứ tư thì gây sự với con mẹ hàng rượu rồi đến nhà BK gây sự : hắn nói với BK :”..Đi ở tù còn có cơm mà ăn , bây giờ về làng nước , một thước cắm dùi không có , chả làm gì nên ăn . Bẩm cụ , con lại đến kêu cụ , cụ lại cho con đi ở tù “” .. bẩm cụ không được thì con phải đâm chết dăm ba thằng , rồi cụ bắt con giải huyện “ . - Bá Kiến khích Chí đến đòi nợ Đội Tảo . Chí đòi được nợ cho BK , được BK cắm cho 5 sào vườn ở bãi sông . Chí Phèo bỗng thành ra có nhà. Hồi ấy hắn mới đâu 27,28.Chí Phèo ra về , cái mặt vênh vệnh . Hắn tự đắc : “ Anh hùng làng này cóc thằng nào bằng ta “ * Ý nghiã : Chí trở thành công cụ đắc lực cho bá Kiến , rồi từ đó bao giờ hắn cũng say “ những cơn say cuả hắn tràn cơn này sang cơn khác , thành một cơn dài , mênh mông… Chưa bao giờ hắn tỉnh và có lẽ hắn chưa bao giờ tỉnh táo , để nhớ rằng hắn có ở đời “. Chí trượt dài tên con đường tha hoá , trở thành “ con quỷ dữ làng Vũ Đại ‘Bao nhiêu việc ức hiếp , phá phách , đâm chem. , mưu hại , người ta giao cho hắn làm ! Những việc ấy chính là cuộc đời hắn “ 3.Lần 3 : - Bị Thị Nở chửi rồi gạt cho ngã xuống đất , chí ngẩn người , rồi hiểu ra , chí uống rượu rồi ôm mặt khóc. - Chí xách dao đến nhà BK đòi “ Tao muốn làm người lương thiện “- Không được . Ai cho tao lương thiện.? Tao không thể là người lương thiện nưã ..” Chí Phèo xông vào giết BK rồi tự sát. * Ý nghiã : Chí giết được kẻ thù nhưng bế tắc tự sát . Chí chết trong bi kịch bị tước đoạt quyền làm người lương thiện . Từ đó bật ra giá trị TP : Số phận người nông dân và tội ác cuả giai cấp thống trị trong xã hội cũ. Lòng xót thương cuả NC .Nam cao lên tiếng kêu cứu cho người lao động lương thiện bị tha hoá (Chí ). Cũng bộc lộ bế tắc cuả NC II. Giá trị hiện thực cuả Chí Phèo : - Chí Phèo đã phản ánh chân thật và sâu sắc một mâu thuẫn cuả giai đoạn 30 45 : mâu thuẫn giưã nhân dân lao động và gc thống trị bóc lột. Cần phải có một cuộc CM, đánh đổ gc thống trị , thay đổi xã hội cũ , mới có thể cứu được nhân dân lao động. à Phản ánh hiện thực ,với hi vọng góp phần cải tạo hiện thực . - Chí Phèo phản ánh xã hội cũ với hai đối tượng : giai cấp thống trị ( điển hình là Bá Kiến ) và cuộc sống cuả người nông dân ( điển hình là Chí Phèo ) và cuộc đấu tranh giai cấp quyết liệt giưã người nông dân và gc thống trị à Nam Cao tố cáo tội ác cuả gc thống trị và lên tiếng kêu cứu cho người nông dân + Tố cáo tội ác giai cấp thống trị Bá Kiến , một tên gian hùng : Bốn đời làm tổng lý , già đời đục khoét với rất nhiều thủ đoạn Thâm hiểm. BK dung sự tàn bạo để thống trị và bóc lột.Đẩy người lương thiện vào con đường tội ác , để rồi biến họ thành công cụ tội ác cho hắn , hắn bao che tội ác cho chúng : Sẵn sàng cho đi tù , dung bọn đầu bò để tác oai tác quái “Khi cần chỉ cho nó dăm hào uống rượu là có thể sai nó đến tác hại bất cứ anh nào 119 không nghe mình .. nó lưà đốt nhà hay cho mấy nhát dao . quăng chai rượu lậu , gây sự rồi kêu làng . Có chúng sinh sự thì mới có dịp mà ăn. ..kể ăn thì cũng dễ ăn nhưng không phải hễ mà làm lý trưởng thì cứ việc ngồi mà khoét .Thầy điạ lý bảo đất làng này vào cái thế quần ngư tranh thực .. mồi thì ngon đấy nhưng năm bè bảy mối. Bá Kiến đặc biệt gây tội ác với Năm Thọ , Binh Chức và Chí Phèo : làm tha hoá người lương thiện , biến người nông dân lương thiện thành công cụ tội ác cuả BK . - Phơi bày tình cảnh bi thảm cuả người nông dân: ( Năm Thọ , Binh Chức , Chí Phèo ) Chí Phèo vô cớ bị Bá Kiến cho đi tù. Về làng không thể sống nổi : “ Đi ở tù còn có cơm mà ăn , bây giờ về làng nước , một thước cắm dùi không có , chả làm gì nên ăn . Bẩm cụ , con lại đến kêu cụ , cụ lại cho con đi ở tù” . Nhà tù TD , cùng với tình cảnh khốn cùng , dưới thủ đoạn hiểm độc cuả BK , Chí bị đẩy vào con đường lưu manh. Chí bị tha hoá , bị huỷ hoại cả nhân hình , nhân tính , bị tước đoạt quyền làm người lương thiện để rồi chết quằn quại trên vũng máu , chết trong bi kịch , chết tuyệt vọng. III. Giá trị nhân đạo cuả Chí Phèo - Chia xẻ , cảm thông sâu sắc nỗi thống khổ cuả người nông dân + Nam Cao chia xẻ nỗi nhục nhã khi Chí bị bà ba kêu lên bóp chân , miêu tả tận cùng nỗi đau đớn trong thẳm sâu tâm hồn khi Chí ngồi ôm mặt khóc rưng rức , thương thân , tuyệt vọng , chia xẻ caí khát vọng cháy bỏng muốn làm người lương thiện .. + Thảm cảnh Binh Chức mất vợ - Điều sâu sắc là Nam Cao phát hiện và khẳng định phẩm chất lương thiện cuả Chí và khát vọng muốn làm người lương thiện cuả Chí ngay cả khi Chí đã bị tha hoá mất cả nhân hình và nhân tính - Nam Cao lên tiếng đòi quyền sống cho người lương thiện: + Nam Cao để cho các nhân vật phản kháng tự phát , họ không còn cách nào khác , họ phải liều mạng Nhưng thực ra , họ lại tiếp tục bị giai cấp thống trị tha hoá : Năm Thọ , Binh Chức , Chí Phèo . + Nam Caoo cũng chỉ ra rằng muốn cứu những con người lương thiện ấy thì phải thay đổi cái xã hội tàn ác ấy. Bởi vì trong cái xã hội ấy “ bằng ấy cánh du lại với nhau để bóc lột con em “ còn “ bọn dân hiền lành chỉ è cổ ra làm nuôi bọn lý hào ..” Thái độ cuả Nam Cao là tiến bộ khi đứng về những kiếp lầm than mà lên tiếng, tuy vậy Nam Cao vẫn bế tắc trong giải pháp. Vì vậy chưa vươn tới chủ nghiã nhân đạo cách mạng *Biên soạn câu hỏi và hướng dẫn học sinh ôn tập dạng câu hỏi 5 điểm thông qua nhận định liên quan đến nhân vật trong tác phẩm hoặc tác phẩm. Câu 1: Bình luận về nhân vật Chí Phèo trong tác phẩm cùng tên cuả Nam cao, có nhà phê bình cho rằng : Chí Phèo vưà là một gã mất trí , vưà là đầu óc sáng suốt 120 nhất cuả làng Vũ Đại , ý kiến anh chị thế nào ? Từ truyện ngắn này cuả Nam Cao, hãy làm sáng tỏ ý kiến cuả mình . ( HSG Đồng Nai 08/11/2005) Dàn ý: 1.Chí Phèo là kẻ mất trí - Mất trí + Là mất trí nhớ ( người bị bịnh tâm thần không còn nhận thức được thực tại ) + Mất trí còn là trạng thái mất lương tri, không còn phân biệt được phải trái , đúng sai, nên làm hay không nên làm. Một lúc nào đó , lương tri bị che mờ, con người phạm vào tội ác - Chí Phèo mất trí: + Chí say bất tận, không biết mình có mặt trên cõi đời + Chí giết Bá Kiến và tự sát. Chỉ kẻ mất lương tri mới giết người. + Chỉ kẻ mất trí mới tự sát 2 Chí Phèo là một đầu óc sáng suốt nhất làng Vũ Đại * Muốn nói Chí Phèo là bộ óc sáng suốt nhất làng Vũ Đại, ta phải so sánh Chí với những bộ óc sáng suốt khác.Làng Vũ Đại có nhiều đầu óc sáng suốt : - Lúc Chí chửi: mọi người đều nghĩ nó chưà mình ra, không ai gây sự với Chí. Đó là sự không ngoan. Dây với thằng say chỉ thiệt - Những người đến xem cái chết cuả hai nhân vật, có người cho rằng, chính chúng nó giết nhau. Bá Kiến bị chính tay sai cuả mình giết. Chơi dao đứt tay. Ác giả ác báo.Người khác bình luận : tre già măng mọc.Bà cô Thị Nở chỉ vào mặt con cháu : Phúc đức con nhá , không ôm lấy chân ông Chí .Nếu ôm lấy Chí thì giờ thành goá phụ. Thị Nở nghĩ, sao nó hiền như đất ( biết phân biệt với ý kiến mọi người ) - Bá Kiến khôn róc đời trong việc nhận định đối phó kẻ thù và mưu đồ thống trị, nhưng Bá kiến chết vì chính tội ác cuả mình trung lúc không còn đủ bình tĩnh * Chí Phèo sáng suốt nhất - Cái sáng suốt cuả dân làng là sáng suốt cầu an, bảo thủ, không dám đấu tranh với cái xấu, cái ác, cam chịu thân phận nô lệ. Cái sáng suốt cuả Bá Kiến là sáng suốt trong mưu đồ thống trị bóc lột. Bá Kiến không hiểu được sức mạnh cuả lương tri và lẽ thiện, phản ứng cuả con người bị dồn đến chân tường - Chí ý thức sâu sắc về nhân phẩm , về giá trị làm người, về khát vọng sống lương thiện. Lúc bị bà Ba kêu lên bóp chân, Chí chỉ thấy nhục và biết mình làm một việc không chính đáng. Chí khát khao trở về đời sống lương thiện - Chí nhận ra kẻ thù và tiêu diệt được kẻ thù.kẻ thù. Chí đứng trên lập trường lẽ thiện ( tao muốn làm người lương thiện ) để kết tội Bá Kiến và xử tội bá Kiến.hành động cuả Chí trong xã hội người lương thiện không được bảo vệ là hành động đúng. Bá Kiến không hiểu được sức mạnh cuả chân lý lẽ thiện * Nam cao xây dựng nhân vật Chí Phèo với mục đích gì ? Có phải chỉ để nói về Chí vưà mất trí vưà sáng suốt không ? 121 - Nhận định Chí vưà mất trí , vưà sáng suốt chỉ là nhận định về những biểu hiện bên ngoài căn cứ vào hành động cuả Chí. Nhật định ấy tạo ra sự mâu thuẫn để gây rồi trí người đọc, để người đọc tập trung vào tìm hiểu Chí - Thực ra Chí là người nông dân lương thiện, bị Bá Kiến làm tha hoá biến thành công cụ tội ác, Nhờ tình yêu cuả thị Nở , chí thức tỉnh và khao khát trở về cuộc sống lương thiện, nhưng chí lâm vào bi kịch và chết trên ngưỡng cưả cuộc đời. à Qua Chí, Nam cao tố cáo tội ác cuả giai cấp đị chủ phong kiến, nói lên tiếngnói nhân đạo kêu cứu cho người lao động lương thiện.Chí trở thành điển hình cho số phận người nông dân bị giai cấp thống trị làm tha hoá. Chí trở thành nhân vật tư tưởng thể hiện giá trị hiện thực và giá trị nhân đạo sâu sắc cuả tác phẩm. Nhận định Chí vưà mất trí vưà sáng suốt không giúp nhân ra giá trị thực cuả hình tượng Chí, nhưng có thể gợi ra những suy nghĩ để tìm hiểu nhân vật này KL : Chí Phèo là một hình tượng tư tưởng đặc sắc cuả Nam Cao Câu 2: “ Tình yêu Thị Nở chẳng những đã thức tỉnh Chí Phèo mà còn hé mở cho anh con đường trở lại làm người trở lại cuộc đời và anh hồi hộp hy vọng ‘. Phân tích mối tình Thị Nở - Chí Phèo để làm sáng tỏ nhận định trên. Dàn ý: I.Tình trạng cuả Chí trước khi gặp Thị Nở : - Không thức tỉnh: Chí say bất tận “ chưa bao giờ hắn tỉnh và có lẽ hắn chưa bao giờ tỉnh táo , để nhớ rằng có hắn ở đời “ Chí gây ra bao nhiêu tội ác trong lúc say ; “ hắn biết đâu hắn đã phá bao nhiêu cơ nghiệp , đập nát bao nhiêu cảnh an vui , đạp đổ bao nhiêu hạnh phúc , làm chảy máu và nước mắt cuả bao nhiêu người lương thiện. Hắn biết đâu hắn làm tất cả những việc ấy trong khi người hắn say “ ( 33) . Chí không hề có ý thức cuả một con người - Bị tha hoá :Chí là con vật lạ , là con quỷ dữ làng Vũ Đại , bị mọi người xa lánh . - Tuyệt vọng: Cuộc đời Chí cứ tuột dốc trên con đường tội ác và tha hoá không sao dừng lại được. Đó là sự tuyệt vọng . Chỉ khi thức tỉnh Chí mới nhận ra ( mất quà khứ , hiện tại là kẻ ác , tương lai cô độc , bệnh tật ) II. Tình yêu cuả Thị Nở : - Gặp gỡ Thị Nở - Chí Phèo: Chí uống rượu say ở nhà Tự Lãng . Trên đường về , Chí gặp TN đang ngủ dưới gốc cây chuối . Thị đi lấy nước ở ngoài sông . Trăng thanh gió mát làm thị ngủ quên. Tai nạn xảy ra với thị. Nưả đêm Chí nôn thốc nôn tháo , TN đưa Chí vào lều , nấu cháo cho Chí ăn. Chí tỉnh rượu , vui đuà với Thị . Chí rủ thị sang ở chung. TN 122 sang nhà Chí ở , làm thành một cặp xứng đôi vưà lưá , được 5 ngày , TN nghe lời bà cô từ chối tình yêu cuả Chí . Chí ôm mặt khóc. - Tình yêu cuả Thị Nở: Gặp gỡ TN,CP khởi đầu là một hành vi bản năng cưỡng bức, nó trở thành tình nghiã .” cái thằng liều lĩnh ấy kể ra thì đáng thương , còn gì đáng thương bằng đau ốm mà nằm còng queo một mình . Giả thử đêm qua không có thị thì hắn chết. Thị thấy như là yêu hắn : đó là một cái lòng yêu cuả người làm ơn. Nhưng cũng có cả lòng yêu cuả người chịu ơn . Một người như thị Nở càng không thể quên được. Cho nên thị nghĩ : bỏ hắn lúc này thì cũng bạc . Dù sao cũng đã ăn nằm với nhau “ Tình nghiã ấy hướng dẫn toàn bộ hành động thị và biểu hiện bằng việc nấu cháo hành cho Chí ăn Ngồi chăm sóc Chí lúc Chí ăn , đáp lại những nguyện vọng yêu cầu cuả Chí .Tô cháo hành kết tinh tất cả tình nghiã ấy . Vì thế khi bị Thị Nở từ chối tình yêu , Chí thấy thoang thoảng mùi cháo hành , tức là tình nghiã , hạnh phúc và tất cả ước mơ cuả Chí đã mất . - Tình yêu cuả Thị Nở làm Chí thức tỉnh: + Tỉnh rượu nhờ ăn cháo hành ( mà cháo hành là tình nghiã cuả Thị Nở ) Chí vã mồ hôi từng giọt , “ hắn thấy long thành trẻ con , Hắn muốn làm nũng với thị như với mẹ “ con người khao khát tình thương yêu. + Thức tỉnh cảnh ngộ Nghe tiếng cuộc sống xung quanh , Chí nhận ra mình mất quá khứ , “ hình như có một thời hắn đã ao ước có một gia đình nho nhỏ . Chồng cuốc mướn cày thuê , vợ dệt vải , chúng lại bỏ một con lợn nuôi để làm vốn liếng. Khá giả thì mua dăm ba sào ruộng “ ( 43) Thực tại và tương lai đen tối: “Tỉnh dậy hắn thấy hắn già mà vẫn còn cô độc.. tuổi già cuả hắn .đói rét và ốm đau và cô độc , cái này còn đáng sợ hơn đói rét và ốm đau ..và có lúc hắn ngẫm mình mà lo .. hắn mơ hồ rằng sẽ có một lúc người ta không thể liều được nưã “ + Thức tỉnh nhân phẩm Thị Nở cho Chí ăn cháo, Chí khóc và ăn năn, vì Chí đã sống ác.” Xưa nay hắn chỉ sống bằng cườp giật và doạ nạt “. Chí phục hồi nhân phẩm dần dần Thị Nở cho Chí ăn Cháo, Chí nhớ đến việc bà ba bắt hắn bóp đùi chỉ thấy nhục vì Chí phải làm một việc không chính đáng “. Đó là bản chất lương thiện cuả Chí.Từ đó chí them lương thiện, Chí khao khát được sống lương thiện. + Hi vọng Từ khát vọng lương thiện , Chí nghĩ Thị Nở có thể giúp Chí : “ Thị Nở sẽ mở đường cho hắn , thị có thể sống yên ổn với hắn thì sao người khác lại không thể được. Họ sẽ thấy rằng hắn cũng có thể không làm hại được ai. Họ sẽ nhận hắn vào cái xã hội bằng phẳng , thân thiện cuả những người lương thiện “ Nghĩ vậy Chí băn khoăn nhìn thị. Thị Nở cười, Chí thấy nhẹ người. ( có lẽ Chí tưởng Thị Nở hiểu và nhận lời giúp hắn )Chí hành động : mở lời với thị : “ Giá cứ thế này mãi thì thích nhỉ “ tức là được sống bình an , được yêu thương và chăm sóc. Chí bước một bước 123 nưã :” Hay là mình sang đây ở với tớ một nhà cho vui “ . Rồi Chí cười khanh khách, lòng hắn rất vui, rồi lại cười ngất khi làm thị thẹn thùng..Chí tỏ tình - Thị Nở mở đường : Thị sang sống chung với Chí. Thị sống với Chí được thì mọi người cũng sẽ sống với Chí được.Đó là bước thứ nhất trên con đường trở về. Thị sẽ là người bảo lãnh cho Chí với mọi người. Chí đã sống được năm ngày lương thiện, tỉnh táo và thực hiện được ước mơ cuả mình : có một gia đình ..Nếu mọi việc cứ vậy mà tiến triển con đường trở về cuả Chí sẽ tốt đẹp.Việc hôn nhân còn nhiều điều ràng buộc, Thị không thể tự quyền mình quyết định, vì còn bà cô. Đúng như thực tế, bà cô TN đã bác hẳn việc TN lấy Chí. Thị không được lấy ,” một thằng không cha ,. Một thằng chỉ có nghề rạch mặt ăn vạ.” điều ấy là nhục nhã giòng họ …Lúc đầu Chí ngẩn người rồi kinh ngạc và hiể ra. Chí ôm mặt khóc. - Giá trị tình yêu Thị Nở với tác phẩm : Đọc Chí Phèo , người đọc nhận ra sự biến đổi này ở nhân vật. + Biến cái xấu thành cái đẹp. Trongmắt Chí , TN xâú ma chê quỷ hờn thành ra là người“ có duyên “, “ tình yêu làm cho người ta có duyên “ + Biến cái ác thành cái thiện. Chí làm ác, “bao nhiêu việc ức hiếp, phá phách, đâm chém,mưu hại, người ta giao cho hắn làm.. Hắn phá bao nhiêu cơ nghiệp, đập nát bao nhiêu cảnh yên vui, đạp đổ bao nhiêu hạnh phúc...” giờ khao khát lương thiện , “hắn thấy lòng thành trẻ con”.”Hắn thèm lương thiện, hắn muốn làm hòa với mọi người” + Biến hai con vật thành người .Chí là con vật lạ , là con quỷ dữ làng Vũ Đại, còn TN bị xa lánh như xa lánh một con vật ghê tởm.Họ đã sống bên nhau như mọi người lương thiện, như một gia đình hạnh phúc mà Chí hằng ước mơ. Chí không còn uống rượu, không còn gây sự, không còn phạm tội ác. + Tình yêu TN làm lộ ra giá trị nhân đạo sâu sáccuả tác phẩm , giúp Chí thức tỉnh khát vọng sống lương thiện .TP vang lên tiếng nói khẳng định Chí là người lương thiện , là đưá hiền như đất, lộ ra bi kịch bị từ chối quyền làm người và tiếng nói kêu cứu cho Chí Phèo. Câu 3: Nói về đoạn đời của Chí Phèo sau đêm gặp thị Nở, nhà nghiên cứu văn học Chu Văn Sơn cho rằng: Tuy chỉ có năm ngày ngắn ngủi, nhưng nó thật sự là một quãng đời khác: Chí được sống rồi chết như một con người. (Chu Văn Sơn- Bình giảng tác phẩm văn học 11, Nxb Giáo dục, 1999). Từ cảm nhận của anh/chị về nhân vật Chí Phèo trong truyện ngắn cùng tên của Nam Cao, hãy làm sáng tỏ ý kiến trên. Dàn ý: 1.Vài nét về tác giả, tác phẩm 124 - Nam Cao là một nhà nhân đạo lớn, một ngòi bút hiện thực xuất sắc, một bậc thầy về nghệ thuật truyện ngắn; các tác phẩm của ông mang ý nghĩa triết lí nhân sinh sâu sắc. - Chí Phèo (1941) là đỉnh cao trong sự nghiệp của Nam Cao. Qua tác phẩm, nhà văn không chỉ thể hiện sâu sắc bi kịch tha hóa của người nông dân mà còn thể hiện xúc động quá trình thức tỉnh về nhân phẩm và quyền sống của họ. 2.Giải thích ý kiến - Tuy chỉ có năm ngày ngắn ngủi là nhận xét về mối tình giữa Chí Phèo với thị Nở diễn ra trong một thời gian ngắn so với quãng đời dằng dặc bóng tối, tội ác mà Chí Phèo đã sống. - Nhưng nó thật sự là một quãng đời khác là đánh giá về ý nghĩa của mối tình Chí Phèo với thị Nở. Mặc dù, mối tình ấy diễn ra trong một thời gian ngắn, nhưng chính tình yêu thương mộc mạc, chân thành của người đàn bà khốn khổ đã giúp Chí Phèo thức tỉnh, được sống với những cảm xúc nhân tính và chết như một con người có ý thức về nhân phẩm, giá trị, quyền sống, quyền làm người. 3. Cảm nhận về nhân vật Chí Phèo làm sáng tỏ ý kiến - Trước khi gặp thị Nở: Chí Phèo là người nông dân hiền lành, lương thiện nhưng đã bị tha hóa trở thành quỷ dữ, sống triền miên trong bóng tối, tội ác. - Ý nghĩa mối tình giữa Chí Phèo với thị Nở: + Chí Phèo được sống như một con người: Chí Phèo nhận ra những âm thanh quen thuộc của cuộc sống thường nhật; nhớ lại quá khứ, suy nghĩ về hiện tại và lo lắng cho tương lai. Được thị Nở chăm sóc, Chí Phèo khao khát hạnh phúc gia đình, khao khát được hoàn lương. + Chí Phèo được chết như một con người: Bị thị Nở cự tuyệt, Chí Phèo hiểu ra nguyên nhân sâu xa dẫn đến bi kịch bị cự tuyệt quyền làm người. Chí nhận ra kẻ thù thực sự của đời mình, dõng dạc đòi quyền sống, kết liễu kẻ thù và tự sát vì bản tính lương thiện trong con người đã trở lại và Chí không thể tiếp tục sống cuộc đời thú vật như trước đây. 4.Đánh giá - Diễn tả quá trình thức tỉnh của Chí Phèo, Nam Cao đã đi sâu vào thế giới tâm hồn nhân vật với những diễn biến tinh tế, phong phú. Ngôn ngữ đa thanh, đa giọng điệu, chi tiết giàu kịch tính. - Xây dựng chi tiết mối tình Chí Phèo, thị Nở, Nam Cao thể hiện lòng yêu thương trân trọng con người và niềm tin vào bản chất tốt đẹp của người nông dân Việt Nam. *Biên soạn câu hỏi và hướng dẫn học sinh ôn tập dạng câu hỏi so sánh(5 điểm) liên quan đến nhân vật trong tác phẩm hoặc giá trị tác phẩm Câu 1: Truyện ngắn Chí Phèo của Nam Cao kết thúc bằng hình ảnh: Đột nhiên thị thấy thoáng hiện ra một cái lò gạch cũ bỏ không, xa nhà cửa, và vắng người lại qua… 125 (Ngữ văn 11, Tập một, NXB Giáo dục Việt Nam, 2011, tr.155) Truyện ngắn Vợ nhặt của Kim Lân kết thúc bằng hình ảnh: Trong óc Tràng vẫn thấy đám người đói và lá cờ đỏ bay phấp phới… (Ngữ văn 12, Tập hai, NXB Giáo dục Việt Nam, 2011, tr.32) Cảm nhận của anh/ chị về ý nghĩa của những kết thúc trên. ( ĐH khối D – 2012 ) Dàn ý: 1. Vài nét về tác giả, tác phẩm - Nam Cao là một nhà nhân đạo lớn, một ngòi bút hiện thực xuất sắc, một bậc thầy về nghệ thuật truyện ngắn; sáng tác mang triết lí nhân sinh sâu sắc. Chí Phèo là đỉnh cao trong sự nghiệp của Nam Cao; truyện có kết thúc độc đáo, tô đậm được chủ đề tư tưởng của tác phẩm. - Kim Lân là nhà văn có sở trường về truyện ngắn; chuyên viết về nông thôn và đời sống của người dân nghèo với ngòi bút đôn hậu và hóm hỉnh. Vợ nhặt là truyện ngắn tiêu biểu của Kim Lân; kết thúc truyện đặc sắc, khắc sâu được chủ đề tư tưởng của tác phẩm. 2. Về ý nghĩa của kết thúc truyện ngắn Chí Phèo - Ý nghĩa nội dung : + “Cái lò gạch cũ” vốn là nơi Chí Phèo bị bỏ rơi lúc lọt lòng, giờ đây khi Chí Phèo vừa chết lại xuất hiện trong ý nghĩ của thị Nở ở kết thúc truyện, đã gợi ra được sự quẩn quanh, bế tắc trong tấn bi kịch tha hóa và bị cự tuyệt quyền sống lương thiện của người nông dân. + Kết thúc truyện thể hiện tư tưởng nhân đạo sâu sắc của Nam Cao: đồng cảm với nỗi thống khổ của người nông dân dưới ách thống trị tàn bạo của bọn địa chủ phong kiến, trân trọng khát vọng được sống lương thiện của họ. - Ý nghĩa nghệ thuật : + Truyện kết thúc bằng cách lặp lại hình ảnh ở phần mở đầu tạo nên kết cấu đầu cuối tương ứng gợi ra vòng tròn luẩn quẩn của thân phận Chí Phèo, giúp tô đậm chủ đề tư tưởng: cuộc đời Chí Phèo tuy kết thúc nhưng tấn bi kịch Chí Phèo sẽ vẫn còn tiếp diễn. + Kết thúc truyện vừa khép vừa mở dành nhiều khoảng trống cho người đọc tưởng tượng và suy ngẫm, tạo ra được dư âm sâu bền đối với sự tiếp nhận 3. Về ý nghĩa của kết thúc truyện ngắn Vợ nhặt - Ý nghĩa nội dung : + Hình ảnh “đám người đói và lá cờ đỏ” hiện lên trong tâm trí Tràng vừa gợi ra cảnh ngộ đói khát thê thảm vừa gợi ra những tín hiệu của cuộc cách mạng, cả hai đều là những nét chân thực trong bức tranh đời sống lúc bấy giờ. + Kết thúc truyện góp phần thể hiện tư tưởng nhân đạo của Kim Lân: trân trọng niềm khát vọng sống ngay bên bờ vực cái chết của người lao động nghèo; niềm 126 tin bất diệt vào tương lai tươi sáng. - Ý nghĩa nghệ thuật : + Hình ảnh dùng để kết thúc truyện là triển vọng sáng sủa của hiện thực tăm tối, đó là tương lai đang nảy sinh trong hiện tại, vì thế nó quyết định đến âm hưởng lạc quan chung của câu chuyện. + Đây là kiểu kết thúc mở giúp thể hiện xu hướng vận động tích cực của cuộc sống được mô tả trong toàn bộ câu chuyện; dành khoảng trống cho người đọc suy tưởng, phán đoán. 4. Về sự tương đồng và khác biệt của hai kết thúc truyện a. Điểm tương đồng - Hai kết thúc truyện cùng phản ánh hiện thực tăm tối của con người trước Cách mạng tháng Tám - Góp phần thể hiện tư tưởng nhân đạo của mỗi nhà văn - Đều là những kết thúc có tính mở, giàu sức gợi b. Điểm khác biệt: - Kết thúc: + Truyện Chí Phèo phản ánh hiện thực luẩn quẩn, bế tắc của người nông dân lao động, được thể hiện qua kết cấu đầu cuối tương ứng hàm ý tương lai sẽ chỉ là sự lặp lại của hiện tại + Truyện Vợ nhặt phản ánh xu hướng vận động tất yếu của số phận con người, được thể hiện qua kết cấu đối lập hàm ý tương lai sẽ mở lối cho hiện tại. - Bút pháp: + Nam Cao viết theo khuynh hướng hiện thực phê phán + Kim Lân viết theo khuynh hướng hiện thực xã hội chủ nghĩa. III. Kết luận + Khắc họa chân thực, sinh động đời sống đáng thương của nhân dân ta. + Tố cáo xã hội sâu sắc + Tấm lòng của nhà văn + Tài năng trong sáng tạo qua những hình ảnh giàu ý nghĩa góp phần nổi bật tư tưởng chủ đề nhân đạo của tác phẩm. Câu 2: Chí Phèo và Vợ Nhặt đều viết về tình cảnh người nông dân trước cách mạng tháng 8 /1945. Anh chị hãy : a. Phân tích những khám phá riêng cuả mỗi tác giả về số phận và cảnh ngộ cuả người nông dân trong từng tác phẩm b.Chỉ ra sự khác nhau trong cách kết thúc cuả hai thiên truyện . Giải thích vì sao có sự khác nhau ấy . Nêu ý nghiã cuả mỗi cách kết thúc 127 c. Phân tích nét đặc sắc trong tư tưởng nhân đạo cuả mỗi tác phẩm ( ĐHSP Hanoi . khối C, N – 2001-2002 ) Dàn ý: I. Giới thiệu khái quát về Nam Cao và truyện ngắn Chí Phèo ,Kim Lân với truyện Vợ Nhặt II Những khám phá riêng cuả mỗi tác giả : 1. Nam Cao trong Chí Phèo : - Cảnh ngộ: ( Cảnh ngộ là hoàn cảnh đang gặp phải – Trong tác phẩm , từ lúc Chí xuất hiện đến khi chết là tình trạng huỷ hoại cả nhân hình nhân tính ) Chí bị BK đẩy vào tình trạng tha hoá , bị huỷ hoại cả nhân hình , nhân tính, trở thành công cụ tội ác cho BK . Nhờ Thị Nở , Chí thức tỉnh và hy vọng , nhưng tuyệt vọng trong bi kịch bị tước đoạt quyền làm người lương thiện . - Số phận cuả Chí: Chí là một đưá trẻ mồ côi , lớn lên làm canh điền , là người nông dân lương thiện , bị Bá Kiến cho đi tù , ra tù không sống nổi , thành công cụ tội ác cho Bá Kiến , chết trong bi kịch .Đó là một số phận khốn khổ , bi thảm , bị thống trị bóc lột , bị huỷ hoại nhân tính , bị tha hóa , bị tước đoạt quyền làm người lương thiện . Số phận Chí phản ánh một bộ phận nông dân trước cách mạng tháng Tám bị đẩy vào con đường lưu manh hoá , bị tha hoá , bế tác. 2. Kim Lân trong Vợ Nhặt : - Cảnh ngộ : người nông dân trong thảm trạng chết đói 1945 , lại phải cưu mang thêm miệng ăn . Một nỗi âu lo bao trùm .Cái đói huỷ hoại mọi giá trị nhân phẩm . - Số phận : Nghèo khó suốt đời , ngụ cư , hậu quả cuả thống trị bóc lột : “ đằng nó bắt giồng đay , đằng nó bắt đóng thuế “ .Tràng không lấy nổi vợ. Người vợ nhặt không gia cư , thất nghiệp , đói tơi tả , theo không Tràng về làm vợ . nhưng số phận cuả họ mở ra , Tràng hướng về Việt Minh III. Sự khác nhau trong cách kết thúc : - Cách kết thúc: + Nam cao.kết thúc Chí Phèo là cảnh người ta bàn tán về cái chết cuả Chí Phèo và Bá Kiến, Thị Nở nhìn xuống bụng.Thị thấy thấp thoáng cái lò gạch cũ bỏ không và vắng người qua lại + Kim Lân kết thúc bằng chi tiết : trong óc Tràng hiện lên hình ảnh đoàn người đi phá kho thóc Nhật . Hình ảnh này train ngược với cảnh lo âu và chết đói trong truyện - Giải thích : Do hoàn cảnh sáng tác và phương pháp sáng tác : + Chí Phèo được viết (1940, in 1941) trước 1945, tình cảnh xã hội Việt Nam đang bị thực dân phong kiến thống trị , cuộc sống nhân dân lao động tăm tối . Phương 128 pháp sáng tác cuả Chí Phèo là phương pháp hiện thực phê phán , nhà văn miêu tả bề trái hiện thực nhằm mục đích phê phán xã hội + Vợ Nhặt được viết sau 1945, khi nhân dân lao động đã được giải phóng. Phương pháp sáng tác là phương pháp hiện thực xã hội chủ nghĩa , nhà văn miêu tả hiện thực cách mạng kết hợp với lãng mạn cách mạng , chỉ ra hướng đi lên cuả xã hội. - Ý nghiã mỗi cách kết thúc + Chí Phèo kết thúc tạo nên kết cấu vòng tròn , thể hiện sự bế tắc cuả số phận người nông dân , đồng thời cho thấy hiện tượng Chí Phèo vẫn tồn tại trong xã hội cũ . + Kết thúc Vợ Nhặt mở ra hướng giải thoát số phận các nhân vật . Chỉ ra con đường sống cuả nhân dân lao động là đi theo cách mạng. IV. Tư tưởng nhân đạo cuả mỗi tác giả : Cả hai tác phẩm đều yêu thương , cảm thông người lao động , đều khẳng định những phẩm chất khát vọng cuả họ , đều lên tiếng tố cáo sự chà đạp lên nhân phẩm..Nhưng mỗi tác phẩm có đặc sắc riêng. Nam Cao : - Chí Phèo là tiếng kêu cứu cho người lao động lương thiện đang bị chà đạp nhân tính ,bị tước quyền sống làm người lương thiện . à NC chỉ rõ chính xh cũ , chính giai cấp thống trị bóc lột là kẻ gây ra tội ác đối với người lao động , đấu tranh tự phát như Chí không thay đổi được gì .Tre gia măng mọc.Cần phải thay đổi xh cũ. - Kim Lân : Lòng nhân ái, có tình người là có tất cả : “ Tôi muốn độc giả thấy dù hoàn cảnh thế nào đi nưã thì tình người vẫn vượt lên tất cả. Có tình người là có cuộc sống , có tình người là có hy vọng tương lai..Đó là chủ đề , là bản chất nhân đạo ..( Kim Lân , Văn Nghệ Trẻ 26/3/2000 ) Câu 3:Nét riêng trong cảm hứng nhân đạo thể hiện qua hai truyện ngắn “Hai đứa trẻ” của Thạch Lam và “Chí Phèo” của Nam Cao. Dàn ý: 1.Giới thiệu chung: + Thạch Lam là cây bút truyện ngắn xuất sắc trước cách mạng tháng Tám 1945. “Hai đứa trẻ” là một trong những truyện ngắn đặc sắc nhất của Thạch Lam. Tác phẩm được in trong tập “Nắng trong vườn”. Cũng như nhiều truyện ngắn khác của tập truyện, “Hai đứa trẻ” thể hiện cảm hứng nhân đạo rõ nét. + Nam Cao là nhà văn hiện thực tiêu biểu của văn học Việt Nam hiện đại. Tác phẩm của ông chân thực và thấm đượm triết lý nhân sinh.. “Chí Phèo” là truyện ngắn xuất sắc tiêu biểu cho sáng tác của Nam Cao trước cách mạng tháng Tám có giá trị nhân đạo sâu sắc. 129 2. Nét riêng trong cảm hứng nhân đạo trong truyện ngắn “Hai đứa trẻ” cuả Thạch Lam và “Chí Phèo” của Nam Cao a. Hai đứa trẻ (Thạch Lam) - Thể hiện niềm xót thương của nhà văn với những kiếp người nghèo khổ, sông mòn mỏi, lay lắt nơi phố huyện nghèo (Mấy đứa trẻ, mẹ con chị Tí, Bác Xẩm … và chị em Liên) - Phát hiện những phẩm chất tốt đẹp của người dân nghèo ở phố huyện (Cần cù, chịu thương chịu khó, giàu lòng yêu thương …) - Trân trọng ước mơ nhỏ bé, bình dị của người dân nghèo về một cuộc sống tươi sáng, có ý nghĩa hơn. b. Chí phèo (Nam Cao) - Lên án xã hội tàn bạo làm tha hoá con người, chặn mọi đường sống của người dân vô tội. - Thể hiện niềm thông cảm trước số phận bi kịch của con người bị xã hội tước quyền sống, quyền hạnh phúc, đấy vào con đường lưu manh tội lỗi. - Khẳng định bản chất lương thiện, khát vọng hoàn lương của họ. c. Mỗi tác giả thể hiện cảm hứng nhân đạo bằng nghệ thuật độc đáo - “Hai đứa trẻ” (Thạch Lam) không có cốt truyện, nhân vật không nhiều, biến cố ít …, diễn tả nội tâm sâu sắc, giọng điệu nhẹ nhàng, sâu lắng … - “Chí Phèo” (Nam Cao): Bút pháp hiện thực sắc sảo, miêu tả nội tâm nhân vật sâu sắc, giọng văn lạnh lùng như dồn nén cảm xúc. 3. Đánh giá chung: Thạch Lam và Nam Cao thể hiện cùng đề tài, cùng có cảm hứng nhân đạo nhưng bằng phong cách riêng đã tạo ra hai tác phẩm độc đáo- “Hai đứa trẻ” và “Chí Phèo” - làm giàu có nền văn học Việt Nam trước cách mạng tháng Tám. Để giúp học sinh ôn thi đại học đối với tác phẩm Chí Phèo con có thể có nhiều cách ôn tập khác, có thể biên soạn được nhiều câu hỏi, cách hỏi…Trong phạm vi chuyên đề nhỏ này tôi đã chia sẻ cùng các đồng nghiệp một số kinh nghiệm của cá nhân tôi trong quá trình giảng dạy, hướng dẫn học sinh ôn thi đại học mà tôi đúc rút được. Rất mong nhận được sự đóng góp ý kiến của các đồng chí. Tôi xin chân thành cảm ơn các quí vị đại biểu, các thầy giáo, cô giáo đã chú lắng nghe những chia sẻ của tôi trong bài viết này. Cuối cùng tôi xin gửi tới quí thầy cô lời chúc sức khỏe, hạnh phúc, thành công! Xin trân trọng cảm ơn! Vĩnh Yên ngày 5/3/2014 Người viết chuyên đề Lê Thu Hà Giáo viên trường THPT Vĩnh Yên 130 “DẠY KỸ NĂNG LÀM VĂN –MỘT GIẢI PHÁP QUAN TRỌNG GIÚP HỌC SINH LỚP 12 THI ĐẠI HỌC ĐẠT KẾT QUẢ CAO.” - Người viết: Nguyễn Văn Lự - Giáo viên Ngữ văn, trường THPT Vĩnh Yên, TP Vĩnh Yên, tỉnh Vĩnh Phúc - Đối tượng học sinh bồi dưỡng: lớp 12. - Số tiết bồi dưỡng phần kỹ năng làm văn: 24 trong 80 ca A. ĐỀ CƯƠNG BÁO CÁO THAM LUẬN Báo cáo tham luận gồm bốn phần: I. Vài nét về công tác ôn thi đại học, cao đẳng (ÔTĐH,CĐ) Ngữ văn Lớp 12. II. Những giải pháp giúp học sinh lớp 12 ÔTĐH, CĐ môn Ngữ văn hiệu quả. III. Những kết quả triển khai chuyên đề tại trường THPT Vĩnh Yên, Vĩnh Phúc. IV. Một vài kiến nghị. B. NỘI DUNG I. VÀI NÉT VỀ CÔNG TÁC ÔN THI ĐẠI HỌC MÔN NGỮ VĂN LỚP 12. 1. Phương pháp dạy kỹ năng làm văn chưa được quan tâm đúng mức - Hiện nay, phương pháp dạy Ngữ văn ở trường phổ thông, phần lớn các thầy, cô giáo tập trung giảng bình tác phẩm, đoạn trích theo cách áp đặt cảm nhận của mình hoặc theo tài liệu giảng dạy của Bộ Giáo dục và Đào tạo (Bộ GDĐT). Chúng ta ít chú trọng dạy học sinh (HS) hình thành các kĩ năng khám phá, tìm hiểu và cảm nhận chủ động theo năng lực và mục đích đọc văn của người học. Học sinh hiểu theo kiến giải của thầy, theo tài liệu nên dễ quên, không để lại trong tâm tưởng những rung cảm hay ấn tượng từ hình tượng văn học trong tác phẩm. Giờ học văn và viết văn trở nên nhàm chán và nặng nề trong tâm trí học trò là câu chuyện không ai muốn nhắc lại. - Trong phân môn Làm văn, giáo viên bộ môn quan tâm dạy lý thuyết kiểu bài, ra đề học sinh làm. Các giờ thực hành, luyện tập kỹ năng viết đoạn, phân tích đề, lập dàn bài, nhất là giờ trả bài chưa được thực hành nghiêm túc và hiệu quả. Học sinh làm theo, viết theo những gì đã học, đã có trong tài liệu mà chưa dám bày tỏ hiểu biết mới theo chủ quan. Không dám sáng tạo, không dám phát hiện nên có thể tạo ra lối nghị luận thiên về học thuộc, sao chép và nhất nhất theo thầy dạy. - Từ năm 2010, việc Bộ GDĐT thay đổi căn bản đề Ngữ văn Đại học, Cao đẳng (ĐH, CĐ) gồm ba câu, trong đó, nghị luận xã hội (NLXH) chiếm 30 % điểm bài và nghị luận văn học (NLVH) chiếm 70% điểm bài, theo cấu trúc ngày một thoát li văn mẫu học thuộc, học tủ, học đối phó, thoát dần lối học đọc chép. - Làm văn nghị luận văn học và nghị luận xã hội theo một số đề bài nhất định, dùng những đề có sẵn, những ý kiến có sẵn làm cho tư duy nghị luận gò bó, xáo mòn, thiếu tranh luận, phản bác hay nêu ý riêng. Những vấn đề mới, kiến giải mới gắn với thực tế, gần gũi với trình độ học trò, đôi khi trái ngược với cách hiểu trong đáp án, vẫn chưa được thầy cô chấp nhận. Một số giám khảo, do nhiều nguyên nhân, không đánh giá đúng và khách quan bài làm của thí sinh theo đề bài mở. Cũng còn không ít giáo viên xem nhẹ câu chữ và kỹ năng viết nghị luận của thí sinh, quan trọng hóa độ dài của bài mà không 131 coi trọng kiến giải và văn phong đã dẫn đến tệ trạng học sinh viết càng dài càng tốt. Bài nghị luận xã hội cũng chấm qua loa, chỉ quan tâm đến các ý, chưa xem xét nhận thức và tình cảm, thái độ của học sinh bàn luận. - Việc ra đề, tổ chức viết bài, chữa bài, trả bài và rút kinh nghiệm chưa làm tốt vẫn còn phổ biến trong dạy làm văn. Khâu cuối cùng của quá trình rèn dạy kỹ năng giúp điều chỉnh hành vi bày tỏ nhận biết của học sinh về vấn đề nghị luận thông qua sử dụng ngôn ngữ và diễn đạt cần phải làm chu đáo hơn. - Phần trau dồi ngôn từ, mở rộng vốn từ trong nói và viết qua những giờ tiếng Việt cũng còn nhiều hạn chế, góp phần làm giờ học văn và bài viết văn kém hấp dẫn và hiệu quả. 2. Dạy ôn thi Ngữ văn ĐH, CĐ như thế nào đạt hiệu quả. - Thực tế nước ta vẫn rất khó tìm cơ hội việc làm các ngành Khoa học xã hội; học sinh không hào hứng học và viết văn; công việc ôn thi Ngữ văn ĐH, CĐ gặp không ít khó khăn. Nhà giáo Ngữ văn của các trường THPT vẫn mạnh ai nấy dạy theo chủ nghĩa kinh nghiệm. Những đơn vị có đông học sinh thi Văn ĐH, CĐ (huyện Vĩnh Tường, Lập Thạch, Xuân Hòa...); các lớp ôn thi cũng chưa thống nhất được chương trình và trọng tâm kiến thức. Việc bố trí thêm giờ cho các môn thi ĐH, CĐ, tăng thời gian học ôn có thể mang lại những kết quả đáng kể. Tuy nhiên, vấn đề phương pháp học và làm bài vẫn là vấn đề mấu chốt quyết định điểm của bài thi văn. Học thêm và học nhiều nhưng việc học sinh viết bài thường xuyên lại chưa được chú trọng nên chất lượng bài làm thấp, nhất là các đối tượng HS không thuộc trường trọng điểm có đầu vào cao... - Nội dung ôn thi trọng tâm trong các bài học chính khóa và đọc thêm không thay đổi. Đến 2014, đề bài vẫn xoay quanh chừng ấy tác phẩm, chừng ấy tác giả văn học. Sự đổi mới hoàn toàn của đề thi năm 2013 đặt ra vấn đề có tính sống còn. Không có sự lặp lại đề thi và không thể dùng văn mẫu làm bài. Đề thi ĐH, CĐ đòi hỏi người thi hiểu và bày tỏ sự cảm thụ của mình về vấn đề đặt ra. Nếu không hiểu tác phẩm và không chắc về kỹ năng, thí sinh không thể làm bài nhưng hiểu tác phẩm mà kỹ năng yếu làm bài chắc chắn kém hiệu quả.. - Thực tế một số Trường THPT phân công giáo viên dạy từ lớp 10 đến hết lớp 12, phải ba năm sau họ mới có dịp dạy ôn thi ĐH, CĐ (trừ các thầy cô giáo có lớp ở ngoài), thành thử, những bài học kinh nghiệm không thể được thực thi. Mặt khác, văn chương chưa dễ thống nhất được ngay cách hiểu và cũng chưa nhất trí được ngay cách dạy. Những khó khăn và tồn tại như thế làm công việc dạy ôn thi ĐH, CĐ Ngữ văn càng nan giải. Trong ba năm gần đây, tôi đã thay đổi cách dạy ôn thi ĐH, CĐ và bước đầu thấy hiệu quả. Bên cạnh việc trang bị tri thức đọc hiểu văn bản văn học, tác giả văn học, đảm bảo những kiến thức cơ bản cần thiết, tôi dành nhiều thời gian ôn tập, thực hành kỹ năng làm văn cho học sinh. Trong tham luận này tôi xin trao đổi thêm ý tưởng “DẠY KỸ NĂNG LÀM VĂN – MỘT GIẢI PHÁP QUAN TRỌNG GIÚP HỌC SINH LỚP 12 THI ĐẠI HỌC ĐẠT KẾT QUẢ CAO”. 132 II. NHỮNG GIẢI PHÁP GIÚP HỌC SINH LỚP 12 ÔN THI MÔN NGỮ VĂN HIỆU QUẢ 1.Chủ động trong kế hoạch dạy kỹ năng làm văn. - Theo phân công chuyên môn, tôi chủ động lập Kế hoạch ôn tập từ năm lớp10 đến lớp 12. Theo các trọng tâm kiến thức: + Lớp 10, 11 dành khoảng 30 % số giờ học chuyên đề dạy lý thuyết và kỹ năng tiếng Việt, làm văn. + Lớp 12 khoảng 30% số giờ học chuyên đề dạy lý thuyết và kỹ năng làm văn. Nôi dung bài học được thực hiện đầy đủ các giờ làm văn, viết bài nghiêm túc trên lớp, chấm bài cẩn thận, trả bài có nhận xét tới từng học sinh, yêu cầu sửa lỗi ở nhà và khuyến khích học sinh viết lại bài theo dàn ý bài chữa. - Lớp 12 được đầu tư 3 ca ôn thi một tuần, tổng số 80 ca (mỗi ca = 2,5 tiết học). Tôi dành 24 ca ôn kỹ năng làm văn, 45 ca ôn tác phẩm, 4 ca giới thiệu bài khái quát văn học sử, 2 ca lý luận văn học, 2 ca giới thiệu cấu trúc đề thi và giới hạn kiến thức. Trong số 24 ca kỹ năng chia ra: NLXH: 10; NLVH: 10; chữa bài: 4 ca; trong đó phần kỹ năng làm bài: 11 ca. ( Tổng hợp chương trình: Phụ lục 6). Kiến thức về tác phẩm, tác giả đã được học theo chuyên đề nên tôi chú ý những vấn đề cơ bản mới nhất, không giảng lại tác phẩm. (Kế hoạch chuyên đề: Phụ lục 7) 2. Nội dung giảng dạy phần Kỹ năng (24 ca x 2,5 tiết = 60 tiết) (Tất cả các nội dung trình bày dưới đây tôi đã soạn thành giáo án chuyên đề theo kế hoạch, được nhà trường ký duyệt ). a. Ôn tập các kỹ năng nhận thức đề. - Ôn tập các thao tác nhận thức đề bài: Đọc thật kỹ đề bài, gạch chân từ ngữ quan trọng, phân tích ngữ pháp nhận định (ngữ liệu); xác lập 3 yêu cầu ( thao tác NL, nội dung NL, kiến thức sử dụng) - Giờ trả bài, chép đề và yêu cầu HS thực hiện công việc này nghiêm chỉnh, chỉ ra những thiếu sót và nhầm lẫn của các học trò. b. Ôn tập các kỹ năng tìm ý, lập dàn ý. - Xây dựng những câu hỏi tìm ý (Vấn đề này là gì, cái gì, ai? Vấn đề đúng hay chưa, tại sao, có thể hiểu thế nào, ...) - Hướng dẫn HS cách ghép vào từng đề bài, nhất thiết phải đặt các câu hỏi tìm ý và tìm ý nào chính, ý trọng tâm để dành thời gian và tri thức cho ý đó. Yêu cầu HS nhất thiết làm dàn ý sơ lược rồi hãy viết. c. Ôn tập các kỹ năng hành văn, lập luận, dùng từ, tạo dựng câu... - Ôn tập các kỹ năng viết câu, dùng từ hợp phong cách, đúng nghĩa, đúng chính tả. (mỗi lỗi chính tả, dùng từ, các em sẽ được viết lại 200 lần và tôi nhân đôi nếu không viết). Các kỹ năng dựng đoạn theo từng cách trình bày được dạy lồng ghép khi viết bài giúp HS hiểu và lựa chọn. Các lớp từ và loại từ bình giá, từ dùng liên kết cũng theo cách vừa học bài vừa luyện tập nhận biết để vận dụng. - Trong bài kiểm tra viết, tôi cố gắng sửa những lỗi phát hiện và khi trả bài phân tích HS để cả lớp nhận biết cùng tránh. 133 d. Ôn tập các kỹ năng lập luận theo kiểu bài NLXH, NLVH. - Mỗi kiểu bài NLXH, NLVH có những đặc điểm riêng, tạm gọi là dạng bài. Bài phân tích đoạn thơ, đoạn văn khác bài phân tích cả bài thơ, cả tác phẩm văn xuôi; bài cảm nhận văn học khác bài bày tỏ ý kiến văn học; bài phân tích khác bình luận văn chương. Mỗi bài có cách tiếp cận và kiến giải theo những thao tác lập luận chủ yếu so sánh, phân tích, bình luận, phản bác hay thuyết minh... - Từng bước, thầy cô giáo giúp HS phân biệt sự khác nhau đó nhằm chọn được những kỹ năng trình bày phù hợp giải quyết vấn đề đặt ra. Khi nào cần giảng giải, khi nào phân tích, bình luận...Điều quan trọng là HS chọn cách dùng ngôn ngữ linh hoạt trong khi bàn giảng vấn đề sao cho thuyết phục. e. Ôn tập cách viết mở bài, kết bài. (Kỹ năng viết mở bài, kết bài: phụ lục 9) - Ôn tập cách viết mở bài, kết bài đã học qua việc xây dựng các thiết kế mẫu cho đối tượng HS trung bình. Các em chỉ cần thuộc, khi viết bài, chỉ cần thay đổi chủ thể là xong phần mở bài. - Mở bài sáng tạo dựa trên hai ý cơ bản (ý giới thiệu khái quát, ý nêu vấn đề NL) để thiết kế theo các dạng nêu trực tiếp, gián tiêp, tương liên, phản luận.... Mở bài ngắn gọn và trúng ý tạo sự hấp dẫn theo công thức có nhiều ích lợi. 3. Hướng dẫn học sinh làm bài theo công thức (khung) dàn bài mẫu cho từng kiểu bài (Dàn bài mẫu: phụ lục 10) a. Kiểu bài tái hiện kiến thức ( câu 2 điểm) - Ôn tập các dạng câu hỏi tái hiện kiến thức: tác giả, nhan đề, chi tiết và văn học sử - Hướng dẫn cách tìm chi tiết, tìm ý nghĩa chi tiết và làm thế nào diễn đạt ý hiểu đó. Thống kê các dạng câu hỏi tái hiện, học sinh cần nhớ: + Câu hỏi về tác giả và sự nghiệp sáng tác, phong cách nghệ thuật (Cần học kỹ 5 tác gia trong chương trình văn học 11 & 12 là: Nguyễn Tuân, Xuân Diệu, Hồ Chí Minh, Tố Hữu Nam Cao). + Hoàn cảnh sáng tác. + Kiến thức văn học sử, đặc điểm nội dung, nghệ thuật... + Nội dung và nghệ thuật của tác phẩm, đoạn trích. + Chi tiết, hình ảnh, ý nghĩa chi tiết văn xuôi + Phong cách nghệ thuật, quan điểm sáng tác của tác giả. + Ý nghĩa nhan đề, tình huống truyện. Đây là câu lí thuyết: Cần làm chính xác, rõ ràng những kiến thức trong sách giáo khoa. Tuy nhiên, không đơn giản chỉ có vậy, thí sinh cần trình bày cả cách hiểu, cảm nhận của mình về một vấn đề. b. Kiểu bài NLXH gồm 3 dạng cơ bản: - Nghị luận về tư tưởng đạo lí. - Nghị luận về về hiện tượng đời sống. - Nghị luận về vấn đề xã hội trong tác phẩm văn học. Phần nghị luận xã hội xoay quanh các chủ đề tư tưởng- đạo lý và những hiện tượng trong đời sống. Lưu ý các kiểu đề có nội dung : quan niệm về sống đẹp; sống có trách nhiệm; tình yêu Tổ quốc; tình yêu thương con người, giữ gìn bản sắc văn hoá dân tộc,… các hiện tượng đời sống như nghiện Internet; sống vô cảm; bạo lực giữa các đối tượng học sinh với học sinh; lối sống buông thả,…, các vấn đề xã hội 134 trong văn học như lẽ sống, lòng tốt, đạo làm người,... Phần NLXH tuy thao tác giải thích rất quan trọng, nhưng cảm thấy từ ngữ đó có thể nêu được khái niệm mà không thể nào giải thích được thì nên nêu nội dung chung, tránh giải thích sai. Bởi vì khi đã giải thích sai, thì mọi việc lập luận hoặc mô tả sau đó đều đi sai hướng. Các dạng đề bài NLXH nên quy chiếu về dạng bài cơ bản: vấn đề tư tưởng (nhận thức, lý tưởng, quan điểm nhìn nhận, thái độ...); vấn đề đạo lí (đạo đức, tình người, quan hệ con người với nhau, cách xử sự...) ; hiện tượng đời sống thuộc lĩnh vực nào... Mỗi dạng bài cần chỉ ra cách nhận biết và phân tích, bình luận. c. Kiểu bài NLVH - Kiểu bài về bài thơ, đoạn thơ (nêu cảm nhận, phân tích, bình luận...) - Kiểu bài về về tác phẩm văn xuôi, về nhân vật, về nhận định ...(nêu cảm nhận, phân tích, bình luận...) - Kiểu bài về so sánh văn học, nhóm tác phẩm, một tác phẩm...(nêu cảm nhận, phân tích, bình luận, nêu ý kiến,...) Đây là câu có số điểm nhiều nhất, nhưng rất nhiều học sinh thường chỉ tập trung vào chương trình 12 và bỏ hẳn chương trình 11, hoặc chỉ học văn xuôi và hoàn toàn bỏ phần thơ. Những HS có năng khiếu cảm thụ văn học và có kỹ thuật hành văn tốt, cần học thuộc những tư liệu văn học, nắm vững các chi tiết, hình ảnh để giải quyết trên cơ sở biết khái quát và tổng hợp vấn đề. Các dạng đề bài NLVH nên quy chiếu về dạng bài cơ bản. Bài NLVH về thơ cần chú trọng ngôn ngữ, các biện pháp nghệ thuật để làm rõ ý thơ, chủ đề... Bàn về nhân vật văn xuôi chú ý đến chi tiết, cử chỉ, hành động để thấy những vẻ đẹp của nhân vật... Chúng tôi soạn những gợi ý và đưa ra dàn ý mẫu như những công thức. Học sinh sau khi phân tích đề bài chắc chắn, phác thảo dàn bài trong giấy nháp, tìm ý điền dần vào khoảng trống, tiến hành viết bài đảm bảo nội dung và thời gian. 4. Luyện tập viết bài theo yêu cầu ngắn gọn, đủ ý và hấp dẫn. - Ôn tập các thao tác nghị luận phân tích, giải thích, so sánh, bình luận, phản bác, thuyết minh. - Cách viết văn linh hoạt, hiệu quả. Thực hành nhiều trên lớp viết câu, dùng từ viết đoạn. Những vấn đề nêu trên, có thể không mới. Vấn đề có giá trị nhất là chúng ta làm như thế nào. Mỗi nội dung tôi đều soạn giáo án và theo từng đối tượng điều chỉnh phù hợp. Học sinh viết nhanh, hiểu nhanh tôi chữa ngay trên bài. Học sinh viết chậm, tôi làm mẫu và hướng dẫn cụ thể hơn. Có khi phải làm nhiều lần viết mẫu, đọc mẫu, thậm chí, cần hướng dẫn HS so sánh với bài văn trong tài liệu để đối chiếu, rút ra bài học. Hiểu vấn đề rồi nhưng làm thế nào diễn đạt được sự hiểu ấy? Tôi thường cố gắng soạn đề mới hoặc tham khảo đồng nghiệp đặt ra những câu hỏi dễ hiểu và lạ giúp HS có những nhìn nhận đa dạng và thực chất vấn đề về tác phẩm. Các giá trị của hình tượng, của chi tiết nghệ thuật cần hiểu đúng chuẩn kiến thức kỹ năng. Mỗi thầy cô giảng dạy có thể lí giải khác nhau, đề thi có thể hỏi khác đề đã có nhưng kiến thức cơ bản không thay đổi. Tôi chủ động giúp HS hiểu bản chất vấn đề rồi diễn đạt ngắn gọn, đủ ý và chính xác. Yêu cầu HS không viết dài, kể lể lan man. Tất nhiên, đề bài chưa thấy bao giờ, nhưng học sinh không bị lúng túng, hoang mang, lần lượt làm hết cả ba câu. Nắm thật tốt kỹ năng, hoàn thành bài thi, đúng và 135 trúng hướng đề yêu cầu tin rằng thí sinh đó đã có nửa số điểm mỗi câu (khoảng 5 điểm cả bài). Nếu viết giỏi hơn, điểm tăng lên. Năm 2013, lớp 12A6 khối C, em Nguyễn Thu Trang (không phải HS giỏi văn) thi ĐH đạt 7,5, CĐ đạt 7,5; em Nguyễn Vân Anh, Lê thị Thúy Mai thi ĐH 7,5...Nhiều em đạt điểm 5,5 - 6,5. Nhiều kỹ năng cần thiết khác, trước và trong khi làm bài, thầy cô nên trang bị cho HS như tâm thế, cách chọn đề, cách huy động tri thức, cách viết ngắn, cách chọn và đưa dẫn chứng trong NLXH và NLVH, phân chia thời gian hợp lí... Sự thành thục kỹ năng làm văn và chủ động xử lí đề thi Ngữ văn thật sự quan trọng không kém kiến thức hiểu biết về tác phẩm và xã hội. Tôi đã rất vui khi hết giờ thi, nhận được tin của học trò “em làm xong cả 3 câu nhưng không biết có đúng không”. Tôi muốn học trò đề nào cũng làm được, cứ theo cách thầy dạy, tự tin, bình tĩnh, nỗ lực và sáng tạo, thành công sẽ đến. 5. Lập bảng tổng hợp theo dõi đề thi ĐH, CĐ các năm của Bộ Giáo dục và Đào tạo Tôi sưu tầm đề thi các năm gần đây của Bộ nhằm theo dõi và đi sâu hơn các trọng tâm ôn tập về kiến thức và kỹ năng.( Tổng hợp đề thi các năm: phụ lục 8) III. NHỮNG KẾT QUẢ TRIỂN KHAI CHUYÊN ĐỀ TẠI NHÀ TRƯỜNG 1. Năm học 2011-2012, nhóm học sinh tôi dạy phụ đạo 9 em 12 A6 của trường, có 1 học sinh đạt 8,5 điểm, số còn lại đạt từ 5,5 điểm trở lên, không có điểm dưới 5. 2. Năm học 2012-2013, lớp 12 A6, khối C, do tôi phụ trách, điểm bình quân đạt 5,87. Một số học sinh nắm vững cách làm bài, các kỹ năng làm văn nên điểm bài thi cao hơn điểm tổng kết môn cuối năm. Các em tự tin và chủ động làm bài hiệu quả. (Phụ lục 5). 3. Năm 2013-2014, lớp 12 A7, khối D, tôi cùng dạy ghép, điểm thi Khảo sát theo đề của Sở lần 1 và 2 điểm bình quân đạt yêu cầu (Phụ lục 4) IV. NHỮNG KIẾN NGHỊ 1.Đối với cơ quan quản lí: - Tổ chức Hội thảo cấp Sở thường xuyên, mỗi năm nên chọn một chủ điểm, một nội dung. - Các đơn vị nhà trường chủ động tổ chức hội thảo cấp trường, nhóm trường. - Có chế độ đãi ngộ thù lao cho cá nhân nghiên cứu, sáng kiến kinh nghiệm thỏa đáng. 2.Đối với nhà giáo tham gia ôn thi Đại học, Cao đẳng - Chủ động lập kế hoạch, vận dụng linh hoạt các nghiên cứu, trao đổi kinh nghiệm sau Hội thảo. - Tăng cường trao đổi và học tập kinh nghiệm ôn thi ĐH, CĐ của đồng nghiệp. Vĩnh Yên, ngày 03 tháng 03 năm 2014 Người viết Nguyễn Văn Lự 136 137 Phụ lục 6: TỔNG HỢP KẾ HOẠCH ÔN THI ĐẠI HỌC, CAO ĐẲNG LỚP 12 A7 NĂM 2013-2014 - NGỮ VĂN KHỐI D I. CẤU TRÚC THEO PHÂN MÔN ( 80 ca) Stt Bài Bài Giới thiệu Phần kỹ năng Phần Lý Phần tác Văn nước Giới hạn Kiểm tra Kháiquát tổng kết cấu trúc đề Làm văn luận văn phẩm VH ngoài kiến thức và chữa VH sử chương ĐH học thiĐH bài NLX NLVH H 1 4 3 1 10 10 2 45 x 1 4 II .CẤU TRÚC THEO CHƯƠNG TRÌNH LỚP 11( Tên bài và số ca dạy). St Khái Hai Chữ Hạnh Chí Đời Nam Vội Xuân Đây t quát đứa người. phúc. Phèo thừa Cao vàng Diệu thôn. trẻ . . . 1 2 2 1 1 2 1 1 1 1 1 St t Chiều Thơ xuân duyên 1 1 Đây mùa thu.. Tống biệt hành 1 Nhớ đồng Giải đi sớm x Một thời. . Vĩnh biệt cửu.. 1 Hầu trời x Tiến g mẹ đẻ.. Tràn Tương g tư giang 1 x Vi hành x II .CẤU TRÚC THEO CHƯƠNG TRÌNH LỚP 12 ( Tên bài và số ca dạy) St Khái TNĐ Tác Tây Việt Tố Đất Sóng Đàn Ngườ Nguyễ t quát L gia Tiến Bắc Hữu Nướ ghit i lái .. n Tuân HC c a .. M NKĐ 1 2 1 1 2 2 1 2 1 2 2 1 St t 1 Nhữn g đứa .. 2 Rừng xànu 2 Chiếc thuyền. . 1 Một Nguyễ người.. n Đình Chiểu.. 0,5 x Mấy Thôn ý g nghĩ.. điêp.. x x Nghệ thuật băm.. x NKT T Tinh thần TD.. x Chiều tối Lai Tân Tổng số ca 80 Từ ấy 1 x Cha con nghĩa.. x Tổng số ca 16 Ai đã.. Nhì n về.. Hồn Trươn g VChồn g APhủ Vợ nhặt 2 x 1 2 2 Đất Dọn Tiếng Bác Bên Nhữn Bắt Mùa Tổng nước về hát..Đò ơi kia g ngày sấu.. lá số ca NĐT làng lèn sông.. đầu.. rụng 0,5 1 0,5 x x 0,5 29 138 Ghi chú: Tác phẩm không in nghiêng đọc thêm. Dấu (x) dạy tích hợp với bài khác. Phụ lục 8: TỔNG HỢP ĐỀ THI ĐH, CĐ MÔN NGỮ VĂN KHỐI C.D TỪ NĂM 2009 ĐẾN 2013 Câu NLVH– ĐẠI HỌC, CAO ĐẲNG STT Lớp Tên tác phẩm 2đ 1 2 3 4 5 6 7 8 9 10 11 11 12 13 14 15 16 17 STT Khái quát VH (TK XX- 45) Hai đứa trẻ Chữ người tử tù Hạnh phúc của một Chí Phèo Đời thừa Tác gia Nam Cao Hầu trời Vội vàng Tác gia Xuân Diệu Đây thôn Vĩ Dạ 2 3 4 5 6 2010 3đ 5đ 2đ 2011 3đ 5đ 2đ 2012 3đ 3aD 2013 3đ CĐ 5đ CĐb 3bC CĐ 3aD CĐ 3b C 3bD 3b C 3b C 5đ 2đ 3aD 3aC 2009 3đ 5đ C 3bD 3bD Tên tác phẩm Khái quát VH (1945 - 75) Tuyên ngôn ĐL Tác gia HCM Tây Tiến Việt Bắc Tố Hữu 2đ 3aD 3aC Tràng giang Tương tư Nhât kí trong tù Chiều tối Lai Tân Từ ấy Lớp 5đ C 2đ 1 2009 3đ 2đ 2010 3đ 5đ 2đ 2011 3đ 5đ 2đ 2012 3đ 5đ 2đ 2013 3đ 5đ D C C 3aC D 139 12 7 8 9 10 11 12 13 14 15 16 17 18 19 20 21 22 Tiếng hát con tàu Đất nước-NKĐ Sóng Đàn ghi ta Lor ca Con đường trở ... Người lái đò SĐà Tác gia Nguyễn Tuân Ai đã đặt tên... Nhìn về vốn VH... Hồn Trương Ba... Vợ chông APhủ Vợ nhặt Những đứa con... Rừng Xà nu Một người Hà Nội Chiếc thuyền... 3bD 3aC CĐa CĐa CĐa 3aD 3bC D 3bC 2.C 2.D 3a C D 3aD CĐ CĐa 3a C CĐ 3aD CĐb 3bC 3bD Câu NLXH – ĐẠI HỌC, CAO ĐẲNG STT Năm 2009 Năm 2010 Năm 2011 Năm 2012 Khối C Người đánh mất niềm tin... mất nhiều thứ quý giá hơn Thói đạo đức giả.... Biết tự hào... nhưng biết xấu hổ còn quan trọng hơn Kẻ cơ hội nôn nóng tạo ra thành tích, người chân chính ... nên thành tựu Khối D Xin cho cháu biết chấp nhận thi rớt.... Thói vô trách nhiệm Đừng cố trở thành nổi tiếng ...mà trước hết là người có ích. Ngưỡng mộ thần tượng là nét đẹp văn hóa, mê muội ... là thảm họa Năm 2013 Không ca tụng trí tuệ mà ca tụng sự khôn khéo... biết thủ thế giữ mình, gỡ được tình thế khó khăn. Người Việt Nam ... Tính thụ động... đi theo con đường vẽ sẵn. 140 Khối CĐ Mối quan hệ tài và đức Thói ích kỷ...thì sự chia sẻ trở thành lạc lõng Nghề nghiệp ko làm nên sự cao quý...mà chính con người... Khi có lỗi, người tử tế thì sẵn sàng nhận lỗi, kẻ ti tiện chỉ tìm cách đổ lỗi. 141 Phụ lục 9: KỸ NĂNG VIẾT MỞ BÀI, KẾT BÀI PHẦN I. VIẾT MỞ BÀI 1.Trong phần đặt vấn đề cần đạt 2 yêu cầu: - Nêu được vấn đề ngắn gọn nhất, hấp dẫn và gây hứng thú cho người đọc, người nghe. - Nêu lên hướng giải quyết, phạm vi giải quyết vấn đề, tầm quan trọng, ý nghĩa của vấn đề để chuẩn bị tư tưởng cho người đọc theo dõi phần nội dung. Mục đích của mở bài là giới thiệu về vấn đề mà ḿình sẽ viết, thực chất là trả lời câu hỏi: Ở bài viết này, ḿình định viết về điều gì? 2. Cấu trúc của một mở bài Cấu trúc của một mở bài gồm 2 nội dung chính và 1 nội dung phụ : Ý chính : + Dẫn dắt vấn đề: Nêu một vài vấn đề liên quan đến vấn đề cần bàn, chuẩn bị tư tưởng dẫn người đọc, người nghe vào vấn đề bàn luận hay tình huống có vấn đề đặt ra ở đề bài. + Nêu vấn đề: Nêu vấn đề một cách ngắn gọn, nêu đúng vấn đề đặt ra trong đề bài và phải nêu một cách khái quát. Vấn đề mà mở bài nêu ra chính là vấn đề mà nội dung bài viết đề cập tới. Vấn đề này được nêu ra ở dạng khái quát, nêu một cách ngắn gọn và gây được sự chú ý của người đọc. Mở bài có nhiệm vụ thông báo chính xác, rơ ràng, đầy đủ vấn đề, dẫn dắt sao cho việc tiếp cận đề tài được tự nhiên nhất. Ý phụ: (có thể lồng ghép vào ý chính) + Nêu giới hạn vấn đề: nêu được phạm vi bàn luận trong khuôn khổ nào (1đề tài, 1 tác phẩm hay nhiều tác phẩm...) + Nêu nhận định về tầm quan trọng của vấn đề, ý nghĩa của vấn đề đối với cuộc sống, xã hội, dòng văn học ; với trước đó và đương thời... (phần này không nhất thiết phải có, tuỳ thuộc vào từng vấn đề cụ thể). 3. Một số mẫu mở bài ứng dụng từ thực tế a. Đặt vấn đề (mở bài) trực tiếp + Mở thẳng vấn đề : - Dẫn dắt ngắn gọn bằng câu văn liên quan trực tiếp tới vấn đề - Nêu rõ vấn đề định bàn luận là gì. - Nêu giới hạn vấn đề. + Mở trực tiếp có thêm phần dẫn dắt (thời gian, không gian và hoàn cảnh sáng tác của tác phẩm). - Dẫn dắt bằng cách nêu bối cảnh làm vấn đề xuất hiện như : thời gian, không gian, địa điểm xảy ra sự kiện gì liên quan đến tác phẩm/vấn đề ; Xuất xứ của tác phẩm văn học. - Nêu rõ vấn đề định bàn luận là gì. - Nêu giới hạn vấn đề. b. Đặt vấn đề (mở bài) với Nghị luận văn học theo cách gián tiếp Giới thiệu từ vấn đề liên quan, dẫn dắt đến nội dung nêu vấn đề hoặc ấn tượng. Cách này dành cho HS khá giỏi biết làm chủ kiến thức. 4.Tóm tắt một số công thức viết mở bài : Cách 1: Dẫn theo nhận định của đề bài Đoạn dẫn + nêu vấn đề + giới hạn vấn đề + nhận định về tầm quan trọng của vấn đề, ý nghĩa Cách 2: Dẫn theo tư liệu tác giả 142 Nêu tên tác giả + vị trí tác giả trong nền văn học hoặc phong cách + đề tài tiêu biểu, tác phẩm tiêu biểu + nêu vấn đề Cách 3: Đoạn dẫn theo lối so sánh (hai vấn đề tương tự). Tìm 1 vấn đề tương tự (đề tài, chủ đề, hình ảnh , tác phẩm...) làm cầu nối so sánh với vấn đề của đề bài để tạo đoạn dẫn + nêu vấn đề. Cách 4. Đoạn dẫn theo lối so sánh (hai vấn đề đối lập) Tìm 1 vấn đề đối lập tạo thế bắc cầu để giới thiệu vấn đề cần bàn + nêu vấn đề Cách 5: Đoạn dẫn dựa vào lời đánh giá ấn tượng của một tác giả nào đó. Lấy 1 đánh giá của một tác giả uy tín có nội dung trùng với vấn đề đã xác định được làm điểm tựa để phát triển tiếp + nêu vấn đề. Cách 6: . Đoạn dẫn dựa vào xuất xứ và những thông tin khác về tác phẩm Lấy các thông tin liên quan đến vấn đề sau khi chọn lọc các chi tiết quan trọng hấp dẫn bố trí thành đoạn dẫn + nêu vấn đề. PHẦN II: PHƯƠNG PHÁP KẾT BÀI Kết bài rất quan trọng đối với bài văn nghị luận, là phần kết thúc vấn đề đặt ra ở mở bài và đã giải quyết ở phần thân bài. Tuy nhiên vì nhiêu lí do khác nhau, kết bài thường là phần phần cuối cùng, gần hết giờ nên làm rất vội. Giống như phần mở bài, phần này chỉ nêu ý khái quát, không trình bày lan man, dài dòng hoặc lặp lại sự giảng giải, minh họa, nhận xét một cách chi tiết như ở phần thân bài. Phần kết bài có nhiệm vụ tổng kết, đánh giá chung vấn đề đã nghị luận. Kết thúc vấn đề hay sẽ tạo được “âm vang”, “dư ba” cho bài văn. Các kiểu kết bài: 1-Tổng kết, tóm lược những ý chính đã trình bày ở phần thân bài: Đây là cách kết bài thông thường nhất vì dễ làm nhất 2. Chiết trung, dung hòa: Đây cũng là kết bài theo cách tổng hợp từ đó rút ra thái độ, tình cảm không phủ định, không tán thành . (thường dùng cho những luận đề không đúng hẳn mà cũng không hoàn toàn sai hoặc những luận đề có hai, ba ý kiến đối nghịch nhau nhưng xem ra ý kiến nào cũng đều có lí của nó, đặc biệt là những vấn đề thuộc quan điểm cá nhân) 3. Phát triển mở rộng thêm vấn đề: Nêu đánh giá khẳng định và dự báo tiếp theo. 4.Vận dụng vào cuộc sống, rút ra bài học áp dụng: Nêu tổng kết bằng những số liệu hay thành công của vấn đề trong cuộc sống. 5. Liên tưởng: là cách kết bài thông qua sự liên tưởng, tức là mượn ý kiến của dân gian, của một danh nhân, một người có uy tín hoặc của sách để làm kết luận 6. Hỗn hợp: Kết hợp 2, 3 ...kiểu kết bài trên làm thành phần kết thúc vấn đề. Nhưng dù kết bài theo kiểu nào đi chăng nữa thì cũng nhằm khắc sâu kết luận của người viết để 143 lại ấn tượng đậm đà cho người đọc và nhằm nhấn mạnh ý nghĩa của vấn đề đã được nghị luận. Kết bài hay phải vừa đóng lại, chốt lại, phải vừa mở ra, nâng cao. 144 Phụ lục 10: DÀN BÀI MẪU A. BÀI NGHỊ LUẬN Xà HỘI I . Xây dựng dàn bài nghị luận xã hội về tư tưởng đạo lí. Xây dựng dàn ý sơ lược 3 phần a+ Mở bài : Ngắn gọn khoảng 5 câu - Dẫn luận xuất xứ vấn đề: Nguồn gốc hay hoàn cảnh không gian vấn đề được nêu lên. - Nêu vấn đề bàn luận: Vấn đề gì, thuộc lĩnh vực nào của tư tưởng đạo lí. b+ Thân bài: 3 ý lớn - Ý 1: Giảng giải các từ ngữ, khái niệm, các vấn đề. Giải thích ý nghĩa gốc và nghĩa chuyển các từ, các vế và cả câu để thống nhất nội dung bàn luận. - Ý 2: Phân tích, bình luận làm rõ các biểu hiện trong từng khía cạnh chính của vấn đề. Vấn đề này có những biểu hiện nào là chủ yếu? Thể hiện trong lĩnh vực nào? Hiểu vấn đề trong mối quan hệ chung và riêng ra sao. Có thể hiểu vấn đề theo quan điểm nào, tầng lớp giai cấp nào của cuộc sống hiện nay. Tại sao hiểu như thế? Mối quan hệ giữa các ý như thế nào. Xem xét vấn đề trên những bình diện đúng/ sai, tốt /xấu, tích cực/ tiêu cực, đẹp /xấu .. Có thể chỉ ra nguyên nhân, dẫn chứng, cơ sở thực tiễn để minh họa. Không phân tích cụ thể ví dụ tránh lan man. Khẳng định ý đúng, bác bỏ ý sai, phê phán hoặc đồng tình.. - Ý 3: Nêu ý nghiã và tác dụng của vấn đề - Vấn đề bàn luận có quan hệ như thế nào đến nhận thức và tầm hồn tình cảm của con người, nhất là tuổi trẻ học đường. - Nêu vài biểu hiện của những tác động đa chiều khái quát, điển hình và toàn diện của vấn đề. Không nêu quá nhiều biểu hiện tác động tiêu cực hoặc tích cực, cần cân đối hài hòa. c+ Kết bài: ( khoảng 5 câu) - Tổng kết các nội dung chính đã bàn ở trên ( lược ghi các ý chính) - Có thể liên hệ suy nghĩ, nhận thức của bản thân trong tình hình cụ thể của mình. 2 . Xây dựng dàn bài nghị luận xã hội về hiện tượng đời sống Xây dựng dàn ý sơ lược 3 phần a+ Mở bài : Ngắn gọn khoảng 5 câu - Dẫn luận xuất xứ vấn đề: Nguồn gốc hay hoàn cảnh không gian vấn đề được nêu lên. - Nêu vấn đề bàn luận: Vấn đề gì, thuộc lĩnh vực nào của hiện tượng đời sống . b+ Thân bài: 3 ý lớn - Ý 1: Giảng giải các từ ngữ, khái niệm, các vấn đề. Giải thích ý nghĩa gốc và nghĩa chuyển các từ, các vế và cả câu để thống nhất nội dung bàn luận. - Ý 2: Phân tích, bình luận làm rõ các biểu hiện trong từng khía cạnh chính của vấn đề. Vấn đề này có những biểu hiện nào là chủ yếu? Thể hiện trong lĩnh vực nào? Hiểu vấn đề trong mối quan hệ chung và riêng ra sao. Có thể hiểu vấn đề theo quan điểm nào, tầng lớp giai cấp nào của cuộc sống hiện nay. Tại sao hiểu như thế? Mối quan hệ giữa các ý như thế nào. Những tác động tốt và không tốt, hệ lụy và ảnh hưởng của vấn đề đến cộng đồng. 145 Xem xét vấn đề trên những bình diện đúng/ sai, tốt /xấu, tích cực/ tiêu cực, đẹp /xấu .. Có thể chỉ ra nguyên nhân, dẫn chứng, cơ sở thực tiễn để minh họa. Không phân tích cụ thể ví dụ tránh lan man. Khẳng định ý đúng, bác bỏ ý sai, phê phán hoặc đồng tình.. -Ý 3: Nêu ý nghiã và tác dụng của vấn đề - Vấn đề bàn luận có quan hệ như thế nào đến nhận thức và tầm hồn tình cảm của con người, nhất là tuổi trẻ học đường. - Nêu vài biểu hiện của những tác động đa chiều khái quát, điển hình và toàn diện của vấn đề. Không nêu quá nhiều biểu hiện tác động tiêu cực hoặc tích cực, cần cân đối hài hòa. c+ Kết bài: ( khoảng 5 câu) - Tổng kết các nội dung chính đã bàn ở trên ( lược ghi các ý chính) - Có thể liên hệ suy nghĩ, nhận thức của bản thân trong tình hình cụ thể của mình. Về dạng bài vấn đề đời sống trong TPVH dựa theo dàn ý trên. B. BÀI NGHỊ LUẬN VĂN HỌC 1 . Xây dựng dàn bài câu hỏi tái hiện kiến thức a+ Mở bài : Ngắn gọn khoảng 5 câu - Dẫn luận xuất xứ vấn đề tác giả, tác phẩm và vị trí TP. - Nêu vấn đề: Vấn đề gì, chi tiết nghệ thuật hay nội dung nào. b+ Thân bài: 3 ý lớn - Ý 1: Ghi chính xác các chi tiết (câu chữ, dấu câu...); hoặc đặc điểm, nội dung đề hỏi. - Ý 2: Phân tích, bình luận làm rõ các biểu hiện trong từng khía cạnh chính của vấn đề. Gắn với nhan đề hay nhân vật, hoàn cảnh để nhận xét ý nghĩa hoặc đánh giá thành công hạn chế. -Ý 3: Nêu ý nghiã và tác dụng của vấn đề khi tìm hiểu tác phẩm hay tác giả... c+ Kết bài: ( khoảng 5 câu) - Có thể liên hệ suy nghĩ, cảm xúc của bản thân khi nhận thức vấn đề. 2 . Xây dựng dàn bài nghị luận văn học: tác phẩm thơ a+ Mở bài : Ngắn gọn khoảng 5 câu - Dẫn luận xuất xứ vấn đề tác giả, tác phẩm và vị trí TP, nội dung và nghê thuật khái quát.. - Nêu vấn đề: Vấn đề gì gây ấn tượng nhất đề bài nêu ra. Dựa phần dẫn hay nhận định của đề. b+ Thân bài: 3 ý lớn - Ý 1: Xác lập không gian nghệ thuật, xuất xứ hay vị trí đoạn thơ, bài thơ. - Ý 2: Phân tích, bình luận làm rõ các biểu hiện trong từng khía cạnh chính của vấn đề. Tùy gợi ý của đề, chọn cách phân tích, bình luận theo trình tự từng câu hay theo khổ thơ, ý thơ, theo diễn biến tâm trạng nhân vật trữ tình... -Ý 3: Nêu nhận xét ý nghĩa hoặc đánh giá thành công hạn chế về nội dung, nghệ thuật. Nêu ảnh hưởng của tác phẩm trong sự phát triển văn thơ. c+ Kết bài: ( khoảng 5 câu) - Tổng kết các ý đã trình bày. - Có thể liên hệ suy nghĩ, cảm xúc của bản thân khi nhận thức về tác phẩm, về tác giả. 3 . Xây dựng dàn bài nghị luận văn học: tác phẩm văn xuôi a+ Mở bài : Ngắn gọn khoảng 5 câu - Dẫn luận xuất xứ vấn đề tác giả, tác phẩm và vị trí TP, nội dung và nghê thuật khái quát.. 146 - Nêu vấn đề: Vấn đề gì gây ấn tượng nhất đề bài nêu ra. Dựa phần dẫn hay nhận định của đề. b+ Thân bài: 3 ý lớn - Ý 1: Xác lập không gian nghệ thuật, xuất xứ hay vị trí đoạn trích, nhân vật, cốt truyện.. - Ý 2: Phân tích, bình luận làm rõ biểu hiện trong từng khía cạnh chính của vấn đề. Tùy gợi ý của đề, chọn cách phân tích, bình luận theo trình tự từng nội dung, diễn biến cốt truyện, theo đặc điểm ngoại hình, tính cách, vẻ đẹp, số phận..., hay nội dung nhận định của đề bài. -Ý 3: Nêu nhận xét ý nghĩa hoặc đánh giá thành công hạn chế về nội dung, nghệ thuật. Nêu ảnh hưởng của tác phẩm trong sự phát triển văn học. Giá trị tư tưởng, phong cách nhà văn.. c+ Kết bài: ( khoảng 5 câu) - Tổng kết các ý đã trình bày. - Có thể liên hệ suy nghĩ, cảm xúc của bản thân khi nhận thức về tác phẩm, về tác giả. Các dạng đề thường dùng cách nêu yêu cầu qua các từ khóa: bày tỏ, cảm nhận, phân tích, bình luận, suy nghĩ, nhận xét, ý kiến tranh luận, làm sáng tỏ...Tất thảy đề hướng về các dạng bài cơ bản trên. Sự khác biệt về yêu cầu nội dung và cách bàn luận không nhiều ./. 147 CHUYÊN ĐỀ LỊCH SỬ THẾ GIỚI CÁC NƯỚC Á-PHI-MĨ LA TINH (1945-2000) 1.Tác giả: Nguyễn Tuyết Mai. Tổ: Văn-Sử- Địa-CD.Trường THPT Vĩnh Yên 2. Đối tượng bồi dưỡng: Học sinh lớp 12. 3. Dự kiến số tiết bồi dưỡng: 8 tiết. 4.Hệ thống kiến thức và bài tập sử dụng trong chuyên đề. * Học sinh năm và hiểu được những kiến thức cơ bản sau: - Nét chung về khu vực Đông Bắc Á. - Sự thành lập nước Cộng hòa nhân dân Trung Hoa, ý nghĩa. Nội dung, thành tựu của công cuộc cải cách, mở cửa từ năm 1978. - Đông Nam Á: quá trình đấu tranh giành độc lập, thành tựu phát triển kinh tế, tổ chức ASEAN. - Ấn Độ: cuộc đấu tranh giành độc lập, những thành tựu trong công cuộc xây dựng đất nước. - Châu Phi và Mĩ La Tinh: quá trình đấu tranh giành độc lập. * Học sinh giải được các đề Đại học - Cao đẳng. 5.Hệ thống kiến thức và bài tập sử dụng trong chuyên đề. - Kiến thức truyền đạt theo phân phối chương trình, bám sát chương trình giảm tải của Bộ GD. PHONG TRÀO GIẢI PHÓNG DÂN TỘC Ở CÁC NƯỚC Á-PHI-MĨ LA TINH. 148 PHẦN I: KIẾN THỨC CƠ BẢN A-CÁC NƯỚC ĐÔNG BẮC Á I. Nét chung về khu vực Đông Bắc Á. - Là khu vực rộng lớn với diện tích 10,2 triệu km2, dân số 1,47 tỷ người. - Trước 1945, các nước Đông Bắc Á đều bị thực dân nô dịch (trừ NB). - Sau 1945 Đông Bắc Á có nhiều biến chuyển quan trọng: + Cách mạng Trung Quốc thắng lợi, nước Cộng hoà Nhân dân Trung Hoa ra đời (1/10/1949). Chính quyền Tưởng Giới Thạch phải rút chạy ra Đài Loan và tồn tại ở đó nhờ sự giúp đỡ của Mĩ.Hồng Công và Ma Cao vẫn là những vùng đất thuộc địa của Anh và Bồ Đào Nha,cho đến cuối thập niên 90 (1997&1999), Hồng Công và Ma Cao đã trở về chủ quyền với TQ. + Sau năm 1945, bán đảo Triều Tiên đã bị chia cắt thành hai miền theo vĩ tuyến 380: Đại Hàn Dân quốc ở phía Nam (8/1948) và Cộng hoà DCND Triều Tiên ở phía Bắc (9/1948). Quan hệ giữa 2 nước này đối đầu căng thẳng, từ năm 2000, đã có những cải thiện bước đầu theo chiều hướng tiếp xúc và hoà hợp dân tộc. + Trong nửa sau thế kỷ XX, khu vực Đông Bắc Á đạt tăng trưởng nhanh chóng về kinh tế, đời sống nhân dân được cải thiện rõ rệt như: Hàn Quốc, Hồng Kông, Đài Loan và đặc biệt những thành tựu to lớn của Nhật Bản, Trung Quốc từ cuối những năm 70. II. Trung Quốc 1. Sự thành lập nước CHND Trung Hoa và thành tựu 10 năm đầu xây dựng chế độ mới (1949- 1959). - Sau khi chiến tranh chống Nhật kết thúc, từ năm 1946 đến 1949 ở Trung Quốc đã diến ra cuộc nội chiến giữa Quốc dân đảng và Đảng Cộng sản Trung Quốc. +Ngµy 20-7-1946 Tëng Giíi Th¹ch huy ®éng 160 v¹n qu©n chÝnh quy tÊn c«ng toµn diÖn vµo c¸c vïng gi¶i phãng. + Tõ th¸ng 7-1946 ®Õn th¸ng 6-1947, qu©n gi¶i phãng nh©n d©n Trung Quèc thùc hiÖn chiÕn lîc phßng ngù tÝch cùc, kh«ng gi÷ ®Êt mµ chñ yÕu tiªu diÖt sinh lùc ®Þch, ph¸t triÓn lùc lîng m×nh. +Tõ th¸ng 6-1947, qu©n gi¶i phãng chuyÓn sang ph¶n c«ng, gi¶i phãng c¸c vïng do Quèc d©n ®¶ng thèng trÞ. - Cuối năm 1949, nội chiến kết thúc, ngày 1-10-1949, níc Céng hoµ nh©n d©n Trung Hoa tuyªn bè thµnh lËp do Mao Tr¹ch §«ng lµm Chñ tÞch. -Sự ra đời của nước CH nhân dân Trung Hoa ®¸nh dÊu th¾ng lîi cña c¸ch m¹ng d©n téc d©n chñ Trung Quèc, chấm dứt hơn 100 năm nô dịch và thống trị của đế quốc, xóa bỏ tàn dư phong kiến, đưa Trung Quốc bước vào kỷ nguyên độc lập tự do và tiến lên CNXH, ảnh hưởng sâu sắc tới phong trào giải phóng dân tộc trên thế giới. 3. Công cuộc cải cách – mở cửa (từ 1978) - Tháng 12-1978, Trung ương Đảng Cộng sản Trung Quốc đã vạch ra đường lối cải cách kinh tế - xã hội, do Đặng Tiểu Bình khởi xướng. 149 - Nội dung cơ bản của đường lối cải cách: lấy phát triển kinh tế làm trung tâm, tiến hành cải cách và mở cửa, chuyển sang nền kinh tế thị trường xã hội chủ nghĩa, tiến hành bốn hiện đại hóa nhằm mục tiêu biến Trung Quốc thành nước giàu mạnh, dân chủ và văn minh. - Sau 20 năm cải cách (1978- 1998), đất nướcTrung Quốc đã diễn ra những biến đổi căn bản và đạt nhiều thành tựu to lớn, đó là: + GDP tăng trung bình hằng năm 8%. Năm 2000, GDP đạt 1080 tỉ USD, thu nhập bình quân đầu người tăng, đời sống nhân dân cải thiện rõ rệt. + Đạt nhiều thành tựu trong khoa học – kỹ thuật :1964, thử thành công bom nguyên tử; 10/2003 phóng thành công tàu “Thần Châu 5” vào không gian đưa nhà du hành Dương Lợi Vĩ vào không gian vũ trụ. - Về đối ngoại: Trung Quốc mở rộng quan hệ ngoại giao với các nước, hợp tác giải quyết các vụ tranh chấp quốc tế. Vai trò và địa vị quốc tế của Trung Quốc không ngừng được nâng cao trên trường quốc tế. B- Các nước Đông Nam Á và Ấn Độ 1. Sự thành lập các quốc gia độc lập sau chiến tranh thế giới thứ hai. a. Khái quát cuộc đấu tranh giành độc lập. - Khu vực Đông Nam Á rộng 4,5 triệu km2, hiện nay gồm 11 nước, dân số 528 triệu người. +Trước CTTG II: hầu hết là thuộc địa của các quốc gia Âu – Mỹ (trừ Xiêm) +Trong CTTG II: là thuộc địa của Nhật -Những nét chính về quá trình giành độc lập: +Tận dụng thời cơ Nhật đầu hàng, các nước ĐNA đã đứng lên đấu tranh, nhiều nước đã giành được độc lập, hoặc giải phóng phần lớn lãnh thổ: Inđônêxi 17/8/45, VN 2/9/45, Lào 12/10/45 +Sau đó, thực dân Âu – Mĩ tái chiếm ĐNA, nhân dân ĐNA đã đấu tranh kiên cường, bền bỉ, buộc ĐQ công nhận độc lập ĐNA (Philipin – 1946, Miến Điện – 1948…) +Ba nước Đông Dương thực hiện cuộc kháng chiến chống Pháp thắng lợi 1954, nhưng tới 1975, sau thắng lợi của cuộc kháng chiến chống Mỹ, ba nước Đông Dương mới giành độc lập hoàn toàn. b Cuộc đấu tranh giành độc lập dân tộc ở Lào từ 1945 – 1975 * Giai đoạn 1945 – 1954: Kháng chiến chống Pháp - Tháng 8/1945, Nhân dân Lào nổi dậy giành chính quyền. 12/10/1945, Lào tuyên bố độc lập. - 3/1946, Pháp trở lại xâm lược Lào. Dưới sự lãnh đạo của Đảng Cộng sản Đông Dương và sự giúp đỡ của quân tình nguyện Việt Nam, cuộc kháng chiến chống Pháp của nhân dân Lào ngày càng phát triển. - Sau chiến thắng Điện Biên Phủ, Pháp kí Hiệp định Giơnevơ (7/1954) công nhận độc lập, chủ quyền và toàn vẹn lãnh thổ của Lào. * Giai đoạn 1954 -1975: Kháng chiến chống Mĩ - Sau hiệp định Giơnevơ Mĩ xâm lược Lào. Dưới sự lãnh đạo của Đảng nhân dân Lào cuộc đấu tranh chống Mĩ trên 3 mặt trận (quân sự, chính trị, ngoại giao), giành được nhiều thắng lợi, lần lượt đánh bại các kế hoạch chiến tranh 150 của Mĩ. Đến đầu những năm 70 vùng giải phóng được mở rộng đến 4/5 lãnh thổ. - 2/1973 Hiệp định Viêng Chăn được kí kết, lập lại hòa bình và thực hiện hòa hợp dân tộc ở Lào. - 2/12/1975 nước Cộng hòa Dân chủ Nhân dân Lào chính thức được thành lập. Từ đó Lào bước sang thời kì mới xây dựng đất nước và phát triển kinh tế - xã hội. c. Những nét chính về tình hình Campuchia từ (1945 – 1993): - Từ cuối năm 1945 đến năm 1954, nhân dân CPC tiến hành cuộc kháng chiến chống Pháp. Ngày 9-11-1953, Pháp ký Hiệp ước trao trả độc lập cho Campuchia. - Tháng 7/1954 Hiệp định Giơnevơ được ký công nhận độc lập, chủ quyền của Lào - Từ 1954 – 1970: Chính phủ CPC do Xihanuc lãnh đạo đi theo đường lối hòa bình, trung lập, không tham gia các liên minh quân sự để xây dựng đất nước. - Tháng 3-1970, Mĩ dùng tay sai đảo chính lật đổ Xihanuc. - Ngày 17-4-1975, thủ đô Phnôm Pênh được giải phóng, kết thúc thắng lợi cuộc kháng chiến chống Mĩ. Tập đoàn Khơ-me đỏ do Pôn-Pốt cầm đầu đã phản bội cách mạng, thi hành chính sách diệt chủng, giết hại hàng triệu người vô tội. - Tháng 12/1978 mặt trận dân tộc cứu nước Campuchia thành lập, ngày 7-11979, thủ đô Phnôm Pênh được giải phóng, nước Cộng hoà Campuchia ra đời. - Từ 1979 đến năm 1991, diễn ra cuộc nội chiến kéo dài hơn một thập niên kết thúc với sự thất bại của Khme đỏ, 10-1991 Hiệp định hòa bình về Campuchia được ký kết. - Sau cuộc tổng tuyển cử 1993, Campuchia trở thành Vương quốc độc lập do Xihanúc (Sihanouk) làm quốc vương, Campuchia bước sang thời kỳ phát triển mới. 2. Quá trình xây dựng và phát triển của các nước Đông Nam a. Nhóm năm nước sáng lập ASEAN *Chiến lược kinh tế hướng nội: - Sau độc lập, nhóm 5 nước sáng lập ASEAN (Inđônêxia, Malaixia, Philippin, Xingapo và Thái Lan) thực hiện đường lối công nghiệp hóa thay thế nhập khẩu (chiến lược kinh tế hướng nội). - Mục tiêu: nhanh chóng thoát khỏi nghèo nàn lạc hậu, xây dựng kinh tế tự chủ. - Nội dung: Đẩy mạnh sản xuất hàng tiêu dùng nội địa, thay thế hàng nhập khẩu. Lấy thị trường trong nước làm chỗ dựa để phát triển sản xuất. - Thành tựu: Sản xuất đáp ứng được nhu cầu cơ bản của nd, góp phần giải quyết nạn thất nghiệp, đời sống nd được cải thiện. - Hạn chế: Thiếu vốn, nguyên liệu và công nghệ, chi phí cao dẫn đến thua lỗ, tham nhũng, quan liêu... * Chiến lược kinh tế hướng ngoại: - Từ những năm 60-70 trở đi chuyển sang chiến lược công nghiệp hóa hướng về xuất khẩu (chiến lược kinh tế hướng ngoại). 151 - Nội dung: Tiến hành “mở cửa” thu hút vốn đầu tư nước ngoài, đẩy mạnh xuất khẩu, phát triển ngoại thương - Thành tựu: Tỉ trọng công nghiệp và mậu dịch đối ngoại tăng trưởng nhanh. Singapo trở thành con rồng kinh tế của Châu Á. Năm 1980, tổng kim ngạch xuất khẩu của 5 nước đạt tới 130 tỉ USD – chiến 14% ngoại thương của các nước đang phát triển. - Hạn chế: Phụ thuộc vào vốn và thị trường nước ngoài, cơ cấu đầu tư bất hợp lí. 3.Sự ra đời và phát triển của tổ chức ASEAN * Hoàn cảnh ra đời: - Sau khi độc lập, các nước trong khu vực cần có sự hợp tác với nhau để phát triển - Hạn chế ảnh hưởng của các cường quốc bên ngoài đối với khu vực. - Trên thế giới xuất hiện nhiều tổ chức hợp tác mang tính khu vực, tiêu biểu là liên minh Châu Âu - EU đã thúc đẩy sự liên kết giữa các nước ĐNA. - Ngày 8/8/1967, Hiệp hội các quốc gia ĐNA (ASEAN) thành lập tại Băng Cốc với sự tham gia của 5 nước là Inđônêxia, Malaixia, Xingapo, Thái Lan và Philippin. * Mục tiêu: Tiến hành sự hợp tác giữa các nước thành viên nhằm phát triển kinh tế và văn hóa trên tinh thần duy trì hòa bình và ổn định khu vực. * Quá trình phát triển: - Từ năm 1967 đến 1975 ASEAN còn là tổ chức non trẻ, hợp tác lỏng lẻo, chưa có vị trí trên trường quốc tế. - Tháng 2/1976, ký hiệp ước thân thiện và hợp tác ở ĐNÁ (Hiệp ước Bali), nhằm xác định những nguyên tắc cơ bản trong quan hệ giữa các nước: +Tôn trọng chủ quyền và toàn vẹn lãnh thổ; +Không can thiệp vào công việc nội bộ; +Không dùng vũ lực đe dọa nhau; + Giải quyết tranh chấp bằng hòa bình; +Hợp tác có hiệu quả trên các lĩnh vực kinh tế, văn hoá, xã hội... - Quan hệ giữa các nước Đông Dương và ASEAN bước đầu được cải thiện. - Kinh tế các nước tăng trưởng - Mở rộng thành viên từ 5 nước lên 10 nước: Năm 1984 Brunây gia nhập ASEAN, Việt Nam ( 28/7/1995), Lào và Mianma (9/1997), Campuchia (1999) => Từ đây ASEAN đẩy mạnh hoạt động hợp tác kinh tế, văn hóa nhằm xây dựng một ĐNA thành khu vực hòa bình, ổn định và phát triển. II. Ấn Độ 1.Cuộc đấu tranh giành độc lập. - Là nước lớn, đông dân thứ 2 Châu Á: 3,3 triệu km2, DS 1 tỉ 20 triệu người (2000) - Sau CTTG II, cuộc đấu tranh giải phóng dân tộc do Đảng Quốc Đại lãnh đạo phát triển mạnh mẽ. ( năm 1946 có 848 cuộc bãi công.....) - Kết quả: thực dân Anh thực hiện kế hoạch Mao bát tơn 15/8/1947, chia Ấn Độ 2 quốc gia theo tôn giáo : Ấn độ (Ấn Độ giáo) và Pakixtan (Hồi giáo) 152 - Không thỏa mãn với chế độ tự trị, Đảng Quốc đại do Nêru đứng đầu đã lãnh đạo nhân dân tiếp tục đấu tranh. - Ngày 26/1/1950, Ấn Độ tuyên bố độc lập và nhà nước cộng hòa được thành lập. Ý nghĩa: đánh dấu thắng lợi to lớn của nhân Ấn Độ, cổ vũ mạnh mẽ PT GPDT trên thế giới . 2.Công cuộc xây dựng đất nước. - Nông nghiệp: nhờ tiến hành cuộc “Cách mạng xanh” nên Ấn Độ tự túc được lương thực, 1995 là nước xuất khẩu gạo đứng hàng thứ ba trên thế giới. - Công nghiệp: đứng thứ 10 trên thế giới về sản xuất công nghiệp, chế tạo được máy móc hiện đại - Khoa học kỹ thuật: Là cường quốc về công nghệ phần mềm, công nghệ hạt nhân và công nghệ vũ trụ. + 1974 chế tạo thành công bom nguyên tử + 1975 phóng vệ tinh nhân tạo… - Về đối ngoại: thực hiện chính sách hòa bình, trung lập, luôn ủng hộ cuộc đấu tranh giành độc lập dân tộc của các nước . C.Các nước Châu Phi và Mĩ La Tinh I.Những nét chính về đấu tranh giành độc lập của nhân dân châu Phi sau CTTG II - Sau CTTG II, đặc biệt là những năm 50 Thế kỉ XX, cuộc đấu tranh chống chủ nghiã thực dân ở Châu Phi phát triển mạnh mẽ mở đầu là khu vực Bắc Phi sau đó lan ra các khu vực khác). - Năm 1960, có 17 nước giành được độc lập được gọi là Năm châu Phi - Năm 1975, Môdămbích và Ănggôla giành được độc lập. Đánh dấu sự sụp đổ căn bản của CNTD cũ ở Châu Phi - Từ năm 1980, nhân dân Nam Rôđêdia tuyên bố thành lập nước Cộng Hòa Dimbabuê (1980) và Cộng Hòa Namibia ra đời( 1990) - Đặc biệt năm 1993, tại Nam Phi đã chính thức xóa bỏ chế độ phân biệt chủng tộc (Apacthai). Tháng 4/1994, Nenxơn Manđêla trở thành vị tổng thống da đen đầu tiên của Cộng Hòa Nam Phi -> đánh dấu sự sụp đổ hoàn toàn của chủ nghĩa thực dân. II.Những nét chính về quá trình giành và bảo vệ độc lập ở khu vực Mĩ La Tinh. - Khu vực Mỹ Latinh sớm giành độc lập (đầu thế kỉ XIX), nhưng sau đó lệ thuộc Mĩ - Sau CTTG II cuộc đấu tranh chống chế độ độc tài thân Mỹ bùng nổ và phát triển. Tiêu biểu là thắng lợi của cách mạng Cu Ba do Phiđen Caxtơrô lãnh đạo vào 1/1959. - Dưới ảnh hưởng của cách mạng Cuba, phong trào đấu tranh chống Mĩ và chế độ độc tài thân Mĩ diễn ra sôi nổi ở nhiều nước trong thập kỷ 60 – 70: Vênêxuêla, Goatêmala, Côlômbia, Pêru, Nicaragoa… - Kết quả: chính quyền độc tài nhiều nước bị lật đổ, các chính phủ dân tộc dân chủ được thiết lập -> Mĩlatinh là “lục địa bùng cháy 153 PHẦN II :CÁC DẠNG CÂU HỎI LIÊN QUAN : C©u 1: Cuéc néi chiÕn c¸ch m¹ng ë Trung Quèc (1946-1949) diÔn ra nh thÕ nµo ? Nguyªn nh©n dÉn tíi th¾ng lîi trong cuéc néi chiÕn c¸ch m¹ng 1946-1949 ë Trung Quèc. Hướng dẫn trả lời * Diễn biến - Ngµy 20-7-1946 Tëng Giíi Th¹ch huy ®éng 160 v¹n qu©n chÝnh quy tÊn c«ng toµn diÖn vµo c¸c vïng gi¶i phãng. Do so s¸nh lùc lîng lóc ®Çu cßn chªnh lÖch, tõ th¸ng 7-1946 ®Õn th¸ng 61947, qu©n gi¶i phãng nh©n d©n Trung Quèc thùc hiÖn chiÕn lîc phßng ngù tÝch cùc, kh«ng gi÷ ®Êt mµ chñ yÕu tiªu diÖt sinh lùc ®Þch, ph¸t triÓn lùc lîng m×nh. Sau 1 n¨m qu©n gi¶i phãng ®· tiªu diÖt 1.112.000 qu©n Quèc d©n ®¶ng, ph¸t triÓn lùc lîng chñ lùc m×nh lªn 2 triÖu ngêi. - Tõ th¸ng 6-1947, qu©n gi¶i phãng chuyÓn sang ph¶n c«ng, gi¶i phãng c¸c vïng do Quèc d©n ®¶ng thèng trÞ. Tõ th¸ng 4-1948 ®Õn th¸ng 1 -1949, qu©n gi¶i phãng lÇn lît më ba chiÕn dÞch, tiªu diÖt h¬n 1.540.000 qu©n Quèc d©n ®¶ng, lµm cho lùc lîng cña ®Þch vÒ c¬ b¶n ®· bÞ tiªu diÖt. -Th¸ng 4-1949, qu©n gi¶i phãng vît Trêng Giang, ngµy 23-4, Nam Kinh - trung t©m thèng trÞ cña tËp ®oµn Quèc d©n ®¶ng ®îc gi¶i phãng, nÒn thèng trÞ cña Quèc d©n ®¶ng sôp ®æ. Tëng Giíi Th¹ch bá lôc ®Þa ch¹y ra ®¶o §µi Loan. - Ngµy 1-10-1949, níc Céng hoµ nh©n d©n Trung Hoa tuyªn bè thµnh lËp do Mao Tr¹ch §«ng lµm Chñ tÞch, ®¸nh dÊu th¾ng lîi cña c¸ch m¹ng d©n téc d©n chñ Trung Quèc. - Ý nghÜa lÞch sö: + KÕt thóc 100 n¨m Trung Quèc bÞ ®Õ quèc, phong kiÕn vµ t s¶n m¹i b¶n thèng trÞ vµ ®a nh©n d©n Trung Quèc bíc vµo kÜ nguyªn míi, kÜ nguyªn ®éc lËp, tù do tiÕn lªn CNXH. + Víi diÖn tÝch 1/4 diÖn tÝch ch©u ¸, vµ chiÕm gÇn 1/4 d©n sè toµn nh©n lo¹i. Th¾ng lîi cña C¸ch m¹ng Trung Quèc ®· t¨ng cêng lùc lîng cña CNXH trªn ph¹m vi thÕ giíi vµ cã ¶nh hëng s©u s¾c ®Õn sù ph¸t triÓn cña phong trµo gi¶i phãng d©n téc trªn thÕ giíi . * Nguyên nhân thắng lợi - Sù l·nh ®¹o cña §¶ng Céng s¶n Trung Quèc.... - Tinh thÇn yªu níc, c¨m thï bÌ lò Tëng Giíi Th¹ch, tinh thÇn ®oµn kÕt, tinh thÇn chiÕn ®Êu hi sinh dòng c¶m... cña nh©n d©n Trung Quèc. C©u 2: Cuéc c¸ch m¹ng d©n téc d©n chñ ë Trung Quèc (1946-1949) thµnh c«ng ®· cã ¶nh hëng nh thÕ nµo ®Õn sù nghiÖp c¸ch m¹ng Trung Quèc nãi riªng vµ c¸ch m¹ng thÕ giíi nãi chung. Hướng dẫn trả lời - Sù kiÖn ®ã cã ¶nh hëng ®Õn Trung Quèc: 154 + §¸nh dÊu c¸ch m¹ng d©n téc d©n chñ nh©n d©n ë Trung Quèc ®· thµnh c«ng. Víi th¾ng lîi nµy ®· kÕt thóc sù n« dÞch vµ thèng trÞ cña ®Õ quèc, phong kiÕn vµ t b¶n m¹i b¶n kÐo dµi h¬n 100 n¨m qua. + Më ra cho nh©n d©n Trung Quèc mét kû nguyªn míi: kû nguyªn ®éc lËp, tù do vµ tiÕn lªn CNXH. + Tõ sau th¾ng lîi ®ã, nh©n d©n Trung Quèc díi sù l·nh ®¹o cña §¶ng céng s¶n Trung Quèc liªn tiÕp giµnh nhiÒu th¾ng lîi trong c«ng cuéc x©y dùng ®Êt níc. §Æc biÖt, tõ n¨m 1978 ®Õn nay, víi ®êng lèi ®æi míi, §¶ng céng s¶n vµ nh©n d©n Trung Quèc ®· thu ®îc nhiÒu th¾ng lîi trong sù nghiÖp c«ng nghiÖp ho¸, hiÖn ®¹i ho¸ ®Êt níc. Cã thÕ nãi Trung Quèc lµ níc XHCN ®Çu tiªn tiÕn hµnh c¶i c¸ch më cöa thµnh c«ng. - Thµnh c«ng cña c¸ch m¹ng Trung Quèc cã ¶nh hëng ®Õn sù nghiÖp c¸ch m¹ng thÕ giíi. + Víi diÖn tÝch b»ng 1/4 diÖn tÝch ch©u ¸ vµ chiÕm gÇn 1/4 d©n sè toµn thÕ giíi, th¾ng lîi cña c¸ch m¹ng Trung Quèc cã t¸c ®éng lín ®Õn c¸ch m¹ng thÕ giíi mµ tríc hÕt t¨ng cêng lùc lîng cho phe XHCN vµ ®éng viªn cæ vò phong trµo gi¶i phãng d©n téc trªn toµn thÕ giíi, ®Æc biÖt lµ c¸c níc ¸, Phi, MÜ latinh. + ViÖc Trung Quèc thu ®îc nhiÒu th¾ng lîi tõ sau cuéc c¸ch m¹ng d©n téc d©n chñ(1946-1949) ®· ®Ó l¹i nhiÒu bµi häc cho c¸ch m¹ng c¸c níc, ®Æc biÖt lµ ViÖt Nam: Mét níc gÇn Trung Quèc ®ang tiÕn hµnh c¶i c¸ch, më cöa vµ ®æi míi ®Êt níc. Câu 3:Tóm tắt sự ra đời của các quốc gia độc lập ở Đông Nam Á trong năm 1945(ĐH 2011) TL -Inđônêxia + Ngày 17 – 8 – 1945, sau khi quân phiệt Nhật đầu hàng Đồng minh Xucácnô đọc bản Tuyên ngôn Độc lập, tuyên bố thành lập nước Cộng hoà Inđônêxia. Hưởng ứng Tuyên ngôn Độc lập, nhân dân cả nước nổi dậy giành chính quyền. + Ngày 18 – 8 – 1945, Hội nghị của Uỷ ban trù bị độc lập Inđônêxia, thông qua Hiến pháp, bầu Xucácnô làm Tổng thống nước Cộng hoà Inđônêxia. - Việt Nam + Tháng 8 – 1945, Đảng và Mặt trận Việt Minh quyết định phát động Tổng khởi nghĩa, thành lập Uỷ ban Dân tộc giải phóng do Hồ Chí Minh làm Chủ tịch. Cả dân tộc Việt Nam vùng dậy tổng khởi nghĩa thắng lợi + Ngày 2/9/1945, Chủ tịch Hồ Chí Minh đọc Tuyên ngôn độc lập, thành lập nước Việt Nam Dân chủ Cộng hoà. Lào + Tháng 8 – 1945, lợi dụng thời cơ Nhật đầu hàng Đồng minh, nhân dân Lào nổi dậy giành chính quyền. + Ngày 12 – 10 – 1945, Chính phủ Lào ra mắt quốc dân và tuyên bố độc lập 155 .Câu 4:Tóm tắt nội dung các giai đoạn lịch sử Campuchia từ sau chiến tranh thế giới hai đến 1/1979.( CĐ 2010) Hướng dẫn trả lời -1945-1954 + Nhân dân Campuchia tiến hành kháng chiến chống thực dân Pháp xâm lược. Ngày 9 - 11 - 1953, Pháp kí hiệp ước trao trả độc lập cho Campuchia, nhưng quân Pháp vẫn chiếm đóng đất nước này +Năm 1954, sau chiến thắng Điện Biên Phủ, Pháp phải kí Hiệp định Giơnevơ công nhận độc lập, chủ quyền, thống nhất và toàn vẹn lãnh thổ của Campuchia, Lào, Việt Nam - Từ 1954 – 1970: Chính phủ CPC do Xihanuc lãnh đạo đi theo đường lối hòa bình, trung lập, không tham gia các liên minh quân sự để xây dựng đất nước. - 1970-1975: + ND Campuchia sát cánh cùng nhân dân VN và Lào kháng chiến chống Mĩ + Ngày 17-4-1975, thủ đô Phnôm Pênh được giải phóng, kết thúc thắng lợi cuộc kháng chiến chống Mĩ. - 1975-1979: nhân dân CPC đấu tranh chống chế độ diệt chủng Khơ-me đỏ do Pôn-Pốt cầm đầu Tháng 12/1978 mặt trận dân tộc cứu nước Campuchia thành lập, ngày 7-1-1979, thủ đô Phnôm Pênh được giải phóng, nước Cộng hoà Campuchia ra đời Câu 5: Hãy phân chia các giai ®o¹n cña c¸ch m¹ng Lµo1945-1975 và tóm tắt diễn biến từng giai đoạn. (ĐH 2009) Hướng dẫn trả lời - Tõ 1945-1954: Giai ®o¹n kh¸ng chiÕn chèng Ph¸p x©m lîc. + Th¸ng 8-1945 lîi dông thêi c¬ NhËt ®Çu hµng, nh©n d©n Lµo næi dËy khëi nghÜa thµnh lËp chÝnh quyÒn c¸ch m¹ng ë nhiÒu n¬i. Ngµy 12-10-1945 nh©n d©n thñ ®« Viªn Ch¨n khëi nghÜa giµnh chÝnh quyÒn, chÝnh phñ C¸ch m¹ng Lµo ra m¾t quèc d©n, tuyªn bè nÒn ®éc lËp cña Lµo. + Th¸ng 3-1946, thùc d©n Ph¸p quay trë l¹i x©m lîc, nh©n d©n Lµo ®øng lªn kh¸ng chiÕn chèng Ph¸p. Ngµy 13-8-1950, MÆt trËn Lµo tù do vµ chÝnh phñ kh¸ng chiÕn Lµo thµnh lËp do hoµng th©n Xuphanuvong ®øng ®Çu. + Phèi hîp víi chiÕn trêng ViÖt Nam vµ C¨mpuchia, ®îc sù gióp ®ì cña qu©n t×nh nguyÖn ViÖt Nam, qu©n d©n Lµo ®· giµnh ®îc nhiÒu th¾ng lîi trong nh÷ng n¨m 1953-1954 buéc thùc d©n Ph¸p ph¶i ký hiÖp ®Þnh Gi¬nev¬ c«ng nhËn ®éc lËp, chñ quyÒn vµ toµn vÑn l·nh thæ cña Lµo. - Tõ 1954 - 1975: Giai ®o¹n kh¸ng chiÕn chèng MÜ x©m lîc. + Sau khi Ph¸p thÊt b¹i ®Õ quèc MÜ ph¸t ®éng chiÕn tranh x©m lîc thùc d©n kiÓu míi, nh»m biÕn Lµo thµnh thuéc ®Þa kiÓu míi cña MÜ. + Díi sù l·nh ®¹o cña §¶ng nh©n d©n C¸ch m¹ng Lµo (thµnh lËp th¸ng 31955), qu©n d©n Lµo ®øng lªn kh¸ng chiÕn chèng MÜ. §Õn ®Çu nh÷ng n¨m 60 vïng gi¶i phãng chiÕm 2/3 diÖn tÝch, 1/3 d©n sè trong c¶ níc. Tõ 1964 ®Õn 1973 156 nh©n d©n Lµo ®· ®¸nh b¹i cuéc "ChiÕn tranh ®Æc biÖt" cña MÜ, buéc MÜ vµ tay sai ph¶i ký HiÖp ®Þnh Viªng Ch¨n (21-2-1973), lËp l¹i nÒn hoµ b×nh, thùc hiÖn hoµ hîp d©n téc ë Lµo. + Th¾ng lîi cña c¸ch m¹ng ViÖt Nam 30-4-1975 ®· cæ vò vµ t¹o ®iÒu kiÖn thuËn lîi cho c¸ch m¹ng Lµo tiÕn lªn giµnh th¾ng lîi hoµn toµn. Díi sù l·nh ®¹o cña §¶ng Nh©n d©n C¸ch m¹ng Lµo tõ th¸ng 5 ®Õn th¸ng 12-1975, nh©n d©n Lµo næi dËy giµnh chÝnh quyÒn trong c¶ níc. Ngµy 2-12-1975, níc Céng hoµ D©n chñ nh©n d©n Lµo chÝnh thøc thµnh lËp. - Ý nghÜa lÞch sö c¸ch m¹ng Lµo: Th¾ng lîi cña 30 n¨m ®Êu tranh gian khæ cã ý nghÜa lÞch sö träng ®¹i cña níc Lµo. Díi sù l·nh ®¹o cña §¶ng Nh©n d©n c¸ch m¹nh Lµo, qu©n ®éi vµ nh©n d©n Lµo ®·: + §¸nh th¾ng bän thùc d©n x©m lîc Ph¸p, vµ bän x©m lîc MÜ, giµnh ®éc lËp, chñ quyÒn vµ toµn vÑn l·nh thæ. + ChÊm døt nÒn qu©n chñ phong kiÕn, thµnh lËp Nhµ níc céng hßa d©n chñ nh©n d©n. + Hoµn thµnh th¾ng lîi cuéc c¸ch m¹ng d©n téc d©n chñ, ®a níc Lµo bíc sang thêi kú ph¸t triÓn míi. + §¸nh dÊu th¾ng lîi míi cña t×nh ®oµn kÕt gi÷a ba níc §«ng D¬ng trong cuéc ®Êu tranh chèng ®Õ quèc, giµnh ®éc lËp tù do vµ x©y dùng ®Êt níc. T×nh ®oµn kÕt h÷u nghÞ ViÖt Lµo ®· ®îc thö th¸ch trong khãi l÷a chiÕn tranh, ngµy cµng ph¸t triÓn trong c«ng cuéc x©y dùng hßa b×nh. Câu 6: Nh÷ng biÕn ®æi vÒ kinh tÕ, chÝnh trÞ vµ x· héi ë §«ng Nam ¸ tríc vµ sau chiÕn tranh thÕ giíi thø hai. Hướng dẫn trả lời - Tríc chiÕn tranh: Lµ nh÷ng níc thuéc ®Þa, lÖ thuéc vµo c¸c níc t b¶n ph¬ng T©y, bÞ c¸c níc t b¶n ph¬ng T©y ra søc bèc lét tµn b¹o Phong trµo ®Êu tranh gi¶i phãng d©n téc tuy diÔn ra m¹nh mÏ nhng ®Òu thÊt b¹i... - Tõ sau chiÕn tranh: LÇn lît c¸c níc ®Òu giµnh ®îc ®éc lËp d©n téc víi c¸c chÕ ®é chÝnh trÞ phï hîp cho mçi níc. Tõ sau khi giµnh ®îc ®éc lËp d©n téc c¸c níc ®Òu ra søc x©y dùng ph¸t triÓn nÒn kinh tÕ x· héi cña m×nh, nhiÒu níc ®¹t ®îc nhiÒu thµnh tùu to lín (NIC, con rång); c¸c níc §«ng Nam ¸ chuyÓn tõ ®èi ®Çu sang ®èi tho¹i vµ hîp t¸c, ®Òu trë thµnh thµnh viªn cña ASEAN. C©u 7: Nêu hoàn cảnh ra đời và qu¸ tr×nh ph¸t triÓn cña HiÖp héi c¸c níc §«ng Nam ¸ (ASEAN) ?( CĐ 2009). Hướng dẫn trả lời - Hoµn c¶nh ra ®êi: + Sau khi giµnh ®éc ®éc lËp, c¸c níc §«ng Nam ¸ ra søc kh«i phôc vµ ph¸t triÓn kinh tÕ. Trong khi 3 níc §«ng D¬ng ph¶i tiÕn hµnh cuéc chiÕn tranh cøu níc gian khæ. + Th¸ng 8-1967, "HiÖp héi c¸c níc §«ng Nam ¸" (ASEAN) thµnh lËp t¹i B¨ng Cèc (Th¸i Lan) gåm c¸c níc: In®«nªsia, Malaixia, Xingapo, Th¸i Lan vµ 157 Philippin. HiÖn nay sè thµnh viªn cña ASEAN lµ 10 níc. ViÖt Nam gia nhËp tæ chøc ASEAN vµo th¸ng 7-1995. Trong t¬ng lai, §«ng timo còng sÏ lµ thµnh viªn cña "HiÖp héi c¸c níc §«ng Nam ¸" + Môc tiªu cña ASEAN: N¨m 1976, Héi nghÞ cÊp cao ASEAN häp ë Bali (In®«nªxia) ký hiÖp íc h÷u nghÞ vµ nªu râ môc ®Ých cña ASEAN lµ: Môc ®Ých: X©y dùng mèi quan hÖ hoµ b×nh, h÷u nghÞ vµ hîp t¸c gi÷a c¸c níc trong khu vùc, t¹o nªn mét céng ®ång §«ng Nam ¸ hïng m¹nh trªn c¬ së tù cêng khu vùc. ThiÕt lËp mét khu vùc hoµ b×nh, tù do, trung lËp ë §«ng Nam ¸. Nh vËy, ASEAN lµ tæ chøc Liªn minh chÝnh trÞ - kinh tÕ cña khu vùc §«ng Nam ¸. - Qu¸ tr×nh ph¸t triÓn: Ho¹t ®éng cña ASEAN tr¶i qua c¸c giai ®o¹n ph¸t triÓn chÝnh: + Tõ 1967 ®Õn 1975: ASEAN cßn lµ mét tæ chøc khu vùc non yÕu, ch¬ng tr×nh hîp t¸c gi÷a c¸c níc thµnh viªn cßn rêi r¹c. + Th¸ng 2-1976, c¸c níc ASEAN ®· ký "HiÖp íc h÷u nghÞ vµ hîp t¸c" (t¹i Héi nghÞ cÊp cao ë Bali, In®«nªxia) nªu râ môc tiªu x©y dùng nh÷ng mèi quan hÖ h÷u nghÞ hîp t¸c gi÷a c¸c níc trong khu vùc t¹o nªn mét céng ®ång §NA hïng m¹nh trªn c¬ së tù cêng khu vùc, thiÕt lËp mét khu vùc hoµ b×nh, tù do, trung lËp ë §NA. ASEAN trë thµnh mét tæ chøc chÝnh trÞ - kinh tÕ cña khu vùc §NA. + 1979 ASEAN cã quan hÖ ®èi ®Çu víi ba níc §«ng D¬ng (chñ yÕu xoay quanh vÊn ®Ò Campuchia). §Õn cuèi thËp niªn 80 ASEAN ®· chuyÓn sang ®èi tho¹i, hîp t¸c trong cïng tån t¹i hoµ b×nh víi ba níc §«ng D¬ng. Sau khi vÊn ®Ò Campuchia ®îc gi¶i quyÕt, ASEAN vµ ba níc §«ng D¬ng ®· ph¸t triÓn mèi quan hÖ hîp t¸c trªn c¸c lÜnh vùc kinh tÕ, v¨n ho¸, khoa häc... + N¨m 1995, ViÖt Nam gia nhËp ASEAN, sau ®ã lµ gia nhËp cña c¸c níc Lµo, Mianma (1997), Campuchia (1999). ASEAN gåm 10 níc ®· trë thµnh "ASEAN toµn §NA" - Thêi c¬, th¸ch thøc ViÖt Nam khi gia nhËp tæ chøc nµy: + Thêi c¬: T¹o ®iÒu kiÖn cho ViÖt Nam ®îc hoµ nhËp vµo céng ®ång khu vùc, vµo thÞ trêng c¸c níc §«ng Nam ¸. Thu hót ®îc vèn ®Çu t, më ra c¬ héi giao lu häc tËp, tiÕp thu tr×nh ®é khoa häc - kÜ thuËt, c«ng nghÖ vµ v¨n ho¸.... ®Ó ph¸t triÓn ®Êt níc ta. + Th¸ch thøc: ViÖt Nam ph¶i chÞu sù c¹nh tranh quyÕt liÖt, nhÊt lµ vÒ kinh tÕ. Hoµ nhËp nÕu kh«ng ®øng v÷ng th× dÔ bÞ tôt hËu vÒ kinh tÕ vµ bÞ "hoµ tan" vÒ chÝnh trÞ, v¨n ho¸ x· héi... C©u 8: Tõ sau chiÕn tranh thÕ giíi thø hai ®Õn nay §«ng Nam ¸ cã nh÷ng biÕn ®æi to lín g× ? Theo Anh (chÞ) trong nh÷ng biÕn ®æi lín ®ã th× biÕn ®æi lín nµo lµ quan träng nhÊt ? V× sao ? Hướng dẫn trả lời - Tõ sau chiÕn tranh thÕ giíi thø hai §«ng Nam ¸ cã nh÷ng biÕn ®æi lín sau ®©y: 158 + BiÕn ®æi thø nhÊt: C¸c níc §«ng Nam ¸ tõ th©n phËn c¸c níc thuéc ®Þa, n÷a thuéc ®Þa vµ lÖ thuéc ®· trë thµnh nh÷ng níc ®éc lËp. (KÓ tãm t¾t 10 níc §«ng Nam ¸ vµ cho biÕt thuéc ®Þa cña níc nµo, vµ giµnh ®éc lËp n¨m nµo theo gîi ý sau) - BiÕn ®æi thø hai: Tõ sau khi giµnh l¹i ®éc lËp, c¸c níc §«ng Nam ¸ ®Òu ra søc x©y dùng vµ ph¸t triÓn nÒn kinh tÕ - x· héi cña m×nh vµ ®¹t nhiÒu thµnh tùu to lín: nh Xingapo, In®«nªxia, Th¸i Lan, Malaixia, ®Æc biÖt lµ Xingapo, níc cã nÒn kinh tÕ ph¸t triÓn nhÊt ë khu vùc §NA vµ ®îc xÕp vµo hµng níc ph¸t triÓn trªn thÕ giíi. - BiÕn ®æi thø ba: Cho ®Õn th¸ng 4-1999, c¸c níc §NA ®Òu gia nhËp HiÖp héi c¸c níc §NA, gäi t¾t lµ ASEAN nh»m môc tiªu x©y dùng nh÷ng mèi quan hÖ hoµ b×nh, h÷u nghÞ hîp t¸c gi÷a c¸c níc trong khu vùc. Trong nh÷ng biÕn ®æi ®ã biÕn ®æi nµo lµ quan träng nhÊt: Lµ biÕn ®æi tõ th©n phËn c¸c níc thuéc ®Þa, n÷a thuéc ®Þa vµ lÖ thuéc ®· trë thµnh nh÷ng níc ®éc lËp. Nhê cã biÕn ®æi ®ã c¸c níc §NA míi cã nh÷ng ®iÒu kiÖn thuËn lîi ®Ó x©y dùng vµ ph¸t triÓn kinh tÕ - x· héi cña m×nh ngµy cµng phån vinh. C©u 9: H·y nªu nh÷ng hiÓu biÕt cña m×nh vÒ tæ chøc ASEAN ? Sù gia nhËp cña ViÖt Nam vµo ASEAN ? Hướng dẫn trả lời - Hoµn c¶nh ra ®êi: Sau khi giµnh ®éc ®éc lËp, nhiÒu níc §«ng Nam ¸ dù ®Þnh thµnh lËp mét tæ chøc khu vùc nh»m hîp t¸c, gióp ®ì lÉn nhau vÒ kinh tÕ, khoa häc kÜ thuËt vµ v¨n ho¸, ®ång thêi ®Ó h¹n chÕ ¶nh hëng cña c¸c níc lín ®èi víi c¸c níc trong khu vùc. Ngµy 8- 8-1967, "HiÖp héi c¸c níc §«ng Nam ¸" (ASEAN) thµnh lËp t¹i B¨ng Cèc (Th¸i Lan) gåm c¸c níc: In®«nªsia, Malaixia, Xingapo, Th¸i Lan vµ Philippin. HiÖn nay sè thµnh viªn cña ASEAN lµ 10 níc. ViÖt Nam gia nhËp tæ chøc ASEAN vµo th¸ng 7-1995. - Môc tiªu: T¬ng trî, hîp t¸c vÒ kinh tÕ; x©y dùng mét §«ng Nam ¸ thÞnh vîng, an ninh, hoµ b×nh trung lËp vµ cïng ph¸t triÓn. Nh vËy, ASEAN lµ tæ chøc Liªn minh chÝnh trÞ – kinh tÕ cña khu vùc §«ng Nam ¸. - C¬ cÊu tæ chøc: + Héi nghÞ thîng ®Ønh: 3 n¨m häp 1 lÇn. + Héi nghÞ ngo¹i trëng: mçi n¨m häp 1 lÇn + Uû ban thêng trùc: ®¶m nhËn c«ng viÖc gi÷a hai kú häp cña héi nghÞ ngo¹i trëng. + HÖ thèng c¸c Uû ban thêng trùc phô tr¸ch c¸c ngµnh. - C¸c giai ®o¹n chÝnh: + Giai ®o¹n ®Çu (1967-1975) - Tõ 1967 - 1975: ASEAN cßn non yÕu, cha cã ho¹t ®éng næi bËt, mäi ngêi Ýt biÕt ®Õn. 159 - Tõ 1976 ®Õn nay: ASEAN ngµy cµng ph¸t triÓn, cã nh÷ng ®ãng gãp tÝch cùc trong khu vùc vµ trªn thÕ giíi. HiÖn nay ASEAN trë thµnh mét tæ chøc cña tÊt c¶ c¸c níc trong khu vùc: N¨m 1984, brun©y trë thµnh thµnh viªn thø s¸u cña ASEAN, n¨m 1995 ViÖt Nam lµ thµnh viªn thø chÝnh thøc cña ASEAN, n¨m 1997, Lµo vµ Mianma còng gia nhËp ASEAN, n¨m 1999 Campuchia, thµnh viªn thø 10, thµnh viªn thø cuèi cïng cña ASEAN ®îc kÕt n¹p. * Mèi quan hÖ ViÖt Nam víi ASEAN. - Tõ 1967-1972: Do mét sè níc dÝnh lÝu vµo cuéc chiÕn tranh cña MÜ ë ViÖt Nam nªn quan hÖ ViÖt Nam - ASEAN rÊt h¹n chÕ. - Tõ 1973-1986: T×nh h×nh khu vùc cã nhiÒu chuyÓn biÕn. Quan hÖ ViÖt Nam - ASEAN ®· ®îc c¶i thiÖn. Tuy nhiªn do "vÊn ®Ò Campuchia" mµ mèi quan hÖ gi÷a ViÖt Nam - ASEAN vÉn cßn c¨ng th¼ng. - Tõ 1986, nhÊt lµ tõ cuèi thËp niªn 80, do"vÊn ®Ò Campuchia" ®· ®îc gi¶i quyÕt nªn quan hÖ ViÖt Nam - ASEAN ®· chuyÓn sang ®èi tho¹i, th©n thiÖn hîp t¸c. - HiÖn nay, quan hÖ ViÖt Nam - ASEAN ngµy cµng ph¸t triÓn toµn diÖn, cã hiÖu qu¶. ViÖt Nam trë thµnh thµnh viªn thø 7 cña ASEAN (7-1995) vµ cã vai trß tÝch cùc trªn nhiÒu lÜnh vùc trong ASEAN. * Sù gia nhËp cña ViÖt Nam vµo ASEAN ? - T¸n thµnh nh÷ng nguyªn t¾c cña tæ chøc ASEAN, th¸ng 7-1992, t¹i Manila (Philippin) ViÖt Nam vµ Lµo ®· gia nhËp HiÖp íc Bali, trë thµnh quan s¸t viªn chÝnh thøc cña ASEAN. - Ngµy 28-07-1995, ViÖt Nam ®· gia nhËp ASEAN. ®a sè thµnh viªn cña tæ chøc nµy lªn 7 níc. §©y lµ mét sù kiÖn quan träng trong viÖc thóc ®Èy xu thÕ hoµ b×nh, æn ®Þnh vµ hîp t¸c ë khu vùc §«ng Nam ¸. - Ngµy 23-7-1997, HiÖp héi c¸c níc §«ng Nam ¸ kÕt n¹p thªm Lµo, Mianma. Tõ ngµy 30-4-1999, Campuchia lµ thµnh viªn thø 10 cña tæ chøc nµy. Nh vËy, ASEAN ®· trë thµnh "ASEAN toµn §«ng Nam ¸". H¬n 30 n¨m, kÓ tõ khi ra ®êi, ASEAN ®· ®¹t ®îc nh÷ng thµnh tùu to lín vµ tèc ®é t¨ng trëng kinh tÕ cao, t¹o nªn nh÷ng biÕn ®æi s©u s¾c trong ®êi sèng kinh tÕ x· héi c¸c níc thµnh viªn. MÆc dï cã nh÷ng bíc th¨ng trÇm, vai trß quèc tÕ ASEAN (víi t c¸ch lµ mét tæ chøc chÝnh trÞ kinh tÕ khu vùc) ngµy cµng t¨ng. Câu 10: Từ năm 1950 đến năm 2000, vị thế ngày càng nâng cao trên trường quốc tế của Ấn Độ được thể hiện như thế nào trên lĩnh vực kinh tế, khoa học - kĩ thuật và chính sách đối ngoại. (ĐH 2012) Câu 11: C¸c giai ®o¹n ph¸t triÓn vµ th¾ng lîi cña phong trµo gi¶i phãng d©n téc ë Ch©u Phi tõ 1945 ®Õn nay ? Hướng dẫn trả lời - Nh÷ng nÐt chung: + Ch©u Phi cã 57 quèc gia víi diÖn tÝch 30,3 triÖu km 2 (gÊp 3 lÇn ch©u ¢u, xÊp xØ ch©u MÜ vµ b»ng 3/4 ch©u ¸). Víi d©n sè kho¶ng 650 triÖu ngêi. Ch©u Phi cã tµi nguyªn phong phó vµ nhiÒu n«ng s¶n quý. Nhng díi ¸ch thèng trÞ cña thùc 160 d©n ph¬ng T©y trong nhiÒu thÕ kÜ ch©u Phi trë thµnh nghÌo nµn, l¹c hËu h¬n nhiÒu so víi ch©u lôc kh¸c. + Sau chiÕn tranh thÕ giíi thø hai, phong trµo c¸ch m¹ng gi¶i phãng d©n téc bïng næ vµ ph¸t triÓn m¹nh mÏ ë ch©u Phi. Ch©u phi trë thµnh "Lôc ®Þa míi trçi dËy" trong cuéc ®Êu tranh chèng chñ nghÜa ®Õ quèc, chñ nghÜa thùc d©n. - C¸c giai ®o¹n: Tõ sau chiÕn tranh thÕ giíi thø hai ®Õn nay, phong trµo gi¶i phãng d©n téc ë ch©u Phi ®· tr¶i qua c¸c giai ®o¹n sau: + 1945-1954: Phong trµo bïng næ ®Çu tiªn ë B¾c Phi víi th¾ng lîi më ®Çu lµ cuéc chÝnh biÕn c¸ch m¹ng cña binh lÝnh vµ sÜ quan yªu níc Ai CËp (3-7-1952), lËp ®æ v¬ng triÒu Pharóc vµ nÒn thèng trÞ cña thùc d©n Anh, thµnh lËp níc Céng hoµ Ai CËp (18-6-1953). + 1954-1960: Do ¶nh hëng cña chiÕn th¾ng lÞch sö §iÖn Biªn Phñ n¨m 1954 lµm rung chuyÓn hÖ thèng thuéc ®Þa cña Ph¸p ë B¾c Phi, T©y Phi, nh©n d©n ch©u Phi ®· vïng dËy, më ®Çu b»ng cuéc ®Êu tranh vò trang cña nh©n d©n Angiªri vµo th¸ng 11 -1954. Sau ®ã nhiÒu quèc gia ®· giµnh ®îc ®éc lËp d©n téc nh: Tuyni®i (1956), Marèc (1956), Xu ®¨ng (1956), Gana (1956), Ghinª (1958)... Trong nh÷ng n¨m 1954 ®Õn 1960, hÇu hÕt c¸c níc B¾c Phi vµ T©y Phi giµnh ®îc ®éc lËp. + 1960 -1975: N¨m 1960, 17 níc ch©u Phi giµnh ®îc ®éc lËp - lÞch sö gäi "n¨m ch©u Phi"; tiÕp ®ã lµ th¾ng lîi cña nh©n d©n Angªri (3-1962), £ti«pi (1974), M«d¨mbÝch (1975), ®Æc biÖt lµ th¾ng lîi cña nh©n d©n ¡ng«la dÉn ®Õn sù ra ®êi cña níc Céng hßa (11-1975) ®¸nh dÊu sù sôp ®æ vÒ c¬ b¶n hÖ thèng thuéc ®Þa cña chñ nghÜa thùc d©n. + 1975 - nay: Giai ®o¹n hoµn thµnh cuéc ®Êu tranh chèng ¸ch thèng trÞ cña chñ nghÜa thùc d©n cò ®Ó giµnh ®éc lËp d©n téc víi sù ra ®êi cña níc Céng hßa Namibia (3-1991). Tuy nhiªn sau khi giµnh l¹i ®éc lËp d©n téc, trong sù nghiÖp x©y dùng ®Êt níc, còng cè ®éc lËp d©n téc hiÖn nay, nhiÒu níc ch©u Phi ®ang gÆp nh÷ng khã kh¨n: sù x©m nhËp cña chñ nghÜa thùc d©n míi, nî chång chÊt, n¹n mï ch÷, ®ãi rÐt, bÖnh tËt lu«n x·y ra, d©n sè qu¸ ®«ng, t×nh h×nh chÝnh trÞ kh«ng æn ®Þnh (do xung ®ét c¸c phe ph¸t, bé téc...) - §Æc ®iÓm phong trµo gi¶i phãng d©n téc ë ch©u Phi: So víi ch©u ¸ vµ MÜlatinh, phong trµo gi¶i phãng d©n téc ë ch©u Phi cã 1 sè ®Æc ®iÓm riªng nh sau: - C¸c níc ch©u Phi ®· ®oµn kÕt víi nhau trong cuéc ®Êu tranh gi¶i phãng d©n téc th«ng qua Tæ chøc thèng nhÊt ch©u Phi gi÷ vai trß quan träng trong viÖc phèi hîp hµnh ®éng vµ thóc ®Èy sù nghiÖp ®Êu tranh c¸ch m¹ng cña c¸c níc ch©u Phi. - L·nh ®¹o phong trµo gi¶i phãng d©n téc ë c¸c níc ch©u Phi hÇu hÕt ®Òu do c¸c chÝnh ®¶ng hoÆc c¸c tæ chøc chÝnh trÞ cña giai cÊp t s¶n d©n téc, cßn giai cÊp v« s¶n cha trëng thµnh, hoÆc cha cã chÝnh ®¶ng ®éc lËp. - H×nh thøc ®Êu tranh giµnh ®éc lËp chñ yÕu lµ th«ng qua ®Êu tranh chÝnh trÞ hîp ph¸p ®Ó ®îc c«ng nhËn ®éc lËp: c¸c níc ch©u Phi giµnh ®îc ®éc lËp ë nhiÒu møc ®é kh¸c nhau vµ sù ph¸t triÓn kinh tÕ - x· héi còng rÊt kh¸c nhau tõ sau khi 161 giµnh ®îc ®éc lËp (vïng B¾c Phi ph¸t triÓn nhanh, vïng ch©u Phi xÝch ®¹o ph¸t triÓn chËm...) C©u 12: Tr×nh bµy nh÷ng ®Æc ®iÓm cña phong trµo gi¶i phãng d©n téc ë ch©u Phi tõ sau chiÕn tranh thÕ giíi thø hai ®Õn nay. Ph©n tÝch nh÷ng nÐt kh¸c biÖt c¬ b¶n vÒ ®èi tîng vµ môc tiªu ®Êu tranh gi÷a c¸c phong trµo gi¶i phãng d©n téc ë ch©u Phi, ch©u ¸ víi khu vùc MÜ latinh trong thêi kú lÞch sö nµy. Hướng dẫn trả lời - C¸c níc ch©u Phi ®· ®oµn kÕt víi nhau trong cuéc ®Êu tranh gi¶i phãng d©n téc th«ng qua Tæ chøc thèng nhÊt ch©u Phi gi÷ vai trß quan träng trong viÖc phèi hîp hµnh ®éng vµ thóc ®Èy sù nghiÖp ®Êu tranh c¸ch m¹ng cña c¸c níc ch©u Phi. - L·nh ®¹o phong trµo gi¶i phãng d©n téc ë c¸c níc ch©u Phi hÇu hÕt ®Òu do c¸c chÝnh ®¶ng hoÆc c¸c tæ chøc chÝnh trÞ cña giai cÊp t s¶n d©n téc, cßn giai cÊp v« s¶n cha trëng thµnh, hoÆc cha cã chÝnh ®¶ng ®éc lËp. - H×nh thøc ®Êu tranh giµnh ®éc lËp chñ yÕu lµ th«ng qua ®Êu tranh chÝnh trÞ hîp ph¸p ®Ó ®îc c«ng nhËn ®éc lËp: c¸c níc ch©u Phi giµnh ®îc ®éc lËp ë nhiÒu møc ®é kh¸c nhau vµ sù ph¸t triÓn kinh tÕ - x· héi còng rÊt kh¸c nhau tõ sau khi giµnh ®îc ®éc lËp (vïng B¾c Phi ph¸t triÓn nhanh, vïng ch©u Phi xÝch ®¹o ph¸t triÓn chËm...) * NÐt kh¸c biÖt c¬ b¶n gi÷a phong trµo gi¶i phãng d©n téc ë ch©u ¸, ch©u Phi víi khu vùc MÜ latinh. - Nh©n d©n ch©u ¸, ch©u Phi ®Êu tranh chèng l¹i bän ®Õ quèc, thùc d©n vµ tay sai ®Ó gi¶i phãng d©n téc, giµnh l¹i ®éc lËp chñ quyÒn. -Khu vùc MÜ latinh ®Êu tranh chèng l¹i c¸c thÕ lùc th©n MÜ ®Ó thµnh lËp c¸c chÝnh phñ d©n téc, d©n chñ, qua ®ã giµnh l¹i ®éc lËp vµ chñ quyÒn thùc sù cho d©n téc. Câu 13: C¸c giai ®o¹n ph¸t triÓn vµ th¾ng lîi cña phong trµo gi¶i phãng d©n téc ë MÜ la tinh tõ sau chiến tranh thế giới thứ hai? Hướng dẫn trả lời Khu vùc MÜ latinh bao gåm Mªhic« (B¾c MÜ), toµn bé Trung vµ Nam MÜ; rÊt giµu vÒ n«ng s¶n, l©m s¶n vµ kho¸ng s¶n. - Tríc chiÕn tranh thÕ giíi thø hai vÒ h×nh thøc, h¬n 20 níc céng hoµ ë MÜ latinh ®Òu lµ nh÷ng quèc gia ®éc lËp; trªn thùc tÕ lµ thuéc ®Þa kiÓu míi - trë thµnh "s©n sau" cña MÜ. - Sau chiÕn tranh thÕ giíi thø hai, phong trµo gi¶i phãng d©n téc ë MÜ latinh ph¸t triÓn m¹nh mÏ (®îc gäi lµ "®¹i lôc nói l÷a"), thÓ hiÖn qua ba giai ®o¹n. + 1945-1959: Cao trµo c¸ch m¹ng næ ra hÇu kh¾p c¸c níc MÜ latinh díi nhiÒu h×nh thøc b¶i c«ng cña c«ng nh©n (Chilª), næi dËy cña n«ng d©n (Pªru, Ecua®o, Mªhic«, Baraxin, Vªnªxuªla..., khëi nghÜa vò trang (Panama, B«livia) vµ ®Êu tranh nghÞ viÖn (Goatªmala, Achentina, Vªnªxuªla)... 162 + 1959 ®Õn cuèi nh÷ng n¨m 80: C¸ch m¹ng Cuba th¾ng lîi (1959) ®¸nh dÊu bíc ph¸t triÓn míi cña phong trµo gi¶i phãng d©n téc, cæ vò cuéc ®Êu tranh cña c¸c níc MÜ latinh. TiÕp ®ã phong trµo ®Êu tranh vò trang bïng næ nhiÒu níc... MÜ latinh trë thµnh "lôc ®Þa bïng ch¸y". Do ¸p lùc ®Êu tranh cña quÇn chóng (díi nhiÒu h×nh thøc), c¸c chÝnh quyÒn ph¶n ®éng tay sai cña MÜ lÇn lît bÞ lËt ®æ, c¸c chÝnh phñ d©n téc d©n chñ ®îc thµnh lËp ®Ó còng cè ®éc lËp vµ chñ quyÒn d©n téc míi giµnh ®îc. + Tõ cuèi thËp niªn 80 ®Õn 1991: Do nh÷ng biÕn ®éng kh«ng cã lîi cho phong trµo c¸ch m¹ng thÕ giíi ë Liªn x« vµ §«ng ¢u, MÜ t¨ng cêng chèng l¹i phong trµo c¸ch m¹ng ë MÜ latinh (Grªna®a, Panama...), uy hiÕp vµ ®e do¹ c¸c m¹ng ë Nicaragoa, t×m mäi c¸ch ph¸ ho¹i chñ nghÜa x· héi ë Cuba... Qua h¬n 4 thËp niªn ®Êu tranh, c¸c níc MÜ latinh ®· kh«i phôc l¹i ®éc lËp, chñ quyÒn vµ bíc lªn vò ®µi quèc tÕ víi t thÕ ®éc lËp, tù chñ, kinh tÕ ngµy cµng ph¸t triÓn (Braxin, Mªhic«..) 163 SỞ GD & ĐT TỈNH VĨNH PHÚC TRƯỜNG THPT VĨNH YÊN CHUYÊN ĐỀ ĐỊA LÍ DÂN CƯ HƯỚNG DẪN ÔN LUYỆN PHẦN ĐỊA LÍ DÂN CƯ LỚP 12 BẰNG PHƯƠNG PHÁP SỬ DỤNG ‘’SƠ ĐỒ’’ Người thực hiện : Lê Thị Thuý Oanh Môn/nhóm môn : Địa lí Tổ : Văn - Sử - Địa - GDCD ĐT: 0986 054 792 Email: thuyoanh@thptvinhyen. edu.vn 164 Tháng 2 năm 2014 165 CHUYÊN ĐỀ ĐỊA LÍ DÂN CƯ HƯỚNG DẪN ÔN LUYỆN PHẦN ĐỊA LÍ DÂN CƯ LỚP 12 BẰNG PHƯƠNG PHÁP SỬ DỤNG ‘’SƠ ĐỒ’’ Tác giả thực hiện: Lê Thị Thuý Oanh - Trường THPT Vĩnh Yên Đối tượng thực hiện: - Học sinh lớp 12 - Dự kiến số tiết bồi dưỡng: 6 tiết I. Đặt vấn đề Phần Địa lí dân cư trong chương trình Địa lí lớp 12 có 3 nội dung theo chương trình Cơ bản, 4 nội dung theo chương trình Nâng cao. Theo cấu trúc đề thi đại học, phần này tối thiểu được 1,0 điểm. Đây được đánh giá là phần điểm rất “ngon ăn”. Tuy nhiên phần này lại có rất nhiều ý nhỏ, học sinh dễ bỏ sót ý khi làm bài. Do đó tôi đã hướng dẫn học sinh sử dụng “sơ đồ” thông qua các từ khóa để ghi nhớ chi tiết nội dung phần này. Thông qua các từ khóa, việc ghi nhớ kiến thức sẽ trở nên dễ dàng hơn, đồng thời phát huy được năng lực viết bài của các em. II. Hệ thống kiến thức phần Địa lí dân cư (theo chương trình Cơ bản) Nội dung1: Đặc điểm dân số và phân bố dân cư 1. Đông dân, có nhiều thành phần dân tộc a) Đông dân - Dân động: theo số liệu thống kê năm 2006 dân số nước ta là 84,1 triệu người, đứng thứ 3 trong khu vực Đông Nam Á (Sau Inđônêxia và Philippin), đứng thứ 13 trên thế giới trong tổng số trên 200 quốc gia và vùng lãnh thổ - Thuận lợi: Dân đông là thị trường tiêu thụ rộng lớn, thúc đẩy sự phát triển kinh tế, đồng thời là nguồn nhân lực quan trọng để xây dựng bảo vệ đất nước. - Khó khăn: Trong điều kiện nước ta hiện nay dân đông là trở ngại cho phát triển kinh tế nâng cao đời sống vật chất và tinh thần cho người dân. b) Nhiều thành phần dân tộc - Việt Nam nằm ở vị trí thuận lợi nên nước ta có nhiều dân tộc: 54 dân tộc, nhiều nhất là dân tộc Kinh (86,2%) các dân tộc khác chiếm 13,8% (Thái, Mường, Tày...) 166 - Ngoài ra còn có 3,2 triệu người Việt Nam sinh sống ở nước ngoài (Hoa Kỳ, Ôxtrâylia, Châu Âu, Nam Á...) - Thuận lợi: các dân tộc luôn đoàn kết bên nhau, phát huy truyền thống văn hoá, phong tục tập quán để phát triển và xây dựng đất nước - Khó khăn: sự phát triển kinh tế xã hội của các dân tộc còn có sự chênh lệch. Nhà nước phải có các chính sách phát triển kinh tế - xã hội cùng dân tộc ít người nhằm nâng cao chất lượng cuộc sống cho đồng bảo dân tộc. 2. Dân số còn tăng nhanh, cơ cấu dân số trẻ a) Dân số tăng nhanh - Dân số nước ta tăng nhanh, đặc biệt vào nửa cuối những năm 50 của thế kỷ XX nước ta trải qua quá trình bùng nổ dân số. Tuy nhiên bùng nổ dân số giữa các giai đoạn, các vùng lãnh thổ và các dân tộc có sự khác nhau. Dân số (triệu người) Tỷ lệ tăng dân số (%) 1921 15,6 1921-1926: 1,86 1960 30,2 1939-1943: 3,06 1990 66,2 1954-1960: 3,93 1999 76,3 1999-2002: 1,32 2006 84,1 2002-2005: 1,32 Năm - Do thực hiện chính sách KHHGĐ nên tỷ lệ gia tăng dấn số nước ta có xu hướng giảm nhưng còn chậm, mỗi năm dân số nước ta vẫn tăng thêm hơn 1 triệu người - Hậu quả của gia tăng dân số nhanh: + Đối với sự phát triển kinh tế: nhìn chung gtds chưa phù hợp với tốc tăng trưởng kinh tế: nếu dân số tăng 1% thì kinh tế phải tăng từ 3-4%, lương thực phải tăng ít nhất là 4%. Trong điều kiện nền dân số nước ta còn chậm phát triển dân số đông và mức gia tăng như hiện nay là vẫn cao ảnh hưởng -> tăng trưởng kinh tế. + Đối với chất lượng cuộc sống: Dân số tăng nhanh ảnh hưởng rõ rệt đến chất lượng cuộc sống, cộng đồng cũng chính sách của từng thành viên trong xã hội. Dân số tăng nhanh thu nhập bình quân theo đầu người thấp, mức sống giảm, việc tăng y tế, văn hoá, giáo dục... gặp nhiều khó khăn + Đối với tài nguyên, môi trường: dân số tăng nhanh đòi hỏi phải tác động mạnh vào các nguồn TNTN để tạo ra những sản phẩm cần thiết cho sản xuất và đời sống của con người. Hậu quả là các TNTN đang có nguy cơ bị suy 167 thoái nghiêm trọng (TN rừng, khoáng sản, nước...). Mặt khác dân số, nhanh dẫn đến không gian cư trú ngày càng chật hẹp, môi trường bị ô nhiễm. b) Cơ cấu dân số trẻ - Theo số liệu thống kê năm 2005: 0-14 tuổi chiếm 27%, 15-59 chiếm: 64,0% từ trên 60 tuổi: chiếm 9%. - Cơ cấu dân số nước ta đã thay đổi theo xu hướng già đi: trong độ tuổi lao động và trên độ tuổi lao động có xu hướng tăng dần về tỉ trọng. - Thuận lợi: Nguồn lao động dồi dào và nguồn dự trữ là lớn, lao động trẻ năng động, sáng tạo, nhanh tiếp thu những KH- KT mới. - Khó khăn: vấn đề việc làm cho lao động mới bổ sung 3. Phân bố dân cư chưa hợp lí Mật độ dân số ở nước ta: 254 người/km 2 (2006) nhưng phân bố chưa hợp lý giữa các vùng a) Giữa đồng bằng với trung du miền núi - Đồng bằng tập trung khoảng 75% dân số, trong khi diện tích chỉ chiếm 25% diện tích lãnh thổ, mật độ dân số cao, (ĐBSH mật độ 1225 người/km 2, ĐBSCL: 429 người/km2, DHNTB: 200 người/km2 ) - Miền núi tập trung khoảng 25% dân số (chiếm 3/4 diện tích lãnh thổ) mật độ dân số thấp: Tây Bắc: 69 người/km2, Tây Nguyên: 89 người/km2 b) Giữa thành thị với nông thôn - Cho đến nay năm 2005: 73,1%, dân số sống ở nông thôn; 26,9% đân số sống ở thành thị. - Tỷ lệ dân ở thành thị và dân nông thôn đang có sự chuyển dịch theo hướng giảm nông thôn, tăng thành thị. Đây là sự chuyển động theo hướng tốt phù hợp với quá trình CNH- HĐH đất nước. c) Trong nội bộ từng vùng - Giữa các đồng bằng: ĐBSH có mật độ dân số cao hơn ĐBSCL do nước ta mở rộng lãnh thổ về phía nam. - Giữa các vùng núi: Đông Bắc có mật độ dan số cao hơn Tây Bắc * Nguyên nhân phân bố dân cư chưa hợp lí: Ở đồng bằng có điều kiện tự nhiên thuận lợi (đất, nước, địa hình...) lại là nơi có nghề trồng lúa nước truyền thống, nền kinh tế tăng nhanh, quá trình CNH 168 - HĐH diễn ra mạnh nên dân cư tập trung đông đốc hơn miền núi. Đồng thời do lịch sử khai thác lãnh thổ nước ta về phía nam nên đb miền Bắc có mật độ dân số cao hơn đồng bằng phía Nam, đồng bằng duyên hải miền Trung. * Hậu quả phân bố dân cư chưa hợp lí Dân số phân bố chưa hợp lý làm cho lãng phí nguồn lao động nơi thừa, nơi thiếu. Ở TDMN nơi có nhiều tài nguyên về rừng, khoáng sản, khả năng to lớn về cây công nghiệp, chăn nuôi đại gia súc nhưng dân cư thưa thớt thiếu lao động, nhất là lao động có trình độ gây khó khăn cho khai thác tài nguyên và bảo vệ đất nước. Ở đồng bằng nhất là các thành phố dân cư và lao động đông, tăng nhanh gây sức ép mạnh mẽ lên môi trường tài nguyên, cản trở sự phát triển kinh tế - xã hội, làm nảy sinh nhiều vấn đề nhất là vấn đề việc làm. 4. Chiến lược phát triển dân số hợp lí và sử dụng có hiệu nguồn lao động nước ta - Tiếp tục thực hiện các giải pháp kiềm chế tốc độ tăng dân số, đẩy mạnh tuyên truyền các chủ trương chính sách, pháp luật về dân số và kế hoạch hoá gia đình. - Xây dựng chính sách chuyển cư phù hợp để thúc đẩy sự phân bố dân cư, lao động giữa các vùng. - Xây dựng quy hoạch và chính sách thích hợp nhằm đáp ứng xu thế chuyển dịch cơ cấu dân số nông thôn và thành thị. . - Đưa xuất khẩu lao động thành 1 chương trình lớn có giải pháp mạnh và chính sách cụ thể mở rộng thị trường xuất khẩu lao động. Đổi mới mạnh mẽ phương thức đào tạo người lao động xuất khẩu có tay nghề cao, có tác phong công nghiệp. - Đẩy mạnh đầu tư phát triển công nghiệp ở trung du, miền núi, phát triển công nghiệp ở nông thôn để khai thác tài nguyên và sử dụng tối đa nguồn lao động ở đất nước. Nội dung 2: Lao động và việc làm 1. Nguồn lao động a) Thế mạnh 169 + Số lượng: Nước ta có nguồn lao động dồi đào, năm 2005, số dân hoạt động kinh tế của nước ta: 42,53 triệu người, chiếm 51,2% dân số, đồng thời mỗi năm lại bổ sung thêm trên 1 triệu lao động mới. + Chất lượng: người lao động Việt Nam cần cù, sáng tạo, có kinh nghiệm sản xuất, phương pháp được tích luỹ qua nhiều thế hệ đặc biệt là trong Nông lâm nghiệp - ngư nghiệp, tiểu thủ công nghiệp Chất lượng người lao động ngày càng được nâng cao: số người có trình độ KHKT, trình độ cao đẳng, đại học ngày càng tăng (đã qua đào tạo chiếm 25% dân số, trong đó lao động có trình độ cao đẳng, đại học và trên đại học chiếm 5,3%) . + Phân bố: Lực lượng lao động có kỹ thuật tập chung ở ĐBSH, ĐNB, nhất là các thành phố lớn như Hà Nội, TP Hồ Chí Minh..., đó là điều kiện để phát triển các ngành dịch vụ, công nghiệp đòi hỏi trình độ cao b) Tuy nhiên, nguồn lao động còn nhiều hạn chế + Thừa lao động, thiếu việc làm. + Do nước ta đi lên từ một nền nông nghiệp nên nguồn lao động thiếu tác phong công nghiệp. Đội ngũ công nhân lành nghề, lực lượng lao động có trình độ cao vẫn còn ít, đặc biệt là đội ngũ cán bộ quản lý, công nhân kỹ thuật lành nghề. + Phân bố lao động không đều, tập trung chủ yếu ở các vùng đồng bằng, đô thị gây lãng phí nguồn lao động sử dụng và khai thác tài nguyên gặp nhiều khó khăn. + Chất lượng lao động ở các vùng không đều. + Nhìn chung, năng suất lao động xã hội ngày càng tăng song còn thấp so với thế giới nên phần lớn lao động có thu nhập thấp làm chậm quá trình chuyển biến, phân công lao động xã hội. Quỹ thời gian lao động còn nhiều trong nông nghiệp và các xí nghiệp quốc doanh chưa được sử dụng triệt để. 2. Cơ cấu lao động a) Cơ cấu lao động theo ngành Do tác động của cuộc cách mạng KH-KT và quá trình Đổi mới nền kinh tế đã làm thay đổi cơ cấu lao động theo ngành ở nước ta.: giảm tỷ lệ lao động trong KVI, tăng tỷ lệ lao động trong khu vực II và khu vực III. Tuy nhiên, sự thay đổi này còn chậm. Tính đến năm 2005, khu vực I: 57,3%, Khu vực II: 18,27%; khu vực II: 24,5% (vượt qua khu vực II) b) Cơ cấu lao động theo thành phần kinh tế 170 Phần lớn lao động tập trung ở khu vực ngoài Nhà nước. Năm 2005: khu vực ngoài Nhà nước 88,9%; khu vực Nhà nước: 9,5%; khu vực có vốn đầu tư nước ngoài: 1,6% Cơ cấu sử dụng lao động theo thành phần kinh tế có sự thay đổi: Khu vực Nhà nước và khu vực ngoài Nhà nước tăng giảm không đều, khu vực có vốn đầu tư nước ngoài tăng nhanh. Điều này phù hợp với xu thế trong của nền kinh tế nhiều thành phần trong cơ chế thị trường theo định hướng xã hội chủ nghĩa. b) Cơ cấu lao động theo thành thị và nông thôn Phần lớn lao động tập trung ở nông thôn, chiếm 75%; thành thị chỉ chiếm 25% (2005). Tuy nhiên, tỷ lệ lao động nông thôn đang có xu hướng giảm, tỉ lệ lao động ở khu vực thành thị có xu hướng tăng, phù hợp với quá trình CNHHĐH đất nước. c) Hạn chế - Năng suất lao động thấp. - Phần lớn lao động có thu nhập thấp. - Phân công lao động xã hội còn chậm chuyển biến - Chưa sử dụng hết thời gian lao động 3. Vấn đề việc làm và hướng giải quyết việc làm a) Vấn đề việc làm Hiện nay dân số nước ta đông (2006: 84,1 triệu người) số người trong độ tuổi lao động là 64%, trong đó số người hoạt động kinh tế: 51,2%. Hằng năm bổ sung thêm trên 1 triệu người lao động mới nên mặc dù đa dạng hoá các thành phần kinh tế, các ngành sản xuất, dịch vụ để tạo thêm gần 1 triệu việc làm mới trong mỗi năm. nhưng tình trạng thất nghiệp và thiếu lao động vẫn còn gay gắt: tính trung bình cả nước tỷ lệ thất nghiệp: 2,1%, thiếu việc làm 8,1%. ở khu vực thành thị, tỷ lệ thất nghiệp lf 5,3%, thiếu việc làm 4,5%, ở nông thôn: tỷ lệ thất nghiệp: 1,1%, thiếu việc làm 9,3% b) Hướng giải quyết việc làm: Trong những năm qua, Đảng và Nhà nước đã có nhiều chính sách nhằm giải quyết việc làm cho người lao động theo hướng sau: - Phân bố lại dân cư và nguồn lao động 171 - Thực hiện tốt chính sách dân số, sức khoẻ sinh sản. - Thực hiện đa dạng hoá các hoạt động sản xuất (nghề truyền thống, thủ công nghiệp, tiểu thủ công nghiệp...) chú ý thích đáng đến hoạt động của các ngành dịch vụ. - Tăng cường hợp tác, liên kết để thu hút vốn đầu tư nước ngoài, mở rộng sản xuất hàng xuất khẩu - Mở rộng, đa dạng hoá các loại hình đào tạo các cấp, các ngành nghề, nâng cao chất lượng đội ngũ lao động để họ có thể tự tạo những công việc và tham gia vào các đơn vị sản xuất dễ dàng, thuận lợi hơn. Nội dung 3: Đô thi hóa 1. Đặc điểm a) Quá trình đô thị hóa diễn ra chậm chạp, trình độ đô thị hóa thấp - Thế kỉ III trước công nguyên: thành Cổ Loa kinh đô của nhà nước Âu Lạc được coi là đô thị đầu tiên ở nước ta. - Thời phong kiến, một số đô thị hình thành ở những nơi có vị trí địa lý thuận lợi với chức năng hành chính, thương mại, quân sự (Thế kỉ XI: thành Thăng Long, XVI - XVIII: Phú Xuân, Hội An, Đà Nẵng, Phố Hiến) - Thời Pháp thuộc hệ thống đô thị có quy mô nhỏ với chức năng hành chính, quân sự. Những năm 30 của thế kỷ XX một số đô thị được hình thành như Hà Nội, Hải Phòng, Nam Định. - Từ 8/1945 -> 1954: qua trình đô thị hóa không có sự thay đổi nhiều - Từ 1954 - 1975, đô thị phát triển có sự khác nhau giữa miền Bắc và miền Nam: miền Nam ĐTH như một biện pháp dồn dân phục vụ chiến tranh, có một số đô thị nằm ở vị trí chiến lược quan trọng: Sài Gòn, Đà Nẵng, Biên Hoà. Ở Miền Bắc ĐTH gắn với CNH (Hà Nội, Hải Phòng, Thái Nguyên, Nam Định, Việt Trì...) - Từ 1965 - 1972: các đô thị bị chiến tranh phá hoại, quá trình ĐTH chững lại. - Từ 1975 đến nay: Quá trình đô thị hoá có sự chuyển biến tích cực, tốc độ ĐTH nhanh. Tuy nhiên cơ sở hạ tần vẫn còn thấp sp với các nước trong khu vực và trên thế giới. Có một số đô thị lớn: TP Hồ Chí Minh, Hà Nội, Hải Phòng, Đà Nẵng, Cần Thơ... b) Tỷ lệ dân thành thị tăng 172 Năm 1990, dân thành thị: 12,9 triệu người chiếm 19,5% dân số cả nước. Từ đó đến nay số dân đô thị liên tục tăng. Đến năm 2005: dân số thành thị 22,3 triệu người chiếm 26,9%. Tuy nhiên, tỷ lệ dân thành thị của nước ta còn thấp so với các nước trong khu vực. c) Phân bố đô thị không đều giữa các vùng - Năm 2006, cả nước có 689 đô thị, tập trung nhiều nhất là Trung du miền núi Bắc Bộ và ít đô thị nhất là Đông Nam Bộ. - Số dân đô thị đông nhất ở Đông Nam Bộ và thấp nhất ở Tây Nguyên. 2. Mạng lưới đô thị - Mạng lưới đô thị được phân thành 6 loại: Loại đặc biệt (Hà Nội, TP Hồ Chi Minh), Loại 1,2,3,4,5. - Năm 2007: có 5 thành phố trực thuộc Trung ương (Hà Nội, TP Hồ Chi Minh, Hải Phòng, Cần Thơ, Đà Nẵng), các đô thị trực thuộc tỉnh. 3. Ảnh hưởng cuả đô thị hóa đến phát triển kinh tế - xã hội: - ĐTH có tác động mạnh tới quá trình chuyển dịch cơ cấu kinh tế của nước ta. - Các đô thị có ảnh hưởng rất lớn đến sự phát triển kinh tế - xã hội của các địa phương, các vùng trong nước. Năm 2005, khu vực đô thị đóng góp 70,4% GDP cả nước, 84% GDP CN - XD, 87% GDP dịch vụ và 80% ngân sách Nhà nước. - Các thành phố, thị xã là các thị trường tiêu thụ sản phẩm hàng hoá lớn và đa dạng, là nơi sử dụng đông đảo lực lượng lao động có trình độ chuyên môn kỹ thuật, có cơ sở vật chất kỹ thuật hiện đại, có sức hút đối với đầu tư trong nước và nước ngoài, tạo ra động lực cho sự tăng trưởng và phát triển kinh tế xã hội. - Các đô thị có khả năng tạo ra nhiều việc làm và thu nhập cho người lao động - Tuy nhiên, quá trình ĐTH cũng gây những hậu quả như: vấn đề ô nhiễm môi trường, an ninh trật tự xã hội... cần phải có kế hoạch khắc phục. III. Các dạng bài tập đặc trưng và phương pháp làm bài 1. Các dạng bài tập đặc trưng a) Dạng bài trình bày, phân tích Trình bày một hay nhiều đặc điểm trong dân cư (dân số, lao động, việc làm) 173 Ví dụ: Trình bày đặc điểm nguồn lao động nước ta. Đây là dạng bài tập không khó chỉ yêu cầu học sinh nắm được kiến thức cơ bản b) Dạng bài chứng minh Chứng minh một hay nhiều đặc điểm của dân cư Ví dụ: Chứng minh dân số nước ta tăng nhanh. Dạng bài tập này ngoài yêu cầu học sinh nắm chắc kiến thức, thì học sinh phải ghi nhớ số liệu để chứng minh. c) Dạng bài so sánh Ví dụ: So sánh mạng lưới đô thị của Đồng bằng Sông Hồng và Đồng bằng Sông Cửu Long. Trong phần địa lí dân cư dạng bài này ít gặp. Đây là một dạng bài tương đối khó đòi hỏi học sinh ngoài nắm chắc kiến thức còn phải biết vận dụng kiến thức để tìm ra sự giống nhau và khác nhau. d) Dạng bài giải thích Ví dụ: Giải thích tại sao tỉ lệ dân thành thị nước ta có xu hướng tăng? Đây là dạng bài tập khó nhưng lại thường xuyên gặp trong phần địa lí dân cư, điểm cho câu hỏi này từ 0,25 điểm đến 0,5 điểm. Nên khi giáo viên dạy đến vấn đề gì của dân cư cũng nên hỏi học sinh “Nguyên nhân vì sao”. 2. Phương pháp đặc thù - Giáo viên cung cấp cho học sinh những kiến thức cần thiết. - Cung cấp cho học sinh những dạng bài và hướng dẫn học sinh cách làm từng dạng bài tập. - Sử dụng “sơ đồ” thông qua các từ khóa khi làm bài để học sinh không bỏ sót ý. IV. Bài tập mẫu theo từng dạng bài 1. Dạng bài trình bày, phân tích Phân tích những thế mạnh và hạn chế của nguồn lao động nước ta. - Thế mạnh: 174 + Số lượng: Nước ta có nguồn lao động dồi dào, năm 2005, số dân hoạt động kinh tế của nước ta: 42,53 triệu người, chiếm 51,2% dân số, đồng thời mỗi năm lại bổ sung thêm trên 1 triệu lao động mới. + Chất lượng: người lao động Việt Nam cần cù, sáng tạo, có kinh nghiệm sản xuất, phương pháp được tích luỹ qua nhiều thế hệ đặc biệt là trong nông lâm nghiệp - ngư nghiệp, tiểu thủ công nghiệp Chất lượng người lao động ngày càng được nâng cao: số người có trình độ khoa học - kĩ thuật, trình độ cao đẳng, đại học ngày càng tăng (đã qua đào tạo chiếm 25% dân số, trong đó lao động có trình độ cao đẳng, đại học và trên đại học: 5,3% . + Phân bố: Lực lượng lao động có kỹ thuật tập chung ở Đồng bằng Sông Hồng, Đông Nam Bộ, nhất là các thành phố lớn như Hà Nội, TP Hồ Chí Minh..., đó là điều kiện để phát triển các ngành dịch vụ, công nghiệp đòi hỏi trình độ cao - Tuy nhiên, nguồn lao động còn nhiều hạn chế: + Thừa lao động, thiếu việc làm. + Do nước ta đi lên từ 1 nền nông nghiệp nên nguồn lao động thiếu tác phong công nghiệp. Đội ngũ công nhân lành nghề, lực lượng lao động có trình độ cao vẫn còn ít, đặc biệt là đội ngũ cán bộ quản lý, công nhân kỹ thuật lành nghề. + Phân bố lao động không đều, tập trung chủ yếu ở các vùng đồng bằng, đô thị gây lãng phí nguồn lao động sử dụng và khai thác tài nguyên gặp nhiều khó khăn. + Chất lượng lao động ở các vùng không đều. + Nhìn chung, năng suất lao động xã hội ngày càng tăng song còn thấp so với thế giới nên phần lớn lao động có thu nhập thấp làm chậm quá trình chuyển biến, phân công lao động xã hội. Quỹ thời gian lao động còn nhiều trong nông nghiệp và các xí nghiệp quốc doanh chưa được sử dụng triệt để. 2. Dạng bài chứng minh Chứng minh rằng dân số nước ta tăng nhanh, cơ cấu dân số trẻ. * Dân số tăng nhanh: 175 - Dân số nước ta tăng nhanh, đặc biệt vào những năm 50 của thế kỷ XX nước ta trải qua quá trình bùng nổ dân số. Tuy nhiên giữa các giai đoạn, các vùng lãnh thổ và các dân tộc có sự khác nhau. Dân số Tỷ lệ tăng dân số (triệu người) (%) 1921 15,6 1921-1926: 1,86 1960 30,2 1939-1943: 3,06 1990 66,2 1954-1960: 3,93 1999 76,3 1999-2002: 1,32 2006 84,1 2002-2005: 1,32 Năm - Do chính sách kế hoạch hóa gia đình tỷ lệ gia tăng dấn số nước ta có xu hướng giảm nhưng còn chậm, mỗi năm dân số nước ta vẫn tăng thêm hơn 1 triệu người * Cơ cấu dân số trẻ: - Theo số liệu thống kê năm 2005: 0-14 tuổi chiếm 27%, 15-59 chiếm: 64,07% từ trên 60 tuổi: 9%. - Một nước có cơ cấu dân số trẻ là từ 0-14 tuổi chiếm >35%, từ 15-59: 55%, trên 60 tuổi dưới 10%, mặc dù nhóm 0-14 tuổi chiếm dưới 35%trong khi đó nhóm trên 60 tuổi ở nước ta chiếm 9% (dưới 10%) nên nước ta có cơ cấu dân số trẻ. - Cơ cấu dân số nước ta đã thay đổi theo xu hướng già đi tuy nhiên trong độ tuổi lao động và dưới tuổi lao động còn chiếm tỷ lệ lớn. 3. Dạng bài so sánh So sánh mạng lưới đô thị giữa Đồng bằng Sông Hồng và Đồng bằng Sông Cửu Long. * Giống nhau: - Đây là 2 vùng có mạng lưới đô thị dày đặc của nước ta do dân đông và quá trình đô thị hóa diễn ra mạnh. - Phân bố đô không đều khắp đồng bằng - Quy mô các đô thị không bằng nhau - Các đô thị có chức năng tổng hợp 176 * Khác nhau: Tiêu chí Đồng bằng Sông Hồng Đồng bằng Sông Cửu Long. Số lượng - Có số lượng đô thị ít hơn Có số lượng đô thị nhiều hơn Dân số - Đồng bằng Sông Hồng có dân - Dân số thấp hơn và mật độ số đông hơn và mật độ dân số dân số thưa thớt hơn Đồng cao hơn Đồng bằng Sông Cửu bằng Sông Hồng (Đồng bằng Long (Đồng bằng Sông Hồng Sông 1225 người/km2) Cửu Long 429 người/km2) LS hình - Có lịch sử hình thành sớm - Hình thành muộn hơn đbsh thành hơn đô thị ở đbscl Quy mô - Có một số đô thị và quy mô - Quy mô ĐT nhỏ hơn lớn: Hà Nội, Hải Phòng (Hà Nội là đô thị đặc biệt, trực thuộc trung ương) Phân bố - Mạng lưới đô thị dày, đồng - ĐBSCL tập trung ở ven Sông đều hơn vì có lịch sử khai thác Tiền, Sông Hậu: Vĩnh Long, lâu đời hơn Cần Thơ, Châu Đốc. Trong khi đó vùng bán đảo Cà Mau, mạng lưới ĐT thưa thớt Chức năng: - Có chức năng hành chính, - Chủ yếu mang chức năng trung tâm công nghiệp. Ngoài kinh tế (chủ yếu là chế biến và ra Hà Nội là thủ đô của cả sản xuất hàng tiêu dùng, lương nước, Hải Phòng là cửa ngõ thực, thực phẩm XNK của phía Bắc Cơ tầng sở hạ - Tốt và đồng bộ hơn do được - Cơ sở hạ tầng kém phát triển ht sớm (GTVT, thông tin liên phát triển do hình thành muộn lạc, điện nước) và kinh tế kém phát triển hơn. 4. Dạng bài giải thích: Tại sao vấn đề việc làm là một vấn đề kinh tế xã hội lớn ở nước ta hiện nay nhất là các đô thị. Việc làm là một vấn đề kinh tế - xã hội lớn ở nước ta hiện nay, vì: 177 - Hiện nay dân số nước ta đông (2006: 84,1 triệu người) số người trong độ tuổi lao động là 64%, trong đó số người hoạt động kinh tế: 51,2%. Hằng năm bổ sung thêm trên 1 triệu người lao động mới. - Mặc dù sự đa dạng hoá các thành phần kinh tế, các ngành sản xuất, dịch vụ để tạo thêm gần 1 triệu việc làm mới trong mỗi năm. nhưng tình trạng thất nghiệp và thiếu lao động vẫn còn gay gắt: tính trung bình cả nước tỷ lệ thất nghiệp: 2,1%, thiếu việc làm 8,1%. Ở khu vực thành thị, tỷ lệ thất nghiệp là 5,3%, thiếu việc làm 4,5%, ở nông thôn: tỷ lệ thất nghiệp: 1,1%, thiếu việc làm 9,3% V. Sử dụng “sơ đồ” thông qua các từ khóa trong ôn thi đại học Sau khi cung cấp kiến thức cơ bản, các dạng bài thường gặp trong đề thi. Giáo viên sử dụng “sơ đồ” thông qua các từ khóa hướng dẫn học sinh làm các câu hỏi. Nội dung1: Đặc điểm dân số và phân bố dân cư Câu 1: Chứng minh rằng dân số nước ta đông, nhiều thành phần dân tộc. Ảnh hưởng của dân số đông, nhiều thành phần dân tộc đến sự phát triển kinh tế - xã hội? * Chứng minh - Dân số đông: Số lượng Vị trí so với khu vực và thế giới - Nhiều thành phần dân tộc: Số lượng dân tộc Người Việt sinh sống ở nước ngoài * Ảnh hưởng của dân đông, nhiều thành phần dân tộc: Kinh tế Chất lượng Môi trường, tài nguyên Câu 2. Nguyên nhân, biểu hiện và hậu quả của việc tăng dân số nhanh ở nước ta? * Nguyên nhân: Tỉ suất sinh cao, tỉ suất tử ổn định ở mức thấp Số người trong độ tuổi sinh đẻ lớn Tâm lí xã hội 178 * Biểu hiện: Tăng về quy mô Tăng về tốc độ Hiện nay gia tăng dân số có xu hướng giảm * Tác động: Giảm tốc độ phát triển kinh tế Giảm chất lượng cuộc sống Suy giảm tài nguyên, ô nhiễm môi trường Câu 3: Chứng minh nước ta có cơ cấu dân số trẻ và đang biến đổi nhanh chóng theo xu hướng già hóa. Ảnh hưởng của cơ cấu dân số trẻ dến sự phát triển kinh tế - xã hội? * Chứng minh cơ cấu dân số trẻ: Cơ cấu dân số năm 2005: 0 -14 tuổi 15 -59 tuổi Từ 60 tuổi trở lên * Dân số nước ta có xu hướng già hóa (1990 - 2005): 0 - 14 tuổi: giảm dần về tỉ trọng 5 - 59 tuổi: tăng dần về tỉ trọng Từ 60 tuổi trở lên: tăng dần về tỉ trọng * Ảnh hưởng: Tích cực: Tạo ra nguồn lao động dồi dào Lao động trẻ năng động, sáng tạo Tiêu cực: Thiếu việc làm cho lao động mới bổ xung Câu 4: Chứng minh dân số nước ta phân bố không đều và chưa hợp lí. Nguyên nhân làm cho dân cư nước ta phân bố không đều và chưa hợp lí. Ảnh hưởng của phân bố dân cư đến sự phát triển kinh tế - xã hội? * Chứng minh dân cư phân bố không đều và chưa hợp lí Giữa đồng bằng với trung du, miền núi Giữa thành thị với nông thôn Ngay trong một vùng * Nguyên nhân: Trình độ phát triển kinh tế và tính chất của nền kinh tế Điều kiện tự nhiên Lịch sử khai thác lãnh thổ 179 * Ảnh hưởng: Đồng bằng thừa lao động, thiếu việc làm Miền núi thiếu lao động Hạn chế sự phát triển kinh tế - xã hội Câu 5: Tại sao dân số đang là một vấn đề đang được quan tâm hàng đầu ở nước ta hiện nay? Dân số đang là một vấn đề đang được quan tâm hàng đầu ở nước ta hiện nay, vì: Dân số nước ta tăng nhanh Dân số nước ta đông lại tăng tương đối nhanh làm cho quy mô dân số nước ta càng thêm lớn Tốc độ gia tăng dân số chưa phù hợp với tăng trưởng kinh tế Dân số gây sức ép lớn đối với sự phát triển kinh tế - xã hội và môi trường. Nội dung 2: Lao động và việc làm Câu 1. Trình bày đặc điểm nguồn lao động nước ta. Số lượng: Đông Mỗi năm tăng thêm hơn 1 triệu lao động. Chất lượng: Cần cù, sáng tạo, có kinh nghiệm trong sản xuất Chất lượng người lao động ngày càng được nâng lên Phân bố: Không đều Câu 2. Chứng minh rằng nước ta có tiềm năng lớn về lao động, nhưng chưa được sử dụng hợp lí Nước ta có tiềm năng lớn về lao động: Chưa được sử dụng hợp lí: Số lượng Chất lượng Trong các ngành kinh tế bất hợp lí Trong các thành phần kinh tế Năng suất lao động còn thấp Câu 3 Giải thích vì sao việc làm đang là vấn đề găy gắt iử nước ta hiên nay? Việc làm là vấn đề găy gắt vì: 180 Nguồn bổ xung lao động hàng năm lớn Tỉ lệ thất nghiệp cao: Trung bình cả nước Nông thôn Thành thị Nội dung 3: Đô thi hóa Câu 1. Nêu đặc điểm của quá trình đô thị hóa ở nước ta. Vì sao quá trình đô thị hóa ở nước ta diễn ra chậm chạp * Đặc điểm: Diễn ra chậm chạp, trình độ đô thị hóa thấp: Tỉ lệ dân thành thị thấp Cơ sở hạ tầng của các đô thị ở mức thấp. Tỉ lệ dân thành thị tăng: Số người Tỉ lệ Phân bố đô thị không đều theo vùng: Số đô thị cả nước Vùng nhiều đô thị nhất Vùng ít đô thị nhất * Quá trình đô thị hóa ở nước ta diễn ra chậm chạp, vì Xuất phát thấp của nền kinh tế Chiến tranh tàn phá Công nghiệp hóa diễn ra chậm Câu 2. Ảnh hưởng của ĐTH đến sự phát triển kinh tế - xã hội ở nước ta Tích cực: Tác động đến chuyển dịch cơ cấu kinh tế Đóng góp lớn vào nền kinh tế Tiêu thu hàng hóa lớn, cơ sở vật kĩ thuật hiện đại, thu hút đầu tư Tạo ra việc làm Tiêu cực: Môi trường Nhà ở…. Câu 3. So sánh mạng lưới đô thị giữa Đồng bằng Sông Hồng và Đồng bằng Sông Cửu Long - Giống nhau: Số lượng 181 Quy mô Phân bố Chức năng - Khác nhau: Số lượng Quy mô Phân bố Chức năng… VI. Các bài tập tự giải Câu 1. Tại sao chúng ta phải chú trọng đầu tư hơn nữa đối với việc phát triển kinh tế - xã hội ở vùng đồng bào dân tộc ít người? Câu 2. Tại sao tỉ lệ gia tăng dân số của nước ta hiện nay đã giảm nhưng quy mô dân số vẫn tiếp tục tăng? Cho ví dụ minh họa. Câu 3. Trình bày đặc điểm nguồn lao động ở nước ta hiện nay. Đặc điểm đó có thuận lợi và khó khăn gì đối với sự phát triển kinh tế - xã hội. Câu 4. Trình bày mạng lưới đô thị ở nước ta. Câu 5. Chứng minh rằng quá trình đô thị hóa ở nước ta diên ra chậm chạp, trình độ đô thị hóa thấp. Câu 6. Tại sao tỉ lệ dân thành thị của nước ta có xu hướng tăng? Kính mong sự tham khảo và chia sẻ của đồng nghiệp Xin chân thành cảm ơn! 182 SỞ GIÁO DỤC VÀ ĐÀO TẠO VĨNH PHÚC TRƯỜNG T.H.P.T VĨNH YÊN ˜–µ—™ CHUYÊN ĐỀ ÔN THI ĐẠI HỌC MÔN: TIẾNG ANH REPORTED SPEECH ( LỐI NÓI GIÁN TIẾP) GIÁO VIÊN: TriÖu ThÞ H»ng TỔ: Hoá- Sinh_ TD- Ngoại ngữ TRƯỜNG: THPT Vĩnh Yên _ TP Vĩnh Yên _ Vĩnh Phúc Vĩnh Yên, tháng 3 năm 2014 CHUYÊN ĐỀ: REPORTED SPEECH (LỐI NÓI GIÁN TIẾP) 183 Tác giả: Triệu Thị Hằng Giáo viên trường THPT Vĩnh Yên Đối tượng học sinh: Lớp 12, Ôn thi ĐH – CĐ Số tiết dự kiến: 4 tiết A. PHẦN MỞ ĐẦU I. Lý do chọn chuyên đề: Ngữ pháp là một hệ thống những quy tắc kết cấu của một ngôn ngữ. Muốn nắm bắt ngôn ngữ tiếng Anh, ngoài những yếu tố khác, người học cần phải hiểu đầy đủ, sâu sắc những kiến thức ngữ pháp cơ bản và thực hành thành thạo. Trong những chuyên đề ngữ pháp, học sinh đặc biệt hứng thú với chuyên đề thể tường thuật ( Reported speech). Tuy nhiên, họ vẫn gặp không ít khó khăn trong khi thực hành lối nói gián tiếp này, đặc biệt với các dạng tường thuật câu hỏi, tường thuật hành động lời nói... Tôi nhận thấy rằng học sinh cần nắm vững kiến thức trước khi làm bài tập thì việc luyện tập mới đạt hiệu quả. Với nhận thức như vậy nên tôi tiến hành trình bày chuyên đề về “câu gián tiếp” trong tiếng Anh. II. §èi tîng häc sinh båi dìng - Häc sinh líp : 12. - Sè tiÕt båi dìng: 4 - Trong ®Ò tµi nµy, t«i chØ tËp trung vµo mét sè vÊn ®Ò lý thuyÕt vÒ c¸c c¸ch thay ®æi tõ c©u trùc tiÕp sang c©u gi¸n tiÕp, cung cÊp mét sè bµi tËp ®Ó cñng cè c¸c kiÕn thøc trªn vµ su tÇm c¸c c©u hái trong ®Ò thi tuyÓn sinh §¹i häc m«n tiÕng Anh khèi D cña Bé GD nh»m gióp häc sinh hiÓu vµ n¾m ®îc nh÷ng kiÕn thøc c¬ b¶n vµ n©ng cao vÒ lèi nãi gi¸n tiÕp trong tiÕng Anh. * HÖ thèng kiÕn thøc sö dông trong chuyªn ®Ò: - Reported speech: + Reporting statements + Reporting questions( Wh- questions and Yes-no questions) + Reporting with special verbs. * Hệ thống các dạng bài tập đặc trưng của chuyên đề. - Các dạng bài tập trắc nghiệm khách quan bám sát SGK và một số tài liệu bài tập ngữ pháp trong sách nâng cao lớp 12. - Một số bài tập sưu tầm trong các đề thi Đại học Cao đẳng môn tiếng Anh khối D của Bộ GD. * Hệ thống các phương pháp cơ bản, đặc trưng để giải các dạng bài tập trong chuyên đề. - Vận dụng cấu trúc ngữ pháp đã được trình bày cụ thể theo từng phần để làm bài tập. - Mỗi một dạng bài tập đều nêu đầy đủ ví dụ minh hoạ để giúp học sinh dễ hiểu và áp dụng trong việc thực hành. 184 * Hệ thống các ví dụ, bài tập cụ thể cùng lời giải minh họa cho chuyên đề. * Các bài tập tự giải ----------------&&&------------------ B: PHẦN NỘI DUNG CHỦYÊN ĐỀ: REPORTED SPEECH * Definition: 1. Lời nói trực tiếp: là sự lặp lại chính xác những từ của người nói. - Lời nói trực tiếp được đặt trong dấu ngoặc kép và sau động từ chính có dấu phẩy(,) hoặc dấu hai chấm(:). - Đôi khi mệnh đề chính cũng có thể đặt sau lời nói trực tiếp. Ex: “ I don’t like this party” Bill said. 2. Lời nói gián tiếp (indirect/reported speech) là lời tường thuật lại ý của người nói, đôi khi không cần phải dùng đúng những từ của người nói. Ex: Bill said that he didn’t like that party. 185 * Thể tường thuật ( reported speech) dùng để thuật lại một lời phát ngôn trước đó. Bao gồm các loại câu tường thuật sau: - Reporting statements - Reporting questions - Reported speech with special verbs REPORTING STATEMENTS (Tường thuật câu phát ngôn) * Khi động từ chính của câu ( động từ tường thuật) ở thì hiện tại ( present tense), thì của câu phát ngôn được tường thuật lại không thay đổi. - Direct speech: “I always get up early in the morning”, my mother often says. Reported speech: My mother often says that she always gets up early in the morning. - Direct speech: difficult”. He says/ he is saying/ he has said/ he will say, “The text is Reported speech: difficult. He says/ is saying/ has said/ will say (that) the text is * Khi động từ chính của câu ở thì quá khứ, thì ngoại trừ các thì quá khứ hoàn thành và quá khứ hoàn thành tiếp diễn, thì của câu phát ngôn được tường thuật phải được lùi về quá khứ một thì so với câu trực tiếp. Direct speech: “We haven’t been to Africa ”, they said. Reported speech: They said that they hadn’t been to Africa. A. Các bước thay đổi từ câu trực tiếp sang câu gián tiếp: 1. Bắt đầu câu tường thuật bằng: * S + said + (that) + clause * S + told + O + (that) + clause Note: Trong câu tường thuật “told” (tell) có thể được dùng thay cho “said” (say). Tuy nhiên “told” luôn đòi hỏi có tân ngữ đi kèm. Ví dụ: They said (that) they had not been to Africa. They told me (that) they had not been to Africa. *Note: Có thể sử dụng một số động từ tường thuật sau: thought, announced, explained, complained, believed… 2. Bỏ dấu ngoặc kép khi tường thuật Ví dụ: He said, “They might win the game,” 186 He said that they might win the game. 3. Các đại từ phải thay đổi: Subject Object I-àhe/she YouàI/we He She It Weà they They You Meàhim/her you him her it usàthem them you-àme/him Possessive Adjective Myàhis/her your his her its our-àtheir their your-àmy Possessive Pronoun Mine-àhis/hers yours his hers its ours-àtheirs theirs yours-àmine Reflexive Pronoun myself yourself himself herself itself ourselves themselves yourslves 4. Thay đổi một số cách nói thời gian và nơi chốn: Direct today tonight now tomorrow yesterday next (week) last (week) (three days) ago this / these here Reported that day that night then/ at once/immediately the next day / the following day the day before / the previous day the next (week) / the following (week) / the week after the (week) before / the previous week (three days) before that / those there 5. Thay đổi thì: Direct speech Present simple: Tom said,”I never eat meat” Present cont.:am/is/are + Ving He said, “I’m waiting for Ann Persent pecfect: She said, “I’ve seen that film” Present pecfect progressive Andrew said, “I’ve been learning Chinese for 5 years Past simple: They said, “We came by car” Past progressive: He said, “I was sitting in the park at 8 o’cock” Future simple: will/shall + V Judy said,”I’ll phone you” Reported speech Past simple: Tom said he never ate meat Past cont.: was/were + Ving He said he was waiting for Ann Past perfect: She said she had seen that film Past perfect progressive Andrew said he had been learning Chinese for 5 years Past perfect: They said they had come by car Past perfect progressive He said he had been sitting in the park at 8 o’clock Future in the past Judy said she would phone me 187 Future progressive He said,”I’ll playing golf at three o’clock tomorrow Modal verbs + Can She said,”You can sit here” + May Claire said, “I may got to Bali again” + Must He said, “I must finish this report” Future progressive in the past He said he would be playing golf at three o’clock tomorrow. Modal verbs in the past + Could She said we could sit here + Might Claire said she might go to Bali again + Must / Had to He said he must / had to finish this report. B. Những trường hợp không cần thay đổi khi chuyển từ câu trực tiếp sang câu gián tiếp: 1. “must” có thể giữ nguyên hoặc đổi thành “had to” - “I must finish this report by Monday,” Jill said  Jill said that she must finish that report by Monday  Jill said that she had to finish that report by Monday 2. “should” và “ought to” có thể giữ nguyên hoặc tường thuật bằng cách dùng động từ “advise sb (not) to do sth” - “You should practice your English every day,” said Miss Hoax  Miss Hoai said that I should practice my English every day  Miss Hoai advised me to practice my English every day - “You ought not to lie in the sun for long hours, Nam,” said Bad  Bad said that Nam ought not to lie in the sun for long hours  Bad advised Nam not to lie in the sun for long hours 3. Khi động từ trong câu trực tiếp là các động từ khiếm khuyết sau : could, should, would, might, ought to, had to, used to. - “You needn’t do it if you don’t want,” she said  She said that I needn’t do it if I didn’t want - “It would be nice if he could see you again, Pam” Mike said  Mike said it would be nice if he could see Pam again - “It might be too dangerous for us to go now,” said Jack  Jack said it might be too dangerous for them to go then - She said; “I could do the homework → She said the she could do the homework 4. Khi tường thuật mệnh đề ước muốn (wish); câu điều kiện loại 2( có thể thay đổi hoặc không), loại 3; và cấu trúc it’s (high/ about ) time... - He said, “I wish I were richer. ” He told me he wished he were richer. - “If I knew her number, I would tell you,” Peter said  Peter said if he knew / had knew her number, he would tell / would have told me - “If you had come , you would have met many old friends,” Liz said  Liz said if I had come, I would have met many old friends - He said, “It’s time we went. ”  He said it was time they went. 188 5. Khi câu nói trực tiếp thể hiện một chân lý hoặc một hành động lặp lại thường xuyên, thì của động từ trong câu gián tiếp vẫn không thay đổi - My teacher said “The sun rises in the East” → My teacher said (that) the sun rises in the East. - He said, ‘My father always drinks coffee after dinner’ → He said (that) his father always drinks coffee after dinner 6. Động từ trong câu nói trực tiếp có thời gian xác định: - He said, “I was born in 1980” → he said that he was born in 1980. 7. Mệnh đề trạng ngữ chỉ thời gian trong câu phức có thì quá khứ đơn và quá khứ tiếp diễn - “I saw him when he was going to the cinema” → She said she saw him when she was going to the cinema REPORTED QUESTIONS (Tường thuật câu hỏi) Về cơ bản khi tường thuật lại một câu hỏi thì cũng có những thay đổi như tường thuật câu phát ngôn: thay đổi về thì, trạng từ và đại từ. Tuy nhiên đối với từng loại câu hỏi có thêm các thay đổi cụ thể như sau: 1. Tường thuật câu hỏi dạng Wh- questions và Yes- no questions: - Wh- questions: * S + asked + (O) + wh-… + clause * S + wondered + wh-… + clause * S + wanted to know + wh-… + clause - Yes- no questions: * S + asked + (O) + if / whether + clause * S + wondered + if /whether + clause * S + wanted to know + if / whether + clause 2. Không đảo ngữ khi tường thuật câu hỏi. 3. Bỏ dấu chấm hỏi và dấu ngoặc kép. 4. Thay đổi thì, động từ, cách nói thời gian, nơi chốn và đại từ cho phù hợp. 5. Đối với câu hỏi “Wh-questions” ta lặp lại các từ hỏi: what, where, when, who, which, how, how much, how many, how long, …. NOTE: không dùng trước chữ “that” 6. Đối với câu hỏi “Yes-No questions” dùng “if / whether/ whether......or not”. NOTE: không dùng “that” trước “if / whether” Example of wh-questions: - “What is this man saying?”  He asked me what that man was saying - “When are you leaving for London” 189  He wanted to know when I was leaving for London - “Where did you buy this bag?”  She wondered where I had bought that bag - “Who phoned you last night?”  She asked who ahd phoned me the night before - “Why is Mary crying?”  He wondered why Mary was crying - “How long have you been working here?” - “How far is it to your hometown?”  He wanted to know how har it was to my hometown. Example of yes-no questions: - “Do you often go out in the evening?”  He asked if/whether I often went out in the evening - “Is your wife a good cook?”  “She wanted to know if/whether my wife was a good cook. - “Have you ever been to South America?”  He asked if/whether I had ever been to South America. - “Did your father go to college?”  He wondered if/whether my father had gone to college - “Was Kate waiting for you at the airport?”  He asked if/whether Kate had been waiting for me at the airport - “Can you swim across this river?”  He wondered if/whether I could swim across that river - “Will you come home late today?”  She wanted to know if/whether I would come home late that day. - “Have you locked the door yet ?”, the mother asked her son.  The mother asked her son if he had locked the door yet.  The mother asked her son whether he had locked the door or not. • Note: Câu hỏi đuôi được tường thuật giống như câu hỏi Yes- no nhưng phải bỏ phần đuôi phía sau: Ví dụ: She asked me, “You will stay here, won’t you ?”  She asked me if/ whether I would stay there. Tóm tắt Statements (Câu phát biểu) Commands (Câu mệnh lệnh) Wh-questions (Câu hỏi nội dung) Yes-no questions (Câu hỏi có không) Câu gián tiếp * S + said + (that) + clause * S + told + O + (that) + clause * S + told + O + to infinitive * S + told + O + not + to infinitive * S + asked + (O) + wh-… + clause * S + wondered + wh-… + clause * S + wanted to know + wh-… + clause * S + asked + (O) + if / whether + clause * S + wondered + if /whether + clause * S + wanted to know + if / whether + clause 190 REPORTED SPEECH WITH SPECIAL VERBS (Câu tường thuật với các động từ đặc biệt) Để tường thuật chính xác các câu thể hiện hành động lời nói ( các câu phát ngôn quen thuộc dùng để diễn tả một chức năng giao tiếp cụ thể. Ví dụ như lời khuyên( advice), lời đề nghị( suggestion), lời xin lỗi ( apology), lời mời ( invitation)...thì chúng ta cần nắm chắc các cấu trúc thường được dùng để diễn tả các chức năng giao tiếp này. A. Câu tường thuật với động từ theo sau bằng “to-V” 1. Tường thuật mệnh lệnh: told/ ordered/ commanded/ asked/ requested sb (not) to do sth. “Put your books away ,” said the teacher.  The teacher told us to put our books away. “Please wait for a minute.” The man asked me The man asked me to wait for a minute. 2. Tường thuật lời yêu cầu: asked sb (not) to do sth. “Please, don’t smoke in this room,” said the clerk.  The clerk asked me not to smokr in that room. 3. Tường thuật lời khuyên: advised sb (not) to do sth. “If I were you, I wouldn’t drink so much wine,” he said.  He advised me not to drink so much wine. 4. Tường thuật lời hứa: promised to do sth. “ I’ll give you a hand, if you like,” said Darian.  Darain promised to give me a hand, if I liked. 5. Tường thuật lời đe dọa: threaten to do sth’. “Get out or I’ll call the police,” said the woman.  The woman threatened to call the police if he didn’t get out. 6. Tường thuật lời cảnh báo: warned sb (not) to do sth. “Don’t touch that wire,” he said.  He warned me not to touch that wire. 191 7. Tường thuật lời mời: invited sb to do sth. “Come for inner with us tonight, will you?” Bill said.  Bill invited me to come for dinner with them that night. 8. Tường thuật lời nhắc nhở: reminded sb to do sth. “Remember to pot my letter on your way,” Wendy said.  Wendy reminded me to post her letter on my way. 9. Tường thuật lời động viên: encouraged sb to so sth. “Go heard, you must enter for the contest, Jill!” said Pam.  Pam encouraged Jill to enter for the contest. 10. Tường thuật lời khẩn cầu: begged/implored sb to do sth. “Please, do me a favor,” said the beggar to Carol.  The beggar beggared/imployed Carol to do him a favor. 11. Tường thuật lời tự nguyện: offered to do sth. “Shall I help you with the housework?” said Tim to hiss wife.  Tim offered to help his wife with the housework. 12. Tường thuật sự đồng ý: agreed to do sth. “OK, I’ll take you to work in my car, Sue” said Carl.  Carl agreed to take Sue to work in his car. B. Câu tường thuật với động từ theo sau bằng “gerund/Ving 1. Tường thuật lời buộc tội: accused sb of doing sth. “you damaged my new laptop, Dan,”said Susan.  Susan accused Dan of damaging her new laptop. 2. Tường thuật lời thú nhận: admitted doing/having done sth. “I didn’t tell you he truth, Ron,” said Kim.  Kim admitted not telling/not having told Ron the truth. 3. Tường thuật lời phủ nhận: denied doing/having done sth. “I didn’t break that vase,” said Tom.  Tom denied breaking /having broken that vase. 4. Tường thuật lời xin lỗi: apolozied (to sb) for doing sth. “I’m sorry I’ve kept you waiting,” said Amanda  Amanda apolozied for having kept me waiting. 5. Tường thuật lời khen: congratulated sb on doing sth. “Congratulations! You won the game!” said the principal. The principal congratulated the students on winning the game. 6. Tường thuật lời nài nỉ, khăng khăng: insisted on doing sth. “I must pay for this damage,” the man said. 192  The man insisted on paying for that weekend. 7. Tường thuật lời đề nghị: suggested doing sth. “Let’s have a pinic this weekend,” Muad suggested.  Maud suggested having a pinic that weekend. 8. Tường thuật lời cảm ơn: thanked sb for (doing sth). “Thank you very much for your advice,” he said.  He thanked me for my advise. 9. Tường thuật lời cảnh báo: warned sb against (doing) sth. “Don’t invest in that business,” said my lawyer.  My lawyer warned me against investing in that business. 10. Tường thuật lời đỗ lỗi: blamed sb for (doing) sth. “You are responsible for this failure,” said the director.  The director blamed me for that failure. 11. Tường thuật lời thú nhận: confessed to (doing) sth. “It was me who stole the money,” said Jack.  Jack confessed to stealing the money. C. Tường thuật câu cảm thán trong lời nói gián tiếp: (Exclamation in reported speech) Động từ tường thuật là: exclaim/shout - He said, “ What a lovely garden they have” ->He exclaimed that they had a lovely garden. - Các hình thức cảm thán bắt đầu bằng “what’ và “how” chuyển sang gián tiếp bằng: He said that it was … / hoặc He exclaimed that it was … She said; “What a pity!” (Direct speech) -> She exclaimed that it was a pity I said; “What a nice horse” -> I exclaimed that it was a nice hoese He said; “How beautiful she is!” -> He exclaimed that she was beautiful! “What a big egg” he said -> He said that it was a big egg. “ How dreadful!” he said -> He exclaimed that it was dreadful D. Các hình thức hỗn hợp trong lời nói gián tiếp: (mixed forms in reported speech) Lời nói trực tiếp có thể bao gồm nhiều hình thức hỗn hợp: câu khẳng định, câu hỏi, câu mệnh lệnh, câu cảm thán. Khi chuyển sang câu gián tiếp thì câu ở dạng nào ta chuyển về gián tiếp ở từng phần của dạng đó: - He said, “Can you play the guitar?” and I said “No” 193 → He asked me if I could play the guitar and I said that I couldn’t. - “I don’t know the way. Do you? He asked. → He said that he didn’t know the way and asked her if she knew it. - “I’m going to shopping. Can I get you something? She said → She said that she was going to shopping and asked if she could get me anything. E. Some special cases: 1. Indirect speech with Let’s/ Let him/ Let her/ Let them: + Let’s/ Let him/ let her/ Let them do something → suggest that somebody should do something (dùng cho mọi trường hợp) - “Let’s eat out tonight” she said. → She suggested that they should eat out that night. - “Let him prepare all the lesson.” She said → She suggested that he should prepare all the lesson. + Let’s/let him/ let her … → suggest + V-ing (dùng cho let’s) → suggest + somebody’s + V-ing ( dùng cho let him/ let her/ let them - “Let’s go camping next weekend” he said →He suggested going camping the following weekend. - “Let them do their exercises this morning” she said → She suggested their doing exercises that morning. 2. + “Yes, let’s” → agree - “Let’s go to the theatre tonight? He said; “Yes, let’s, ” said one of his friends. → He suggested going to the theatre that night and one of his friends agreed. + “No, Let’s not” → object to the idea/ be against the idea - “No, let’s not. It is hot” said another friend → Another friend objected to that idea/ was against it because it was hot. 3. Let = allow - : “Let him go with me, mother” she said → She asked her mother to let him go with her - “Let her do it herself, teacher” he said to his teacher. → He asked his teacher to let her do it herself. 4. Indirect speech with ‘must” ‘need” “have to” + I must do … = unchanged/ had to Eg: ‘I must be there at 6 a.m” he said → He said he must/ had to be there at 6 a.m + Must I/ you/ he … do …? = had to Eg: “Must you get ready for the party” he said → He asked if I/ she had to get ready for the party. + I must not do … = unchanged Eg: “I must not get up early tomorrow”, she said → she said that she must not get up early the next day 194 + You/ She/ He must not do … = unchanged/ weren’t to do/ wasn’t to do Eg: “He must not be present at the meeting today” she said → She said that he must not be/ wasn’t to be present at the meeting that day. + I needn’t do = needn’t have to/ wouldn’t have to … Eg” “I needn’t do my homework” she said → She said that she needn’t have to do her homework/ wouldn’t have to do her homework. + You he/ she needn’t do … = unchanged Eg: “You needn’t lock the door” she said → She said that he needn’t lock the door. + Need I/ you/ he do …? = unchanged / had to Eg: “Need I prepare everything for the meal” she said → She asked if she need/ had to prepare everything for the meal. 5. Indirect speech with some expressions: + “Thanks” he said → He thanked me + “Good luck” he said → He wished me good luck “Happy Christmas” he said → he wished me happy Christmas + “Congratulation” she said → He congratulated me + The notice said ‘Welcome to Bacgiang” →The notice welcomed visitors to Bac giang + “No/ Yes” → Subject + Auxiliary verb Eg: “Did you have a good time” he said; “yes’ I said → He asked me if I had had a good time and I said I had “Have you done your work” he said “ No’ I said → He asked me if I had done my work and I said I hadn’t + Good morning/ evening/ hello → Subject + greet … EXERCISES: REPORTED SPEECH I. Choose the best answer : 1. He said he……………..at the “Ritz” Hotel. A.is staying B. has stayed C. was staying 2. He asked me where I……………. A. have studied B. study C. am studying 3. I thought that I ………….my work at that time. D. will stay D. studied 195 A. shall finish B. will finish C. would finish D. will finished have 4. He wondered if she lived…………….with her parents. A. here B. there C. in there D. in here 5. They wondered………………she would come if it rained. A. what B. who C. whether D. then 6. He says he…………..at school two years ago. A. had worked B. works C. had been working D. worked 7. Victor said he………….very busy. A. is B. will be C. was D. may be 8. I asked Hoa if she……………… Chinese. A. could speak B. speaks C. can speak D. has spoken 9. Tina asked me how long I……………..a teacher. A. have been B. had been C. am D. will be 10. She asked me…………….I liked pop music. A. if B. whether C. that D. A&B 11. The students asked if I was going to teach them physics……………… A. the next day B. next day C. day next D. the day next 12. I want to know if he…………for her birthday. A. has bought B. buys C. had bought D. would buy 13. She said she………….come to the party on Friday. A. won’t B. can’t C. doesn’t D. couldn’t 14. Chris asked me where my car……………. A. is B. will be C. was D. has been 15. Mr Jim said that he……………….happy with his new assistant. A. weren’t B. isn’t C. wasn’t D. aren’t 16. He said that John……………….up his job. A. has given B. have given C. had give D. had given 17. She said that she ……………to learn to drive. A. is going B. was going C. go D. goes 18. I knew that he……………….a very clever man. A. was B. will be C. has been D. is 19. They said they…………….us if we needed. A. help B. helped C. had gone D. would help 20. She said she…………..the next week, but I never saw her again. A. will be B. had been back C.would be D. is back back back 21. She said that she……………..there the year before. A. had gone B. went C. would go D. goes 22. I wonder………………the tickets are on sale yet. A. what B. whether C. where D. when 23. She said she……………….the book. A. already finds B. has already found C. had already found D. would already find 24. I was sure he…………….the letter. 196 A. posted B. had posted C. will post D. is posting 25. He said he…………….to the station to see me off. A. came B. will come C. would come D. is coming 26. The policeman asked George where he……………so early. A. is running B. ran C. was running D. runs 27. He says that he……………..the laws of the country. A. knew B. is knowing C. knows D. had known 28. They realised that they…………..their way in the dark. A. had lost B. lose C. lost D. was lost 29. My friend asked me who……………….the piano in the sitting room. A. plays B. was playing C. is playing D. has played 30. “Go home,” said the teacher to us. A. The teacher told us to go home B. The teacher said us to go home C. The teacher told us go home D. The teacher told to us to go home 31. “Don’t forget to clean your teeth,” said Granny to Helen. A. Granny told Helen not forget to clean her teeth. B. Granny reminded Helen to clean her teeth. C. Granny told Helen to not forget to clean her teeth. D. Granny said Helen not to forget to clean her teerh. 32. “Please bring me some fish soup,” he said to the waitress. A. He asked the waitress bring him some fish soup. B. He asked the waitress to bring her some fish soup. C. He asked the waitress to bring him some fish soup. D. He asked the waitress bring her some fish soup. 33. “This man spoke to me on the road,” said the woman. A. The woman said that man had spoken to me on the road. B. The woman said that man has spoken to her on the road. C. The woman said that man spoke to her on the road. D. The woman said that man had spoken to her on the road. 34. I said to Nick: “Where are you going?” A. I asked Nick where was he going. B. I asked Nick where he was going. C. I asked Nick where is he going. D. I asked Nick where he is going. 35. “If I were you, I’d try to get a room on the top of floor,” he said. A. He advised me to try to get a room on the top floor. B. He advised me to try getting a room on the top floor. C. He offered me to try to get a room on the top floor. D. He suggested me to try to get a room on the top floor. 36. “I’ll wait for you if you like,” she said. A. She offered to wait for me. B. She said she’d wait for me if I liked. C. She suggested she waited for me. 197 D. A&B are correct. 37. “Don’t touch this pot. It is still very hot” she said to me A. She told me to touch that pot as it was still very hot B. She told me not to touch that pot as it was still very hot C. She told me to not touch that pot as it was still very hot D. She told me not to touch that pot as it is still very hot 37. “Remember to switch off when you’ve finished,” he said. A. He reminded me to switch off when I’ve finished. B. He reminded me to switch off when I’d finished. C. He offered me to switch off when I’d finished. D. He suggested me to switch off when I’d finished. 38. “You’d better apologise for being late,” said my mother. A. My mother advised me to apologise for being late. B. My mother suggested me to apologise for being late. C. My mother suggested apologising for being late. D. My mother warned me to apologise for being late. 39. “Could you check the oil, please?” I asked the mechanic. A. I invited the mechanic to check the oil. B. I asked the mechanic to check the oil. C. I reminded the mechanic to check the oil. D. I warned the mechanic to check the oil. 40 Kate said: “Mike, do you like my dress?” A. Kate asked Mike if he liked her dress. B. Kate asked Mike if you liked her dress. C. Kate asked Mike if he liked your dress. D. Kate asked Mike if he likes her dress. 41. “Would you like to have lunch with me on Sunday?” he said to me. A. He offered me to have lunch with him on Sunday. B. He suggested me to have lunch with him on Sunday. C. He invited me to have lunch with him on Sunday. D. He reminded me to have lunch with him on Sunday. 42. “Don’t open your books,” the teacher said to the pupils. A. The teacher told the pupils to not open their books. B. The teacher told the pupils did not open their books. C. The teacher told the pupils not open their books. D. The teacher told the pupils not to open their books. II. Chose the best answer 1. She asked me……..the seat………..or not a. If / had occupied b. Whether / was occupied c. If / has been occupied d. Whether / occupied 2. The policeman asked us……….. a. had any of us seen the accident happen. b. If had any of us seen the accident happen c. Whether any of us had seen the accident happen 198 d. that if any of us had seen the accident happen 3. I asked him………..,but he said nothing a. what the matter was b. what was the matter c. the matter was what d. what’s the matter was 4. James………..him up when the bus reached the square a. told me wake b. asked me to wake c. said me to wake d. requested me waking 5. I wonder………….we’ll catch the bus………we’ll take a taxi a. if / and whether b. whether / or that c. if / or that d. whether / or whether 6. Excuse me. Could you tell me………………? a. what time is it b. what is the time c. what time it is d. it is what time 7. We wondered……….from his office after that scandal a. why did he not resign b. why he did not resign c. why he not resign d. why didn’t he resign 8. Jeff wanted to know…………….. a. that why were his friends laughing b. why were his friends laughing c. why his friends were laughing d. the reason why his friends laughing 9. Mr Hawk told me that he would give me his answer the………..day a. previous b. following c. before d. last 10. She…………him whether he liked the steak she cooked a. asks b. wondered c. wanted to know d. asked 11. His neighbours sometimes wondered……….he did for a living a. why b. when c. where d. what 12. Could you please tell me……………? a. It is how far to the nearest bus stop b. how far is it to the nearest bus stop c. how far to the nearest bus stop is it d. how far it is to the nearest bus stop 13. Marigold wondered………..Kevin and Ruth would be at the party a. that b. whether c. if d. b& c 14. He asked me……………. a. How long you have studied English. b. How long had you studied English c. How long you had study English d. How long you had studied English 15. The passenger asked…………….we landed a. what b. when c. if d. why 16. “I’ll tell you about this tomorrow, Mary.” said Tom. a. Tom said to Mary that he will tell her about that the next day. b. Tom told Mary that I would tell you about that the next day. c. Tom told Mary that he would tell her about that the next day. 199 d. Tom told Mary that she would tell him about that the next day. 17. “I have something to tell you” Kerry said to Cheryl. a. Kerry told Cheryl I had something to tell her. b. Kerry told Chery he had something to tell her. c. Kerry told Cheryl she had had something to tell him. d. Kerry told Cheryl he had had something to tell her. 18. He said, “My wife has just bought a diamond ring.” a. He said that his wife had just bought a diamond ring. b. He said that my wife had just bought a diamond ring. c. He said that his wife has just bought a diamond ring. d. he said that his wife just bought a diamond ring. 19. “I will come with you as soon as I am ready”, she said to Philip. a. She said to Philip he will come to see you as soon as he I am ready. b. She told Philip she will come to see her as soon as she was ready. c. She told Philip she would come to see you as soon as she was ready. d. She told Philip she would come to see him as soon as she was ready. 20. “I wrote to him yesterday” a. She said to me I wrote to him the day before. b. She told me she wrote to him yesterday. c. She told me she had written to him yesterday. d. She told me she had written to him the day before. 21. “He is talking to your sister”, She said to me. a. She told me he was talking to your sister. b. She told me she was talking to my sister. c. She told me he was talking to my sister. d. She told me he was talking to her sister. 22. “It is the time to check what you have done”, the father said to the boys. a. The father said to the boys it was time to check what they had done. b. The father told the boys it was time to check what they had done. c. The father told the boys it was time to check what they have done. d. The father told the boys it is time to check what they had done. 23. “You will like my sister when you meet her” a. He told me you will like her sister when you meet her. b. He told me I will like his sister when I met her. c. He told me I would like his sister when I met her. d. He told me I would be liked his sister when I met her. 24. “I didn’t meet Susan last week” a. He said he didn’t meet Susan the week before. b. He said he hasn’t met Susan last week. c. He said he hadn’t met Susan last week. d. He said he hadn’t met Susan the week before. 25. “I didn’t break your watch” a. The boy told the girl he hadn’t broken her watch. 200 b. The boy asked the girl he hadn’t broken her watch. c. The boy told the girl he didn’t break her watch. d. The boy told the girl he hadn’t broken your watch. 26. “I found a lot of mistakes in your plan”. Catherine said to Kevin. a. Catherine told Kevin she found a lot of mistakes in his plan. b. Catherine told Kevin she has found a lot of mistakes in his plan. c. Catherine told Kevin she had found a lot of mistakes in his plan. d. Catherine told Kevin she had found a lot of mistakes in her plan. 27. “I have just seen your mother this morning”. a. Laura told Lewis I have just seen your mother this morning. b. Laura told Lewis she had just seen his mother that morning. c. Laura told Lewis she has just seen his mother that morning. d. Laura told Lewis he had just seen her mother that morning. 28. “We are ready to come with our friends” a. They told us they are ready to come with their friends. b. They told us they were ready to come with our friends. c. They told us we were ready to come with our friends. d. They told us they were ready to come with their friends. 29. “I couldn’t get into the house because I had lost my key” a. He said he couldn’t get into the house because he had lost my key. b. He said he hadn’t been able to get into the house because he had lost his key. c. He said he had been able to get into the house because he had lost his key. d. He said he hadn’t been able to get into the house because he lost his key. 30. “I was intending to meet you tomorrow”. a. She told me she was intending to meet me tomorrow. b. She told me she had intending to meet me the next day. c. She told me she had been intending to meet me tomorrow. d. She told me she had been intending to meet me the next day. 31. Paker: “ Help me” a. Paker asked not to help him b. Paker asked to help me c. Paker advised me to help him d. Paker asked us to help him 32. Carol: “ Listen to that song” a. Carol said to listen that song b. Carol told us to listen to that song c. Carol asked us to listen to this song d. Carol talked me to listen to that song 33. Maria: “ Leave me alone!” a. Maria told her friend to leave her alone b. Maria told her friend to leave me alone c. Maria said her friend to leave me alone 201 d. Maria told her friend leaving me alone 34. Mr Stephens: “ Buy me some milk” a. Mr Stephens suggested to buy him some milk b. Mr Stephens asked me to buy him some milk c. Mr Stephens asked to buy him some milk d. Mr Stephens spoke to buy me some milk 35. George: “ Feed the cat” a. George asked his girlfriend to feed the cat b. George asked his girlfriend feed the cat c. George asked his girlfriend feeding the cat d. George asked his girlfriend fed the cat 36. Heather: “ Don’t say that” a. Heather tells to you not to say that b. Heather told you not to say that c. Heather advised you to say that d. Heather ordered not you to say that 37. You: “ Come with us” a. They told John to come with us b. They told to John to come with us c. They told John to come with them d. They told John came with us 38. Tom: “ Call me in the evening” a. Tom told us calling him in the evening b. Tom told us to called him in the evening c. Tom told not us to call him in the evening d. Tom told us to call him in the evening 39. Your mother: “ Come back in one hour” a. My mother told me come back in one hour b. My mother asked me come back in one hour c. My mother reminded to come back in one hour d. My mother told me to come back in one hour. 40. Mrs Smith: “Don’t play in front of my windows” a. Mrs Smith told us not to play in front of her windows b. Mrs Smith told us not to play in front of my windows c. Mrs Smith told us to not play in front of her windows d. Mrs Smith said us not to play in front of her windows 41. Jame: “Don’t eat so much junk food !” a. Jame reminded me not to eat so much junk food b. Jame asked me to not eat so much junk food c. Jame reminded me not to ate so much junk food d. Jame says me not to eat so much junk food 42.Walter: “Don’t ring Tony on Sunday” a. Walter told me don’t ring Tony on Sunday b. Walter told not to ring Tony on Sunday c. Walter told me not ring Tony on Sunday 202 d. Walter told me not to ring Tony on Sunday 43. Jane: “Don’t watch the new film” a. Jane advised me not to watch the new film b. Jane advised not me to watch the new film c. Jane advised me do not to watch the new film d. Jane advise me not to watch the new film 44. “Don’t make noise because I am listening music now” a. He asked me not to make noise because I am listening music now b. He asked me not to make noise because I was listening music then c. He asked me not to make noise because he was listening music then d. He asked me to make noise because I was listening music then 45. Marcel: “Don’t sing this song” a. Marcel asked me not to sing this song b. Marcel asked me not to sing that song. c. Marcel said me not to sing that song d. Marcel asked me not to sang that song 46. Teacher: “Don’t forget your homework” a. Teacher reminded me not to forget my homework. b. Teacher asked me if not to forget my home work c. Teacher reminded whether I not to forget my home work. d. Teacher reminded me not to forget your home work 47. John: “Don’t shout at your son” a. John asked me to not shout at my son b. John asked me not to shout at my son. c. John asked me not to shout at your son d. John wondered if I do not shout at my son 48. Tom: “Why don’t you study hard for the coming exam, Ba” a. Tom advised me not to study hard for the coming exam b.Tom advised me to study hard for the coming exam c. Tom advised Ba not to study hard for the coming exam d. Tom advised Ba to study hard for the coming exam. 49. Father: ‘ Brush your teeth before going to bed, please” a. Father said to his son brushing your teeth before going to bed b. Father said to his son to brush his teeth before going to bed c. Father told his son to brush my teeth before going to bed d. Father told his son to brush his teeth before going to bed 50. The man: “ Come in but don’t bring anything” a. The man said to come in but not bring anything b. The man told come in but not to bring anything c. The man told the boy to come in but not to bring anything d. The man asked to come but not to bring anything 51. They said “ Hurrah! We have won the contest!” a. They said “ Hurrah” that they had won the contest. b. They said they had won the contest. c. They exclaimed with joy that they had won the contest. 203 d. They said with joy that they have won the contest. 52. Miss White said to him, “ Why are you so late? Did your car have a flat fire?” a. Miss White told him why was he so late and did your car have a flat fire b. Miss White asked him why was he so late and did your car have a flat fire. c. Miss White asked him why was he so late and if your car have a flat fire. d. Miss White asked him why he was so late and whether his car had a flat fire. 51. His friend said to Peter “ I’m sad. Let me alone” a. His friend said that I am sad and let me alone. b. His friend said that I was sad and let me alone. c. His friend said that he was sad and told Peter to let him alone. d. His friend said that he was sad and if Peter to him alone. 52. “ What does she like?” he asked me. a. He asked me what she likes b. He asked me what she liked. c. He asked me what do I like. d. He asked me what did he like. 53. She said, “Don’t tease me, John.” a. She said not to tease me to John. b. She said John not to tease me. c. She told John not to tease her. d. She told John do not tease her. 54. He asked me what……………………………………….. a. time was it b. time is it c. time it was d. none is correct. 55. He told me that…………………………………………… a. his father has sold the house. b. his father had sold the house yesterday. c. his father will sell the house. d. his father don’t sell the house. 56. He told me to rest for a while. “…………….for a while”, he said. a. To rest b. Rest c. Do you rest d. Resting 57. He asked “Why did she take my pen?” - He asked why…………………………. a. she took his pen. b. did he take his pen. c. she had taken his pen d. she has taken his pen 58. He said, “Hurrah! We have won the match!” a. He said, “Hurrah” that they had won the match. b. He said with joy that they have won the match. c. He exclaimed with joy that they had won the match. d. He said that they have won the match. 59. My father said to me, “Why are you late? Did you miss the train?” 204 a. My father told me why was I late and did I miss the train. b. My father asked me why was I late and did I miss the train. c. My father asked me why you were late and if you missed the train. d. My father asked me why I was late and whether I missed the train. 60. I said to him “ I’m very angry with you . Go away” a. I said that I am very angry with you and go away. b. I said that I was very angry with you and went away. c. I said that I was very angry with him and told him to go away. d. I said that he was very angry with me and told me to go away. 61. “ What do you want ?” he asked me. a. He asked me what I want. b. He asked me what I wanted. b. He asked me what do I want d. He asked me what did I want. 62. She said, “ Don’t smile, John. Be serious.” a. She said not to smile and be serious. b. She said John not to smile and be serious. c. She told John not to smile and asked him to be serious. d. She told John do not smile and be serious. 63. . He said "If I had enough money, I could buy that dictionary." a. He said if he had enough money, he could buy that dictionary b. He said if he had had enough money, he could buy that dictionary. c. He said if he had enough money, he could have bought that dictionary. d. He said if he had had enough money, he could have bought that dictionary. 64. "Good morning, Mary! How are you?" Henry said a. Henry greeted Mary and asked how she was. b. Henry greeted Mary and asked how is she. c. Henry said good morning Mary and asked how was she. d. Henry said good morning and asked Mary how she is. 65. Cindy said that “I haven’t seen John since last month.” a. Cindy said she doesn’t see John since the previous month. b. Cindy said she wasn’t seen John since the previous month. c. Cindy said she hasn’t seen John since the previous month. d. Cindy said she hadn’t seen John since the previous month MỘT SỐ BÀI TẬP LIÊN QUAN ĐẾN “CÂU GIÁN TIẾP” TRONG ĐỀ THI ĐẠI HỌC MÔN TIẾNG ANH KHỐI D CÁC NĂM CỦA BỘ GD Choose the best answer : Question 1: “Why don’t we go out for dinner ? ” said Marry 205 A. Marry requested a dinner out. B. Mary suggested a dinner out. C.Mary demanded a dinner out. D.Mary ordered a dinner out. Question 2: “I will let you know the answer by the end of this week” Tom said to Janet. A. Tom suggested giving Janet the answer by the end of the week. B. Tom insisted on letting Janet know the answer by the end of the week. C.Tom promised to give Janet the answer by the end of the week. D.Tom offered to give Janet the answer by the end of the week. Question 3: My friend told me: “If I were you , I would not smoke so much” A. My friend advised me not to smoke so much. B.My friend warned me against smoking so much. C. My friend suggested not smoking so much. D. My friend prohibited me from smoking so much. ( Đề Đại học năm 2007) Question 4: The captain to his men: “Abandon the ship immediately” A. The captain invited his men to abandon the ship immediately. B. The captain suggested his men abandon the ship immediately. C. The captain ordered his men to ababdon the ship immediately. D.The captain requested his men to abandon the ship immediately. Question 5: “Shall I make you a coffee” the girl said to the lady. A. The girl wanted to make a coffee to the lady. B. The girl offered to make a coffee to the lady. C.The girl refused to make a coffee to the lady. D.The girl promised to make a coffee to the lady. ( Đề Đại học năm 2008) Question 6: “Why don’t you reply to the President’s offer right now?” said Mary to her husband. A. Mary suggested that her husband should reply to the President’s offer without delay. B. Mary told her husband why he didn’t reply to the President’s offer then. C. Mary ordered her husband to reply to the President’s offer right now. D. Mary wondered why her husband didn’t reply to the President’s offer then. Question 7: “Please don’t drive so fast, Tom,” said Lisa. A. Lisa complained about Tom’s driving too fast. B. Lisa pleaded with Tom not to drive too fast. C. Lisa insisted on Tom’s driving on. D. Lisa grumbled to Tom about driving slowly. Question 8: She said, “John, I’ll show you round my city when you’re here.” A. She made a trip round her city with John. B. She promised to show John round her city. C. She planned to show John round her city. D. She organized a trip round her city for John. 206 ( Đề Đại học năm 2009) Question 9: “Would you like some more beer?” he asked. A. He asked me if I wanted some beer. B. He wanted to invite me for a glass of beer. C. He offered me some more beer. D. He asked me would I like some more beer. Question 10: “Stop smoking or you’ll be ill,” the doctor told me. A. I was warned against smoking a lot of cigarettes. B. The doctor suggested smoking to treat illness. C. I was ordered not to smoke to recover from illness. D. The doctor advised me to give up smoking to avoid illness. ( Đề Đại học năm 2010) Question 11: “Be careful! Don’t do that again,” he said. A. He encouraged me to do that again. B. He advised me to be careful and do that again. C. He told me to be careful, so I didn’t do that again. D. He warned me not to do that again. ( Đề Cao đẳng năm 2010) Question 12: “My company makes a large profit every year. Why don’t you invest more money in it?” my friend said to me. A. My friend persuaded me to invest more money in his company. B. My friend suggested his investing more money in his company. C. My friend instructed me how to put more money into his company. D. I was asked to invest more money in my friend’s company. Question 13: “If you don’t pay the ransom, we’ll kill your boy,” the kidnappers told us. A. The kidnappers pledged to kill our boy if we did not pay the ransom. B. The kidnappers threatened to kill our boy if we refused to pay the ransom. C. The kidnappers ordered to kill our boy if we did not pay the ransom. D. The kidnappers promised to kill our boy if we refused to pay the ransom. Question 14: “You shouldn’t have leaked our confidential report to the press, Frank!” said Jane. A. Jane suspected that Frank had leaked their confidential report to the press. B. Jane accused Frank of having cheated the press with their confidential report. C. Jane blamed Frank for having flattered the press with their confidential report. D. Jane criticized Frank for having disclosed their confidential report to the press. Question 15: “Don’t forget to tidy up the final draft before submission,” the team leader told us. 207 A. The team leader reminded us to tidy up the final draft before submission. B. The team leader asked us to tidy up the final draft before submission. C. The team leader ordered us to tidy up the final draft before submission. D. The team leader simply wanted us to tidy up the final draft before submission. Question 16: “Mum, please don’t tell dad about my mistake,” the boy said. A. The mother was forced to keep her son’s mistake as a secret when he insisted. B. The boy earnestly insisted that his mother tell his father about his mistake. C. The boy requested his mother not to talk about his mistake any more. D. The boy begged his mother not to tell his father about his mistake. ( Đề Đại học năm 2011) Question 17: “We lost the last game because of the referee,” said the team captain. A. The team captain said that without the referee, they might have lost the last game. B. The team captain admitted to the referee that they had lost the last game. C. The team captain refused to tell the referee about their loss in the last game. D. The team captain blamed the referee for their loss in the last game. Question 18: “If I were you, I would not choose to write about such a sensitive topic,” the teacher said. A. The teacher advised me against writing about such a sensitive topic. B. The teacher advised me on writing about such a sensitive topic. C. I was ordered by the teacher not to write about such a sensitive topic. D. I was blamed for writing about such a sensitive topic by the teacher. ( Đề Đại học năm 2012) SOME MORE EXERCISES (BÀI TẬP TỰ GIẢI) I. Choose the best answer : 1. Peter said he was leaving for Paris ______. A. next week B. the week previous C. following week following week 2. “I don’t usually drink milk when ______ ” Mrs. Pike said. A. she was hungry B. I was hungry C. I am hungry be 3. They said that their house had been broken into ______. A. the two days before B. two days ago C. two days before two days 4. She advised me _______ an apple everyday to stay healthy. A. eating B. I should eat C. to eat eat D. the D. I will D. since D. please, 208 5. “I’ll come with you as soon as _______” she replied. A. I was ready B. I am ready C. I ready ready D. am I ready 6. “_____ today as it was yesterday.” She remarked. A. it wasn’t so foggy B. it isn’t so foggy C. it hadn’t been so foggy D. it isn’t such foggy 7. “I wish _____ eat vegetables”, he said. A. my children will B. my children would C. whether my children would D. my children must 8. “I _____ late again”, she promised. A. I am not B. I won’t be C. I wouldn’t D. I was not late 9. “How long _____ here ?”, asked Jane. A. you have been B. you had been C. had you been D. have you been 10. She wanted to know ______. A. what was happening B. was happening C. it was happening D. was happening what 11. “Do you know ______ ?” asked the man. A. where is the ticket office B. where the ticket office is C. the ticket office is D. where was the ticket office 12. The teacher asked Mary ______ the previous match. A. who wins B. who is winning C. who won D. that won 13. My father asked me _____ of the film. A. what do you think B. what I think C. what did you think D. what I thought 14. The doctor ____ him to take more exercise. A.told B.tell C. have told D. are telling 15. I wanted to know_____ return home. A.when would she B. when will she C. when she will D. when she would 16. Claire told me that her father____ a race horse. A.owns B.owned C. owning D. A and B 17. What did that man say ______? A. at you B. for you C. to you D. you 18. I rang my friend in Australia yesterday, and she said it _______ raining there. A. is B. were C. has been D. was 19. The builders have ______ that everything will be ready on time. 209 A. promised B. promise C. promises D. promising 20. The doctor _______ him to take more exercise. A. told B. tell C. have told D. are telling 21. The last time I saw Linda, she looked very relaxed. She explained she’d been on holiday the ______ week. A. ago B. following C. next D. previous 22. Yesterday, Laura ______ him to put some shelves up. A. asked B. is asking C. ask D. was asked 23. Tom has ______ this story wasn’t completely true. A. admitting that B. was admitted that C. admitted that D. admit that 24. When I rang Tessa some time last week, she said she was busy ______ day. A. that B. the C. then D. this 25. I wonder _____ the tickets are on sale yet. A. what B. when C. where D. whether 26. Mathew _____ Emma that her train was about to leave. A. has reminded B. has reminded that C. reminded D. reminded that 27. Hello, Jim. I didn’t expect to see you today. Sophie said you _____ ill. A. are B. were C. was D. should be 28. Ann ______ and left. A. said goodbye to me B. says goodbye to me C. tell me goodbye D. told me goodbye 29. I told you ______ to switch off the computer, didn’t I ? A. don’t B. not C. not to D. to not 30. Bill was slow, so I ________ hurry up. A. tell him B. told him for C. told to D. told him to 31. Sarah was driving to fast, so I ______ to slow down. A. asked her B. asked C. ask D.have asked her 32. Someone ______ me there’s been an accident on the motorway. A. asked B. said C. spoke D. told 33. Sue was very pessimistic about the situation. I advised her _____. A. no worry B. not worry C. no to worry D. not to worry 210 34. I couldn’t move the piano alone, so I asked Tom_______. A. giving a hand B. gave a hand C. to give a hand D. give a hand 35. Tom said that New York _______ more lively than London. A. is B. be C. was D. were 36. When he was at Oliver’s flat yesterday, Martin asked if he ______ use the phone. A. can B. could C. may D. must 37. George couldn’t help me. He ______ me to ask Kate. A. tell B. said C. told D. say 38. Judy ______ going for a walk, but no one else wanted to. A.admitted B. offered C. promised D. suggested 39. I said that I had met her ______ . A. yesterday B. the previous day C. the day D. the before day. 40. The man asked the boys ______ . A. why did they fight B. why they were fighting C. why they fight D.why were they fighting II. Choose the best answer: 1. “Thank you very much for your help, John,” said Daisy. A. Daisy thanked John for helping her. B. Daisy told John to help her. C. Daisy wanted John to help her and said thanks. D. Daisy would like John to help her. 2. “It is you that stole my purse,” Mrs. Pike said to the young man. A. Mrs. Pike told the young man that it was you that stole her purse. B. Mrs. Pike denied the young man of stealing her purse. C. Mrs. Pike accused the young man of stealing her purse. D. Mrs. Pike asked the young man to steal her purse. 3. “You mustn’t get into the area,” the policeman said. A. The policeman stopped them from getting into the area. B. The policeman denied from getting into the area. C. The policeman wanted them to get into the area. D. The policeman suggested getting into the area. 4. “Whose composition haven’t we heard yet?” The teacher asked us to tell her .......................... A. whose composition we haven’t heard yet. B. whose composition we hadn’t heard yet. C. whose composition hadn’t we heard yet. D. whose composition we hadn’t heard yet? 211 5. “You look beautiful in this dress.” Last night she told me......................................... A. you look beautiful in that dress. B. you looked beautiful in that dress. C. I’ll look beautiful in this dress. D. I looked beautiful in that dress. 6. “Hi, Bob. Did you take the job?” Bob’s friend asked him ...................................... A. did he take the job B. whether you took the job. C. if he had taken the job D. had he taken the job. 7. “How many people are there in your family?” The interview asked me ..................................... A. how many people there were in your family. B. how many people were there in your family. C. how many people were there in my family. D. how many people there were in my family. 8. “Don’t stay up late. You should go to bed early,” Peter told me. A. Peter told me not to stay up late and promised to go to bed early. B. Peter promised not to stay up late and told me to go to bed early. C. Peter told me do not to stay up late and advised me to go to bed early. D. Peter told me not to stay up late and advised me to go to bed early. 9. “I will bring you some food.” He said to me A. He invited me to eat some food B. he offered to bring me some food C. He told me to bring some food D. he advise me to bring some food 10. “Go on, Susan! Apply for the job,” the father said. A. The father invited Susan to apply for the job. B. The father denied applying for the job. C. The father encouraged Susan to apply for the job. D. The father wanted Susan not to apply for the job. III. Change the following sentences into reported speech: 1. “Let’s give a party” said Ann “Let’s not” said her husband 2. “Let’s take a tent and camp out” said Bill “ Good idea” said Nina 3. “Let the children stay indoor tonight.” She said 4. “Let me go” he said to the policeman. 5. “Let’s go swimming in the lake” he said “No, it is against law” said his friend. 6. “Let’s prepare everything for the party” she said. “Yes, let’s” said her friend. 7. “Let’s stay here till the storm has passed” he said 8. “Don’t drive too close the car in front” said my mother (warn) 9. “Let’s go swimming. It is too hot to learn our lesson now.” He said 212 10. “You ought to wear a safety helmet when you drive motorbike” she said to her husband. IV. Rewrite the following sentences using given words or phrases: 1. ‘What’s your job?” Mary asked Tom. Mary wanted to … 2. The factory inspector asked me where I kept the fine fighting equipment ‘Where … 3. She said “It’s good if you make your own decision” (advise) She … 4. “Why don’t you read that book, Peter” she said She suggested … 5. “What a mischievous boy!” he said. He exclaimed … 6. “How wonderful the play was!” said John John exclaimed … 7. ‘Me? No, I didn’t break your glasses.’ Peter said. Peter … 8. “Don’t move or I’ll shoot”, said the bank robber to the clerk. The bank robber threatened. ………………………………………………………………… 9. “Don’t forget to phone the police”, she told him. She reminded... ………………………………………………………………………… 10. “Don’t swim out too far, boys”, said the coach. The coach…....………………………………………………….. …………………… V. Change into reported speech 1. “Open the safe,” the robber said to the bank clerk. -> The robber ordered.... 2. “ Leave this spaces clear,” the notice said.-> The noticed told....... 3. “Mr.Brown, this is Miss White,” he said.-> He introduced .......... 4. “ Shall I open the window for you?” he said.-> He told.... 5. “ I’ll wait for you. I promise,” he said to me.-> He promised ...... 6. “ Would you like to come with me?” John said.-> He invited........ 7. “ Don’t worry. Of course you can take a holiday,” John said.-> John ressured me that..... 8. “ Don’t forget to take the holiday,” John said. -> John reminded .............. 9. “ You should take a holiday,” John said.-> John advised ............ 10.“Are you going to take a holiday?” John said.-> John asked whether........ 11.“ It’s all right by me if you take a holiday,” John said. John agreed that ........ 12.“ Actually, I’d rather you took a holiday,” John said. -> John prefered that...... 213 13.“I thought you were going to take a holiday” John said-. -> John expected ..... 14.“ Taking a holiday would be a good idea” John said-->John proposed that..... 15.“ You really must take a holiday,” John said.->John insisted that....... -------------------------&&&--------------------------C : PHẦN KẾT LUẬN Trong chuyên đề trên tôi đã giới thiệu và đưa ra một số kiến thức cơ bản về câu gián tiếp trong tiếng Anh và một số bài tập ứng dụng. Hy vọng chuyên đề này sẽ giúp các em học sinh hiểu hơn về nội dung ngữ pháp này và áp dụng được trong quá trình học tập. Hệ thống bài tập được chọn lọc, mang tính khái quát cao để học sinh có thể luyện tập đạt hiệu quả. Mặc dù tôi đã rất cố gắng, song sẽ không tránh khỏi thiếu sót. Rất mong sự đóng góp ý kiến của quý thầy cô và các bạn. Xin chân thành cảm ơn ! SỞ GIÁO DỤC VÀ ĐÀO TẠO TỈNH VĨNH PHÚC TRƯỜNG THPT VĨNH YÊN ˜™ CHUYÊN ĐỀ ÔN THI ĐẠI HỌC MÔN TIẾNG ANH : TEACHING SKILLS TO DO 214 READING COMPREHENSION TESTS Giáo viên : Đỗ Thị Thu Minh Trường THPT Vĩnh Yên CHUYÊN ĐỀ: TEACHING SKILLS TO DO READING COMPREHENSION TESTS Tác giả:Đỗ Thị Thu Minh Giáo viên trường THPT Vĩnh Yên Đối tượng học sinh: Lớp 12, Ôn thi ĐH – CĐ Số tiết dự kiến: 10 tiết 1. Đối tượng bồi dưỡng : Học sinh lớp 12 2. Số tiết dạy dự kiến : 10 3. Hệ thống kiến thức : Skill 1-Answer main idea questions correctly. Skill 2. Answer stated detail questions correctly. Skill 3 :Find “unstated” details Skill 4. Find pronoun referents. Skill 5. Answer implied detail questions correctly. Skill 6. Find definitions from structural clues. Skill 7. Use context to determine meanings of difficult words. Skill 8. Use context to determine meanings of simple words. Skill 9. Determine the tone, purpose, or course 215 TEACHING SKILLS TO DO READING COMPREHENSION TESTS: Each reading passage is often followed by 10 questions of reading comprehension and vocabulary. Topics of the reading passage are varied, but they are often informational subjects such as history, literature, art, architecture, geology, geography, and astronomy. Time is definitely a factor in the Reading Comprehension. Some students, especially average students, note that they are unable to finish all the questions in this section. Others cannot understand what the passage is about, so they do without intention and don’t know surely if their answer is true. Therefore, they need to make the most efficient use of their time in this section to get the highest score. The following method is the best way of attacking a reading passage to get the most questions correct in a limited amount of time. 216 STRAGIES FOR THE READING COMPREHENSION QUESTIONS 1. Skim the reading passage to determine the main idea and the overall organizations of ideas in the passage. You do not need to understand every detail in each passage to answer the questions correctly. It is therefore a waste of time to read the passage with the intent of understanding every single detail before you try to answer the questions. 2. Look ahead at the questions to determine what types of questions you must answer. Each type of question is answered in a different way. 3. Find the section of the passage that deals with each question. The question – type tells you exactly where to look in the passage to find correct answers. - For main ideas questions, look at the first line of each paragraph. - For directly and indirectly answered detail questions, choose a key word in the question, and skim for that key word (or a related idea) in order in the passage. - For vocabulary questions, the questions will tell you where the word is located in the passage. - For overall review questions, the answers are found anywhere in the passage. 4. Read the part of the passage that contains the answers carefully. The answer will probably be in the same sentence (or one sentence before or after) the key word or idea. 5. Choose the best answer to each question from the four answer choices listed in the textbook. The best answer can be chosen according to what is given in the appropriate section of the passage, eliminate definitely wrong answers, and mark the best guess on the answer sheet. The following skills will help students to implement these strategies in the Reading passage of the test. A. QUESTIONS ABOUT THE IDEAS OF THE PASSAGE_________________ It is very common for reading passages of the test to have questions about the overall ideas in the passage. The most common type of question asks about the main idea, topic, title, or subject. Skill 1. Answer main idea questions correctly. Students may be asked to identify the topic, subject, title, primary idea, or main idea. These questions are all really asking what primary point that the author is trying to get across in the passage, and it is not difficult to find the main idea by 217 studying the topic sentence, which is most probably found at the beginning of a paragraph. If the passage consists of only one paragraph, students should study the beginning of that paragraph to determine the main idea. If a passage consists of more than one paragraph, students study the beginning to determine the main idea. Example 1. In the Second Semester Test, Grade 12, year 2010-2011, Code AV 121, the Reading comprehension is that: The Asian Beach Games is a multi – sport event held every two years among athletes representing countries from Asia. The games are regulated by the Olympic Council of Asia. The first Games were held in 2008 in Bali, Indonesia. This Games will be composed of sports with strong television appeal such as windsurfing, kite boarding, swimming, beach volleyball, beach handball, beach soccer, and dragon boat racing. The Asian Indoor Games is a multi – sport event held every two years among athletes representing countries from Asia. The Games are regulated by the Olympic Council of Asia. The first Games were held in Bangkok, Thailand. This Games will be composed by sports with TV broadcasting potential and not included in the Asian Games and Winter Asia Games Programs and are not Olympic sports. The sports program will comprise from six to eight exciting sports with strong television appeal, including electronic sports, extreme sports, aerobics, acrobatics, indoor athletics, dance sports, inline hockey, fin swimming, and 25 meters short course swimming. Question 27. The passage is about ………………………… A. The Asian Beach Games and the Asian Indoor Games. B. The Olympic Council of Asia. C. The Asian Games and Winter Asian Games Programs. D. The Asian Indoor Games and Winter Asian Games Programs. In the above given passage, there are four paragraphs. Students should be sure to read the first sentence of each paragraph to determine the subject, title, or main idea. In Example 1, the first sentence of the first paragraph of this passage discusses the Asian Beach Games, and the second paragraph begins with “this Games”, which means that it is about the Asian Beach Games. The first sentence of the third paragraph tells of the Asian Indoor Games, about which the fourth paragraph tells, also beginning with “this Games”. So the whole passage discusses the two kinds of games: the Asian Beach Games and the Asian Indoor Games. Therefore, the best choice is answer A. 218 The following chart outlines the key information that students should remember about main idea questions: MAIN IDEA QUESTIONS HOW TO What is the topic of the passage? IDENTIFY THE What is the subject of the passage? QUESTIONS What is the main idea of the passage? What is the author’s main point in the passage? With what is the author primarily concerned? Which of the following would be the best title? WHERE TO The answer to this type of question can generally be FIND THE determined by looking at the first sentence of each ANSWER paragraph. HOW TO 1. Read the first line of each paragraph. ANSWER THE 2. Look for a common theme or idea in the first lines. QUESTION 3. Pass your eyes quickly over the rest of the passage to check that you really have found the topic sentence(s). 4. Eliminate any definitely wrong answers and choose the best answer from the remaining choices. B. DIRECTLY ANSWERED QUESTIONS______________________________ Skill 2. Answer stated detail questions correctly. A stated detail question asks about one piece of information in the passage rather than the passage as a whole. The answers to these questions are generally given in order in the passage, and the correct answer is often a restatement of what is given in the passage. This means that the correct answer often expresses the same idea as what is written in the passage, but the words are not exactly the same. Example 2: In English 10, page 45 and 46, Unit 4 “Special Education”, Part Reading, the second paragraph: At first, there was a lot of opposition from the parents of the disabled children. They used to believe that their children could not learn anything at all. In the first week, only five children attended the class. Gradually more children arrived. Their parents realized that the young teacher was making great efforts to help their poor kids. Task 2: 219 Question 2: At first the parents were ____________ the idea of sending their children to the special class. A. interested in B. opposed to C. satisfied with D. worried about Since in the first sentence of the paragraph, there is the noun “opposition”, we can see from the four answers the verb “opposed” in the answer B, they are in a family. So the answer B is the best one. The following chart outlines the key information that students should remember about stated detail questions: STATED DETAIL QUESTIONS HOW TO According to the passage, … IDENTIFY THE It is stated in the passage … QUESTION The passage indicates that … The author mentions that … Which of the following is true …? WHERE TO The answers to these questions are found in order in the FIND THE passage ANSWER HOW TO 1. Choose a key word in the question. ANSWER THE 2. Skim in the appropriate part of the passage for the QUESTION key word or idea. 3. Read the sentence that contains the key word or idea carefully. 4. Look for the answer that restates an idea in the passage. 5. Eliminate the definitely wrong answers and choose the best answer from the remaining choices. Skill 3. Find “unstated” details. Students will sometimes be asked to find an answer that is not stated or not mentioned or not true in the passage. This type of question really means that three of the answers are stated, mentioned, or true in the passage, while one answer is not. The actual job is to find the three correct answers and then choose the letter of the one remaining answer. The teacher asks students to notice that there are two kinds of answers to this type of question: 1) There are three true answers and one answer that is not discussed in the passage. 220 2) There are three true answers and one that is false according to the passage. Example 3: In English 12, page 163 and 164, Unit 15 “Woman in Society”, Part Reading, the last paragraph: Today, although their status varies in different countries, women in most parts of the world have gained significant legal rights. The most important of these are: the right to have equal work opportunities and pay to men, the right to vote, and the right to formal education. Task 2: Question 5: Which of the following is NOT mentioned in the last paragraph as one of women’s legal rights today? A. the right to hold property B. the right to equal pay C. equal employment opportunity rights D. the right to vote The question asks for the one answer that is not mentioned, so three of the answers are listed in the passage and one is not. Since the right to equal pay, equal employment opportunity rights, the right to vote, are listed in the passage, answers B, C, and D are incorrect. However, the right to hold property is not listed in the passage, so answer D is the best answer to this question. The following chart outlines the key information that students should remember about “unstated” detail questions: HOW TO IDENTIFY THE QUESTION WHERE TO FIND THE ANSWER HOW TO ANSWER THE QUESTION “UNSTATED” DETAIL QUESTIONS Which of the following is not stated …? Which of the following is not mentioned …? Which of the following is not discussed …? All of the following are true except …? The answers to these questions are found in order in the passage. 1. Choose a key word in the question. 2. Scan in the appropriate place in the passage for the key word (or related idea). 3. Read the sentence that contains the key word or idea carefully. 4. Look for answers that are definitely true according to the passage. Eliminate those answers. 221 5. Choose the answer that is not true or not discussed in the passage. Skill 4. Find pronoun referents. In the Reading comprehension of the test, you will sometimes be asked to determine which noun a pronoun refers to. In this type of question it is important to understand that a noun is generally used first in a passage, and the pronoun that refers to it comes after. Whenever you are asked which noun a pronoun refers to, you should look before the pronoun to find the noun. Example 4: In the Second Semester Test, Grade 12, year 2011-2012, Code AV 101, the Reading comprehension is that: Arctic Ice May Disappear in a Decade A new report on global warming predicts the frozen Arctic Ocean will soon be like a normal sea in the summers. There are shocking changes happening in the polar environment and its fragile eco-systems. For millions of years, the sea around the North Pole has been frozen all year round. Recent research from the World Wildlife Fund (WWF) and the Catlin Atlantic Survey show things are changing fast. They predict that within a decade, the Arctic will be largely icefree in the summer. They base their prediction on the rate at which the sea ice is currently thinning. The lead researcher, professor Peter Wadhams said, “The area is now more likely to become open water each summer, bringing forward the potential date when the summer sea ice will be completely gone.” An ice-free Arctic will have consequences for the whole world’s weather patterns. The Arctic sea ice is a key part of the Earth’s climate system. Experts call it Earth’s “refrigerator”. They said that as it disappears, the world will become a lot warmer. Scientists are still unclear exactly what changes there will be to our weather. Forecasters predict an increase in all kinds of disasters and extreme weather events. These include massive flooding, much more dangerous hurricanes and the spread of the world’s deserts. These new findings provide an urgent call for world leaders to act. The timing of the WWF report is a reminder to those attending the UN climate summit in Copenhagen in December. Rich countries will face pressure to agree to reduce their carbon emissions by40 per cent by 2020. Question: 33. What does the word “it” in paragraph 2 refer to? A. the Arctic sea ice B. climate system C. refrigerator D. the Earth 222 To answer this question, students should look before the pronoun it for singular nouns that pronoun could refer to. An ice sea Arctic, the Arctic sea ice, the Earth’s climate system come before the pronoun, so they are possible answers. Then students should try the three possible answers in the sentence in place of the pronoun. From the following sentence students should notice that as it disappears, the world will become a lot warmer , so the best answer to this question is answer A. The following chart outlines the key information that students should remember about pronoun referents: HOW TO IDENTIFY THE QUESTION WHERE TO FIND THE ANSWER HOW TO ANSWER THE QUESTION PRONOUN REFERENTS The pronoun “…” in line X refers to which of the following? The line where the pronoun is located is generally given in the question. The noun that the pronoun refers to is generally found before the pronoun. 1. Find the pronoun in the passage. (The line where the pronoun can be found is generally stated in the question.) 2. Look for nouns that come before the pronoun. 3. Read the part of the passage before the pronoun carefully. 4. Eliminate any definitely wrong answers and choose the best answer from the remaining choices. C. INDIRECTLY ANSWER QUESTIONS______________________________ Some questions of the Reading Comprehension section of the test will require answers that are not directly stated in the passage. To answer these questions correctly, you will have to draw conclusions from information that is given in the passage. Two common types of indirectly answer questions are: (1) implied detail questions and (2) transition questions. Skill 5. Answer implied detail questions correctly. You will sometimes be asked to answer a question by drawing a conclusion from a specific detail are details in the passage. Questions of this type contain the words implied, inferred, likely, or probably to let you know that the answer to the question is not directly stated. In this type of question it is important to 223 understand that you do not have to “pull the answer out of thin air.” Instead, some information will be given in the passage, and you will draw a conclusion from that information. Example 5: In English 10, page 45 and 46, Unit 4 “Special Education”, Part Reading, the second paragraph is : At first, there was a lot of opposition from the parents of the disabled children. They used to believe that their children could not learn anything at all. In the first week, only five children attended the class. Gradually more children arrived. Their parents realized that the young teacher was making great efforts to help their poor kids. Task 2: Question 3: It can be inferred from the second paragraph of the reading passage that there has been ______________________. A. a change in the parents’ attitude towards the class B. a lot of protest from the parents against the class C. a feeling of doubt in the teacher’s ability D. a belief in the parents’ opposition To answer this question, students should refer to the part of the passage where it states that in the past the parents had a lot of opposition, but now they don’t, so the answer B is incorrect. It also says that their parents realized that the young teacher was making great efforts to help their poor kids, which means the parents don’t have the feeling of doubt in the teacher’s ability, then the answer C is incorrect, too. The sentence “Gradually more children arrived” shows that the parents have a change in the parents’ attitude towards the class, so the answer D can’t be correct and the best answer is A. The following chart outlines the key information that students should remember about implied detail questions: HOW TO IDENTIFY THE QUESTION WHERE FIND ANSWER HOW TO THE TO IMPLIED DETAIL QUESTION It is implied in the passage that … It can be inferred from the passage that … It is most likely that … What probably happened …? The answers to these questions are found in order in the passage. 1. Choose a key word in the passage. 224 ANSWER THE QUESTION 2. Scan the passage for the key word (or a related idea). 3. Carefully read the sentence that contains the key word. 4. Look for a answer that could be true, according to that sentence. D. VOCABULARY QUESTIONS_____________________________________ In the Reading Comprehension section of the test, there will be a number of vocabulary questions. To answer this type of question, it is of course useful if you know the meaning of the word that the test is testing. However, it is not always necessary for you to know the meaning of the word; often there are skills that you can use to help you find the correct answer to the question: (1) finding definitions from structural clues; (2) determining meanings from the word parts, and (3) using context clues to determine meanings. Skill 6. Find definitions from structural clues. When you are asked to determine the meaning of a word in the Reading Comprehension section of the test, it is possible (1) that the passage provides information about the meaning of the word, and (2) that there are structural clues to tell you that the definition of a word is included in the passage. Example 6: In law, a nuisance is an act that has no legal justification and which interferes with the safety or comfort of other people. Public nuisances, those which are injurious to the community, may be prosecuted as crimes. Question: A nuisance is _______________. A. a protective law B. an injurious act C. a legal justification D. a safety precaution We should choose answer B because a “nuisance” is an act which interferes with the safety of others, that is, an injurious act. The following chart outlines the key information that students should remember about structural clues to help you understand unknown vocabulary words: 225 HOW TO IDENTIFY THE QUESTION TYPES OF CLUES WHERE TO FIND THE ANSWER HOW TO ANSWER THE QUESTION STRUCTURAL CLUES What is …? What is the meaning of …? What is true about …? Punctuation: comma, parentheses, dashes Restatement: or, that is, in other words, i.e. Examples: such as, for example, e.g. Information to help you determine what something means will generally be found after the punctuation clue, the restatement clue, or the example clue. 1. Find the word in the passage. 2. Locate any structural clues. 3. Read the part of the passage after the structural clue carefully. 4. Eliminate any definitely wrong answer and choose the best answer from the remaining choices. Skill 7. Use context to determine meanings of difficult words. On the test you will sometimes be asked to determine the meanings of the difficult words, a word that you are not expected to know. In this case, the passage will give you a clear indication of what the word means. Example 7: In the Second Semester Test, Grade 12, year 2011-2012, Code AV 101, the Reading comprehension is as follows: Arctic Ice May Disappear in a Decade A new report on global warming predicts the frozen Arctic Ocean will soon be like a normal sea in the summers. There are shocking changes happening in the polar environment and its fragile eco-systems. For millions of years, the sea around the North Pole has been frozen all year round. Recent research from the World Wildlife Fund (WWF) and the Catlin Atlantic Survey show things are changing fast. They predict that within a decade, the Arctic will be largely icefree in the summer. They base their prediction on the rate at which the sea ice is currently thinning. The lead researcher, professor Peter Wadhams said, “The area is now more likely to become open water each summer, bringing forward the potential date when the summer sea ice will be completely gone.” An ice-free Arctic will have consequences for the whole world’s weather patterns. The Arctic sea ice is a key part of the Earth’s climate system. Experts call it Earth’s “refrigerator”. They said that as it disappears, the world will become a lot warmer. Scientists are still unclear exactly what changes there will 226 be to our weather. Forecasters predict an increase in all kinds of disasters and extreme weather events. These include massive flooding, much more dangerous hurricanes and the spread of the world’s deserts. These new findings provide an urgent call for world leaders to act. The timing of the WWF report is a reminder to those attending the UN climate summit in Copenhagen in December. Rich countries will face pressure to agree to reduce their carbon emissions by40 per cent by 2020. Question 32: The word “fragile” in the passage mostly means ………………….. A. easily broken B. easily damaged C. physically weak ________D. extremely complicated To answer this question, students should look at the sentence that contains the word fragile. It comes before the word eco-system, which means that it is an adjective of the latter word. So it cannot be easily broken, physically weak or extremely complicated, then answers A, C, and D are incorrect. As a result, answer B is the best answer to this question. The following chart outlines the key information that students should remember about vocabulary questions containing difficult words: VOCABULARY QUESTIONS CONTAINING DIFFICULT WORDS HOW TO “What is the meaning …?” IDENTIFY THE “Which of the following is closest in meaning to …?” QUESTION The word is a difficult word, one that you probably do not know. WHERE TO The question usually tells you in which line of the passage FIND THE the word can be found. ANSWER HOW TO 1. Find the word in the passage. ANSWER THE 2. Read the sentence that contains the word carefully. QUESTION 3. Look for context clues to help you understand the meaning. 4. Choose the answer that the context indicates. Skill 8. Use context to determine meanings of simple words. You will sometimes be asked to determine the meaning of a simple word, a word that you see often in everyday English. In this type of question, you should not give the normal, everyday meaning of the word; instead, the test wants to know the meaning of the word in this situation. 227 Example 8: A line from the passage: ….. He put his answer this way …. Question: The word “put” is closest in meaning to which of the following? A. placed B. set C. expressed D. handed In this type of question, students should understand that put is a normal, everyday word, and they are not being asked to give the regular meaning of a normal, everyday word. Because the primary meaning of to put is to place, answer A is not the correct answer. To answer this type of question students must see which of the answers best fits into the sentence in the passage. We cannot place an answer or set an answer or hand an answer, but we can express an answer, so answer C is the best answer to this question. The following chart outlines the key information that students should remember about vocabulary questions containing simple words. VOCABULARY QUESTIONS CONTAINING SIMPLE WORDS HOW TO “What is the meaning …?” IDENTIFY THE “Which of the following is closest in meaning to …?” QUESTION The word is a simple word, one that you see often in everyday English. WHERE TO The question usually tells you in which line of the passage FIND THE the word can be found. ANSWER HOW TO 1. Find the word in the passage. ANSWER THE 2. Read the sentence that contains the word carefully. QUESTION 3. Look for context clues to help you understand the meaning. 4. Choose the answer that the context indicates. E. OVERALL REVIEW QUESTIONS__________________________________ Often in the Reading Comprehension section of the test, the last question (or two) for a particular reading passage is an overall question, one that asks about the passage as a whole rather than one small detail. The overall review questions are generally not main idea questions; instead they ask about some other aspect of the passage as a whole. The most common types of overall review questions are (1) questions that ask where in the passage something is found; (2) questions about the tones of the passage; (3) questions about the author’s purpose in writing the passage, or 228 (4) questions about which course the passage might be a part of. Skill 9. Determine the tone, purpose, or course. Other types of overall review questions occur occasionally in the Reading Comprehension section of the test. Possible questions of this type are those that ask about: (1) the tone of the passage; (2) the author’s purpose in writing the passage, and A question about the tone is asking if the author is showing any emotion in his or her writing. The majority of the passages on the test are factual passages presented without any emotion; the tone of this type of passage could be simply informational, explanatory, or factual. Sometimes on the test, however, the author shows some emotion, and you must be able to recognize that emotion to answer a question about tone correctly. If the author is being funny, then the tone might be humorous; if the author is making fun of something, the tone might be sarcastic; if the author feels strongly that something is right or wrong, the tone might be impassioned. A question about purpose is asking what the author is trying to do in the passage. You can draw a conclusion about the author’s purpose by referring to the main idea and the organization of details in the passage. For example, if the main idea is that George Washington’s early life greatly influenced his later career and if the details give a history of his early life, the author’s purpose could be to show how George Washington’s early life influenced his later career. However, the answer to a purpose question is often considerably more general than the main idea. A more general author’s purpose for the main idea about George Washington would be to demonstrate the influence of early experiences on later life (without any mention of George Washington). Example 9: In English 10, page 45 and 46, Unit 4 “Special Education”, Part Reading: A TEACHER IN A SPECIAL CLASS Like other teachers, Pham Thu Thuy enjoys her teaching job. However, her class is different from other classes. The twenty-five children, who are learning how to read and write in her class, are disabled. Some are deaf, some dumb and others mentally retarded. Most of the children come from large and poor families, which prevents them from having proper schooling. 229 At first, there was a lot of opposition from the parents of the disabled children. They used to believe that their children could not learn anything at all. In the first week, only five children attended the class. Gradually more children arrived. Their parents realized that the young teacher was making great efforts to help their poor kids. Watching Thuy taking a class, one can see how time-consuming the work is. During a maths lesson, she raised both arms and opened up her fingers one by one until all ten stood up. She then closed the fingers one by one. She continued the demonstration until the children realized they had just learned how to add and subtract. The children have every reason to be proud of their efforts. They know a new world is opening up for them. Question 5: The writer’s attitude towards Thuy’s work in the passage can be described as_____________. A. humorous B. angry C. suspicious D. admiring The question asks about the tone (writer’s attitude) of the passage. To determine the tone of a passage, students should look for any indications of emotion on the part of the writer. In this passage, the writer shows the great efforts the teacher has made to help the poor kids who later see a new horizon opening up for them. Therefore, the best answer to this question is answer D. There is nothing in the passage to indicate any humor A, anger B, or suspicion C on the part of the writer. The following chart outlines the key information that students should remember about tone, purpose, or course questions: TONE, PURPOSE, OR COURSE HOW TO Tone: What is the tone of the passage? IDENTIRY THE Purpose: What is the author’s purpose in this passage? QUESTION WHERE TO Tone: There will be clues throughout the passage that FIND THE the ANSWER author is showing some emotion rather than just presenting facts. Purpose: Draw a conclusion about the purpose form the main idea and supporting details. HOW TO Tone: 1. Skim the passage looking for clues that the ANSWER THE author is showing some emotion . QUESTION 2. Choose the answer that identifies the emotion. Purpose: 1. Study the main idea in the topic sentence and the details used to support the main idea. 2. Draw a conclusion about the purpose. 230 B. READING COMPREHENSION EXERCISES Reading Exercise 1: Read the two passages and choose the best answer for each of the questions that follow. A. Before ballpoint pens or fountain pens, pens were made from goose faethers. These goose feathers, called quills, were sharpened and dipped into inkwells, where they absorbed enough ink to write a few words. It wass necessary to keep an inkwell very close by, as frequent dipping was necessary. These quill pens were one of the earliest products “designed”specifically for left and right- handed people. Fathers from the left wing of the goose worked best for the right- handers because of the way that the feathers arched. Feathers from the right wing were preferred by left- handers. 1. Which of the following is the best title for this passage? a.Early Ballpoint and Fountain pens b. Quill Pens for Lefties and Righties c. Where quill Pens Came from. d. Various Users for goose Feathers. 2. The passage indicates that a quill pen could hold enough ink to write……….. a. one or two pages b. for about one hour c. a couple of words d. numerous sentences 3. Look at the word designed in paragraph two. It could be best the replaced by…………. a. defined b. devised c. deformed d. deferred 4. Which of the following is NOT true about quill pens, according to the passage? a. Left- handers were unable to use quill pens. b. Left- handers people generally preferred quills from the right wing. c. Right- handers could use quill pens d. Right- handed – people generally preferred quills from the left wing. 5. It can be inferred from the passage that quill pens……….. a. are still used regularlytoday b. are referred over ballpoint pens c. are the best pens for the left- handers. d.are not longer used much B. 231 When you can’t touch the car door handle or sit on the seat because they are so hot, you are meeting solar energy. We can use this heaat from the sun to keep us warm and give us electricity. We know that we will someday run out of the fossil fuels, such as oil, gas and coal. And we are now spending lots of money to learn how to put nuclear energy to work for us. But we knoe that the radioactive wastes that result from using nuclear energy can be very dangerous. We need a supply of energy that will not pollute our air or water, that will not cost too much and thatwill not run out. One of the sources we are considering is the sun. Enough sunshine falls on the United States in one minute to give the whole country all the energy, we’ll be able to make as much electricity as we need. And it wouldn’t make smog in our air or poison our water. We could collect the sun’s heat in big sun farms. A good place for a sun farm is on the desert, where the sky is clear and there is plenty of sunshine and only a little rain. People could set up acres of glass mirrors to focus the sun’s rays on a special surface that would become very hot. Or they could put out acres and acres of glass- covered collectors called solar energy traps to gather the sun’s heat. The heat would melt a special metal;this melted metal would below to a largetank of melted salt; the melted salt would then take the heat from the metal and hold it. This stored heat could then be used like the heat we get rom burning fossil fuels. It would make steam and turn generators to make electricity. There would even be enough heat to make steam during the night and on cloudy days. The trouble is that collecting the sun’s heat in these ways is expensive right now. We need to find out how to make these methods cost les before we can have big sun farms on our deserts. Scientists are trying to make a solar energy collector that they hope will be cheap enough for anybody to put on a house- either on the roof or in the wall. The inventors are making these solar collectors in kits for people who want to do- it themselves. 6. What is the subject of this selection? a.Kinetic energy b. nuclear energy c. Mechanical enegy d. Solar energy 7. According to this article,…….. a. the sun is a good source of energy b. smog does not pollute the air c. nuclear energy is safe and clean d. many people use solar energy. 8. The best place for a sun farm is the……… a. desert b. seashore c. moutains d. forest 9. Scientists…… a. have final found a safe way to use nuclear energy. b. Know that fossil fuels willlast forver 232 c. Are still experimenting with energy from the sun d. Have found the cheap way to collect the sun’s heat 10. The writer feels that nuclear energy…… a. will soon run out b. is cheap c. is clean d. can be harmful Exercise 2: Read the two passages and choose the best answer for each of the questions that follow. A. From the start of his career as a writer, Mark Twain was fascinated with inventions. As a result, he chose to spend much of his available income on various types of inventions, generally without much success. In fact, he lost most of the money he invested. By 1887, the 49- year- old author had invested most of his money in one project, the Paige typesetting machine. Twain expected the Paige machine to be completed in 1989. Unfortunately, the machine was never completely finished, and by the early 1980s, Twain kneww he was in serious financial situation. He worked hard to take care of his problems by producing a number of books is a row he wrote A Connecticut Yankee in King Arthur’s Court (1892), An American Claimant (1892), Tom Sawyer Abroad (1893), and Pudd’nhead Wilson 91894). However, all of this work did not solve his financial problems, and he was forced to declare bankruptcy. 1.This passage mainly discusses how an author…. a.invested poorly in inventions b. wrote so many books c. created his famous work d. created a successful invention 2. According to the passage, what interested Mark Twain? a. Giving investment advice b. Work as a typesetter c.The world of banking d. New ideas and products 3. Where was the majority of Twain’s money in 1887? a.In a bank account b. Invested in various inventions c.Invested in one invention d. Invested in his writings 4. It can be inferred from the passage that the Paige machine was……… a. created by Mark Twain b. ready in 1890s c. close to being completed d. a great success 5. Look at the expression in a row in paragraph 2. This expression could best be replaced by. a. one after another b. very quickly c. on time d. in a straight line B. The English names of the last months of the Gregorian calendar ( September, October, November, December) have rather interesting histories. The Gregorian calendar is a twelve- month calendar, so these months are the ninth, tenth, eleventh, and twelfth months respectively. However their name do not reflect their positioning in the calendar. The name September comes from the Latin word septum, which means seven. 233 This month was originally the name of the seventh rather than the ninth month. Similarly, the name October comes from the Latin oct( eight); the name November comes from the Latin novem (nine); the name December comes from the Latin decem(ten) 1.The main topic of this passage is… a. the origin of certain month names b. the Gregorian calendar c. the number in Latin d. ten- and twelve- month calendar 2. The first month on the Gregorian calendar is probably. a. March b. May c. January d. December 8. The passage states that in the orginal version of the calendar, September was the name of…. a. the sixth month b. the seventh month c. the eighth month d. the ninth month 9. It can be inferred from the passage that November… a. used to be the ninth month of the year b. is no longer part of the Gregorian calendar c. has always been the eleventh month d. was not part of the orginal Gregorian calendar 10. Which of the following is NOT mentioned in the passage about December? a. it is the twelfth month of the Gregorian calendar b. Its name is derived from a Latin word c. Its meaning comes from the number ten d. It has thirty- one days Exercise 3: Read the following two passages and choose the best answer A, B, C or D for each of the questions. A. I get a lot of letters at this time of the year from many people who have a cold which won’t go away. There are many different stories about how to prevent or cure a cold. That’s why it is difficult to know what to do. We know that colds arerarely “ dangerous”, except for weak people such as the elderly or young babies. These people are always uncomfortable and usually most unpleasant. Of course, you can buy a lot of medicines. They will help to make your cold less unpleasant. However, you must remember that nothing can actually cure a cold or make it go away faster. Any strong medicine used to make you feel bettercould be dangerous if you are already taking drugs for some other illness. Thus, check with your chemistor doctor to see whether they are all right for you. And remember they might make yo sleepy. Please don’t try to drive if they do! Lastly, whatever you may be told about magic foods or drinks, the best answer is to keep yourself strong and healthy. You’ll have less chance of catching a cold. And if you do, it sholdn’t be so bad. 1.The writer of the pasage wants……… A to write in anamusing way 234 B. to give generaladvice C. to complain about his/ her health D. to persuade people to have more foods and drinks 2.Colds are……… A. very dangerous B. not dangerous at all C. unpleasant D. usually fatal 3. What is the key to avoid catching a cold? A. Having magic foods and drinks B. Sleeping a lot C. Taking a lot of medicine D. Keeping yourself strong and healthy 4. The word “ most” as used in line 5 is closest in meaning to………… A. very B. in the highest degree C. greatest D. absolutely 5. The writer of the passage is probably………….. A. a nurse B. a health care specialist a novelist D. a reporter B. If you were to stop people in the street and ask them to name a ship that had been sunk, it is likely that nearly all of them would say the Titalic,. For the sinking of the Titalic was, if not the most tragic, certainly the most famous sea disaster in the history of ocean travel. The Titalic was built as luxuryliner, intended to be the fastest in the world, and a great deal of publicity had surrounded it. The last point probably explains why so many important people from all walks of life were on the boat when it went down. The Titalic was on its mainden voyage to America in 1912 when it struck an iceberg anh sank. Of its, 2,300 passengers, more than two- third were drowned. Because the Titalic was thought to be virtually unsinkabl, no one was prepared for the tragedy. There was total panic as very fewof the passengers had botheredto learn the necessary drill in the event of trouble. There was serve shortage of lifeboats and those that were launchedwere still half- empty. The one point of calm was to be foundin the ballroom where the band carried on playing right to the end. What makes the sinking of the Titalic even more tragic is the fact that warnings of icebergs had been sent, yet the linerwas still continuing at full distress signal. One good thing did, however, result from the disaster. The whole questions of safety at sea was looked into, resulting measure, including stricter lifeboat regulations and the establishment of an iceberg patrol. 6. The sinking of the Titalic was…………. A. the most tragic sea disaster B. the most historic sea disaster C. the most famous sea disaster D. the first great disaster 7. Why were so many important people in board? A. The Titalic had received a lot of publicity. 235 B. The Titalic was luxury liner C. It was the fastest liner in the world. D. They wanted to go to America. 8. The lifeboats were.. A. too short B. launched too soon C. half finished D. poorly enquipped 9. The chances of disaster were increased because………… A. no warnings had been sent B. the dance band was playing too loud C. the Titalic wastravalling too fast D. there was no distress calls. 10. The positive result of the disaster was that…… A. a full inquiry was made B. lifeboats were made larger C. a program of iceberg destruction was started D. sea travel was made safer Exercise 4: Read the two passages and choose the best answer for each of the questions that follow. A. TEA OR COFFEE The Boston Tea Party did more than just helped start a war.It helped change the habits and taste of the North American people. It turned tea drinkers into coffee drinkers. Coffee and tea are both products of plants that grow in the tropics. When these beverages were introduced in Europe during the 1600s, they became very profitable busunesses. Coffee reached England before tea did, but once the English tasted tea, it became favorite mealtime drink. English people were soon buying about two pounds of tea leaves per person per year. English people who went to North America as colonists took along their taste for tea. They preferred it to coffeeuntill about 1773. In that year, the British Parliament passed the Tea Act, a law that helped to push the cololists into war againstthe mother country. Among cololists, tea became a symbol of unfair taxation and of England’s attempts to contgrol all trading with the colonies. The resentment of the cololists was so strong that once several groups of them boarded English ships in BostonHArbour and threw bales of tea into the ocean. After this ‘ Boston Tea Party’ drinking tea was called unpatriotic by the colonists. They turned to coffee as their beverage. Today tea is still the favorite drink in England. On the other side of the Atlantic, things are different. The people of the United States drink about three times as many cups of coffee as cups of tea. 1.While it is not directly started, the article suggests that… A. tea drinkers might go to war B. the tastes of people can change C. the colonists were very strong D. both tea nd coffee are good for the health 236 2. Coffee and tea plants grow in…………. A. Great Britain B. The tropics C. Boston D. both A and C 3. Tea became a a symbol of………….. A. a good, hot drink B. parties in Boston C. unfair taxation D. a wealthy country 4. Why was the Tea Act not a good idea? A. Because it helped to push the colonists into war B. Because it made coffee become too popular. C. Because it made English people visit the tropics. D. Because people didn’t obey it. 5. On the whole, the article tells about………… A. beverages in Europe in the 1600s B. England’s resentment of taxes C. the favorite drink in England. D. one reason why Americans drink coffee. B. In 776 B. C, the first Olympic Games were held at the food of Mount Olympus to honour the Greek’s chief god, Zerus. The warn climate for outdoor activities, the need for preparedness in war, and their lifestyle caused the Greeks to create competitive sports. Only the elite and military could prticipate at first, but later the games were open to all free Greek maleswho had no criminal record. The Greeks emphasized physicals fitness and strengthin their educationof yourths. Therefore, contests in jumping, running, discus and javelinthrowing, boxing, and horse and chariot racing were held individual cities, and the winners competed every four yearsat Mount Olympus. Winnerswere greatlyhonoured by having olive wreaths placed on their heads and having poems sung about their deeds. Originally these contests were held as games of friendship, and any wars in progress were halted to allow the games to take place. They also helped to strengthen bonds among competitors and the different cities represented. The Greeks attached so much importance to the games that they calculated time in four year cycles called “Olympiads”, dating from 776 B.C. The contests coicided with relgious festivities and constitued an all- out effort on the part of the participants to please the gods. Anyone who disobeyed the rules were dismissed and seriously punished. These athletes brought shame not only to themselves but also to the city they represented. 6. Why were the Olympic Games held? A.To stop wars B. To crown the best athletes C. To honour Zeus D. To sing song about the athletes 7. Approximately how many years ago did these games originate? A. 800 years B. 1,200 years 237 C. 2,300 years D. 2,800 years 8. The word “halted” in line 11 means nearly the same as……….. A. started B. fixed C. stopped D. encouraged 9. What is an “ Olympiad”? A. The time it took to finish a war B. The time took to finish the games C. The time between games D. The time it took the athletes to train. 10. What is the main idea of this passage? A. Physiccal fitness was anintegral part of the lives of the acient Greeks. B. THE Greeks severely punished those who did not participate in the games C. The Greeks had always encouraged everyone to participate in the games D. The Greeks had the games coincide with religious festivities. Exercise 5: Read the two passages and choose the best answer for each of the questions that follow. A. Jan Dibbets may someday have his work revered as much as his Dutch predecessors: Verneer, Van Gogh, and Rembrandt. At a time when the trends in art are toward abstraction and minimalism, Dibbetes’ work intergrates these two dispatate trends into one remarkable whole. In one series of composite works, he arranged sections of architectural photographs into rounded patterns. Then, with pen and ink and watercolors, he connected the segments together into 360degree circular forms blending the abstract with the real and the imagined. The imaginary images resemble the views of ceiling in gothic, baroque, and neoclassical buildings as they might be seen from the ground floor. 1.As used in line 2, what is the meaning of the word “time”? A.season B. period C. cetury D. invetal 2.The word “ disparate” in line 3 is closest in meaning to…….. A. similar B. unspoken C. different D. unknown 3. The word “ neo- classical” as used in the last sentence refers to…… A. trends in art B. museum decorations C. artistic pattens D. architectural styles 4. As decribed in the passage, Dibbetes used all of the following elememts except. A. pen and ink B. architectural photographs C. water colors D. still- life drawing 5. What does this passage mainly discus? A. the life of Jan Dibbetes B. new forms of art created by Dibbetes C. an exhibite of abstract art D. an art that follows the style of Verneer, Van Gogh, and Rembrandt 238 B. When I open the first “Body Shop”in 1976 my only object was to earn enough to feed my children. Tgoday “ The Body Shop” is an international company rapidly growing all around the world.. In the yeaars since we began I have learned a lot. Much of what I have learned will be found in this book, for I believe that we, ad a company, have something worth saying about how to run a successful business without giving up what we really believe in. It’s npot a normal business book, nor is it just about life. The massage is that to succeed in business you have to be different. Business, as in life, I need to enjoy myself, to have a feeling of family and to feel excited by the unexpected. I have always wanted the people who work for “ The Body Shop” to feel the same way. Now this book sends these ideas of mine out into the world, make them public. I’d like to think there are no limits to our ‘ family’, no limits to what can be done. I find that an exciting thought. I hope you do, too. 6. What is the writer’s main purpose in writing this text? A. To tell the reader her life. B. To introduce her ideas to the reader. C. To explain how international companies operate. D. To tell the reader how she brought up her family. 7. What would someone learn from this text? A. How to make a lot of money B. how to write a book about business C. What’s the writer’s family like D. What’s the writer’s book is about 8. How does the writer feel about the business she run? A.She doesn’t care about success if her children are fed B. She just runs it for her own entertainment. C. It is not like any other company. D. It is likely to become even more successful. 9. What kind of workers does the writer like to employ? A. Workers who can explain her ideas B. Workers who get on well with the public. C. Workers who have the same attitudes as she does D. Workers who have their own families 10. What kind of person does the writer seem to be? A. She seems to be someone with strong opinions B. She doesn’t seem to be confident C. She is mainly interested in making money D. She sees running a business as just a job Exercise 6: Read the following two passages and choose the best answer A, B, C or D for each of the questions. The World Trade Organization (WTO), founded of January 1, 1995, aims to encourage international trade to flow as freely as possible, making sure that 239 trade agreements are respectes and that any disputes can be settled. In the five years since its founding, the WTO has become well known as one of the world’s most powerful economic organizations, taking its place alongside the World Bank and International Monetary Fund. The system of global rules for international trade , however, dates back half a century to 1948 when the General Agreement on Tarriffs and Trade ( GATT) was formed after World War II. As time went by, it became clear that the GATT had two major drawbacks – the limited areas of trade it covered, and the lack of an effective system to settle disputes. After seven years of trade talks ending in 1994, the so- called Uruguay Round finally gave birth to the WTO, complete with an effective system to settle disputes and to form new rules covering trade in services and intellectual property. Even after seven years of talks and with 22, 500 pages of agreements reached, problems remained, especially in the difficult- to- deal- with areas of agriculture and services. 1.WTO and GATT are…… A.acronyms B. synonyms C. antonyms D. homonyms 2. What is the purpose of the WTO? A. to encourage international trade to flow freely B. To make sure that trade agreements are respected C. to settle any disputes D D. All of the above 3. The writer thinks that the WTO has become A. more powerful than any other economic organizations B. the world’smost powerful economic organization. C. one of the world’s most powerful economic organizations. D. less powerful than World Bank and International Monetary Fund. 4.According to the passage, the GATT stopped working……. A. in 1948 B. in 1994 C. after the World War ended D. during the seven years of the Uruguay Round talk 5. the word “it’ in bold refers to…………. A. the WTO B. the GATT C. the World Bank D. theInternational Monetary 6. The WTO is better than the GATT in that…… A. it can settle disputes in more areas of international trade. B. it enables its members to sign agreements more easily C. it deals with problems in agriculture and sevices more effectively. D. it pays more attention to serviceand intellectual properly 7. The word” went by” could best replaced by……… A. flew B. flowed C.passed D. past 8. The word “drawbacks” in paragraph 1 means…… A. weak points B. strong points C. improvements D. changes 9. According to the last paragraph, which of the following statements is true? A. The WTO is the world’s most powerful economic organization. B.The GATT was founed in Uruguay in 1995 240 C. The WTO has an effectivesystem to settle disputes and to form new rules. D. All the countries in the world are member nations of the WTO. 10. Which of the following is not true about the WTO? A. It has an effective system to settle disputes. B. It can solve all problems in the world. C. It has new rules to cover trade in services and intellectual properly. D. It makes international trade agreements respected. Exercise 7: Read the following passage and choose the best answer A, B, C or D for each of the questions. When the first white men came to America, they found vast amounts of natural resources of tremendous value. Forests covered a large part of the nation; later gas, oil andminerals were found in unbelievable amounts. There was a great abundance of very fertile soil. Forests , prainries, steams and rivers abounded with wildlife. So vast were these sources that it seemed that they could never be used up. So forests were destroyed to make way for farmland. Grasslands and prainries were plowe and harrowd. Minerals and oil were used in great quantitiesto supply a young industrial nation. Almost every river became the scene of factories, mills and power companies. Mammals and birds were slaughtered for food and sport. Within a short time, the result of obvious. Flood caused millionsof dollars worth of damageyearly. The very fertile soil washed away or blew up in great clouds. The seemingly inexhaustible oil and minerals showed signs of depletion. Rivers were filled with silt from eroding farms and wastes from disappeared, and some mammals seemed on the verge of going. Future timber shortages were predicted. In short, Americans soon came to realize that some of conservation program must be set up, if future as well as present Americans were to share in the resources that are the heritage of every American. 1.The title that best expresses the main theme or subject of this election is………. A.What the first whote men found in America. B. The loss of topsoil C. The cause of timber shortage D. the story of America’s natural recourses. 2.It seemed to be early American settles that………………… A. fertile sol was scarce B. the natural resources were inexhaustible C. forests should not be cut D. there was a shortage ofminerals. 3. The use of America’s natural recourses by the early settles was……….. A. careless B. scientific C. unbelievable D. predicted 4. much of the fertile soil of America has…….. A. sunk deep into the earth B.been covered by lake 241 C. been eroded by wind and wate D. become the sense of factories 5. According to the passage, all the followings are true EXCEPT. A. They early American settlers used a lot of minerals and oil. B. They kille animals for food and sport. C. They grew different kinds of plants in prairies. D. They plowed and harrowed grasslands and prairies. 6.the word “abounded with” could best replaced by……. A. were abundant in B. were rich with C. were plentful of D. were a lot of 7. The word “ silt” in paragraph 2 is closest in meaning to……… A. dust B. soil C. mud D. land 8. One reason why many of our rivers are no longer suitable living places for fish is that A. too manyfish have been caught B. a conservation program has been set up C. floods have caused much damage. D. factories have dumped waste into the rivers. 9. Some species of birds and mammmals seemed……… A. to become ixtinct B. to die C. to be killed D. to be slaughtered 10. Americans soon came to realize that………… A. They should stop killing animals for food. B. They must give up exploiting minerals. C. They shouldn’t reclaim the land D. they must establish a coservation program. Exercise 8: Read the following passage and choose the best answer for each of the questions. Aging is the process of growing old. It occurs eventualy in every living thing provided , of course, that an illness or acident does not kill it prematurely. The most familiar outward signs of aging may be seen in old paople, such as the graying of the hair and the wrinkling of the skin. Signs of aging in a pet dog or cat include lass of playfullness and energy., a decline in hearing and eyesight, or even a slight graying of the coat. Plants age too, but the signs are much harder to detect. Most body parts grow bigger and stronger, and function more efficiently during childhood. They reach their peak at the time of maturity, or early adulthood. After that, they begin to decline. Bones, for example, gradually become lighter and more brittle. In the aged, the joints between the bones also become rigid and more inflexible. This can make moving very painful. All the major organs of the body show signs of aging. The brain , for example, works less effeciently, and even gets smaller in size. Thinking processes of all sorts are slowed down. Old people often have trouble in remembering recent events. 242 One of the most serous changes of old age occurs in the arteries, the blood vessels that lead from the heart. They become thickened and constricted, allowing less blood to flow the rest of body. This condition accounts, directly or indirectly,for many of the diseasesof the aged. It may, for example, result in heart attack. Aging is not a uniform process. Different parts of the body wearout at diffeent rates. There are great differences among people in their rate of aging. Even the cells of the body differ in the way they age. The majority of cells are capable of reproducing themselves may times during the course of a lifetime. Nerve cells and muscle fibers can never be replace once they wear out. Gerontologists- scientists who study the process of aging- believe this wearing out of the body is controlled by a bult- in biological time clock. They are trying to discover how this clock works so that they can slow down the process.This could give man a longer life and a great number of productive years. 1.What is the main idea of the first paragraph? A. Signs of aging are easier to detect in animals than in plants. B. Aging occurs in every living thing after it has reached maturity. C. not all signs of aging are visible D. the ouward signs of aging may be seen in old people. 2. What does the word “it” in line 2 refer to? A. aging B. a living thing C. an illness D. an accident 3. All the followings may be the outward signs of aging EXCEPT……… A. the graying of the hair B. the wrinkling of the skin C. the decline in hearing and eyesight. D. the loss of appetite 4. When does the human body begin to losevigor and the ability to function effeciency? A. Soon after reaching adulthood B. During childhood C. Early adulthood D. Past middle age 5. What happens to memorization when the brain begins to age? A. It works less B. It becomes forgetful C. It declines D. It slows down 6. What does “ Aging is not a uniform pracess” mean? A. Not all living thins age B. Not all people age have the same age C. Not all people have signs of aging D. Aging doesn’t occur in all people. 7. The word “brittle” as used in the second paragraph means ? A. soft and easily bent B. hard and enduable C. hard but easily broken D. rigid nd inflexible 8. According to the passage, what condition is responsible for many of the diseases of the old? A. the arteries become thickened and constricted B. the blood vessels lead from the heart. 243 C. the brain gets smaller in size D. bones become lighter and brittle 9. What is the main idea of the least paragraph? A. Gerontologists have controlled the process of aging. B. Gerontologists are working hard to help people live longer and more healthily. C. Gerontologists are trying to give people an enternal life D. Gerontologists are now able to slow down the process of aging. 10. According to the passage, which of the following is NOT true? A. All lving things grow old. B. Aging is unavoidable in any livings things. C. Plants show less signs of aging than any other living things. D. Most body parts wear out during the courses of a lifetime. Exercise 9: Read the following passage and choose the best answer A, B, C or D for each of the questions. “ We would like to interview you…” Joyful words for the job- seeker, but my letter carried a warning: “ you will be required to take a psychomestric test.” More than seven percent of companies now use these “ objectove” tests for potential employees. They are meant to give a true picture of candidates that removes the unfairness that may result from the personal opinions of interviewers. On the day of my interview for the job of assistant to a company Public Relations consultant, my nerves were made worse by finding that the office was close to a hospital with particularly unhappy associations. Luckily, I had deliberately got there early so that I was able to calm myself down before a secrectary rushed me upstairs for my test. Keeping to a strict time limit, I had to ACCESS groups of adjectives, marking which most and which least matched my ideas of myself at work. Choosing one quality out of four when all seemed appropriate was difficult, more difficult than the interview that followd- thought I felt I hadn’t impresed in that either. Confirmation of this arrived a week later. My rejection letter was accompanied by a copy of the Private and Confidential Personal Profile Anaylist- two and a half sides of paper, based on that 10- minute test. The Profile’s rude inaccuracy and ITS JUDGEMENTAL tone were harder to eccept than the fact that I had been turned down for a job. Apparently, I have “no eye for detail”; I am also “ a foceful individual… who leads rather than directs” and I am “ motivated by finacial reward to pay for good living.” The words “impatient”, “ restless” and “ strong- willed” also came up. “ A portrait of an ambitious, power- mad person,” said a psychologist friend of 15 years old to whom I showed the Profile. She said it didn’t apply to me at all. I know myself to be careful, industrious checker. I am shy but cheerful and a bit over- anxious to be thought rceative. I am not a power- crazed person. 244 What would I do, I worried, I had to take another test for another job, and this unattractive personality emerged again? I sent the company a polite disagreement with the Pofile, purely for therecord. Meanwhile, I made a few enquires. Had my emotional state of mind mae the results untypical of me? I had been disturbed to find the officeso close to a hospital that held unhappy memories for me. ‘ State of mind will have an impact,” says Shane Pressey, an occupational psychologists,” but on thr whole its effect will be relatively minor. It appears that the test was an inadequate tool for the amount of information they were trying to get out of it, and it is not suprising that there was inaccuracies. Too late for thatparticular job. I arranged to sit another psychometric test. This one took much longer and was more thorough: the profile was also more detailed and accurate- it showed my eye for detail and the fact that I have a problem meeting deadlines. In my experience is anything to go by, the ob candidate should be suspicious of 10-minute tests that result in generalised- and possibly wildly inaccuratejudgements. I accept that it would be costly to arrange for face- tofacediscussion of test results with all job candidates, but a telephone call would bepreferable to simply receiving a written “profile” through the post and having no apportunity to discuss its contents. 1.Before the writer took the test….. A.felt that she was unlikelyto do it very wel. B. made sure that she was mentally prepared for it. C. believed that such tests were fair to candiates. D. did some research into tests of that kind. 2. What did the writer think when she took the test? A. She couln’t understand some of the questions. B. She found that there was not enough time to do it C. She felt that she had not done it very well D. She decided that it would not prove anything. 3. What does he writer mean by “judgemental” in the fifth paragraph? A. critical B. impatient C. impersonal D. thoughtful 4. When the writer received the Personal Profile Anaylysis, she……… A. was offended by the comments made about her answers. B. was glad that she had not been offered the job. C. regretted some of the answers she had given in the test. D. realisd that her personality wold not have suitd the job. 5. Why did the Profile worry her? A. It made her feel that she had been to self- confident before. B. It indicated that she might have trouble getting a job in the future. C. It did not show that she was capable of being a creative person D. it told her thins about heself that she hadn’t noticed before. 6. What did she find out after taking the test for a job? 245 A. the way she ws feelig had badly effected her performance in it B. Spychometric tests seldom provide reliable information about people. C. Many job candidates are unwilling to take psychometric tests. D. Employers should sop asking cadidates to take psychometric tests. 7. What does the writer recommend? A. Candidates should be able to talk about their test results with employers. B. Employers should pay no attention to the result of psychometric tests C. Candidates shold notbe concerned about taking psychometric tests D. Employers should stop asking cadidates to take psychometric tests 8. Wh does the writer describe her experience? A. It is tyoical of experiences that a great many other people have. B. it shows that no method of selecting job candidates can ever be fair. C. It is an example of how difficult it can be for someone to get a job. D. It illustrates faults in a new method of assessing job candidates. 9. The word “access” in the third paragraph stands for: A. study B. reach C. use D. memoriese 10. the writer’s attitude towards the psychometric test is……… A. positive B. negative C. sastifying D. admiring Exercise10: Read the following two passages and choose the best option ( A, B, C or D) to answer each of the questions. A. There was one thing that I fond strange on my first day as housekeeper at Monk’s House. The floors in the house were very thin. The bathroom was dirrectly above the kitchen, and when Mrs. Woolf was having her bath before breakfast, I could hear her talkng to herself. On and on she went, talk tlk talk, asking questions and iving herself the answers. I thought there must be two or three people up there with her. When Mr. Woolf saw that I looked supprised he told me that MRS. Woolf always said the sentences out lound that she had written during the night. She needed to know if they sounded right and the bath was a god place for trying them out. I was not allowd to make coffee at Monk’s Houe- Mr and Mrs. Woolf were very particular about coffee and always made it themselves- so Mr Woolf came into the kitchen at eight o’clock every morning to make it. When we carriedthe breakfast tray to Mrs. Woolf’s room I noticed that she had always been working during the night. There were pencils and paper beside her bed so that when she woke up she could work, and somes it seems as though she had had very little sleep. Mrs. Woolf’s bedroom was outside the house in the garden. I usd to thnk how inconvenient it must be to have go out in the rain to go to bed. Her bedroom had been added on to the back of the house; the door faced the grden and awindoe at the side openned ot on to the field. Because the writing room was 246 small, he had had a large one built for her at the end of the garden against the church wall. I can always remember her coming to the ouse each day from the writing room., when I rang the bell for lunchat 1 o’clock she usd walk down through the garden smoking one of he rfavorite ciggarettes in a long holder. She was tall and thin and very graceful. She had large, deep- set eyes and a wide curving mouth- I think perhapsit was this that made her face seem particularly beautiful. She ware long skirt- usually blue or brown- in the fashion of the day, and silk jacket of the same color. Her cigarettes were made from a special tobacco called My Mixture. Mr. Woolf brought for her in Londonand, in the evenings, they used to sit by the fire and make cigarrretes themselves. Mrs. Woolf wore clothes that suited her well. I pressed them for her and did any sewing that was necessary- she was not ablt to sew, although sometimes she liked to try. There was one thing in the kitchen that Mrs. Woolf was very good at doing; she could make beautiful bred. 1.What was particularly unusual about Monk’s House? A. The bathroom was next to the kitchen. B. Mrs. Woolf’sbedroom door opened on to the garden. C. The kitchen window looked out over fields. D. The breakfast room was upstairs. 2. What did the writer observe about Mrs. Woolf’s writing habbits? A. She did the great deal of writing at night. B. She worked in the garden whenever she could. C. She preferred to write in the house. D. She sometimes wrote in thebath. 3. Mrs. Woolf wore clothes were……… A. in matching colors. B. designed for her individually. C. suitable for country life. D. rather dull and unattractive. 4. Mr. Woolf’s attitude toward his wifeseeed to be that…… A. He found her strange ways difficult to except. B. Hde led a very separate life C. He did what he could for her. D. He watched over her all time. 5. As far as the housework was concerned, Mrs. Woolf….. A. was too busy to do any. B. disliked dong any C. was very bad at it D. liked one or two particular tasks. B. A Japanese construction copany plans to create a huge independent citystate, akin to the legendary Atlantic, in the middle of the Pacific Ocean. The city, dubbed “ Marinnation”, would have about one million inhabitants, two airports, and possibly even a space sport. Mainnation, if built, would be separate country but could serve as home for international organization as the nited Nations and the World Bank.Aside from the many political social problems that would have to be solved, the engineering task envisaged is monumental. The 247 initial stage requires the building of a circular da 18 miles in diameterattached to the bed relativelyshallow place in international waters, Then, several hundred powerful pumps, operating for more than a year, would suck out the sea water from within the dam. When empty and dry, the area would be about 300 feet below sealevel. According to designers, the hardest task from engineering point of view would be to ensure that the dam is leak proof and earthquake proof. If all goes well, it is hoped that Marinnation could be ready for habitation at the end o the second decade of the twenty- first century. Whenever anyone would want to live in such an isolate and artifical community, however, it will remain an open question until that time. 6.To what does the underlined phrase “the city” refer? A. Japenese construction company .B. Atlantic C. The United Nations D. A future city. 7. What kind of city will Marinnation be? A. underground B. underwater C. marine D. legendary 8. The tone of thepassage is… A. sarcastic B. humous C. judgemental D. informative 9. The problem of Marinnation focused on her mainly……. A. human B. engineering C. political D. social 10. By refering to Atlntic in the passage, the author is saying that A. Marinnation wll never be built B. Marinnation is a city in the ocean C. even if, Marinnation will fail D. Marinnatin is only a dream Answerkey Exercise 1: 1.b 6. d 2. c 7. a 3. b 8. a 4. a 9. c 5. d 10. c 2. d 7. c 3. c 8. b 4. c 9. a 5. d 10. d 2. C 7. A 3. D 8. A 4. A 9. C 5. B 10. D Exercise 2: 1.a 6. a Exercise 3: 1.B 6. C Exercise 4: 248 1.B 6.C 2. B 7. D 3. C 8. C 4. A 9.C 5. D 10. A 2.C 7. D 3. D 8. C 4. D 9. C 5. B 10. A 2. D 7. C 3. C 8. A 4. B 9. C 5. B 10. A 2. B 7. C 3. A 8. D 4. C 9. A 5. C 10. D 2.B 7. C 3. C 8. A 4. A 9. B 5. C 10. C 2. C 7. A 3. A 8. D 4. A 9. C 5. B 10. B 2. A 7. B 3. A 8. D 4. C 9. B 5. D 10. D Exercise 5: 1.B 6. B Exercise 6: 1.A 6. A Exercise 7: 1.D 6. A Exercise 8: 1.B 6. B Exercise 9: 1.B 6. D Exercise 10: 1.B 6. D 249 SỞ GD & ĐT VĨNH PHÚC TRƯỜNG THPT VĨNH YÊN CHUYÊN ĐỀ ÔN THI ĐẠI HỌC, CAO ĐẲNG MÔN SINH HỌC Tên chuyên đề : PHƯƠNG PHÁP GIẢI MỘT SỐ DẠNG BÀI TẬP DI TRUYỀN LIÊN KẾT VỚI GIỚI TÍNH Người viết chuyên đề : Nguyễn Thị Thúy Chức vụ : Giáo viên Đơn vị công tác : Trường THPT Vĩnh Yên 250 Vĩnh Yên, 03/2014 A. LÝ DO CHỌN ĐỀ TÀI Sinh học là môn khoa học chuyên nghiên cứu các quá trình sống ,các hiện tượng di truyền và biến dị ,các quy luật di truyền của sinh vật .Đặc thù của bộ môn là nghiên cứu từ thực nghiệm ,từ đó rút ra được các quy luật chung cho toàn bộ sinh giới. Trong quá trình giảng dạy chương các quy luật di truyền ( Sinh học 12 ) và đặc biệt là phần di truyền liên kết với giới tính, đây là phần có kiên thức khá đa dạng . Do đó khi giải các bài tập trong chương này nói chung và bài tập di truyền liên kết giới tính nói riêng học sinh đã gặp rất nhiều khó khăn. Đây cũng là phần bài tập thường có trong các đề thi tuyển sinh vào các trường cao đẳng và đại học Mặt khác số tiết học môn sinh trong 1 tuần rất ít mà theo phân phối chương trình sinh học 12 số tiết giải bài tập không nhiều và thường ra bài tập cụ thể, rời rạc, chưa có tính hệ thống, do đó phần lớn các em còn lúng túng trong phương pháp giải bài tập. Hiện nay, môn sinh là một trong các môn thi theo hình thức trắc nghiệm nên việc phân loại các dạng bài tập để đưa ra phương pháp giải hợp lí là điều rất cần thiết Vì vậy để học sinh có phương pháp tư duy tốt, biết vận dụng kiến thức linh hoạt trong việc giải được nhanh các dạng bài tập liên quan đến di truyền liên kết giới tính , tôi đã chọn chuyên đề: “PHƯƠNG PHÁP GIẢI MỘT SỐ DẠNG BÀI TẬP DI TRUYỀN LIÊN KẾT VỚI GIỚI TÍNH” B. PHẠM VI CHUYÊN ĐỀ Chuyên đề áp dụng cho học sinh lớp 12 Số tiết thực hiện chuyên đề : 8 tiết C. NỘI DUNG I. Hệ thống kiến thức sách giáo khoa sử dụng trong chuyên đề Nhiễm sắc thể (NST) giới tính và cơ chế xác định giới tính bằng nhiễm sắc thể Nội dung thí nghiệm của MoocGan Đặc điểm di truyền của gen nằm trên NSTGT X và Y II. Nội dung kiến thức sử dụng để bồi dưỡng 1. Nhiễm sắc thể giới tính (NST-GT) ở các đối tượng: + Người, ĐV có vú, cây chua me, cây gai: ♀ XX, ♂XY + Chim, bướm, gia cầm, bò sát, ếch nhái, 1 số loài cá, loài tằm dâu, dâu tây: ♀ XY, ♂XX + Bọ xít, rệp, châu chấu: ♀ XX, ♂XO + Bọ nhậy: ♀ XO, ♂XX .2. Nhận dạng quy luật di truyền: 251 + Dựa vào kết quả các phép lai thuận và lai nghịch: - Nếu khác nhau à Gen NST GT - Tính trạng của bố chỉ xuất hiện ở con ♂à di truyền (DT thẳng)à gen NST GT Y - Tính trạng lặn chỉ xuất hiện con ♂à DT chéoà Gen NST-GT X (tính trạng chỉ xuất hiện ở giới dị giao) + Dựa vào di truyền chéo: - Dấu hiệu: tính trạng từ ông ngoại biểu hiệnà con gái không biểu hiệnàcháu trai biểu hiệnà gen NST-GT X + Tính trạng biểu hiện không đồng đều ở 2 giới: - Cùng 1 thế hệ: tính trạng nào đó chỉ xuất hiện ở con ♂ còn giới ♀ không có và ngược lạià gen NST-GT III. Một số dạng bài tập liên quan đến giới tính và NST giới tính: 1. Dạng bài tập thuận: a. Phương pháp giải Biết kiểu hình (KH) của bố, mẹ (P), biết gen liên kết trên NST-GT, biết trội lặnà Xác định kết quả lai. Bước 1: Từ KH của P , tính trội lặn và gen liên kết trên NST-GTà kiểu gen (KG) P Bước 2: Viết sơ đồ lai (SĐL) để xác định kết quả. Lưu ý: Với những bài toán thuận có liên quan đến hoán vị gen thì khi viết giao tử chú ý tỷ lệ phụ thuộc vào f (tần số hoán vị) b. Các ví dụ Ví dụ 1: Ở 1 giống gà, các gen xác định lông trắng và lông sọc vằn nằm trên NST-GT X. Tính trạng sọc vằn là trội so với tính trạng lông trắng. Tại 1 trại gà khi lai gà mái trắng với gà trống sọc vằn thu được đời con bộ lông sọc vằn ở cả gà mái và gà trống. Sau đó, người ta lai những cá thể thu được từ phép lai trên với nhau và thu được 594 gà trống sọc vằn 607 gà mái trắng và sọc vằn. Xác định KG bố mẹ và con cái thế hệ thứ 1 và 2. Bài giải Quy ước: A -sọc vằn a -lông trắng. Gà trống có KG XX, gà mái có KG XY. Gà trống sọc vằn có KG XAXA hoặc XAXa Gà mái lông trắng có KG XaY F1 thu đc toàn bộ gà có lông sọc vằn → Ptc P: XAXA (Sọc vằn) x XaY (lông trắng) Gp XA Xa,Y F1: XAXa : XAY (100% sọc vằn) F1 x F1 : XAXa (Sọc vằn) x XAY (Sọc vằn) 252 GF1: XA, Xa XA,Y F2: KG: XA XA (Sọc vằn): XA Xa (Sọc vằn): XaY(lông trắng): XA Y (Sọc vằn) KH: 2 trống sọc vằn:1 mái sọc vằn:1 trống sọc vằn 2. Dạng bài tập ngược: Biết KH P,kết quả của phép laià Xác định KG P và viết SĐL. Bước 1: Tìm trội lặn và quy ước gen Bước 2: Nhận dạng QL di truyền chi phối và từ tỷ lệ PL KH của thế hệ con lai và đặc điểm của gen trên NST-GTà KG của P Bước 3: Viết SĐL 2.1: Dạng 1 : Bài tập di truyền lien kết với giới tính thuần a. Phương pháp giải: + Bước 1: Tìm trội lặn và quy ước gen + Bước 2: Nhận dạng QL di truyền chi phối và từ tỷ lệ PL KH của thế hệ con lai và đặc điểm của gen trên NST-GTà KG của P + Bước 3: Viết SĐL b. Các ví dụ: Ví dụ 1: (CĐ 2010) Ở ruồi giấm, tính trạng màu mắt do một gen gồm 2 alen quy định. Cho (P) ruồi giấm đực mắt trắng x ruồi giấm cái mắt đỏ, thu được F1 100% Ruồi giấm mắt đỏ. Cho F1 giao phối tự do với nhauà F2 tỉ lệ kiểu hình (TLKH): 3 mắt đỏ:1 mắt trắng, trong đó mắt trắng là con đực. Cho mắt đỏ dị hợp F2 x đực đỏàF3. Biết không có đột biến, theo lý thuyết trong tổng số ruồi F3 ruồi đực mắt đỏ chiếm tỷ lệ bao nhiêu. A.50% B.75% C.25% D.100% Bài giải Từ TLKH F2: 3:1 (mắt trắng chỉ biểu hiện ở đực) à gen quy định màu mắt trên NST –GT. Mắt đỏ-D, mắt trắng-d P: (Đỏ) XDXD x XdY (Trắng) F1: (Đỏ) XDXd x XDY (Đỏ) F2: XDXD XDXd XDY XdY Đỏ Đỏ Đỏ Trắng D D F2: X Xd (Đỏ) x X Y (Đỏ) F3: XDXD XDXd XDY XdY Ở F3 ruồi đực mắt đỏ chiếm 25% (Đ/A C) Ví dụ 2 (ĐH 2013): Ở ruồi giấm, gen quy định màu mắt nằm trên vùng không tương đồng của nhiễm sắc thể giới tính X có 2 alen, alen A quy định mắt đỏ hoàn toàn so với alen a quy định mắt trắng. Lai ruồi cái mắt đỏ với ruồi đực mắt trắng (P) thu được F1 gồm 50% ruồi mắt đỏ, 50% ruồi mắt trắng. Cho F1 giao phối tự do với nhau thu được F2. Theo lí thuyết, trong tổng số ruồi F2, ruồi cái mắt đỏ chiếm tỉ lệ 253 A. 6,25% B. 31,25% C. 75% D. 18,75% Bài giải . P: XA Xa x Xa Y F1: ¼ XA Xa: 1/4 Xa Xa:1/4 XA Y:1/4 Xa Y Tần số alen Xa ớ giới cái là: ¾; Tần số alen Xa ở giới đực: ½ tần số kiểu gen Xa Xa ở giới cái là: ¾ x ½ = 3/8 = 37.5% Tỷ lệ kiểu hình mắt đỏ ở giới cái là: 100% - 37,5% = 62,5% Tỷ lệ kiểu hình mắt đỏ trong quần thể là: 62,5%/2 = 31,25% (vì tỷ lệ đực cái là 1:1) à Đáp án B A. XAY × XA Xa B. XAY × Xa Xa C. Xa Y × XAXA D. Xa Y × Xa Xa 2.2. Dạng 2: Quy luật di truyền liên kết với giới tính tích hợp với quy luật phân li độc lập a. Phương pháp giải: + Bước 1: Tìm trội lặn và quy ước gen (nên xét từng tính trạng, xem nằm trên NST thường hay giới tính. Nếu tính trạng nào đều có ở đực và cáià nằm trên NST thường, tính trạng nằm trên NST GT có đặc điểm của gen trên NST GT) + Bước 2: Nhận dạng QL di truyền chi phối (Nhân 2 tỷ lệ riêng nếu thấy kết trùng với TLPL KH F2 theo đầu bàià Tuân theo QL PLĐL, có 1 cặp gen nằm trên, NST GT và Từ TLPL KH F+gen trên NST-GTà KG P + Bước 3: Viết SĐL b. Các ví dụ Ví dụ 1: Trong 1 thí nghiệm lai ruồi giấm con cái cánh dài, mắt đỏ x đực cánh ngắn, mắt trắngàF1: 100% cánh dài-mắt đỏ. Cho F1x ngẫu nhiênà F2 ♀:306 Dài -đỏ:101 Ngắn-Đỏ và ♂: 147 Dài- đỏ:152 Dài trắng:50 Ngắn đỏ:51 Ngắn Trắng. Mỗi gen quy định 1 tính trạng. Giải thích kết quả thu được và viết SĐL Bài giải: Giải thích: + Bước 1: Tìm trội lặn và quy ước gen (nên xét từng tính trạng, xem nằm trên NST thường hay giới tính. Nếu tính trạng nào đều có ở đực và cáià nằm trên NST thường, tính trạng nằm trên NST GT có đặc điểm của gen trên NST GT) - Ruồi giấm: ♂ XY, ♀XX; F1 100% Dài đỏà Dài trội ngắn; đỏ trội trắng - Xét riêng tính trạng hình dạng cánh F2: Dài:Ngắn= (306+147+152):(101+50+51)=3:1 Ở ♂: Dài : Ngắn=(147+152):(50+51)=3:1; Con ♀; Dài : Ngắn=(306):(101)=3:1 => Gen quy định TT hình dạng cánh nằm trên NST-thường và tuân theo QL PL A-Dài, a-Ngắn - Xét riêng tính trạng màu sắc mắt F2: Đỏ :Trắng= (306+101+147+50):(152+51)=3:1 và có sự phân bố khác nhau ở 2 giới mà ta thấy tính mắt trắng chỉ có ở con ♂ nên gen quy định màu mắt phải nằm trên NST-GT X và trên Y không có alen tương ứng. B-Đỏ, b-Trắng + Bước 2: Nhận dạng QL di truyền chi phối (Nhân 2 tỷ lệ riêng nếu thấy kết trùng với TLPL KH F2 theo đầu bàià Tuân theo QL PLĐL, có 1 cặp gen nằm trên NST GT và Từ TLPL KH F2 gen trên NST-GTà KG P 254 - Ta có (3 dài:1 ngắn)x(3 đỏ:1 trắng)=9:3:3: 1 trùng với tỷ lệ PLKH ở F2 nên sự di truyền đồng thời của 2 cặp tính trạng trên tuân theo QL PLĐL và có 1 cặp gen nằm trên NST GT - F1 Đồng tính=> P t/c và từ lập luận trên à KG P ♀ dài-mắt đỏ: AAXBXB ♂ Ngắn-mắt trắng: aaXbY SĐL PàF2 P: AAXBXB x aaXbY Gp: AXB aXb=aY=1/2 F1: AaXBXb x AaXBY GF1: AXB =AXb= AXB =AY= aXB = aXb=1/4 aXB = aY=1/4 F2: kẻ khung pennet TLKH: 3 cái Dài đỏ:1 cái Ngắn đỏ:3 đực Dài đỏ:3 đực dài trắng:1 đực Ngắn đỏ:1 đực ngắn trắng 2.3. Dạng 3: Quy luật di truyền liên kết với giới tính tích hợp với liên kết gen hoàn toàn a. Phương pháp giải: - Nếu phép lai gồm nhiều cặp tính trạng ta tách riêng từng cặp tính trạng và xét sự di truyền của từng cặp tính trạng. * Từ THPL KH của từng cặp=> KG tương ứng của nó * Khi kết hợp sự di truyền của cả 2 cặp tính trạng, trường hợp hai cặp gen cùng liên kết với NST GT X, ta phải biện luận để xác định chúng liên kết gen hay hoán vị. * Muốn vậy ta căn cứ vào sự xuất hiện KH giới tính XY ở thế hệ sau để suy ra tỷ lệ giao tử của giới cái XX của thế hệ trước, từ đó suy ra liên kết gen hay hoán vị gen. * Biết tỷ lệ giao tử cái ta suy ra KG và viết SĐL - Nếu phép lai ở từng cặp là dị hợp lai với dị hợp (gen tồn tại trên NST thường) * Có thể áp dụng công thức sau: (A-B-) – (aabb)=50% A-bb + aabb= 25%; aaB- + aabb = 25% (A-B-) + A-bb =75%; (A-B-) + aaB- =75% * Aa x Aa à ¾ A-: ¼ aa - Nếu là phép lai phân tích thì lưu ý tỷ lệ của đồng hợp lặn luôn bằng 1 X D X d × X D Y à2/4 X D X − + 1/4 X D Y +1/4 XdY b. Các ví dụ Ví dụ 1: Ở ruồi giấm, alen A quy định thân xám trội hoàn toàn so với alen a quy định thân đen; alen B quy định cánh dài trội hoàn toàn so với alen b quy định cánh cụt; alen D quy định mắt đỏ trội hoàn toàn so với alen d quy định mắt trắng. AB AB D d Thực hiện phép lai P: ab X X × ab X Y thu được F1. Trong tổng số các ruồi ở F1, ruồi thân xám, cánh dài, mắt đỏ chiếm tỉ lệ là 52,5%. Biết rằng không xảy ra đột biến, theo lí thuyết, ở F1 tỉ lệ ruồi đực thân xám, cánh cụt, mắt đỏ là A. 3,75% B. 1,25% C. 2,5% D. 7,5% D 255 Bài giải: D Bài yêu cầu tìm tỷ lệ ruồi đực thân xám, cánh cụt, mắt đỏ A-bb X Y * Tìm tỷ lệ A-bb như sau D − D Ta có các ruồi ở F1, ruồi thân xám, cánh dài, mắt đỏ (A-B-( X X + X Y D d D − D D )) = 52,5% mà trong phép lai X X × X Y à X X + X Y = 3/4 => Vậy A-B- = 0,525/0,75 = 0,7 Luôn có (A-B-) + A-bb =75% => A-bb = 0,75 - A-B- = 0,05. D * Tìm tỷ lệ X Y như sau X D X d × X D Y à Đực mắt đỏ chiếm tỉ lệ ¼ D Vậy ruồi thân xám, cánh cụt, mắt đỏ (A-bb X Y )= là 0,05x1/4 = 0,0125 ( Đáp án B) 2.4. Dạng 4: Quy luật di truyền liên kết với giới tính tích hợp với liên kết gen không hoàn toàn a. Phương pháp giải: + Có nhiều dấu hiệu cho thấy các tính trạng nghiên cứu được xác định bởi các gen liên kết với nhau như: - Tỷ lệ PL ở đời lai khác với tỷ lệ mong đợi đối với hai bên PLĐL cho thấy các gen di truyền liên kết với nhau. - Các tính trạng được xác định bởi các gen liên kết luôn được di truyền cùng nhau. - Liên kết gen hoàn toàn làm giảm số KG và KH ở đời con lai. Ngược lại, trao đổi chéo giữa các gen làm tăng số KG và KH ở thế hệ sau. - Tỷ lệ của các loại giao tử mang gen liên kết luôn bằng nhau, tỷ lệ của các giao tử mang gen trao đổi chéo cũng bằng nhau và nhỏ hơn tỷ lệ của các giao tử mang gen liên kết. - Trong một phép lai phân tích, việc có hai lớp KH có tần số lớn bằng nhau và hai lớp KH có tần số nhỏ bằng nhau cho biết trong đó có gen liên kết không hoàn toàn. + Với các gen liên kết không hoàn toàn nằm trên nhiễm sắc thể giới tính X mà không có các gen tương ứng trên Y, tần số hoán vị gen hoặc tỷ lệ KG, KH được xác định như trong trường hợp có trao đổi chéo một bên + TLPL KH ở 2 giới đực và cái khác nhau: Nếu XX cho 2 KH + nếu XY cho 4 lớp KH với TL không bằng nhauà Có hoán vị gen ở cá thể XX. Tính f dựa vào KH lặn nhất cá thể XY. + Nếu Đầu bài cho 100% Con đực và 100% con cáià khi tạo giao tử X-=Y=1 Nếu Đầu bài cho tổng số Đực và cái là 100% thì khi tạo giao tử X- =Y=1/2 + f=2 x giao tử hoán vị, giao tử liên kết=0,5-f/2 > 25%, giao tử hoán vị có tổ hợp gen là (A-,bb) - KH mắt đỏ ở F1 --> gồm các KG là (XDXD, XDXd, XDY) chiếm tổng tỷ lệ là 3/4 - Mà gen quy định tính trạng màu mắt PLĐL với gen quy định tính trạng màu sắc thân và hình dạng cánh 3,75% xám, cụt, đỏ => 3,75 (A-bb x D-) --> KH thân xám, cánh cụt ở F1 --> có tổ hợp gen là (A-bb)chiếm tỷ lệ = 3,75% : 3/4 = 5% = 0,05 AB D d AB D d X X X X - Vì P: ♀ ab x ♂ ab --> F1 có : (A-bb) + (aabb) = 0,25 (vì cộng hai tổ hợp gen này sẽ là tỷ lệ KH của KG bb =1/4) --> Tổ hợp gen (aa,bb) = 0,25 – 0,05 = 0.2 --> Đây là tỷ lệ KH thân đen cánh cụt ở F1 - Tỷ lệ ruồi cái F1 mắt đỏ gồm các KG là (XDXD, XDXd)= 1/4 + 1/4 = 1/2 --> tỉ lệ ruồi cái F1 có KH thân đen, cánh cụt, mắt đỏ là: 0,2 x 1/2 = 0,1 = 10% --> Đáp án A 2.5. Dạng 5: Di truyền lien kết với giới tính tích hợp quy luật tương tác gen a. Phương pháp giải: Các gen có thể tương tác với nhau để quy định một tính trạng. Phổ biến là hai gen không alen (và thường nằm trên các nhiễm sắc thể tương đồng khác nhau) tương tác với nhau. Vì vậy, tỷ lệ PL ở F2 thường là tỷ lệ biến đổi của phép lai hai tính (9:3:3:1) của Mendel. Ví dụ tỷ lệ 9:6:l. Có các kiểu tương tác chủ yếu sau: - Tương tác bổ trợ: Hai gen trội cùng có mặt trong một KG tương tác với nhau làm xuất hiện tính trạng mới, khác bố mẹ. Ngoài cơ chế tương tác, các gen còn có thể có các chức năng riêng. Vì vậy, kiểu tương tác bổ trợ có thể cho các tỷ lệ PL 9:3:3:1, 9:6:1 hoặc 9:7. - Tương tác át chế. Kiểu tương tác trong đó một gen ức chế sự biểu hiện của gen kia. Gen ức chế được gọi là gen át, còn gen bị ức chế dược gọi là gen khuất. Tuỳ thuộc vào gen át là gen trội hay gen lặn mà F2 có thể có các tỷ lệ PL 13:3, 12:3:1 hoặc 9:3:4. - Tương tác cộng gộp: Kiểu tương tác trong đó mỗi alen trội (hoặc lặn) của mỗi gen đóng góp một phần vào sự hình thành tính trạng. Kiểu tương tác này đặc trưng cho các tính trạng số lượng. Với hai gen tương tác cộng gộp, F2 sẽ có tỷ lệ PL KH là 1:4:6:4:1. Nếu KH không phụ thuộc vào số lượng alen trội trong KG, ta có tỷ lệ PL 15:1 ở F2. Tuy nhiên cũng có những trường hợp hai gen tương tác nhưng lại cùng nằm trên một nhiễm sắc thể. Khi đó, ngoài QL tương tác, các gen còn chịu sự chi phối của QL liên kết và hoán vị gen. b. Các ví dụ: 257 Ở ruồi giấm, tính trạng mắt trắng do gen lặn nằm trên NST giới tính X ở vùng không tương đồng với Y, alen trội tương ứng qui định mắt đỏ. Thế hệ xuất phát cho giao phối ruồi cái mắt đỏ dị hợp với ruồi đực mắt đỏ sau đó cho F1 tạp giao. Tỉ lệ phân tính ở F2 là A. 13 đỏ: 3 trắng B. 11đỏ: 5 trắng C. 5 đỏ: 3 trắng D. 3 đỏ: 1 trắng Bài giải Gọi a là gen qui định tính trạng mắt trắng (gen lặn nằm trên NST giới tính X ở vùng không tương đồng với Y), alen trội tương ứng A qui định mắt đỏ P: XA Xa x XA Y F1: 1 XA XA: 1 XA Xa : 1 XA Y : 1XaY Cho F1 tạp giao, tỉ lệ giao tử cái là 3/4 XA : 1/4 Xa tỉ lệ giao tử đực là 1/4 XA : 1/4 Xa : 2/4 Y F2 thu được tỉ lệ mắt trắng là : (1/4 Xa) x (1/4 Xa ) + (1/4 Xa ). (2/4 Y) = 1/16 + 2/16 = 3/16 Đáp án A. 13 đỏ: 3 trắng IV. Hệ thống các bài tập tự giải D d Bài 1 (ĐH 2011) : Trong quá trình giảm phân ở một cơ thể có KG AaBbX e X E đ đã xảy ra hoán vị gen giữa các alen D và d với tần số 20%. Cho biết không xảy d ra đột biến, tính theo lí thuyết, tỉ lệ loại giao tử abX e được tạo ra từ cơ thể này là : A. 2,5% B. 5,0% C.10,0% D. 7,5% Bài 2 (ĐH 2011): Ở ruồi giấm, alen A quy định thân xám trội hoàn toàn so với alen a quy định thân đen; alen B quy định cánh dài trội hoàn toàn so với alen b quy định cánh cụt. Các gen quy định màu thân và hình dạng cánh đều nằm trên một nhiễm sắc thể thường. Alen D quy định mắt đỏ trội hoàn toàn so với alen d quy định mắt trắng nằm trên đoạn không tương đồng của nhiễm sắc thể giới tính X. Cho giao phối ruồi cái thân xám, cánh dài, mắt đỏ với ruồi đực thân xám, cánh dài, mắt đỏ (P), trong tổng số các ruồi thu được ở F1, ruồi có KH thân đen, cánh cụt, mắt trắng chiếm tỉ lệ 2,5%. Biết rằng không xảy đột biến, tính theo lí thuyết, tỉ lệ KH thân xám, cánh dài, mắt đỏ ở F1 là: A.7,5% B. 45,0% C.30,0% D. 60,0% Bài 3: (ĐH 2012) Ở ruồi giấm, alen A quy định thân xám trội hoàn toàn so với alen a quy định thân đen; alen B quy định cánh dài trội hoàn toàn so với alen b quy định cánh cụt; alen D quy định mắt đỏ trội hoàn toàn so với alen d quy định mắt trắng. AB AB Thực hiện phép lai P: ab X X × ab X Y thu được F1. Trong tổng số các ruồi ở F1, ruồi thân xám, cánh dài, mắt đỏ chiếm tỉ lệ là 52,5%. Biết rằng không xảy ra đột biến, theo lí thuyết, ở F1 tỉ lệ ruồi đực thân xám, cánh cụt, mắt đỏ là A. 3,75% B. 1,25% C. 2,5% D. 7,5% Bài 4 (ĐH 2009): Ruồi giấm A thân xám, a thân đen, B cánh dài, b cánh cụt cùng nằm trên một cặp NST. D mắt đỏ, d mắt trắng nằm trên X, không có alen tương ứng trên Y. Phép lai AB//ab XDXd x AB//ab XDY cho F1 thân đen, cánh D d D 258 cụt, mắt đỏ chiếm tỉ lệ 11,25%. Tính theo lí thuyết, tỉ lệ ruồi đực thân xám, cánh cụt, mắt đỏ là: A. 3,75% B. 5% C. 15% D. 2,5% Bài 5. Ở chim, P thuần chủng: lông dài, xoăn x lông ngắn, thẳng, F1 thu được toàn lông dài, xoăn. Cho chim trống F1 giao phối với chim mái chưa biết KG, F2 xuất hiện 20 chim lông ngắn, thẳng: 5 lông dài, thẳng: 5 lông ngắn, xoăn. Tất cả chim trống của F2 đều có lông dài, xoăn. Biết 1 gen quy định 1 tính trạng. Tần số HVG của chim trống F1 là: A. 5% B. 25% C. 10% D. 20% Bài 6 (Thi thử ĐH Nguyễn Huệ 2011): Cho biết mỗi cặp tính trạng do một cặp gen quy định và di truyền trội hoàn toàn; tần số hoán vị gen giữa A và B là 20%, Ab Ab d D d X giữa D và E không có hoán vị gen. Xét phép lai aB X E e × ab X E Y, tính theo d lý thuyết, các cá thể con có mang A, B và có cặp nhiễm sắc thể giới tính là X E X de ở đời con chiếm tỉ lệ A. 7,5%. B. 12,5%. C. 18,25%. D. 22,5%. Bài 7. Cho biết mỗi cặp tính trạng do một cặp gen quy định và di truyền trội Ab Ab D d hoàn toàn; tần số hoán vị gen giữa A và B là 20%. Xét phép lai aB X E X E × ab X dE Y, KH A-bbddE- ở đời con chiếm tỉ lệ A. 45%. B. 35%. C. 40%. D. 22,5%. DE M m Bài 8: Trong quá trình giảm phân ở cơ thể có KG AaBB de X n X N đã xảy ra hoán vị giữa alen D và d với tần số 40%; giữa alen M và m với tần số 20%. Cho M biết không xảy ra đột biến, tính theo lý thuyết thì loại giao tử ABDE X N được tạo ra từ cơ thể này chiếm tỉ lệ: A. 1,5% B. 4,5% C. 1% D. 3% Bài 9 (thi thử ĐH Lam Kinh 2011): ở ruồi giấm gen A quy định thân xám là trội hoàn toàn so với a thân đen, gen B quy định cánh dài là trội hoàn toàn so với b cánh cụt, gen D quy định mắt đỏ là trội hoàn toàn so với d mắt trắng? Phép lai AB AB giữa ruồi giấm ab XDXd với ruồi giấm ab XDY cho F1 có KH thân đen, cánh cụt, mắt trắng chiếm tỷ lệ = 5%. Tần số hoán vị gen là: A. 35%. B. 20%. C. 40%. D. 30%. Bài 10: Ở ruồi giấm, alen A quy dịnh mắt đỏ là trội hoàn toàn so với alen a quy định mắt trắng. Tính theo lí thuyết, phép lai nào sau đây cho đời con có tỉ lệ KH là 3 ruồi mắt đỏ : 1 ruồi mắt trắng? A a a A A a a a A A.XAXa x XAY. B. X X ×X Y . C. X X ×X Y . D. X X ×X Y Bài 11: Ở ruồi giấm, allele A quy định thân xám trội hoàn toàn so với allele a quy định thân đen; allele B quy định cánh dài trội hoàn toàn so với allele b quy định cánh cụt; allele D quy định mắt đỏ trội hoàn toàn so với allele d quy định mắt trắng. Theo lí thuyết, phép lai: 259 Ab aB X DY × Ab ab X D X d cho đời con có tỉ lệ ruồi đực thân xám, cánh cụt, mắt đỏ là A. 12,5% B. 6,25% C. 18,75% D. 25% Bài 12. Ở một loài động vật, alen A quy định lông xám trội hoàn toàn so với alen a quy định lông hung; alen B quy định chân cao trội hoàn toàn so với alen b quy định chân thấp; alen D quy định mắt nâu trội hoàn toàn so với alen d quy định mắt đen. Phép lai P : ♀ AB D d Ab d X X ×♂ X Y thu được F1. Trong tổng số cá ab aB thể F1, số cá thể cái có lông hung, chân thấp, mắt đen chiếm tỉ lệ 1%. Biết quá trình giảm phân không xảy ra đột biến nhưng xảy ra hoán vị gene ở cả hai giới với tần số như nhau. Theo lí thuyết, số cá thể lông xám dị hợp, chân thấp, mắt nâu ở F1 chiếm tỉ lệ A. 8,5% B. 17% C. 2% D. 10% Bài 13.Giả sử mỗi gen quy định một tính trạng, gen trội là trội hoàn toàn. Tính theo lí thuyết, trong các phép lai sau đây, phép lai cho đời con có tỉ lệ phân li kiểu hình giống với tỉ lệ phân li kiểu gen là: A. AaXBXb x AaXbY B. XAXa x XAY C. Aabb x aaBb D. Ab/ab x AB/ab Bài 14. Ở ruồi giấm, allele A quy định thân xám trội hoàn toàn so với allele a quy định thân đen; allele B quy định cánh dài trội hoàn toàn so với allele b quy định cánh cụt; allele D quy định mắt đỏ trội hoàn toàn so với allele d quy định mắt trắng. Thực hiện phép lai P: AB ab X DX d × AB ab X DY thu được F1. Trong tổng số các ruồi ở F1, ruồi thân xám, cánh dài, mắt đỏ chiếm tỉ lệ là 52,5%. Biết rằng không xảy ra đột biến, theo lí thuyết, ở F 1 tỉ lệ ruồi đực thân xám, cánh cụt, mắt đỏ là A. 3,75% B. 1,25% C. 2,5% D. 7,5% Bài 15. Ở gà, gene qui định màu sắc lông nằm trên vùng không tương đồng của nhiểm sắc thể giới tính X có hai allele: allele A qui định lông vằn trội hoàn toàn so với allele a qui định lông đen. Cho gà trống lông vằn thuần chủng giao phối với gà mái lông đen thu được F1. Cho F1 giao phối với nhau thu được F2. Khi nói về kiểu hình ở F2, theo lý thuyết, kết luận nào sau đây không đúng? A.Gà trống lông vằn có tỉ lệ gấp đôi gà mái lông đen. B. Gà trống lông vằn có tỉ lệ gấp đôi gà mái lông vằn. C. Tất cả các gà lông đen đều là gà mái. D.Gà lông vằn và gà lông đen có tỉ lệ bằng nhau D. KẾT LUẬN Qua vài năm thực hiện giảng dạy và hướng dẫn học sinh ôn thi đại học, cũng như tham gia bồi dưỡng học sinh giỏi, khả năng tiếp thu và vận dụng vào giải các bài tập liên quan đến qui luật di truyền liên kết với giới tính đạt những kết quả đáng mừng : Số học sinh hiểu bài và vận dụng giải bài tập có hiệu quả cao 260 Đa số HS tỏ ra khá tự tin khi giải quyết các bài tập về qui luật di truyền liên kết với giới tính sau khi đã được tiếp cận với nội dung phương pháp giải các dạng bài tập nêu trong chuyên đề này. Chuyên đề trên chỉ là ý tưởng của cá nhân tôi dựa trên những kinh nghiệm của bản than và tham khảo một số đồng nghiệp nên không thể tránh khỏi thiếu sót. Rất mong được sự đóng góp ý kiến của các đồng nghiệp khác để giúp tôi hoàn thiện chuyên đề. Tôi xin chân thành cảm ơn! 261 262 [...]... Số cách phân công các thanh niên tình nguyện về tỉnh thứ nhất là C31C124 Với mỗi cách phân công các thanh niên tình nguyện về tỉnh thứ nhất ta có số cách phân công các thanh niên tình nguyện về tỉnh thứ hai là C21C84 Với mỗi cách phân công các thanh niên tình nguyện về tỉnh thứ nhất và tỉnh thứ hai ta có số cách phân công các thanh niên tình nguyện về tỉnh thứ ba là C11C44 Vậy có tất cả các cách... sau: Học sinh lớp A Học sinh lớp B Học sinh lớp C Số cách chọn tương ứng 2 1 1 1 2 1 1 1 2 C52 C41 C31 = 120 cách C51.C42 C31 = 90 cách C51.C41.C32 = 60 cách Vậy có tất cả là 120+90+60=270 cách b/ Số cách chọn 4 học sinh từ 12 học sinh đã cho là C124 = 495 Số cách chọn 4 học sinh mà mỗi lớp có 1 học sinh là 270 cách chọn Nên số cách chọn 4 học sinh thuộc không quá 2 trong 3 lớp phải là : 495-270=225 cách...  4 ( ) 17 - Tên chuyên đề: CÁC DẠNG TOÁN VỀ PHÉP ĐẾM - Tác giả chuyên đề: NGUYỄN THỊ THANH HẢI - Chức vụ : Giáo viên Toán - Đơn vị công tác: Trường THPT Vĩnh Yên - Đối tượng học sinh bồi dưỡng: lớp 11, lớp 12 - Dự kiến số tiết bồi dưỡng: 10 tiết I HỆ THỐNG KIẾN THỨC SỬ DỤNG TRONG CHUYÊN ĐỀ: 1 Quy tắc cộng: Một công việc A được chia ra k công việc A1 , A2 , , Ak để thực hiện, mỗi công việc độc lập... lập nhau Trong đó: + Công việc A1 có n1 cách thực hiện 18 + Công việc A2 có n2 cách thực hiện ………………………………… + Công việc Ak có nk cách thực hiện Khi đó số cách thực hiện công việc A là : ( n1 + n2 + + nk ) cách 2 Quy tắc nhân: Một công việc A được thực hiện lần lượt qua k giai đoạn A1 , A2 , , Ak , với mỗi cách thưck hiện ở giai đoạn này không trùng với bất kỳ cách thực hiện nà ở các giai đoạn còn lại... bao nhiêu cách tặng? *Lời giải : a/ Cách 1: Chọn 6 quyển sách bất kỳ từ 5 quyển sách văn học và 4 quyển âm nhạc là một tổ hợp chập 6 của 9 phần tử => có C96 cách chọn Với mỗi cách chọn như vậy sẽ có 6! cách tặng Vậy số cách tặng là: C96 6!=60480 cách tặng Cách 2: Số cách tặng 6 quyển sách theo yêu cầu bài toán là một chỉnh hợp chập 6 của 9 phần tử Vậy số cách tặng là: A96 =60480 cách tặng b/ Cách 1:... gồm 5 học sinh lớp A, 4 học sinh lớp B và 3 học sinh lớp C Cần chọn 4 học sinh đi làm nhiệm vụ sao cho: a/ 4 học sinh này thuộc cả 3 lớp trên b/ 4 học sinh này thuộc không quá 2 trong 3 lớp Hỏi có bao nhiêu cách chọn như vậy? *Phân tích: Số học sinh được chọn từ 12 học sinh không sắp xếp thứ tự gì nên ta có thể sử dụng bài toán 2 *Lời giải: a/Vì 4 học sinh được chọn cần ở cả 3 lớp nên ta có các trường... là tập gồm 5 chữ số khác 0 Số cách chọn P là C95 Với mỗi cách chọn P như trên thì: c có 1 cách chọn abde có 4! cách chọn Do đó với mỗi tập P như trên ta được 4! số x cần tìm Vậy số x được lập trong trường hợp này là : C95 4! (số) *Gọi Q là tập chứa chữ số 0 và 4 chữ số khác 0 Số cách chọn Q là C94 29 Với mỗi cách chọn Qnhư trên thì: c có 1 cách chọn a có 3 cách chọn bde có 3 cách chọn Do đó với mỗi tập. .. và cô Bình không thể rời nhau là: 3 5 C28 + C28 =101556 cách chọn Cách 2: -Chọn 5 người tuỳ ý trong 30 người =>có C305 cách chọn -Chọn 5 người trong đó có Tâm mà không có Bình =>có C284 cách chọn -Chọn 5 người trong đó có Bình mà không có Tâm =>có C284 cách chọn Vậy có tất cả số cách chọn ban cán sự để Cậu Tâm và cô Bình không thể rời nhau là: C305 -( C284 + C284 )=101556 cách chọn c/ Cách 1: 5 -Chọn... -Chọn 5 người trong đó không có An mà không có Hà =>có C28 cách chọn -Chọn 5 người trong đó có An mà không có Hà =>có C284 cách chọn -Chọn 5 người trong đó có Hà mà không có An =>có C284 cách chọn Vậy có tất cả số cách chọn ban cán sự để Cậu An và Hà không thể làm việc chung với nhau là: 5 C28 + C284 + C284 =139230 cách chọn Cách 2: -Chọn 5 người tuỳ ý trong 30 người =>có C305 cách chọn 3 -Chọn 5 người... văn học và 4 quyển âm nhạc là một tổ hợp chập 6 của 9 phần tử => có C96 cách chọn Với mỗi cách chọn như vậy sẽ có 6! cách tặng Vậy số cách tặng là: C96 6!=60480 cách tặng Cách 2: Số cách tặng 6 quyển sách theo yêu cầu bài toán là một chỉnh hợp chập 6 của 9 phần tử Vậy số cách tặng là: A96 =60480 cách tặng 2 Bài toán đếm không có sắp xếp Ví dụ 4: 23 Đội thanh niên xung kích của trường X có 12 học ... ễN THI I HC MễN: TON NG DNG HM S TRONG GII PHNG TRèNH, H PHNG TRèNH V BT PHNG TRèNH GIO VIấN: Đại Văn Hải T: TON L- KT TRNG: THPT VNH YấN-TP VNH YấN-VNH PHC Vnh Yờn, thỏng nm 2014 CHUYấN ễN THI. .. Vic biờn son h thng hng dn cho hc sinh ụn thi i hc tỏc phm CHớ Phốo ca Nam Cao Ng Vn Nguyn Vn L Dy k nng lm mt s gii phỏp quan trng giỳp hc sinh lp 12 thi i hc t kt qu cao Lch s Nguyn Tuyt Mai... xut hin thng xuyờn cỏc thi i hc-Cao ng cú li gii dựng phng phỏp hm s Phng phỏp hm s thng da vo mt s tớnh cht ca hm s v th hm s nh vo dựng o hm ( lp 12) kho sỏt chiu bin thi n ca hm s xột nghim

Ngày đăng: 23/10/2015, 09:37

Từ khóa liên quan

Mục lục

  • SỞ GD & ĐT VĨNH PHÚC

  • A. LÝ DO CHỌN ĐỀ TÀI

  • Sinh học là môn khoa học chuyên nghiên cứu các quá trình sống ,các hiện tượng di truyền và biến dị ,các quy luật di truyền của sinh vật .Đặc thù của bộ môn là nghiên cứu từ thực nghiệm ,từ đó rút ra được các quy luật chung cho toàn bộ sinh giới.

Tài liệu cùng người dùng

Tài liệu liên quan